05.03.2015 Views

clerkship handbook - University of Hawaii – Department of Medicine

clerkship handbook - University of Hawaii – Department of Medicine

clerkship handbook - University of Hawaii – Department of Medicine

SHOW MORE
SHOW LESS

You also want an ePaper? Increase the reach of your titles

YUMPU automatically turns print PDFs into web optimized ePapers that Google loves.

UNIVERSITY OF HAWAII<br />

JOHN A. BURNS SCHOOL OF MEDICINE<br />

DEPARTMENT OF MEDICINE<br />

THIRD-YEAR CLERKSHIP IN<br />

INTERNAL MEDICINE<br />

MEDICINE 531 (6B)<br />

MEDICINE 532 (6L)<br />

CLERKSHIP HANDBOOK<br />

Revised 02/25/13<br />

1


TABLE OF CONTENTS<br />

CONTACT INFORMATION ................................................................................................................................................... 3<br />

SUMMARY TABLE OF CLERKSHIP REQUIREMENTS ................................................................................................. 4<br />

TOP 10 WAYS TO EXCEL ON THE INTERNAL MEDICINE CLERKSHIP ................................................................. 5<br />

DESCRIPTION OF THE THIRD-YEAR CLERKSHIP IN INTERNAL MEDICINE ...................................................... 6<br />

INTRODUCTION ..................................................................................................................................................................... 6<br />

Goal <strong>of</strong> the Clerkship ........................................................................................................................................ 6<br />

Design <strong>of</strong> the Clerkship .................................................................................................................................... 6<br />

Work Hours, Designated Study Time, Days Off and Holidays ..................................................................... 6<br />

Absences ........................................................................................................................................................... 7<br />

CURRICULUM ....................................................................................................................................................... 8<br />

Learning Objectives .......................................................................................................................................... 9<br />

Learning Strategies .......................................................................................................................................... 9<br />

EVALUATION IN THE THIRD-YEAR CLERKSHIP IN INTERNAL MEDICINE ..................................................... 9<br />

General Guidelines ........................................................................................................................................... 9<br />

Retaking Exams .............................................................................................................................................. 10<br />

Honors.............................................................................................................................................................. 10<br />

SPECIFIC REQUIREMENTS AND GUIDELINES................................................................................................ 12<br />

Inpatient <strong>Medicine</strong> ........................................................................................................................................... 12<br />

Ambulatory <strong>Medicine</strong> ...................................................................................................................................... 16<br />

PBL Tutorial ..................................................................................................................................................... 18<br />

Role Descriptions for <strong>Medicine</strong> 531/532 ....................................................................................................... 19<br />

MISCELLANEOUS CLERKSHIP INFORMATION ......................................................................................................... 24<br />

GUIDELINES FOR APPROPRIATE APPEARANCE AND ATTIRE .................................................................... 25<br />

RECOMMENDED RESOURCES ......................................................................................................................... 26<br />

REQUIRED EQUIPMENT .................................................................................................................................... 27<br />

EXPOSURE TO BLOOD/BODY FLUIDS PROTOCOL ....................................................................................... 28<br />

HIV MEDICINE ..................................................................................................................................................... 29<br />

MEDICINE T-RES INSTRUCTIONS .................................................................................................................... 32<br />

STUDENT EVALUATION OF TEACHERS .......................................................................................................... 34<br />

EVALUATION FORMS ......................................................................................................................................................... 35<br />

OBSERVED HISTORY AND PHYSICAL ............................................................................................................. 36<br />

THE BASIC PHYSICAL EXAMINATION SEQUENCE (BPES) ........................................................................... 37<br />

OBSERVED PATIENT COUNSELING ................................................................................................................ 48<br />

CASE PRESENTATION ....................................................................................................................................... 49<br />

SMALL GROUP LEARNING EXPERIENCE ....................................................................................................... 50<br />

MID-CLEKSHIP FEEDBACK FORM ................................................................................................................... 51<br />

STUDENT EVALUATION FORM ......................................................................................................................... 52<br />

OTHER FORMS ...................................................................................................................................................................... 56<br />

INPATIENT MEDICINE AGREEMENT FORM ..................................................................................................... 57<br />

INPATIENT WORK HOURS LOG ........................................................................................................................ 59<br />

PHYSICIAN’S ORDERS ...................................................................................................................................... 60<br />

APPENDICES .......................................................................................................................................................................... 61<br />

TRAINING PROBLEMS LIST .............................................................................................................................. 62<br />

GENERAL CLINICAL CORE COMPETENCIES IN INTERNAL MEDICINE ..................................................... 192<br />

LIST OF ERROR-PRONE ABBREVIATIONS, SYMBOLS, AND DOSE DESIGNATIONS ............................... 234<br />

COMPREHENSIVE WRITE-UPS ....................................................................................................................... 238<br />

Definition <strong>of</strong> Comprehensive Write-up ....................................................................................................... 238<br />

Submission Requirements <strong>of</strong> Comprehensive Write-ups ........................................................................ 238<br />

Example <strong>of</strong> Inpatient History and Physical ................................................................................................ 239<br />

Example <strong>of</strong> Ambulatory Note ....................................................................................................................... 246<br />

CLINICAL SKILLS EXAMINATION (CSE) ........................................................................................................ 251<br />

NBME SUBJECT EXAM IN INTERNAL MEDICINE .......................................................................................... 256<br />

2


CONTACT INFORMATION<br />

Clerkship Director<br />

Clerkship Coordinator:<br />

Hospital Site Coordinators:<br />

Laurie M. Tam, M.D., F.A.C.P.<br />

Assistant Pr<strong>of</strong>essor <strong>of</strong> <strong>Medicine</strong><br />

1356 Lusitana Street, 7 th Fl.,<br />

Honolulu, HI 96813<br />

Telephone: (808) 586-7460 FAX: 586-7486<br />

e-mail: lmtam@hawaii.edu<br />

Ms. Julieta Rajlevsky<br />

1356 Lusitana Street, 7 th Fl.<br />

Honolulu, HI 96813<br />

Telephone: (808) 586-7478 FAX: 586-7486<br />

e-mail: jlrajlev@hawaii.edu<br />

Queens Medical Center:<br />

Miki Kiyokawa, M.D.<br />

Assistant Pr<strong>of</strong>essor <strong>of</strong> <strong>Medicine</strong><br />

QET9, 1301 Punchbowl Street,<br />

Honolulu, HI 96813<br />

Telephone: 586-2910 FAX: 586-7486<br />

e-mail: kmcsitecoordinator@yahoo.com<br />

Kuakini Medical Center:<br />

Miki Kiyokawa, M.D.<br />

Assistant Pr<strong>of</strong>essor <strong>of</strong> <strong>Medicine</strong><br />

Office <strong>of</strong> Medical Education<br />

347 N. Kuakini St., Honolulu, HI 96817<br />

Telephone: 547-9226 FAX: 547-9867<br />

e-mail: kmcsitecoordinator@yahoo.com<br />

Tripler Army Medical Center:<br />

Bethany Sonobe, MD<br />

Internal <strong>Medicine</strong><br />

Telephone: 433-2638 Pager: 363-1423<br />

e-mail: bethany.sonobe@amedd.army.mil<br />

3


Third-Year Clerkship in Internal <strong>Medicine</strong><br />

SUMMARY TABLE OF CLERKSHIP REQUIREMENTS<br />

Inpatient Ambulatory<br />

1. Call<br />

a. Every time your team is on call 6B & 6L -<br />

6B & 6L<br />

-<br />

b. Overnight call once (preferably Friday night)<br />

at KMC<br />

2. Comprehensive Write-ups<br />

a. 1 Comprehensive Write-up per week<br />

6B & 6L<br />

(3 - 6 total)<br />

b. 2 Comprehensive Write-ups per week (10 total) 6B<br />

c. 2 Comprehensive Write-ups per month (10 total) - 6L<br />

3. Required Clerkship Activities<br />

a. UH <strong>Department</strong> <strong>of</strong> <strong>Medicine</strong> Grand Rounds<br />

(designated Tues, 12:30-1:30 pm)<br />

6B & 6L<br />

(All students<br />

except TAMC<br />

6B<br />

(All students<br />

except TAMC)<br />

b. PBL Tutorial 6B (1x/week)<br />

6L (1x/week)<br />

c. Bedside Clinical Skills (1-2x/week) 6B & 6L -<br />

d. Chief Medical Resident Rounds (1x/week) 6B & 6L -<br />

e. CVPE (usually Tues pm) 6L 6B<br />

f. EBM 1 and 2 (usually Wed pm) 6L 6B<br />

g. EKG Workshop (usually Tues pm) 6L 6B<br />

h. HIPSTER (Sim Session) 6B & 6L -<br />

6B (1x/week)<br />

6L (1x/month)<br />

i. HIV <strong>Medicine</strong> (Tues 1:30-5:00 pm) 6L 6B<br />

j. Neuro 1 and 2 (usually Tues pm) 6L 6B<br />

k. Site-specific conferences 6B & 6L 6B & 6L<br />

4. Learning Objectives<br />

a. Training Problems (33) 6B & 6L 6B & 6L<br />

b. General Clinical Core Competencies in IM (17) 6B & 6L 6B & 6L<br />

5. Evaluation<br />

a. Observed History and Physical (with BPES)<br />

6B & 6L -<br />

(by end <strong>of</strong> 2 nd wk <strong>of</strong> inpatient)<br />

b. Observed Patient Counseling (2) 6B & 6L<br />

c. Mid-Clerkship Feedback Form 6B & 6L 6B & 6L<br />

d. Student Evaluation Forms 6B & 6L 6B & 6L<br />

e. Clinical Skills Exam (Saturday, ____________ ) End <strong>of</strong> Clerkship<br />

f. NBME Exam (Friday, ____________________) End <strong>of</strong> Clerkship<br />

4


TOP 10 WAYS TO EXCEL ON THE INTERNAL MEDICINE CLERKSHIP<br />

1. Find out what your residents and preceptors expect <strong>of</strong> you. Meet and try to exceed their<br />

expectations. Follow through on every assigned task.<br />

2. Be actively involved in the care <strong>of</strong> your patients to the greatest extent possible. Go the<br />

extra mile for your patients. You will benefit as much as they will.<br />

3. Go the extra mile for your team. Additional learning will follow. The more you put in, the<br />

more you will gain.<br />

4. Read consistently and deeply about the problems your patients face. Raise what you<br />

learn in your discussions with your team and in your notes. Educate your team members<br />

about what you learn whenever possible.<br />

5. Learn to do excellent presentations as early as possible. This will make you more effective<br />

in patient care and gain the confidence <strong>of</strong> your supervisors to allow you more involvement<br />

in patient care.<br />

6. Ask good questions.<br />

7. Speak up—share your thoughts in teaching sessions, share your opinions about your<br />

patients’ care, constructively discuss how to improve the education you are receiving and<br />

the systems around you.<br />

8. Actively seek feedback and reflect on your experiences.<br />

9. Keep your goals focused on the right priorities, in the following order: patient care,<br />

learning, and personal satisfaction. You should always strive to meet all three goals.<br />

10. Always be enthusiastic. Be caring and conscientious and strive to deliver outstanding<br />

quality to your patients as you learn as much as you can from every experience.<br />

From: Primer to the Internal <strong>Medicine</strong> Clerkship, Second Edition, A Guide Produced by the<br />

Clerkship Directors in Internal <strong>Medicine</strong> (CDIM), c2008, 2nd edition<br />

Download free <strong>of</strong> charge from:<br />

http://www.im.org/Publications/PhysiciansInTraining/Pages/Primer.aspx<br />

5


DESCRIPTION OF THE THIRD-YEAR CLERKSHIP IN INTERNAL MEDICINE<br />

INTRODUCTION<br />

Goal <strong>of</strong> the Clerkship<br />

The goal <strong>of</strong> the Third-Year Clerkship in Internal <strong>Medicine</strong> is to provide the opportunity to develop<br />

experience and competence in Internal <strong>Medicine</strong>, which serves as the foundation for all<br />

specialties and their respective residencies. To practice competently, a physician must obtain and<br />

retain a large body <strong>of</strong> knowledge, master the skills <strong>of</strong> clinical practice, develop efficient problem<br />

solving techniques and demonstrate compassion, integrity, self-discipline and life-long learning<br />

skills. It is the student’s responsibility to utilize this <strong>clerkship</strong> experience to accomplish these<br />

goals. It is the <strong>Department</strong> <strong>of</strong> <strong>Medicine</strong>’s responsibility to assure that every graduate <strong>of</strong> the John<br />

A. Burns School <strong>of</strong> <strong>Medicine</strong> has obtained graduation level competency in Internal <strong>Medicine</strong>.<br />

The core <strong>of</strong> this <strong>clerkship</strong> is “Patient-Based Learning,” which occurs as students evaluate patients<br />

through history taking and physical examinations, develop comprehensive assessments including<br />

appropriate differential diagnoses, formulate diagnostic, therapeutic and education plans and then<br />

provide care and follow-up appropriate to the inpatient or ambulatory setting.<br />

Design <strong>of</strong> the Clerkship<br />

Third-Year Clerkship in Internal <strong>Medicine</strong> (MED 531/532)<br />

MED 531 for 6B students is 11 weeks in length and consists <strong>of</strong> 5-1/2 or 6 weeks <strong>of</strong> Inpatient<br />

<strong>Medicine</strong> and 5-1/2 or 5 weeks <strong>of</strong> Ambulatory <strong>Medicine</strong>. MED 532 for 6L students consists <strong>of</strong> 6<br />

weeks <strong>of</strong> Inpatient <strong>Medicine</strong> (block) and 25 half days <strong>of</strong> ambulatory medicine (clinics - one half<br />

day weekly for 21-22 weeks plus 3-4 additional half days).<br />

Work Hours, Designated Study Time, Days Off and Holidays<br />

Work hours<br />

No student should work more than eighty (80) hours per week, averaged over the course <strong>of</strong> the<br />

<strong>clerkship</strong>.<br />

The <strong>clerkship</strong> recognizes that excessive work hours do not promote student well-being and may<br />

endanger students, their patients and others with whom they interact.<br />

For inpatient medicine, the earliest time that any student is permitted to arrive at the hospital is<br />

4:00 a.m. and the earliest time that any student is permitted to see any patient, that is, to actually<br />

talk to and examine, is 4:30 a.m. (excluding emergency situations such as Code Blues). (See<br />

Inpatient <strong>Medicine</strong>)<br />

The Hospital Site Coordinators and Chief Medical Residents are aware <strong>of</strong> these work hour<br />

guidelines.<br />

If a student is not able to complete his/her work within these work hour guidelines, the student is<br />

advised to see his/her Hospital Site Coordinator, Chief Medical Resident or Supervising Resident.<br />

6


If a student does not follow these guidelines, the Hospital Site Coordinator and Chief Medical<br />

Resident are required to advise the student and notify the <strong>clerkship</strong>.<br />

It is the <strong>clerkship</strong>'s goal that students will become more efficient with experience, that they will be<br />

able to arrive at the hospital and see patients at reasonable times and that they will be able to<br />

maintain reasonable work hours.<br />

"Designated study time"<br />

Students should have one (1) afternoon <strong>of</strong> "designated study time" per week, averaged over<br />

the course <strong>of</strong> the <strong>clerkship</strong>.<br />

"Designated study time" is defined as time Monday through Friday and morning through<br />

afternoon away from patient care responsibilities that is devoted to studying which includes<br />

reading, completing write-ups, preparing for required <strong>clerkship</strong> activities such as PBL Tutorials,<br />

Bedside Clinical Skills rounds, Chief Rounds and other inpatient or ambulatory activities.<br />

Students will have "designated study time" when their patient care responsibilities and any<br />

required activities are finished. (See Summary Table <strong>of</strong> Required <strong>Medicine</strong> Clerkship Activities)<br />

In both the inpatient and ambulatory settings, students are advised to notify their team and<br />

preceptor(s), respectively, when they are leaving to study.<br />

"Designated study time" should be used for educational activities, as described above, and not for<br />

leisure activities. Therefore, academic action may be initiated against students who abuse the<br />

"designated study time," as determined by the <strong>Department</strong> <strong>of</strong> <strong>Medicine</strong> Student Education<br />

Committee. Likewise, inpatient and ambulatory sites that do not follow the guidelines on<br />

"designated study time" should be brought to the attention <strong>of</strong> the <strong>clerkship</strong> director.<br />

The one day before the NBME Exam is designated a full day <strong>of</strong> “designated study time” for<br />

<strong>clerkship</strong> students.<br />

Days <strong>of</strong>f<br />

Students must have one (1) day <strong>of</strong>f per week, averaged over the course <strong>of</strong> the <strong>clerkship</strong>.<br />

On Inpatient <strong>Medicine</strong>, there is one (1) day <strong>of</strong>f each week, usually a Saturday or a Sunday. The<br />

day <strong>of</strong>f will be determined by the site’s Hospital Site Coordinator and Chief Medical Resident.<br />

On Ambulatory <strong>Medicine</strong>, there are usually two (2) days <strong>of</strong>f each week, usually Saturday and<br />

Sunday. The days <strong>of</strong>f will be determined by the Ambulatory Preceptor.<br />

Holidays<br />

Students will follow the holiday schedule that is observed at their training site.<br />

Absences<br />

Absences from Inpatient <strong>Medicine</strong><br />

On each day that you are absent, you must notify your Chief Medical Resident and your Hospital<br />

Site Coordinator.<br />

You may be required to make up any time missed. This decision will be made together by your<br />

Chief Medical Resident, Hospital Site Coordinator and Clerkship Director and is dependent on the<br />

reason for absence, length <strong>of</strong> absence, <strong>clerkship</strong> activities missed and your <strong>clerkship</strong><br />

7


performance to date.<br />

Absences from Ambulatory <strong>Medicine</strong><br />

On each half day that you are absent, you must notify your Ambulatory Preceptor and the Clerkship<br />

Director (Dr. Laurie Tam at lmtam@hawaii.edu).<br />

Each half day missed must be made up.<br />

6B students usually have 2-4 weekday afternoons and every Saturday <strong>of</strong>f each week. They<br />

should try to schedule their make up during these times, at the convenience <strong>of</strong> their Ambulatory<br />

Preceptor.<br />

6L ambulatory students usually have 3 half days and every Saturday <strong>of</strong>f each week. They should<br />

try to schedule their make up during these times, at the convenience <strong>of</strong> their Ambulatory<br />

Preceptor.<br />

Notes<br />

Attendance is mandatory for all <strong>Medicine</strong> Clerkship orientations and exams.<br />

If you are absent for more than three (3) days, totaled over the course <strong>of</strong> the <strong>clerkship</strong>, the<br />

<strong>clerkship</strong> is required to report this to the Office <strong>of</strong> Student Affairs.<br />

Required make up for time missed must be completed by the end <strong>of</strong> <strong>Medicine</strong> Clerkship in order<br />

to receive Credit for the <strong>clerkship</strong>. Ideally, required make up should be completed in the same<br />

half <strong>of</strong> the <strong>clerkship</strong> that time was missed.<br />

Questions regarding absences should be directed to the Clerkship Director (Dr. Laurie Tam at<br />

lmtam@hawaii.edu).<br />

CURRICULUM<br />

The <strong>Medicine</strong> Clerkship Curriculum is based on the Clerkship Directors in Internal <strong>Medicine</strong><br />

(CDIM)-Society <strong>of</strong> General Internal <strong>Medicine</strong> (SGIM) Core <strong>Medicine</strong> Clerkship Curriculum<br />

Guide Version 3.0, which was released in 2006 to all <strong>Medicine</strong> <strong>clerkship</strong> directors in the United<br />

States. The third edition outlines thirty-three (33) Training Problems and seventeen (17)<br />

General Clinical Core Competencies, which are aligned with the Accreditation Council for<br />

Graduate Medical Education (ACGME) general competencies. The Training Problems and<br />

General Clinical Core Competencies are printed in the <strong>clerkship</strong>’s Student Handbook and<br />

website.<br />

The <strong>Medicine</strong> Clerkship Curriculum, which has been reviewed by JABSOM’s <strong>Department</strong><br />

<strong>of</strong> <strong>Medicine</strong> Student Education Committee (SEC), is aligned with and fulfills JABSOM’s<br />

Graduation Objectives. The curriculum provides third-year medical students with the<br />

opportunity to develop and demonstrate competence and excellence in the medical<br />

knowledge, clinical skills and pr<strong>of</strong>essionalism expected in the evaluation and care <strong>of</strong> the<br />

adult patient.<br />

8


Learning Objectives<br />

a. Training Problems (See Appendix A)<br />

Appendix B)<br />

b. General Clinical Core Competencies in Internal <strong>Medicine</strong> (See<br />

Learning Strategies<br />

a. After seeing each patient, students should identify which Training<br />

Problems were manifested, read and study those Training Problems and assess whether<br />

they are able to meet the specific learning objectives for each (Appendix A).<br />

b. Students should read and study the General Clinical Core<br />

Competencies in Internal <strong>Medicine</strong> and assess whether they are able to meet the<br />

specific learning objectives for each (Appendix B).<br />

c. Students should refer to the Recommended Resources for their<br />

reading and studying. .<br />

EVALUATION IN THE THIRD-YEAR CLERKSHIP IN INTERNAL MEDICINE<br />

General Guidelines<br />

The <strong>clerkship</strong> uses a "competency-based" system to assess the three domains <strong>of</strong> Medical Knowledge,<br />

Skills and Pr<strong>of</strong>essionalism.<br />

For Medical Knowledge, competency is assessed throughout the <strong>clerkship</strong> by faculty and resident<br />

observations <strong>of</strong> students’ clinical performance. It is also assessed by performance on the NBME<br />

Subject Examination in Internal <strong>Medicine</strong> administered at the end <strong>of</strong> the <strong>clerkship</strong><br />

For Clinical Skills, competency is assessed throughout the <strong>clerkship</strong> by faculty and resident<br />

observations <strong>of</strong> students’ clinical performance. It is also assessed by performance on the Clinical<br />

Skills Exam (CSE) conducted by the Center for Clinical Skills near the end <strong>of</strong> the <strong>clerkship</strong>.<br />

For Pr<strong>of</strong>essionalism, competency is assessed throughout the <strong>clerkship</strong> by faculty and resident<br />

observations <strong>of</strong> students’ clinical performance.<br />

a. Hospital Site Coordinators and Ambulatory Preceptors give students mid-<strong>clerkship</strong> feedback on<br />

the students’ clinical performance, reviewing the students’ strengths and weaknesses and<br />

discussing specific strategies to improve. Therefore the <strong>clerkship</strong> expects students to<br />

demonstrate improvement in their clinical performance by the end <strong>of</strong> the <strong>clerkship</strong>.<br />

b. The UH <strong>Department</strong> <strong>of</strong> <strong>Medicine</strong> Student Education Committee reviews each student’s entire<br />

<strong>clerkship</strong> performance, including Clinical Performance (inpatient and ambulatory), CSE and<br />

NBME Exam to determine each student’s grade.<br />

c. The <strong>clerkship</strong> does not use numerical formulas for evaluation and grading.<br />

d. For questions about <strong>clerkship</strong> grades, refer to JABSOM’s Academic Appeals Policy.<br />

9


Credit<br />

a. To earn Credit for the Third-Year Clerkship in Internal <strong>Medicine</strong> (MED 531 for 6B students;<br />

MED 532 for 6L students), students must demonstrate Clerkship Level Competency in all three<br />

domains, as reflected by their Clinical Performance (inpatient and ambulatory), CSE and NBME<br />

Exam.<br />

For Clinical Performance (inpatient and ambulatory rotations), students need to receive ratings <strong>of</strong><br />

Competent or higher.<br />

On the CSE, students need to score 55 or higher.<br />

On the NBME Exam, students need to score 60 or higher.<br />

Students who do not demonstrate Clerkship Level Competency in all three domains by the end <strong>of</strong><br />

the <strong>clerkship</strong> may be required to repeat part or all <strong>of</strong> the <strong>clerkship</strong> and/or retake the CSE and/or<br />

NBME Exam.<br />

b. To earn Credit for Advanced <strong>Medicine</strong> (MED 541), which is a JABSOM graduation requirement,<br />

students must also demonstrate Graduation Level Competency on their CSE and NBME Exam<br />

by December 31 <strong>of</strong> the fourth year.<br />

On the CSE, students need to score 60 or higher.<br />

On the NBME Exam, students need to score 67 or higher.<br />

Students who do not demonstrate Graduation Level Competency on their CSE and/or NBME<br />

Exam by the end <strong>of</strong> the <strong>clerkship</strong> will be required to retake the respective exam. They may also<br />

be required to complete a designated fourth-year <strong>Medicine</strong> elective before retaking the exam.<br />

Retaking Exams<br />

CSE<br />

Students who score 55-59 on their CSE must retake the CSE and score 60 (Graduation Level<br />

Competency) or higher by December 31 <strong>of</strong> their fourth year. Students will have up to two (2)<br />

opportunities to achieve this.<br />

NBME Exam<br />

Students who score 60 - 66 on their NBME Exam must retake the NBME Exam and score 67<br />

(Graduation Level Competency) or higher by December 31 <strong>of</strong> their fourth year. Students will have up<br />

to two (2) opportunities to achieve this.<br />

Honors<br />

The <strong>Department</strong> <strong>of</strong> <strong>Medicine</strong> gives all students the opportunity to earn an "Honors" grade for exceptional<br />

performance in the Third-Year Clerkship in Internal <strong>Medicine</strong>.<br />

To earn Honors for the Third-Year Clerkship in Internal <strong>Medicine</strong> (MED 531 for 6B students; MED<br />

532 for 6L students), students must demonstrate mostly Exceptional performance in all three domains,<br />

as reflected by their Clinical Performance (inpatient and ambulatory), CSE and NBME Exam by the end<br />

<strong>of</strong> the <strong>clerkship</strong>.<br />

10


SUMMARY OF EVALUATION<br />

1. Medical Knowledge:<br />

o Clinical performance (Student Evaluation Form)<br />

(Interns, Residents, Chief Resident, Inpatient and Ambulatory Attendings)<br />

Credit for Credit for<br />

Clerkship Advanced <strong>Medicine</strong><br />

Competency (in 3 rd yr) (by 12/31 <strong>of</strong> 4 th yr) Consequence<br />

- - - Repeat the <strong>clerkship</strong>,<br />

partially or entirely<br />

Clerkship Level + +<br />

o NBME Subject Exam in Internal <strong>Medicine</strong> (Shelf Exam)<br />

Credit for Credit for<br />

NBME Clerkship Advanced <strong>Medicine</strong><br />

Score Competency (in 3 rd yr) (by 12/31 <strong>of</strong> 4 th yr) Consequence<br />


3. Pr<strong>of</strong>essionalism:<br />

o Clinical performance (reflected on Student Evaluation Form)<br />

(Interns, Residents, Chief Resident, Inpatient and Ambulatory Attending)<br />

Credit for Credit for<br />

Clerkship Advanced <strong>Medicine</strong><br />

Competency (in 3 rd yr) (by 12/31 <strong>of</strong> 4 th yr) Consequence<br />

- - - Repeat the <strong>clerkship</strong>,<br />

partially or entirely<br />

Clerkship Level + +<br />

SPECIFIC REQUIREMENTS AND GUIDELINES<br />

Inpatient <strong>Medicine</strong><br />

Schedule:<br />

1) For 6B students, Inpatient <strong>Medicine</strong> is 5-1/2 weeks (if in the first half <strong>of</strong> the academic year) or 6<br />

weeks (if in the second half <strong>of</strong> the academic year).<br />

2) For 6L students, Inpatient <strong>Medicine</strong> is 6 weeks.<br />

Site: The student will be assigned to Kuakini Medical Center (KMC), Queen’s Medical Center<br />

(QMC) or Tripler Army Medical Center (TAMC).<br />

Orientation: The student will be oriented to the site by its Hospital Site Coordinator(s) (HSC) and<br />

Chief Medical Resident (CMR).<br />

Team Assignment: The student will be assigned to a medical team where an Upper Level<br />

Resident will be directly responsible for the student’s supervision. The student will also work with<br />

the Intern(s) on the team.<br />

Call:<br />

1) Students must take call every day that their team is on call. Students may leave earlier if<br />

their patient care responsibilities are complete, with the Upper Level Resident’s approval.<br />

2) Students at KMC must take one (1) overnight call with their team, preferably a Friday night<br />

or a Saturday night.<br />

Work Hours (See Work Hours, Dedicated Study Time and Days Off):<br />

1) The earliest time that students are permitted to arrive at the hospital is 4:00 a.m. at KMC<br />

and 5:00 a.m. at QMC and TAMC.<br />

2) The earliest time that students are permitted to see patients, that is, to actually talk to and<br />

examine patients, is 4:30 a.m. at KMC and 5:30 a.m. at QMC and TAMC (excluding emergency<br />

situations such as Code Blues).<br />

3) The Hospital Site Coordinators, Chief Medical Residents and Upper Level Residents are<br />

aware <strong>of</strong> these work hour guidelines. If a student is not able to complete his/her work within<br />

these work hour guidelines, the student is advised to see his/her Hospital Site Coordinator, Chief<br />

12


Medical Resident or Upper Level Resident. If a student does not follow these work hour<br />

guidelines, the Hospital Site Coordinator, Chief Medical Resident or Upper Level Resident are<br />

required to advise the student and notify the <strong>clerkship</strong>.<br />

Patient Assignment:<br />

1) The Upper Level Resident will be responsible for assigning patients to the student.<br />

Patients should be selected for their ability to cooperate and communicate, as well as for their<br />

specific medical problems.<br />

2) The student has a list <strong>of</strong> Training Problems on which the Third-Year Clerkship in Internal<br />

<strong>Medicine</strong> curriculum and specific learning objectives are based. The student’s goal is to see at<br />

least one (1) inpatient patient with each <strong>of</strong> thirty-two (32) Training Problems. Note that the Training<br />

Problem does not have to be the patient’s Chief Complaint. In fact, a patient may present with many<br />

Training Problems. (See Training Problems section)<br />

Patient Census: The student should admit 1 – 2 patients per call. The student should actively<br />

follow an average <strong>of</strong> two (2) patients at all times (maximum 5 patients).<br />

Patient Care Responsibilities:<br />

1) The student should complete an Initial History and Physical on each and every patient<br />

assigned to the student, whether or not the student was present for the patient’s admission (i.e.<br />

transfer patients, etc). This Initial History and Physical includes interviewing and examining<br />

patients either independently or while being observed by a resident or attending. If the student<br />

observes a resident or attending obtain the History and Physical, this observation does not qualify as<br />

the student’s own History and Physical. The student would have to go back at another time and<br />

perform his/her own independent History and Physical “Group” work-ups are not allowed or<br />

accepted.<br />

2) The student should perform the physical exam which was taught as the Basic Physical<br />

Exam Sequence (BPES) in the pre-clinical years. Note that the funduscopic exam is included in the<br />

BPES. The student will also be expected to perform appropriate Branch Steps as needed. The<br />

Upper Level Resident is responsible for insuring proper supervision <strong>of</strong> the following parts <strong>of</strong> the<br />

physical exam which may be performed by the student if clinically indicated: female breast exam,<br />

female genital and/or pelvic exam, male genital and/or prostate exam, and female and male<br />

rectal exam. This means that the student must be supervised by a physician (interns, upper level<br />

residents, chief residents or attendings) who is certified or has expertise to competently perform the<br />

exam in question.<br />

4) The student is expected to Pre-Round and write daily Progress Notes on all his/her<br />

assigned patients before the Intern or Upper Level Resident(s) write their note(s). The Intern or<br />

Upper Level Resident should review the notes with the student, give constructive feedback and<br />

countersign the notes. Any missed history or physical exam finding should be noted, corrected and<br />

demonstrated as needed.<br />

5) The student should Round with the team. Since the student will be expected to have<br />

detailed knowledge <strong>of</strong> his/her assigned patients, the student is expected to take the lead in<br />

discussing his/her patients, including the student’s assessment <strong>of</strong> the patients’ problems and<br />

the student’s plans. In addition, the student will be expected to have a general knowledge <strong>of</strong> the<br />

other patients on the team so that he/she can be included in the team’s discussions <strong>of</strong> the care <strong>of</strong><br />

13


all <strong>of</strong> the team’s patients and can assist the team in the care <strong>of</strong> all <strong>of</strong> the team’s patients. The<br />

student will be expected to actively participate in teaching attending rounds with the team.<br />

6) The student is expected to assume as much patient care responsibility and functions<br />

as the team feels are appropriate for the individual student’s level <strong>of</strong> training and competence.<br />

The student is expected to participate in patient education and counseling, work with nursing<br />

staff, dieticians, respiratory therapists, physical and occupational therapists, social workers,<br />

hospital chaplains, etc. and to assist in discharge planning.<br />

7) The student may perform Procedures or assist in the performance <strong>of</strong> procedures that the<br />

team feels are appropriate for the student’s level <strong>of</strong> training and competence. The Upper Level<br />

Resident is responsible for insuring proper supervision <strong>of</strong> any procedure performed or assisted by<br />

the student. This means that the student must be supervised by a physician who is certified or has<br />

expertise to competently perform the procedure in question, which includes interns, upper level<br />

residents, chief residents or attendings. Note that there are no required procedures for third-year<br />

medical students to perform.<br />

8) The student should learn how to write Orders on his/her assigned patients. The student will<br />

observe the residents enter orders electronically. The student should practice writing orders in their<br />

Comprehensive Write-ups and daily Progress Notes (in the Plans section) and on the Physician's<br />

Orders forms (See Physician’s Orders). The Hospital Site Coordinator and residents should<br />

review the student’s orders with the student and correct them as needed.<br />

Comprehensive Write-ups:<br />

1) The student is required to submit 3 write-ups by halfway through the inpatient rotation<br />

(averaging 1 write-up each week - schedule to be determined by the Hospital Site Coordinator).<br />

The HSC will decide whether these 3 write-ups are satisfactory. If they are deemed satisfactory,<br />

then the student will not be required to submit any more write-ups. If, however, they are deemed<br />

unsatisfactory, then the student will be required to submit 1 – 3 additional write-ups, as<br />

determined by the Hospital Site Coordinator, up to a maximum <strong>of</strong> 6 write-ups. (See Appendix)<br />

2) The HSC should read and then review each comprehensive write-up with the student and<br />

give constructive feedback.<br />

3) At the end <strong>of</strong> Inpatient <strong>Medicine</strong>, the student will turn in a total <strong>of</strong> 3 - 6 comprehensive<br />

write-ups, (total number to be determined by the HSC), either the originals or copies <strong>of</strong> the originals<br />

with the HSC’s comments, to the <strong>clerkship</strong> <strong>of</strong>fice.<br />

Required Clerkship Activities specific to Inpatient <strong>Medicine</strong>:<br />

1) Bedside Clinical Skills<br />

week.<br />

a) All students on Inpatient <strong>Medicine</strong> will attend Bedside Clinical Skills 1 - 2 times each<br />

b) Each week, one student will prepare and formally present one <strong>of</strong> his/her patients.<br />

c) Patients should be chosen who are unknown to the other students in the group (ideally)<br />

and who have physical exam findings.<br />

d) The student who is presenting should have the case presentation memorized<br />

(written notes for referral only).<br />

14


e) After presenting the case, the student will take the group to see the patient. (The<br />

student should have already gotten the patient’s consent and determined that the patient would<br />

be available for bedside teaching.)<br />

f) The Bedside Clinical Skills attending will teach/review physical exam skills and<br />

review/clarify important physical exam findings.<br />

g) The Bedside Clinical Skills attending may meet individually with the student who<br />

presented to give constructive feedback on his/her case presentation. (See Case Presentation<br />

Evaluation Form)<br />

h) The Bedside Clinical Skills attending may use the Small Group Learning Experience<br />

Evaluation Form to evaluate the students in the group. (See Small Group Learning<br />

Experience Evaluation Form)<br />

2) Chief Medical Resident Rounds<br />

a) All students on Inpatient <strong>Medicine</strong> will attend weekly Chiefs Rounds with the Chief<br />

Medical Resident (CMR). These may include bedside or didactic teaching.<br />

b) Chief Rounds give the students the opportunity to specify which areas in Internal<br />

<strong>Medicine</strong> they would like the CMR to teach in a small group setting.<br />

c) The CMR, together with the Upper Level Resident, are responsible for insuring that<br />

students are instructed in, but not limited to, the following:<br />

o Case presentations<br />

o Interpretation <strong>of</strong> basic EKGs<br />

o Interpretation <strong>of</strong> common imaging, including chest and abdominal x-rays and<br />

CT, head CT and MRI<br />

o Physiology and management <strong>of</strong> fluids and electrolytes<br />

o Interpretation <strong>of</strong> arterial blood gases<br />

o Basic physiology and management <strong>of</strong> shock<br />

o Basic understanding and management <strong>of</strong> ventilators (at KMC)<br />

d) The CMR may use the Small Group Learning Experience valuation Form to evaluate<br />

the students in the group. (See Small Group Learning Experience Evaluation Form)<br />

3) HIPSTER (<strong>Hawaii</strong> InterPr<strong>of</strong>essional Simulation Training for Emergency<br />

Response)<br />

Evaluation and Feedback to Students:<br />

1) The student is expected to regularly (at least once a week) ask for feedback on his/her<br />

performance and progress from his/her residents and attendings. It is especially important that<br />

this occurs before the student rotates <strong>of</strong>f the team and before the residents and attendings rotate<br />

<strong>of</strong>f the team. This feedback should identify the student’s strengths and weaknesses so that the<br />

student will know what to work on improving.<br />

2) Midway through Inpatient <strong>Medicine</strong>, the Hospital Site Coordinator will complete a Mid-<br />

Clerkship Feedback Form and discuss it with the student. The HSC will indicate whether the<br />

student’s progress to date is satisfactory or unsatisfactory, identify the student’s strengths and<br />

weakness and suggest a remediation plan if necessary. (See Mid-Clerkship Feedback Form)<br />

15


3) Interns, Upper Level Residents and Attendings who work with the student for one (1) week<br />

or more are expected to evaluate the student. During the course <strong>of</strong> the student’s inpatient rotation,<br />

the Intern and Upper Level Resident are expected to give the student verbal feedback on his/her<br />

performance and progress. This feedback should be provided on a regular basis, ideally daily but at<br />

least weekly. Then during the last week <strong>of</strong> the Intern and Upper Level Resident’s block or during the<br />

last week <strong>of</strong> the student’s inpatient rotation, the Intern and the Upper Level Resident will complete<br />

independent evaluations <strong>of</strong> the student which will be turned in to the Chief Medical Resident or<br />

Hospital Site Coordinator. Finally, at the end <strong>of</strong> the student’s inpatient rotation, the HSC will<br />

summarize all evaluations and complete a summary Student Evaluation Form for the site, which is<br />

submitted to the <strong>clerkship</strong> <strong>of</strong>fice. (See Student Evaluation Form)<br />

4) The student will be formally observed performing a history and physical on a patient<br />

unknown to the student by the end <strong>of</strong> the student’s second week <strong>of</strong> Inpatient <strong>Medicine</strong>. It is the<br />

student’s responsibility to arrange for a time when both the student and the Chief Medical<br />

Resident are available. It is the CMR’s responsibility to find an appropriate patient. The CMR will<br />

observe the student perform a history and physical. The CMR may also ask the student to make<br />

a brief case presentation. The CMR will complete the Observed History and Physical and<br />

BPES (Basic Physical Exam Sequence) Evaluation Forms (which should be provided by the<br />

student to the CMR) and give the student immediate feedback. If the student’s performance is<br />

not satisfactory, the student must repeat the exercise until his/her performance is satisfactory to<br />

the CMR. Note that this history and physical may be used for one <strong>of</strong> the student’s<br />

comprehensive write-ups. (See Observed History and Physical and BPES Evaluation Forms –<br />

this is a required <strong>clerkship</strong> activity.)<br />

5) The student will have the opportunity to present his/her patients at various hospital rounds<br />

and conferences. The student should ask the Upper Level Resident or Chief Medical Resident in<br />

advance for assistance in preparing for such presentations at a level that is appropriate for the<br />

student’s training and for the particular rounds or conference. After the presentation, the student<br />

should ask the physician who is supervising the rounds or conference for feedback. (See Case<br />

Presentation Evaluation Form)<br />

Ambulatory <strong>Medicine</strong><br />

Schedule: For 6B students, Ambulatory <strong>Medicine</strong> consists <strong>of</strong> 5 half days per week for 5-1/2<br />

weeks (if in the first half <strong>of</strong> the academic year) or for 5 weeks (if in the second half <strong>of</strong> the<br />

academic year). For 6L students, Ambulatory <strong>Medicine</strong> consists <strong>of</strong> 25 half days (one half day<br />

weekly for 21-22 weeks plus 3-4 additional half days).<br />

Location: The Ambulatory <strong>Medicine</strong> sites include Queen Emma Clinics, TAMC Internal<br />

<strong>Medicine</strong> Clinic, VA Clinics, community health clinics, Kaiser HMO and private physician<br />

<strong>of</strong>fices. The sites for MED 531 for 6B students are all on Oahu, primarily in Honolulu,<br />

while the sites for MED 532 for 6L students are either outside <strong>of</strong> Honolulu on Oahu or on<br />

the Neighbor Islands. Although each site has unique features, the <strong>clerkship</strong>’s goal is to<br />

provide students with a uniform learning experience based on the same learning objectives<br />

while allowing students the opportunity to take advantage <strong>of</strong> the strengths <strong>of</strong> each site.<br />

Due to the distant location <strong>of</strong> their assigned sites and their schedules, 6L students<br />

do not attend any <strong>clerkship</strong> activities while on Ambulatory <strong>Medicine</strong> (UH <strong>Department</strong> <strong>of</strong><br />

<strong>Medicine</strong> Grand Rounds, EBM 1 and 2, EKG Workshop, HIV <strong>Medicine</strong>, Neuro 1 and 2).<br />

16


Patient Assignment: The Ambulatory Preceptor(s) will be responsible for assigning patients<br />

to the student. Patients should be selected for their ability to cooperate and communicate, as<br />

well as for their specific medical problems. The student has a list <strong>of</strong> Training Problems on<br />

which the Third-Year Clerkship in Internal <strong>Medicine</strong> curriculum and specific learning<br />

objectives are based. The student’s goal is to see at least one (1) ambulatory patient with<br />

each <strong>of</strong> thirty-three (33) Training Problems. Note that the Training Problem does not have to<br />

be the patient’s Chief Complaint. In fact, a patient may present with many Training Problems.<br />

(See Training Problems section)<br />

Patient Census: The student should work up at least two (2) patients each half day.<br />

Patient Care Responsibilities:<br />

1) The student may see new or returning patients for complete examinations or<br />

problem-focused visits. Ideally, the student will see some <strong>of</strong> the patients when they return<br />

for their follow-up visits.<br />

2) After reviewing the patient’s chart as necessary, the student will perform an<br />

appropriate history and physical examination. The student will present the case to the<br />

preceptor who should correct and demonstrate any missed history or physical exam<br />

findings and review the student’s assessment and plans.<br />

3) The student will write an appropriate write-up or progress note in a timely manner, as<br />

specified by the preceptor. The preceptor should review the write-up or note with the student and<br />

give constructive feedback.<br />

4) The student is expected to carry out whatever patient care responsibilities and functions<br />

the preceptor feels is appropriate for the student’s level <strong>of</strong> training and competence. The student<br />

should try to assume as much responsibility as is appropriate and possible. The student is<br />

expected to participate in patient education and counseling, work with <strong>of</strong>fice/clinic staff, and to<br />

assist in follow-up planning. If the student’s patient requires any consultations or procedures, the<br />

student is highly encouraged to be present if the patient agrees. If the student’s patient is<br />

hospitalized, the student is highly encouraged to follow the patient during the hospitalization.<br />

Comprehensive Write-ups:<br />

1) The 6B student is required to complete an average <strong>of</strong> two (2) comprehensive writeups<br />

each week. The 6L student is required to complete an average <strong>of</strong> two (2)<br />

comprehensive write-ups each month. These comprehensive write-ups may be different<br />

from those initially written – they are expected to be more detailed and thorough, requiring<br />

additional reading and thought. (See Appendix D and E)<br />

2) The Ambulatory Preceptor should read, correct and review each comprehensive writeup<br />

with the student and give constructive feedback.<br />

3) At the end <strong>of</strong> Ambulatory <strong>Medicine</strong>, the student will turn in a total <strong>of</strong> ten (10)<br />

comprehensive write-ups, either the originals or copies <strong>of</strong> the originals with the preceptor’s<br />

comments, to the <strong>clerkship</strong> <strong>of</strong>fice.<br />

Evaluation and Feedback to Students:<br />

1) The student is expected to regularly ask the Ambulatory Preceptor for feedback on<br />

his/her performance and progress, ideally on a weekly basis for 6B students and on a<br />

monthly basis for 6L students, and definitely before the student finishes Ambulatory<br />

<strong>Medicine</strong>. This feedback will give the student the opportunity to correct mistakes and<br />

17


omissions and to improve skills.<br />

2) Midway through Ambulatory <strong>Medicine</strong>, the student should provide the Ambulatory<br />

Preceptor with a Mid-Clerkship Feedback Form to complete and discuss with the<br />

student. The preceptor will indicate whether the student’s progress to date is satisfactory<br />

or unsatisfactory, identify the student’s strengths and weakness and suggest a remediation<br />

plan if necessary. (See Mid-Clerkship Feedback Form)<br />

3) At the end <strong>of</strong> Ambulatory <strong>Medicine</strong>, the Ambulatory Preceptor will complete a Student<br />

Evaluation Form provided by the Third-Year Clerkship in internal <strong>Medicine</strong>. The Student<br />

Evaluation Form should be completed during the last week <strong>of</strong> the student’s Ambulatory<br />

<strong>Medicine</strong>. The preceptor should review the Student Evaluation Form with the student and the<br />

student should then sign the form. The form is then submitted to the <strong>clerkship</strong> <strong>of</strong>fice. (See<br />

Student Evaluation Form)<br />

4) The student will be formally observed counseling a patient and/or family on two<br />

separate occasions. (See Observed Patient Counseling Evaluation Form – this is a<br />

required <strong>clerkship</strong> activity.)<br />

PBL Tutorial<br />

1) All students will attend PBL Tutorial:<br />

a. Inpatient students, whether 6B or 6L, will attend weekly PBL Tutorial led by<br />

their Hospital Site Coordinator who serves as the Inpatient PBL Tutor.<br />

b. Ambulatory 6B students will attend weekly PBL Tutorial led by an Ambulatory<br />

PBL Tutor.<br />

c. Ambulatory 6L students will attend monthly PBL Tutorial led by an<br />

Ambulatory PBL Tutor.<br />

2) One student will prepare and formally present one <strong>of</strong> his/her patients at each PBL<br />

Tutorial.<br />

3) Patients should ideally be chosen who are unknown to the other students in the<br />

tutorial group and who manifest one or more Training Problems (see Appendix A).<br />

4) The student who is presenting should have the case presentation memorized<br />

(written notes for referral only) and should bring and be familiar with all<br />

pertinent data (labs, pathology reports, EKG, imaging, etc.).<br />

6) The group will agree on appropriate Learning Issues that ideally address Training<br />

Problems and their specific Learning Objectives. (See Appendix A)<br />

7) At the following week’s PBL Tutorial, the Learning Issues from the previous week will<br />

be presented and discussed.<br />

8) The PBL Tutor should meet individually with the student who presented to give<br />

constructive feedback on the student’s case presentation. (See Case Presentation<br />

Evaluation Form)<br />

9) The PBL Tutor may use the Small Group Learning Experience Evaluation Form to<br />

evaluate the students in the group. (See Small Group Learning Experience<br />

Evaluation Form)<br />

18


Role Descriptions for <strong>Medicine</strong> 531/532<br />

The Bedside Clinical Skills Attending’s Role<br />

1. The BCS Attending’s role will be to serve as a role model, instructor and facilitator.<br />

The BCS Attending will make formal rounds. Meetings on the inpatient service will<br />

be scheduled regularly and this time should be held inviolable.<br />

2. The BCS Attending will be familiar with and reinforce the method <strong>of</strong> problem based,<br />

self-directed learning, as established in Units 1-5.<br />

3. BCS Attendings will emphasize bedside clinical teaching during their rounds, never<br />

failing to see the patients that are presented. The Attending will serve as resource<br />

faculty at the bedside, teaching techniques and demonstrating abnormalities. In<br />

addition to the general bedside teaching, they are tasked to specifically ensure<br />

competent student performance <strong>of</strong> pulmonary and cardiac bedside examination<br />

skills.<br />

4. The BCS Attending will be responsible for giving feedback to the student on his/her<br />

performance after each presentation. Any problems identified will be discussed and<br />

a plan to (FIX SPACING! remedy difficulties will be constructed. Progress on<br />

previously identified difficulties will also be discussed.<br />

5. BCS Attendings will be responsible for evaluation and will serve on the<br />

<strong>Department</strong>al Student Evaluation Committee.<br />

The Upper Level Resident’s Role<br />

1. Introduction:<br />

The upper level resident (Levels 2 or 3) is ultimately responsible for the third-year medical<br />

student’s experience with the inpatient medical team and should insure that the student has a<br />

good inpatient experience. At the beginning and throughout the student’s inpatient<br />

experience, the resident should clarify what is expected <strong>of</strong> students.<br />

2. Responsibilities:<br />

The Upper Level Resident will be responsible for assigning 2-3 patients per week<br />

for the student work up and continuing care. Patients should be selected for their<br />

ability to cooperate and communicate, as well as for their specific medical problems.<br />

The Upper Level Resident will be responsible also for reviewing the student’s write<br />

ups, for assistance with development <strong>of</strong> learning issues and for help in execution <strong>of</strong><br />

the learning agenda.<br />

The Upper Level Resident will "sign <strong>of</strong>f" on the student’s write up, confirming that<br />

he/she has reviewed the written note and given feedback.<br />

The Upper Level Resident will confirm that the student is pursuing self-directed<br />

learning by taking advantage <strong>of</strong> "Dedicated Learning Time."<br />

19


The Chief Medical Resident’s Role<br />

1. Introduction:<br />

The Chief Medical Resident (CMR) serves as a key faculty resource for the medical<br />

student on the inpatient service. As supervisor for the medical teams and their<br />

students, the CMR helps insure a good experience for the students.<br />

2. Responsibilities:<br />

a. The CMR will supervise and assist the Upper Level Resident in the<br />

inpatient instruction <strong>of</strong> medical students. They are responsible for insuring<br />

that students are instructed in, but not limited to, the following:<br />

o Case presentations<br />

o Interpretation <strong>of</strong> basic EKGs<br />

o Interpretation <strong>of</strong> common imaging, including chest and abdominal x-<br />

rays and CT, head CT and MRI<br />

o Physiology and management <strong>of</strong> fluids and electrolytes<br />

o Interpretation <strong>of</strong> arterial blood gases<br />

o Basic physiology and management <strong>of</strong> shock<br />

o Basic understanding and management <strong>of</strong> ventilators (at KMC)<br />

b. The CMR will observe and evaluate student’s Observed History and<br />

Physical during the student’s first two weeks <strong>of</strong> inpatient medicine, as<br />

previously described.<br />

c. The CMR will lead weekly Chief Rounds with the students as<br />

previously described<br />

d. The CMR will regularly discuss each student’s clinical performance<br />

with the Hospital Site Coordinator.<br />

The Site Coordinator’s Role<br />

1. Introduction:<br />

As a community-based <strong>clerkship</strong>, the Third-Year Clerkship in Internal <strong>Medicine</strong><br />

utilizes several clinical sites for medical student instruction. A major role <strong>of</strong> the<br />

Site Coordinator is to provide students at his/her site with the best possible<br />

educational experience and to insure that that learning experience is equivalent<br />

to the educational experience provided at the other sites utilized by the<br />

<strong>clerkship</strong>.<br />

The Site Coordinator is selected for his/her knowledge and experience at the site and<br />

the ability to work well with physicians, staff and administrators at the site, as well as<br />

the ability to work well with medical students. The Site Coordinator may be inpatient<br />

(Hospital Site Coordinator) or ambulatory (Ambulatory Site Coordinator).<br />

2. Responsibilities <strong>of</strong> Hospital Site Coordinators and Ambulatory Site<br />

Coordinators include, but are not limited to:<br />

a. Selecting qualified faculty to provide a good learning experience for the<br />

20


student(s) assigned to the site. The site coordinator will serve as a resource for<br />

the faculty and chief medical resident (CMR) at the site. This includes orienting<br />

new faculty and CMRs and assisting with faculty development and education<br />

regarding <strong>clerkship</strong> curriculum, expectations and goals.<br />

b. Identifying learning resources at the site that are available for student use (i.e.<br />

library, photocopying, computer, classrooms, etc.).<br />

c. Orientating new students, which include scheduling and leading orientations to<br />

the site and key personnel including faculty, CMR and administrative staff,<br />

making team/clinic assignments, reviewing <strong>clerkship</strong> requirements and student<br />

responsibilities. Orientations should occur on the student’s first day at the<br />

assigned site.<br />

d. Evaluating case presentations. (See Case Presentation Evaluation Form)<br />

e. Evaluating comprehensive write-ups. The Site Coordinator will read students’<br />

comprehensive write-ups and give constructive feedback at least on a weekly<br />

basis. (See Appendix D and E)<br />

f. Meeting with each student regularly to review the student’s performance and<br />

progress and to provide feedback. These meetings should also be used to<br />

review the number and type <strong>of</strong> patients that students are seeing and their level<br />

<strong>of</strong> patient care involvement. (See Training Problems List) Formal meetings<br />

with each student should be scheduled at least twice during the student’s<br />

rotation at the assigned site, once midway and once before the last day (ideally<br />

during the last week).<br />

g. Completing a Mid-Clerkship Feedback Form, midway through the student’s<br />

rotation at the assigned site, indicating whether the student’s progress to date is<br />

satisfactory or unsatisfactory, identifying the student’s strengths and weakness<br />

and suggesting a remediation plan if necessary. The Site Coordinator will<br />

review the completed form with the student to allow the student time to improve<br />

in the identified areas <strong>of</strong> weakness during the remainder <strong>of</strong> the rotation. If the<br />

student’s progress to date is unsatisfactory, the Site Coordinator must<br />

immediately notify the Clerkship Director who may further discuss the student’s<br />

performance and possible remediation with the <strong>Department</strong> <strong>of</strong> <strong>Medicine</strong> Student<br />

Education Committee. (See Mid-Clerkship Feedback Form)<br />

h. Coordinating the site evaluation <strong>of</strong> students. The Site Coordinator will<br />

review the student’s performance with each physician (including Interns, Upper<br />

Level Residents and Attendings) who worked with the student for one (1) week<br />

or more and also with any Attendings who worked with the student on a weekly<br />

basis. The Site Coordinator will collect and review evaluation forms such as the<br />

Observed History and Physical, Observed Patient Counseling, Case<br />

Presentations and Small Group Learning Experiences. The Site Coordinator<br />

will complete a summary Student Evaluation Form for the site at the end <strong>of</strong> the<br />

rotation (ideally, during the last week), summarizing all the evaluations including<br />

his/her own personal evaluation. The Site Coordinator will be careful to insure that<br />

a single evaluation, incident or conflict will not overshadow a student’s overall<br />

performance. The Site Coordinator will review the form with the student and<br />

then submit it to the <strong>clerkship</strong> <strong>of</strong>fice. In addition, the Site Coordinator will complete<br />

the confidential Honors/AOA Evaluation Form which is not shared with the student<br />

but is submitted to the <strong>clerkship</strong> <strong>of</strong>fice. If the student’s performance is marginal in<br />

21


any area(s), the Site Coordinator should immediately notify the Clerkship<br />

Director who may further discuss the student’s performance and possible<br />

remediation with the <strong>Department</strong> <strong>of</strong> <strong>Medicine</strong> Student Education Committee.<br />

(See Student Evaluation Form and Honors/AOA Evaluation Form)<br />

i. Serving as a member <strong>of</strong> the <strong>Department</strong> <strong>of</strong> <strong>Medicine</strong> Student Education<br />

Committee (SEC). The Site Coordinator attends the committee’s monthly<br />

meetings to act as a spokesperson for the students assigned to his/her site,<br />

reporting on students’ performance and progress and recommending<br />

commendation and remediation when appropriate. In addition, SEC members<br />

participate in the oversight, development and implementation <strong>of</strong> all <strong>of</strong> the<br />

<strong>Department</strong> <strong>of</strong> <strong>Medicine</strong>’s 3 rd and 4 th year clinical rotations.<br />

j. Monitoring students’ work hours, dedicated study time, days <strong>of</strong>f and<br />

absences. (See Work Hours, Dedicated Study Time and Days Off section.)<br />

k. Serving as mediator in the event students encounter problems at their<br />

assigned site. Problems may include <strong>clerkship</strong> requirements, <strong>clerkship</strong><br />

performance, absences, mistreatment, harassment and abuse. The Site<br />

Coordinator should refer significant problems to the Clerkship Director and/or the<br />

<strong>Department</strong> <strong>of</strong> <strong>Medicine</strong> Student Education Committee.<br />

3. Additional responsibilities <strong>of</strong> Hospital Site Coordinators include, but are not<br />

limited to:<br />

a. Assisting in Team Assignments - the Hospital Site Coordinator will assist<br />

the Chief Medical Resident with the assignment <strong>of</strong> students to medical<br />

teams.<br />

b. Leading PBL Tutorial. This should occur on average once a week<br />

throughout the <strong>clerkship</strong> and each session should last on average 2 – 3<br />

hours. (See PBL Tutorial description.)<br />

c. Insuring that Bedside Clinical Skills occur 1 – 2 times each week and<br />

Chief Medical Resident Rounds occur 1 time each week and monitoring<br />

students’ attendance at these as well as at other hospital conferences<br />

and rounds deemed to be <strong>of</strong> educational value by the Hospital Site<br />

Coordinator.<br />

The Ambulatory Attending’s Role<br />

1. Introduction:<br />

The ambulatory attending, or preceptor, is a critical element for student learning in<br />

Internal <strong>Medicine</strong> as it is practiced in the ambulatory milieu. Although sometimes<br />

lacking the drama <strong>of</strong> inpatient medicine, the outpatient setting <strong>of</strong>fers special skills sets<br />

in learning and teaching critical thinking, decision making, clinical skills and judgment,<br />

and patient-physician communication in Internal <strong>Medicine</strong><br />

2. Responsibilities:<br />

a) The ambulatory attending/preceptor will serve as a role model for excellence<br />

in ambulatory primary internal medical practice, to include thoughtful, evidence-<br />

22


ased health care that is patient focused, comprehensive, and which maximizes<br />

and restores health, and is preventative.<br />

b) The attending/preceptor will assign patients for student evaluation that will<br />

cover the depth and breadth <strong>of</strong> ambulatory learning issues, as found in the<br />

Learning Objectives listed under Patient’s Presenting Health issues (Appendix<br />

A).<br />

c) The attending/preceptor will monitor and teach data gathering as it applies to<br />

the clinical focus <strong>of</strong> ambulatory patient care; this includes appropriate<br />

consideration <strong>of</strong> the breadth <strong>of</strong> patient health issues, rather than a simple focus<br />

on chief complaint.<br />

d) The attending/preceptor will model and teach evidence-based problem<br />

solving as it applies to the ambulatory experience.<br />

e) The attending/preceptor will critique student write-ups for accuracy, style,<br />

problem solving and give constructive feedback, by applying the criteria <strong>of</strong> the<br />

“comprehensive write-up” and the write-up examples (see Appendix E).<br />

f) The attending/preceptor will give continuous feedback on student clinical and<br />

analytical skills, and stimulate his/her intellectual growth.<br />

g) The ambulatory preceptor will assure completion the “Observation <strong>of</strong> Patient<br />

Counseling form for a least 2 separate patient encounters.<br />

h) The attending/preceptor will evaluate student performance in areas <strong>of</strong><br />

medical knowledge, clinical skills, and pr<strong>of</strong>essionalism and work habits.<br />

23


MISCELLANEOUS CLERKSHIP INFORMATION<br />

24


U.H. John A. Burns School <strong>of</strong> <strong>Medicine</strong><br />

Third-Year Clerkship in Internal <strong>Medicine</strong><br />

GUIDELINES FOR APPROPRIATE APPEARANCE AND ATTIRE<br />

These guidelines are intended to contribute to your overall pr<strong>of</strong>essional development as students<br />

in training to become physicians. The Third Year Clerkship in Internal <strong>Medicine</strong> expects students<br />

to appear and dress in a pr<strong>of</strong>essional manner. Some things, such as your appearance and attire,<br />

which may have been acceptable in college, may no longer be appropriate in our <strong>clerkship</strong>. Your<br />

appearance and attire should show respect to faculty, staff, classmates, patients and the general<br />

public.<br />

It is recognized that different attire will be necessary for different settings, depending on factors<br />

such as student activities and responsibilities, training sites, patient and public contact. Student<br />

attire should always be appropriate and not interfere with the activities and responsibilities<br />

expected <strong>of</strong> students.<br />

General guidelines for all <strong>Medicine</strong> Clerkship students:<br />

Students should wear:<br />

JABSOM name tag with photo ID<br />

Short white medical coat<br />

Closed-toe footwear<br />

Students should maintain an optimum level <strong>of</strong> personal hygiene and grooming<br />

Strong odors and fragrances should be avoided<br />

Clothes, hair, fingernails and footwear should be clean and neat<br />

Clothing should not be suggestive, revealing or tight-fitting<br />

Clothing should not have <strong>of</strong>fensive images or language<br />

Sites-specific (inpatient and ambulatory) guidelines for <strong>Medicine</strong> Clerkship students:<br />

Students should adhere to the dress code/policy in place at their training site<br />

25


U.H. John A. Burns School <strong>of</strong> <strong>Medicine</strong><br />

Third-Year Clerkship in Internal <strong>Medicine</strong><br />

RECOMMENDED RESOURCES<br />

Highly recommended <strong>clerkship</strong> guidebook:<br />

Primer to the Internal <strong>Medicine</strong> Clerkship, Second Edition, A Guide Produced by the Clerkship<br />

Directors in Internal <strong>Medicine</strong> (CDIM), c2008, 2nd edition – download free <strong>of</strong> charge from<br />

http://www.im.org/Publications/PhysiciansInTraining/Pages/Primer.aspx<br />

Online resources:<br />

Internal <strong>Medicine</strong> Essentials for Clerkship Students - online materials such as photographs, tables,<br />

screening tools and other instruments – access free <strong>of</strong> charge at http://www.acponline.org/acp_press/essentials/<br />

UpToDate – access available at some training sites<br />

Textbooks:<br />

Internal <strong>Medicine</strong> Essentials for Students, American College <strong>of</strong> Physicians (ACP) and Clerkship Directors<br />

in Internal <strong>Medicine</strong> (CDIM), c2011 – discount <strong>of</strong>fered for ACP student members at<br />

http://www.im.org/Publications/PhysiciansInTraining/Pages/Essentials.aspx<br />

Kochar’s Clinical <strong>Medicine</strong> for Students, Lippincott Williams & Wilkins, c2008, 5 th edition<br />

Cecil Essentials <strong>of</strong> <strong>Medicine</strong>, Elsevier, c2011, 8th edition<br />

Reference textbooks:<br />

DeGowin’s Diagnostic Examination, McGraw-Hill, c2008, 9th edition<br />

Spiral bound books and pocket guides:<br />

Pocket <strong>Medicine</strong>, Lippincott Williams & Wilkins, c2010, 4 th edition<br />

Practical Guide to the Care <strong>of</strong> the Medical Patient, Elsevier, c2011, 8 th edition<br />

Washington Manual <strong>of</strong> Medical Therapeutics, Lippincott Williams & Wilkins, c2010, 33rd edition<br />

Washington Manual <strong>of</strong> Outpatient Internal <strong>Medicine</strong>, Lippincott Williams & Wilkins, c2010, 2nd edition<br />

The Sanford Guide to Antimicrobial Therapy, c2012, 42nd edition<br />

Highly recommended self-assessment program:<br />

MKSAP (Medical Knowledge Self-Assessment Program) for Students 5 (Book and Digital), American<br />

College <strong>of</strong> Physicians (ACP) and Clerkship Directors in Internal <strong>Medicine</strong> (CDIM), c2011 – discount <strong>of</strong>fered for ACP<br />

student members at http://www.im.org/Publications/PhysiciansInTraining/Pages/MKSAPforStudents.aspx<br />

26


U.H. John A. Burns School <strong>of</strong> <strong>Medicine</strong><br />

Third-Year Clerkship in Internal <strong>Medicine</strong><br />

REQUIRED EQUIPMENT<br />

1. White coat<br />

2. JABSOM nametag<br />

3. Stethoscope<br />

4. Ophthalmoscope and otoscope<br />

5. Penlight<br />

6. Reflex hammer<br />

7. Tuning fork 128 Hz for vibratory exam<br />

8. Tape measure<br />

9. Visual acuity card<br />

10. Small magnifier lens<br />

11. Disposable tongue blades<br />

12. Cotton-tipped swabs for sensory exam<br />

13. Watch with second and minute marks to record vital signs, etc.<br />

All <strong>of</strong> the above listed equipment, except for the JABSOM nametag and watch, are available for<br />

purchase at the Medical Bookstore.<br />

Students are expected to have ALL <strong>of</strong> the equipment and to bring them to their assigned training<br />

sites on the first day. The equipment should be easily and quickly accessible, i.e. worn, carried in<br />

a bag or in/on a white coat.<br />

All students on inpatient medicine are required to carry ALL <strong>of</strong> the listed equipment AT ALL<br />

TIMES while on the wards.<br />

Students on ambulatory medicine may find that their assigned sites may have some, but not all,<br />

<strong>of</strong> the listed equipment available for student use. If the listed equipment are NOT easily and<br />

quickly accessible at the site, it is the student’s responsibility to carry them AT ALL TIMES.<br />

The possible consequences <strong>of</strong> not having the required equipment are (1) being immediately sent<br />

to the medical bookstore to purchase them (requiring the student to make up the missed time)<br />

and (2) receiving unsatisfactory evaluations in the areas <strong>of</strong> (a) Clinical Skills - physical<br />

examinations and (b) Pr<strong>of</strong>essionalism dependability, pr<strong>of</strong>essional appearance and attire.<br />

27


EXPOSURE TO BLOOD/BODY FLUIDS PROTOCOL<br />

1. IMMEDIATELY following the exposure:<br />

a. Flush the exposed skin or mucous membrane with water or saline.<br />

If exposure to the eyes has occurred, use wash station or nearest sink to flush eyes with water for at<br />

least 5 minutes.<br />

b. Wash any needle stick, puncture, cut or abrasion with soap and water.<br />

2. Initiate the host agency protocol for hazardous exposure to blood/body fluids by following the instructions outlined in<br />

the table below.<br />

3. If the exposure is in a non-hospital setting (for example, ambulatory site not associated with a hospital, in a JABSOM<br />

lab, class, or other non-hospital-based exposure), you or your preceptor/supervisor can call Queens ED (547-4311)<br />

to review current protocol for immediate needs in such a circumstance, and begin the process, (AFTER #1). You<br />

may go to an Emergency <strong>Department</strong>, or during open hours, contact the <strong>University</strong> Health Services (Manoa<br />

Campus) 956-8965, and ask for immediate attention.<br />

FACILITY CONTACT or GO TO PHONE<br />

Castle Medical Center Report incident to supervisor. Obtain care from<br />

Employee Health Coordinator or hospital supervisor who<br />

will assist in filing incident report. Contact JABSOM<br />

OSA to report incident.<br />

263-5159 or<br />

hospital supervisor<br />

263-5329<br />

(5 pm-8 am)<br />

HOME Clinic<br />

Notify attending physician and complete incident report. 221-0685<br />

Kaiser Permanente Medical<br />

Center<br />

Kapiolani Medical Center<br />

Kuakini Medical Center<br />

Pali Momi Medical Center<br />

Call Dr. Jill Omori to report exposure.<br />

Report incident within 2 hours <strong>of</strong> exposure. Call<br />

operator in house “0” and ask for infection control<br />

personnel on duty.<br />

Report to Employee Health. Go to Emergency Dept, if<br />

EH closed, also call on-call Employee Health<br />

Coordinator, 983-6000).<br />

Occupational Health Services.<br />

When closed, go to ED, and also notify Nursing<br />

Supervisor (through Operator, dial “O”).<br />

Employee Health during regular work hours or<br />

Emergency <strong>Department</strong> when exposure occurs after<br />

hours. Notify supervisor. Report incident Work Injury<br />

Line.<br />

432-0000<br />

983-8525<br />

547-9531<br />

535-7200<br />

The Queen’s Medical Center Employee Health/PEP Team. 547-4004<br />

Straub Clinic and Hospital Employee Health during business hours, go directly to 522-3481<br />

ED after business hours.<br />

Tripler Army Medical Center Let care team know <strong>of</strong> exposure. Report to the ER.<br />

Report exposure to, or go to, Occupational Health the<br />

next business day.<br />

433-6235<br />

VA Clinic<br />

Contact EHU during business hours. Go to TAMC ER<br />

after hours.<br />

433-0091<br />

Wahiawa General Hospital<br />

Go to ER; also notify Nursing Supervisor (through<br />

operator) <strong>of</strong> exposure.<br />

621-4230<br />

You may also seek care and information from <strong>University</strong> Health Services (956-8965), your personal physician, or any emergency<br />

department, but seek immediate evaluation and counseling. All follow-up care after immediate evaluation services are the<br />

responsibility <strong>of</strong> the student<br />

4. Report exposure to;<br />

a. Your supervising faculty member and course/<strong>clerkship</strong> director<br />

b. Medical School Office <strong>of</strong> Student Affairs @ 692-1000;<br />

c. For URGENT after hours needs, call 692-0912, ask for Dr Smerz or Administrator on-call<br />

5. Students should be knowledgeable about their health insurance coverage, and should know what their plan will<br />

cover related to occupational exposures. Remind anyone billing for follow-up that it is NOT an Occupational<br />

Exposure, but medical follow-up, or the insurer may not want to pay for services.<br />

JABSOM’s Affiliation Agreement with Health Care Facilities (HCF) state:<br />

“Environmental exposure. In the event a medical student is exposed to an infectious, environmental, or occupational hazard at<br />

the HCF, the HCF shall be responsible for providing immediate evaluation and counseling as with employees <strong>of</strong> the HCF. Followup<br />

after the initial evaluation and counseling will not be the responsibility <strong>of</strong> the HCF, and will proceed according to <strong>University</strong><br />

student health policies.”<br />

28


U.H. John A. Burns School <strong>of</strong> <strong>Medicine</strong><br />

Third-Year Clerkship in Internal <strong>Medicine</strong><br />

HIV MEDICINE<br />

1:30 – 5:00 p.m.<br />

Clint Spencer Clinic<br />

Leahi Hospital<br />

All students in the Third-Year Clerkship in Internal <strong>Medicine</strong> are scheduled to attend one <strong>of</strong> the<br />

upcoming HIV <strong>Medicine</strong> sessions.<br />

This is a required <strong>clerkship</strong> activity.<br />

How to prepare for the HIV <strong>Medicine</strong> session:<br />

1. Read the 4 chapters on HIV at http://uhmed.org/images/HIV_Required_Reading.pdf<br />

2. Read about the Clint Spencer Clinic at http://www.hawaii.edu/hacrp/spencer.htm<br />

3. Read about HACRP (<strong>Hawaii</strong> AIDS Clinical Research Program) at<br />

http://www.hawaii.edu/hacrp/home2.htm<br />

4. If you are on inpatient medicine, please notify your team and chief resident <strong>of</strong> your<br />

scheduled absence to attend this required <strong>clerkship</strong> activity. Likewise, you should discuss<br />

with them whether or not you are expected to return to the hospital after this session.<br />

What to bring to the HIV <strong>Medicine</strong> session:<br />

White coat<br />

JABSOM nametag<br />

Temporary parking pass<br />

After attending the HIV <strong>Medicine</strong> session:<br />

Check <strong>of</strong>f HIV as an “S” (Special” patient encounter) on your Training Problems List<br />

Log patient(s)<br />

Evaluate teaching attending(s)<br />

29


Location and Parking:<br />

The Clint Spencer Clinic is located on the basement floor <strong>of</strong> the Young Building on the grounds <strong>of</strong><br />

Leahi Hospital at 3675 Kilauea Ave (on the block bordered by Kilauea Ave, Makapuu Ave,<br />

Maunalei Ave and Pokole St.) in Kaimuki.<br />

There is free parking in an open lot on Maunalei Ave. You may park in any unmarked stall. (Do<br />

not park in the stalls marked “Reserved for Clinical Research.”) Display your temporary parking<br />

pass on your dashboard.<br />

After parking, cross Maunalei Ave, walk on the main floor (actually the basement floor) <strong>of</strong> the<br />

main building (Young Building) and look for a sign that reads “HACRP” (<strong>Hawaii</strong> AIDS Clinical<br />

Research Program) and then the door that reads “Clint Spencer Clinic.” Check in with Joel<br />

Chapman at the reception desk.<br />

Please plan on arriving early at 1:15 p.m., especially if you are not familiar with Leahi Hospital<br />

and to allow time for parking.<br />

If you get lost, call the Clinic at 737-2751.<br />

30


TEMPORARY<br />

PARKING PASS<br />

FOR<br />

THE CLINT SPENCER<br />

CLINIC<br />

Date _______________<br />

1:30 – 5:00PM<br />

CALL 737-2751 for<br />

QUESTIONS<br />

31


MEDICINE T-RES INSTRUCTIONS<br />

General<br />

Logging all <strong>of</strong> your patients is required for JABSOM accreditation and for your <strong>clerkship</strong> grade.<br />

Failure to log properly and on a timely basis may lead to serious consequences for JABSOM<br />

and for you.<br />

You should log your patients regularly – ideally every workday so that you don’t forget and so<br />

that you don’t fall behind.<br />

You should sync your patient log regularly – at least once a week.<br />

T-Res Data Fields<br />

Complete all data fields (except 2 nd Diag which is not always needed).<br />

Date:<br />

In the Inpatient setting, the date is when you first saw the patient. This may or may not be the<br />

date <strong>of</strong> admission.<br />

In the Ambulatory setting, the date is when you see the patient. If you see a patient again for<br />

a follow-up visit, the patient should be logged again using the date <strong>of</strong> the follow-up visit.<br />

Site:<br />

In the Inpatient setting, select the hospital (KMC, QMC, TAMC)<br />

In the Ambulatory setting:<br />

o If you are working in a clinic, select the clinic (QEC, VA, Kaiser-Mapunapuna, Kaiser-<br />

Waipio, Kalihi Palama, Kokua Kalihi Valley, TAMC Clinic)<br />

or<br />

o<br />

If you are working in a physician’s <strong>of</strong>fice, select <strong>Medicine</strong> – Other Amb (Do not select<br />

Private Outpatient).<br />

Do not select Other<br />

Setting:<br />

For the Inpatient block: Select Inpatient or Special (Do not select Other)<br />

Inpatient is the patient for whom you performed a history and physical exam, and wrote daily<br />

progress notes. In other words, you provided “D” (Direct patient care) to this patient.<br />

You should log each patient only 1 time during the patient’s hospitalization. For example: If<br />

Mr. S was hospitalized for three days, although you wrote daily progress notes, you should log<br />

him only 1 time (upon his admission). If Mr. S. was discharged and then comes back to your<br />

team, you should log him again as a new patient encounter (upon his re-admission). If,<br />

however, Mr. S. went home AMA and then comes back with the same problem, you should<br />

not log him as a new patient encounter. If, however, Mr. S. went home AMA and then comes<br />

back with a different problem, you should log him as a new patient encounter.<br />

Special: Special is any patient that contributed to your education in the setting <strong>of</strong> conferences,<br />

rounds, procedures, etc. In other words, you did not provide “D” (Direct patient care) to this<br />

patient. Instead, this was an “S” (Special setting). If you wrote one or two progress notes to<br />

help out your team (but not on a daily basis), this is still an “S” (Special setting).<br />

For the Ambulatory block: Select Ambulatory or Special (Do not select Other)<br />

Ambulatory is the patient for whom you performed a history and physical exam, and wrote a<br />

note. In other words, you provided “D” (Direct patient care) to this patient.<br />

Each and every visit, including follow-up visits, should be considered a new patient encounter<br />

and logged.<br />

32


Special: Special is any patient that contributed to your education in the setting <strong>of</strong><br />

conferences, rounds, procedures, etc. In other words, you did not provide “D” (Direct patient<br />

care) to this patient. Instead, this was an “S” (Special setting).<br />

Supervisor:<br />

In the Inpatient setting, select last name MEDICINE + first name Inpt-KMC, Inpt-QMC-MTC,<br />

Inpt-QMC-UHS or Inpt-TAMC.<br />

In the Ambulatory setting, select the attending physician from the menu - except for the<br />

following situations:<br />

o If you are at QEC, select MEDICINE, QEC<br />

o If you are at TAMC IM Clinic, select MEDICINE, TAMC Clinic<br />

o If you are at VA Honolulu, select MEDICINE, VA<br />

o If you are at another VA, select your attending physician<br />

If the setting is Special, select MEDICINE, Special<br />

Do not select Other<br />

Birth Date:<br />

Enter the patient’s date <strong>of</strong> birth<br />

Sex:<br />

Select the patient’s gender<br />

Problems:<br />

Select up to 5 Training Problems that apply to the patient<br />

If none <strong>of</strong> the Training Problems apply to the patient, select None<br />

Prim Diag:<br />

Select the primary diagnoses that you addressed for this patient. You may enter up to 5<br />

diagnoses.<br />

If the diagnosis is not listed or if you have a more specific diagnosis than is listed, you can<br />

write in the diagnosis by choosing Other.<br />

2 nd Diag:<br />

Use this field if the patient has more than 5 diagnoses. You may enter up to 5 additional<br />

diagnoses, for a total <strong>of</strong> 10.<br />

Printed Reports<br />

Print an activity report according to the following schedule:<br />

o Mid-inpatient feedback meeting<br />

o Mid-ambulatory feedback meeting<br />

o End <strong>of</strong> the inpatient block<br />

o End <strong>of</strong> the ambulatory medicine block<br />

At the end <strong>of</strong> the inpatient block and at the end <strong>of</strong> the ambulatory block, your printed reports<br />

must reviewed, signed and dated by your Hospital Site Coordinator or Ambulatory Preceptor,<br />

respectively, and then turned in to the <strong>clerkship</strong>.<br />

Your Hospital Site Coordinator or Ambulatory Preceptor may request that you report additional<br />

handwritten information on your printed report (to help identify patients). However, the signed<br />

reports that you turn in to the <strong>clerkship</strong> should be copies without any additional handwritten<br />

information.<br />

Instructions for generating and printing your T-Res <strong>Medicine</strong> Activity Report:<br />

o Login to the T-Res web site: www.t-res.net<br />

o Select Lists under Reports in the left column<br />

o View report 038c – Activity Export Details for <strong>Hawaii</strong> Pediatrics<br />

o Activity Type: Internal <strong>Medicine</strong>, then View Report<br />

o Select Export as Acrobat (PDF) File<br />

o You can save a copy if you wish, otherwise select Open, then print the report<br />

33


U.H. John A. Burns School <strong>of</strong> <strong>Medicine</strong><br />

<strong>Department</strong> <strong>of</strong> <strong>Medicine</strong><br />

STUDENT EVALUATION OF TEACHERS<br />

Please complete an evaluation on each <strong>of</strong> the following individuals with whom you interacted during your<br />

inpatient or ambulatory rotation:<br />

INPATIENT MEDICINE<br />

Clerkship Director - Dr. Tam<br />

Hospital Site Coordinator<br />

PBL Tutor<br />

Bedside Clinical Skills Teacher(s)<br />

Chief Medical Resident<br />

Upper Levels<br />

Interns<br />

CV PE - Dr. Fukuyama<br />

EBM - Dr. Kasuya<br />

EKG - Dr. Azuma<br />

HIV <strong>Medicine</strong> Teacher(s)<br />

HIPSTER - Dr. Ganitano<br />

Neurology - Dr. Yee<br />

AMBULATORY MEDICINE<br />

Clerkship Director - Dr. Tam<br />

Ambulatory Preceptor(s)<br />

PBL Tutor<br />

Upper Levels, if at VA<br />

Interns, if at VA<br />

CV PE - Dr. Fukuyama<br />

EBM - Dr. Kasuya<br />

EKG - Dr. Azuma<br />

HIV <strong>Medicine</strong> Teacher(s)<br />

HIPSTER - Dr. Ganitano<br />

Neurology - Dr. Yee<br />

In addition, you are also encouraged to complete evaluations on any other individuals to help the <strong>Department</strong> <strong>of</strong><br />

<strong>Medicine</strong> recognize our most outstanding teachers as well as identify those who may need further training to improve<br />

their teaching.<br />

These evaluations are anonymous. Your honest and thoughtful feedback is integral in helping the <strong>Department</strong> <strong>of</strong><br />

<strong>Medicine</strong> and its teachers improve students’ educational experiences. Your assistance is greatly appreciated. Thank<br />

you!<br />

INSTRUCTIONS<br />

1. Go to uhmed.org/db<br />

2. Login using the following: Account Name: student<br />

Password: jabsom*08<br />

3. Click on “StudentEval”<br />

4. Select “MS3”, the teacher and the academic year for which you are evaluating<br />

5. Select “Evaluation <strong>of</strong> Teacher”<br />

6. Enter your Block Number which will be emailed to you.<br />

7. Complete the evaluation form and then click submit.<br />

8. You will be given a confirmation number and the option to email it to yourself for record keeping (Do not email<br />

this number directly to Julieta, as she will not be able to identify the sender). Save this confirmation number as<br />

a pro<strong>of</strong> <strong>of</strong> your submission.<br />

9. To complete another evaluation, select “”Email & New Eval”.<br />

10. Email your confirmation number(s) to Julieta at jlrajlev@hawaii.edu<br />

34


EVALUATION FORMS<br />

35


U.H. John A. Burns School <strong>of</strong> <strong>Medicine</strong><br />

Third-Year Clerkship in Internal <strong>Medicine</strong><br />

OBSERVED HISTORY AND PHYSICAL<br />

Evaluation Form<br />

INSTRUCTIONS:<br />

o The Observed History and Physical should be scheduled by halfway through the inpatient block.<br />

o The Chief Medical Resident should select a patient who is unknown to the student.<br />

o The student must perform the Observed History and Physical by memory.<br />

o The student should try to finish the Observed History and Physical in 1 hour.<br />

o The student must turn in at least one (1) completed Observed History and Physical Evaluation Form to the<br />

Hospital Site Coordinator.<br />

Student_____________________________________________ Hospital (circle): KMC QMC TAMC<br />

Start Time ____:____<br />

Faculty (Chief Resident) _______________________________ Date_____________ End Time ____:____<br />

TOTAL Time ____:____<br />

A. PATIENT-PHYSICIAN RELATIONSHIP<br />

Observe whether and how the student:<br />

Maximum Score<br />

Student’s Score<br />

1. Introduces self to the patient and explains his/her role correctly 2 ( )<br />

2. Attends to the patient’s privacy and comfort 3 ( ) before beginning<br />

and throughout the history and physical<br />

3. Demonstrates effective listening skills 5 ( )<br />

(appropriate use <strong>of</strong> eye contact, posture, pauses/silence, etc.)<br />

4. Demonstrates effective verbal skills 5 ( )<br />

(appropriate use <strong>of</strong> open- and closed-ended questions,<br />

repetition, clarifying, restating, explanation, interpretation, etc.)<br />

5. Uses language that the patient understands 5 ( )<br />

B. INTERVIEW<br />

Observe whether and how the student:<br />

1. Obtains the patient’s complete history 5 ( )<br />

systematically and completely<br />

2. Balances allowing the patient to speak while 5 ( )<br />

directing questions to efficiently obtain a<br />

clear and accurate history in the time available<br />

3. Elicits the history <strong>of</strong> the present illness (HPI) 10 ( )<br />

systematically and completely<br />

4. Delineates major symptoms systematically and completely 5 ( )<br />

(location, radiation, quality, intensity, setting, onset, duration, frequency<br />

setting, aggravating and alleviating factors, associated manifestations,<br />

functional impairment, patient’s interpretation <strong>of</strong> symptoms, etc.)<br />

36


5. Obtains information for the other major sections <strong>of</strong> the history<br />

systematically and completely:<br />

Maximum Score<br />

THE BASIC PHYSICAL EXAMINATION SEQUENCE (BPES)<br />

Student’s<br />

A. GUIDELINES<br />

a. Past Medical History (PH) 5<br />

( )<br />

b. M<br />

During the physical examination, the following guidelines should be observed:<br />

e<br />

d<br />

ications (dose, route, frequency, last dose) 2 ( )<br />

c. Allergies (specific reactions and date) 2 ( )<br />

d. Family History (FH) 2 ( )<br />

e. Social History (SH) 2 ( )<br />

f. Substance Use (tobacco, alcohol and Illicit drugs) 2 ( )<br />

g. Review <strong>of</strong> Systems (ROS) 5 ( )<br />

C. PHYSICAL EXAMINATION<br />

Observe whether and how the student:<br />

1. Demonstrates proper hygienic practices 5 ( )<br />

2. Positions the patient and self properly 5 ( )<br />

for each part <strong>of</strong> the physical exam, being<br />

observant and respectful <strong>of</strong> the patient’s modesty and comfort<br />

3. Performs the patient’s complete physical exam 5 ( )<br />

systematically, completely and correctly<br />

4. Performs the basic physical examination sequence (BPES) 10 ( )<br />

systematically, completely and correctly<br />

5. Performs the appropriate branch steps 10 ( )<br />

systematically, completely and correctly<br />

Maximum<br />

Total Score<br />

Student’s<br />

Total Score<br />

100 ( )<br />

If the Student’s Total Score is < 85,<br />

the Observed History and Physical must be repeated with another patient.<br />

Comments:<br />

Chief Medical Resident’s name: _____________________________________<br />

Chief Medical Resident’s signature: __________________________________<br />

Student’s signature: ______________________________________________<br />

I received constructive and immediate feedback on my Observed History and Physical (circle): Yes No<br />

37


1. The patient always comes first - be considerate <strong>of</strong> his/her comfort and privacy.<br />

2. Students WILL wash their hands before each examination.<br />

3. While the patient must adequately disrobe, modesty should be respected at all times.<br />

4. The examiner should explain what he/she is doing during the examination.<br />

5. Never force a procedure on any adult patient.<br />

6. Reassure the patient during the examination.<br />

7. Be serious, yet maintain an appropriate sense <strong>of</strong> humor.<br />

8. Always be gentle -- a physical examination may be slightly uncomfortable if an<br />

abnormality is present. It should never be painful.<br />

9. Be systematic and thorough.<br />

Physical examination techniques and skills that are required for MD 1 are outlined in<br />

the MD 1 Handbook and are described in detail below , and in your physical diagnosis<br />

textbooks.<br />

B. STEPS OF THE BASIC PHYSICAL EXAMINATION SEQUENCE (BPES)<br />

These are the steps included in the student Basic Physical Examination Sequence<br />

(BPES). They should be performed by the student in every complete physical<br />

examination. The BPES should be completed in 30 minutes.<br />

Patient sitting: physician facing patient<br />

1. GENERAL APPEARANCE: Inspect for habitus, level <strong>of</strong> consciousness, movement,<br />

comfort, affect.<br />

2. VITAL SIGNS: Determine body temperature - read thermometer.<br />

Determine radial pulse - rate per minute and<br />

compare bilateral radial pulses simultaneously.<br />

Determine respiratory rate per minute (w hile<br />

comparing radial pulses).<br />

Determine blood pressure - place BP cuff 2.5 cm<br />

above antecubital fossa and at level <strong>of</strong> atria.<br />

Determine BP by unilateral palpation <strong>of</strong> radial pulse<br />

and then by auscultation <strong>of</strong> the brachial artery in the<br />

antecubital fossa.<br />

3. SKIN: Inspect for overall color, hair and gross lesions. (Inspect and palpate skin in<br />

detail under each regional exam below).<br />

4. HEENT: HEAD: Inspect face for symmetry, abnormality. Inspect and<br />

palpate scalp.<br />

38


EYES: Test Visual acuity. Check each eye separately (w ith<br />

corrective lenses) w ith Rosenbaum pocket chart or new sprint.<br />

Cover opposite eye. Inspect external ocular structures - inspect<br />

lids, conjunctivae, sclerae, cornea. Test Ocular muscle function<br />

and alignment (III, IV, VI). Note light reflex in neutral position. Ask<br />

patient to direct eyes laterally, superiorly and inferiorly " up and<br />

out," " dow n and out." Test pupillary response in accommodation<br />

(convergence) and to light (direct and consensual). Inspect<br />

corneas, lenses, ocular fundi (retina <strong>of</strong> each eye) w ith<br />

ophthalmoscope. Stabilize head w ith opposite hand. Ask patient<br />

to fix gaze on a specific point on the w all. Change hands and<br />

sides appropriately for examination <strong>of</strong> the other eye.<br />

EARS: Test Hearing. Use w ristw atch or w hispered voice. Inspect<br />

and palpate auricles and mastoids. Inspect canals and tympanic<br />

membranes on each side.<br />

NOSE: Palpate for tenderness and ask patient to " sniff" to test<br />

for patency <strong>of</strong> each nasal passage. Inspect each passage w ith<br />

light, using nasal speculum.<br />

MOUTH: Palpate masseters as you ask patient to " bite dow n"<br />

(V) bilaterally. Inspect for symmetry as patient " show s teeth"<br />

(VII) & " protrudes tongue" (XII). Inspect lips, teeth, gingivae,<br />

oral mucosa, tongue, under tongue, and posterior pharynx w ith<br />

tongue blade and light. Note uvula motion w hen patient says<br />

" ah" .<br />

Patient sitting: physician in front <strong>of</strong>/behind patient.<br />

(Tutor to demonstrate both techniques.)<br />

5. NECK: Inspect neck for neck veins (see below - #7). Inspect and palpate for lymph<br />

nodes -postauricular, anterior and posterior cervical, submandibular and supraclavicular<br />

bilaterally, simultaneously with both examining hands. Palpate for parotid glands bilaterally.<br />

Inspect and palpate for thyroid - each lobe separately, with patient swallowing, and<br />

palpate for trachea, from front or rear <strong>of</strong> patient.<br />

39


6. THORAX:<br />

POSTERIOR THORAX: Inspect thorax and palpate entire spine. Percuss at costovertebral angles -w ith<br />

fore-w arning Percuss for diaphragmatic excursion, from mid-thorax dow n to dullness before and after<br />

deep inspiration on each side. Percuss bilateral chest w all at equivalent areas looking for<br />

hyperresonance or dullness. Auscultate posterior and lateral lung fields -w ith diaphragm <strong>of</strong><br />

stethoscope, w hile patient is deep-breathing w ith mouth open. Compare side to side in equivalent<br />

areas.<br />

Patient sitting: physician facing patient.<br />

ANTERIOR THORAX: Inspect. Auscultate anterior lung fields beginning w ith supraclavicular areas<br />

bilaterally w ith deep breathing.<br />

Patient supine: physician at right.<br />

AXILLAE: Palpate w ith passive abduction-adduction (R hand for L axilla and<br />

vice versa).<br />

BREASTS: Inspect and palpate in all quadrants and nipples bilaterally.<br />

7. CARDIOVASCULAR:<br />

NECK: Inspect veins. Palpate carotid pulses on each side one at a time.<br />

Auscultate carotids bilaterally.<br />

HEART: Inspect precordium. Palpate apical impulse (PMI) and entire precordium w ith R hand.<br />

Auscultate w ith diaphragm <strong>of</strong> stethoscope all cardiac areas (aortic, pulmonic, LSB and apex) w ith<br />

concurrent left finger palpation <strong>of</strong> right carotid pulse for time <strong>of</strong> the heart sounds. Auscultate at<br />

pulmonic area for split S2 during inspiration. Auscultate at apex w ith bell <strong>of</strong> stethoscope (supine,<br />

and in left lateral decubitus position, for presence <strong>of</strong> S3, S4 and murmur).<br />

8. ABDOMEN:<br />

ABDOMEN: Begin by inquiring about pain. Alert patient. Ask patient to flex hips<br />

and knees to relax abdominal wall muscles.<br />

40


Inspect. Auscultate - epigastrium, left paraumbilical area over aorta for bruit and<br />

bowel sounds. Palpate - superficially and deeply in all four quadrants. Palpate deeply<br />

for aorta and for liver and spleen with deep inspiration. Palpate for spleen: turn<br />

patient to R lateral decubitus if spleen not previously felt with patient supine.<br />

INGUINAL AREAS:<br />

Inspect. Palpate for lymph nodes and/or masses<br />

and femoral arterial pulses.<br />

9. EXTREMITIES:<br />

LOWER EXTREMITIES:<br />

Inspect and palpate feet, legs, and thighs for<br />

edema. Palpate dorsalis pedis and posterior tibial<br />

pulses bilaterally and simultaneously.<br />

Patient sitting: physician facing patient.<br />

UPPER EXTREMITIES: Inspect skin, fingernails, and for tremor. Test strength- <strong>of</strong> grip bilaterally by<br />

asking patient to squeeze 2 <strong>of</strong> your fingers. Ask patient to "shrug shoulders" (XI).<br />

DEEP TENDON REFLEXES:<br />

plantar reflexes bilaterally.<br />

Test biceps, triceps, patellar, achilles reflexes and<br />

Patient standing: physician facing patient and then behind patient.<br />

POSTURE:<br />

Inspect for symmetry <strong>of</strong> shoulders, hips, and<br />

curvature <strong>of</strong> the spine.<br />

LOWER EXTREMITIES: Inspect for varicose veins, other deformities.<br />

GAIT: Inspect gait as patient walks barefoot.<br />

The follow ing is usually omitted for the video examination:<br />

MALE GENITALIA: Inspect penis (ask patient to retract foreskin) and scrotum.<br />

Palpate scrotal contents and inguinal canals.<br />

ANORECTUM: Men – have patient bend over examining table or lie in<br />

left lateral decubitus position w ith right hip and knee flexed.<br />

Women - patient in dorsosacral position w ith<br />

knees flexed and abducted (if for pelvic exam), or<br />

lying in left lateral decubitus position w ith right hip<br />

and knee flexed.<br />

Inspect anus, buttocks, and perineum.<br />

41


Palpate anal canal, rectal ampulla (and prostate in<br />

men), after gently inserting gloved and lubricated<br />

index finger.<br />

PELVIC EXAM: Instruction by obstetrician-gynecologist.<br />

C. BRANCH STEPS: (EXTENDED PHYSICAL EXAMINATION)<br />

If an abnormality is detected, it must be fully described and additional examination<br />

(branch steps) performed to clarify and delineate that finding.<br />

Example: If decreased auditory acuity is noted, the Weber and Rinne tests must be<br />

done.<br />

Such branch steps, how ever, are not part <strong>of</strong> the BPES.<br />

Branch steps are taught in separate laboratory sessions. They are useful in further<br />

elucidating abnormalities uncovered in the history and BPES and are generally not<br />

screening maneuvers (Refer to page 24).<br />

42


COMMON ERRORS OF THE BASIC PHYSICAL EXAM SEQUENCE<br />

EYES:<br />

When testing visual acuity for each eye corrected preferred, use some form <strong>of</strong><br />

measurement so that your evaluator knows that the same distance is being used for both<br />

eyes. For most pocket eye-chart cards, the card should be held 14 inches away from the<br />

eye being tested.<br />

When testing pupillary responses in accommodation and to light (direct and<br />

consensual), review the difference between accommodation and convergence. For<br />

accommodation, ask the patient to look at a distant object or the wall (for example) for<br />

distance, then have the patient look at your finger held at 2-4 inches from the bridge <strong>of</strong> his/her<br />

nose. Convergence should be tested along with ocular motor function testing.<br />

When inspecting cornea, lens and retina <strong>of</strong> each eye with ophthalmoscope, remember to<br />

turn <strong>of</strong>f the room lights. Use your right eye and hold the ophthalmoscope in your right hand when<br />

examining the patient’s right eye. Conversely, use your left eye and hold the ophthalmoscope in<br />

your left hand when examining the patient’s left eye.<br />

EARS: When testing for auditory acuity bilaterally, patient should have their eyes closed. Test<br />

one ear at a time. Begin by snapping your fingers 3 feet (arms length) away from the ear being<br />

tested. Ask the patient to indicate if he/she hears the snapping <strong>of</strong> your fingers. If the patient<br />

doesn’t hear, move closer to ear in six-inch increments and retest.<br />

NECK: Inspect and palpate thyroid, each lobe separately, and trachea with swallowing from<br />

front or back. If you choose to examine the patient from the front, identify the thyroid<br />

isthmus and then each lobe <strong>of</strong> the thyroid gland together and separately, asking the patient<br />

to swallow as you examine each side.<br />

THORAX:<br />

Review technique to percuss the lungs, percussing the posterior chest wall in<br />

symmetrical areas.<br />

Review proper technique to percuss diaphragmatic excursion on each side.<br />

Need to estimate the descent <strong>of</strong> the diaphragm when patient inspires.<br />

Review proper technique to palpate axillae with passive abduction-adduction. Your left<br />

hand should be used to examine the patient’s right axillae, using your right hand to support the<br />

patient’s right arm. Your right hand should be used to examine the patient’s left axillae, using<br />

your left hand to support the patient’s left arm.<br />

ABDOMEN:<br />

Review proper use <strong>of</strong> hands. Your left hand should be used to support the patient’s left<br />

flank and your right should be used to feel for the spleen.<br />

EXTREMITIES:<br />

Review proper technique to test for deep tendon reflexes, biceps, triceps, knee, ankle,<br />

and plantar reflexes bilaterally. Student should check patient’s bicep reflexes on each arm then<br />

tricep reflexes on each arm in order to compare bilaterally versus checking the bicep and tricep<br />

reflexes on one arm then moving to the other arm and checking the bicep and tricep reflexes.<br />

When inspecting the spine, please have patient bend over forward in order to check the<br />

curvature <strong>of</strong> their spine.<br />

43


EVALUATION OF PERFORMANCE OF<br />

BASIC PHYSICAL EXAMINATION SEQUENCE<br />

Student: Date: _________ Evaluator: ___________________<br />

Steps Done Not<br />

Done<br />

Seq.<br />

Incor<br />

.<br />

Tech<br />

.<br />

Incor<br />

.<br />

Comments<br />

1. General Appearance: (sitting)<br />

Inspect overall<br />

2. Vital Signs (sitting)<br />

A. Temperature with thermometer<br />

B. Pulse - Measure rate per minute<br />

Compare bilaterally<br />

C. Respiratory - rate per minute<br />

D. Blood pressure - Palpation (unilateral)<br />

(unilateral)<br />

- Auscultation<br />

3. Skin (sitting)<br />

Inspect overall<br />

4. HEENT<br />

A. Head - Inspect face<br />

Inspect/palpate scalp<br />

B. Eyes (sitting)<br />

Test visual acuity for each eye,<br />

corrected preferred<br />

Inspect lids, conjunctivae, sclerae,<br />

corneas<br />

Test extraocular muscle movements<br />

and convergence<br />

Test pupillary responses to light (direct<br />

and consensual) and to accomodation<br />

Inspect cornea, lens and retina <strong>of</strong> each<br />

eye with ophthalmoscope<br />

C. Ears (sitting)<br />

Test for auditory acuity bilaterally<br />

Inspect and palpate auricles and<br />

mastoids<br />

Inspect canals and tympanic<br />

membranes<br />

44


Steps Done Not<br />

Done<br />

Seq.<br />

Incor<br />

.<br />

Tech<br />

.<br />

Incor<br />

.<br />

Comments<br />

D. Nose (sitting)<br />

Palpate nose and sinuses for<br />

tenderness<br />

Test patency <strong>of</strong> each nasal passage by<br />

asking patient to "sniff"<br />

Inspect each nasal passage with<br />

speculum<br />

E. Mouth (sitting)<br />

Test CN V - "bite down" palpate<br />

masseter<br />

Examine CN VII - "show teeth"<br />

Test CN XII - "protrude tongue"<br />

Inspect lips, teeth, gingivae, tongue,<br />

posterior pharynx: "say ah"<br />

5. Neck (sitting)<br />

Inspect for neck veins (see below also)<br />

Inspect/palpate for postauricular, cervical,<br />

sub-mandibular and supraclavicular nodes<br />

Palpate for parotid gland on each side<br />

Inspect and palpate thyroid (each lobe<br />

separately), (and trachea) with swallowing<br />

6. Thorax (sitting)<br />

A. Posterior Thorax (sitting)<br />

Inspect posterior thorax with<br />

respiration<br />

Inspect and palpate entire spine<br />

Percuss costovertebral angles for<br />

tenderness<br />

Percuss posterior thorax bilaterally and<br />

symmetrically<br />

Percuss diaphragmatic excursion<br />

bilaterally<br />

Auscultate posterior lung fields<br />

bilaterally with deep inspiration<br />

Auscultate lateral lung fields bilaterally<br />

B. Anterior Thorax (sitting)<br />

Inspect anterior thorax with respiration<br />

45


Steps Done Not<br />

Done<br />

Seq.<br />

Incor<br />

.<br />

Tech<br />

.<br />

Incor<br />

.<br />

Comments<br />

Auscultate anterior lung fields<br />

bilaterally (starting with supraclavicular area)<br />

with deep inspiration<br />

C. Axillae (supine) Palpate axillae with<br />

passive abduction- adduction<br />

D. Breasts (supine)<br />

Inspect & palpate breasts including<br />

nipples<br />

7. Cardiovascular (supine)<br />

A. Neck Veins/arteries: Inspect, palpate,<br />

auscultate carotid arteries bilaterally one at a<br />

time<br />

B. Heart (supine) Inspect precordium<br />

Palpate PMI (apical impulse) &<br />

precordium<br />

Auscultate with diaphragm at aortic and<br />

pulmonic areas, LSB and apex<br />

Auscultate for splitting <strong>of</strong> S 2 in pulmonic<br />

area<br />

left<br />

Auscultate with bell at apex (supine &<br />

lateral decubitus)<br />

8. Abdomen (supine; hips and knees flexed)<br />

Alert patient to abdominal exam<br />

Inspect abdomen<br />

for<br />

Auscultate epigastrium and over aorta<br />

bruit, and bowel sounds<br />

Palpate superficially/deeply in 4<br />

quadrants<br />

Palpate for liver<br />

Palpate for aorta<br />

Palpate for spleen - supine position<br />

- R lateral decubitus<br />

Inspect and palpate for inguinal nodes<br />

and femoral pulses in inguinal areas<br />

bilaterally<br />

9. Extremities<br />

A. Lower Extremities (supine)<br />

Inspect and palpate legs and feet for edema<br />

46


Steps Done Not<br />

Done<br />

Seq.<br />

Incor<br />

.<br />

Tech<br />

.<br />

Incor<br />

.<br />

Comments<br />

Palpate dorsalis pedis & posterior tibial<br />

pulses bilaterally<br />

B. Upper Extremities (sitting)<br />

Inspect skin and nails. Observe for<br />

tremor<br />

Examine grip<br />

Examine CN XI ("shrug shoulders")<br />

Test for deep tendon reflexes, biceps,<br />

triceps, knee, ankle, and plantar<br />

reflexes bilaterally<br />

C. Posture (standing) - Inspect spine<br />

D. Lower Extremities (standing)<br />

Inspect for varicosities<br />

E. Gait - Inspect walking<br />

Did student respect patient's privacy?<br />

No<br />

Did student arouse patient's anxiety?<br />

No<br />

Yes<br />

Yes<br />

Did student display appropriate affect?<br />

No<br />

Yes<br />

Did student use appropriate verbal/nonverbal behavior? Yes<br />

No<br />

47


U.H. John A. Burns School <strong>of</strong> <strong>Medicine</strong><br />

Third-Year Clerkship in Internal <strong>Medicine</strong><br />

OBSERVED PATIENT COUNSELING<br />

Evaluation Form<br />

Complete 2 during ambulatory medicine and turn in to your Ambulatory Preceptor.<br />

Student: __________________________ Site: ________________ Date: __________<br />

Setting: _____ Inpatient<br />

Counseling was directed at: _____ Patient<br />

_____ Ambulatory (check one or both) _____ Patient’s family<br />

Counseling:<br />

___ Explain diagnoses<br />

___ Explain tests, procedures or surgery<br />

___ Review results <strong>of</strong> tests, procedures or surgery<br />

___ Review medications (indications, dosing, side effects, etc.)<br />

___ Review diets, exercise or other lifestyle changes<br />

___ Discuss smoking cessation<br />

___ Discuss alcohol use<br />

___ Discuss compliance (medications, follow-up, etc.)<br />

___ Review discharge instructions<br />

___ Other (specify) ___________________________<br />

Good<br />

Needs Improvement<br />

Student used clear and understandable language.<br />

Student adapted to patient’s/family’s readiness to learn.<br />

Student adapted to patient’s/family’s comprehension level<br />

All pertinent information was presented accurately.<br />

Student demonstrated empathy and compassion.<br />

Patient’s/family’s comprehension was assessed.<br />

Evaluator Comments:<br />

Evaluator’s name _________________________<br />

Evaluator’s signature _______________________<br />

Student’s signature _______________________<br />

I received constructive feedback on my Observed Patient Counseling (circle): Yes No<br />

48


U.H. John A. Burns School <strong>of</strong> <strong>Medicine</strong><br />

Third-Year Clerkship in Internal <strong>Medicine</strong><br />

CASE PRESENTATION<br />

Evaluation Form<br />

Before presenting, the student should give this form to the attending or resident leading rounds. Afterwards, the<br />

student should turn the form in to the Hospital Site Coordinator.<br />

Student: _______________________________________<br />

Date: _______________<br />

Rounds:<br />

___ Attending Rounds<br />

___ Bedside Clinical Skills<br />

___ ICU Rounds<br />

___ Morning Report<br />

___ PBL Tutorial<br />

___ Other Rounds/Conference __________________<br />

Major problems/diagnoses <strong>of</strong> case presented:<br />

1. ___________________________<br />

2. ___________________________<br />

3. ___________________________<br />

Presentation was clear<br />

Presentation was organized<br />

Presentation was memorized<br />

All pertinent history was presented<br />

accurately<br />

All pertinent physical findings were<br />

presented accurately<br />

All pertinent labs were presented<br />

accurately<br />

All pertinent problems were correctly<br />

identified<br />

Assessment was appropriate for level <strong>of</strong><br />

training<br />

Plan (diagnostic, therapeutic, education)<br />

was appropriate for level <strong>of</strong> training<br />

Comments:<br />

Exceptional<br />

Very<br />

Competent Competent Marginal Unsatisfactory<br />

Evaluator:<br />

Print _____________________________<br />

Signature _________________________<br />

49


U.H. John A. Burns School <strong>of</strong> <strong>Medicine</strong><br />

Third-Year Clerkship in Internal <strong>Medicine</strong><br />

SMALL GROUP LEARNING EXPERIENCE<br />

Evaluation Form<br />

Student: ____________________ Date: _________ Activity: ____ PBL Tutorial<br />

____ Bedside Clinical Skills<br />

____ Chief Rounds<br />

____ Other: ___________<br />

Check appropriate box on this Rating Scale:<br />

Activity<br />

Participates actively<br />

Shares knowledge<br />

Respects opinions and learning<br />

needs <strong>of</strong> others<br />

Asks thoughtful questions<br />

Facilitates group process<br />

Demonstrates appropriate fund<br />

<strong>of</strong> knowledge for MS3<br />

Demonstrates appropriate<br />

clinical skills for MS 3<br />

Is properly prepared for this<br />

activity<br />

Evaluator Comments:<br />

Exceptional<br />

Very<br />

Competent<br />

Competent Marginal Unsatisfactory<br />

Evaluator’s name: ___________________ Evaluator’s signature: _____________________<br />

Student’s signature _________________<br />

50


U.H. John A. Burns School <strong>of</strong> <strong>Medicine</strong><br />

Third-Year Clerkship in Internal <strong>Medicine</strong><br />

MID-CLEKSHIP FEEDBACK FORM<br />

Student’s name: ___________________________________________________<br />

Considering the three domains <strong>of</strong> Medical Knowledge, Clinical Skills and Pr<strong>of</strong>essionalism,<br />

This student’s STRENGTHS are:<br />

This student NEEDS TO WORK AND IMPROVE ON:<br />

Overall, this student’s progress to date is:<br />

___ Satisfactory<br />

___ Unsatisfactory<br />

Evaluator’s name: ___________________________<br />

Student's signature: __________________________<br />

Signature: _____________________________<br />

Date reviewed with student: _______________<br />

Please make a photocopy <strong>of</strong> this form after it is completed, reviewed and signed by both the Evaluator and the<br />

Student.<br />

Instructions for the Student:<br />

Keep a photocopy <strong>of</strong> this form after your Hospital Site Coordinator or Ambulatory Preceptor completes and reviews it with<br />

you.<br />

You should actively work on and improve the areas identified above.<br />

You will be instructed on how to turn in the completed form at the end <strong>of</strong> your inpatient block and at the end <strong>of</strong> your<br />

ambulatory block.<br />

Instructions for the Evaluator:<br />

Please keep a copy and include the information in your final Student Evaluation Form.<br />

Specifically, if you identified any areas to work on and improve, please comment on whether the student successfully<br />

responded to your feedback and improved by the end <strong>of</strong> the block.<br />

51


Exceptional<br />

Very Competent<br />

Competent<br />

Marginal<br />

Unsatisfactory<br />

Not Evaluated<br />

UH John A. Burns School <strong>of</strong> <strong>Medicine</strong><br />

Third-Year Clerkship in Internal <strong>Medicine</strong><br />

STUDENT EVALUATION FORM<br />

Student: _____________________________________<br />

Ambulatory Site: _______________________<br />

Block: _____ Dates <strong>of</strong> rotation: _____________________ Date <strong>of</strong> this report: ___________________<br />

IMPORTANT:<br />

o Comments are MANDATORY for any Exceptional, Marginal or Unsatisfactory ratings.<br />

<br />

I. Life-Long Learning Skills<br />

Graduates will be life-long learners.<br />

Demonstrates enthusiasm, motivation and self-initiative to learn<br />

Identifies relevant learning issues, searches for and critically appraises information<br />

(self-directed learning)<br />

Comments:<br />

II.<br />

The Biological Sciences<br />

Graduates will understand the biological sciences underlying clinical medicine.<br />

Demonstrates basic science knowledge (including anatomy, physiology,<br />

pathology and pathophysiology)<br />

Demonstrates clinical knowledge (including knowledge <strong>of</strong> the illnesses seen in the<br />

adult patient/population in <strong>Hawaii</strong>, Asia-Pacific region and globally)<br />

Applies basic science knowledge to clinical situations<br />

Comments:<br />

52


Exceptional<br />

Very Competent<br />

Competent<br />

Marginal<br />

Unsatisfactory<br />

Not Evaluated<br />

<br />

<br />

III. The Care <strong>of</strong> Patients<br />

Graduates will be able to care for their patients under the supervision <strong>of</strong> faculty.<br />

Demonstrates respect and compassion for patients and their families<br />

Serves as an effective patient advocate<br />

Interviews patients in a systematic, organized and thorough manner<br />

(History Taking)<br />

Examines patients in a systematic, organized and thorough manner<br />

(Physical Exam)<br />

Formulates appropriate Assessments, Problem Lists and Differential Diagnoses<br />

Formulates appropriate Diagnostic Plans<br />

Formulates appropriate Therapeutic Plans<br />

Educates and counsels patients (and their families) about their health and<br />

medical problems<br />

Demonstrates clinical judgment and problem solving skills<br />

Works efficiently<br />

Comments:<br />

IV. Oral and Written Communication Skills<br />

Graduates will be able to communicate effectively with patients, families and other<br />

healthcare providers.<br />

Uses effective verbal and non-verbal communication skills (including listening<br />

and using language appropriate for each patient, family and clinical setting)<br />

Presents cases in a systematic, organized and thorough manner that is accurate<br />

and appropriate for the clinical setting<br />

Writes notes (write-ups and progress notes, etc.) in a systematic, organized and<br />

thorough manner that is accurate, legible and appropriate for the clinical setting<br />

Comments:<br />

53


Exceptional<br />

Very Competent<br />

Competent<br />

Marginal<br />

Unsatisfactory<br />

Not Evaluated<br />

<br />

V. Pr<strong>of</strong>essionalism<br />

Graduates will be pr<strong>of</strong>essional and ethical, demonstrate an enthusiasm for medicine, and value<br />

honor, integrity, altruism, respect, accountability, excellence, scholarship, and leadership<br />

while delivering compassionate care to their patients.<br />

Demonstrates pr<strong>of</strong>essional appearance and attire<br />

Attends all required <strong>clerkship</strong> and training site-specific activities; is punctual<br />

Listens attentively and participates actively<br />

Completes all <strong>clerkship</strong> and patient care responsibilities on time<br />

Demonstrates pr<strong>of</strong>essional behavior<br />

Demonstrates pr<strong>of</strong>essional relationships (with attendings, residents, students,<br />

nurses, other health care workers, patients and their families)<br />

Seeks, accepts and responds positively and effectively to feedback<br />

Works as an effective team member<br />

Comments:<br />

COMMENTS:<br />

Ambulatory Preceptor’s name: ___________________________________________<br />

Ambulatory Preceptor’s signature: ____________________________ Date: _________________<br />

Student’s signature: ______________________________________<br />

Date: _________________<br />

Please return this form to:<br />

Julieta Rajlevsky<br />

UH <strong>Department</strong> <strong>of</strong> <strong>Medicine</strong><br />

1356 Lusitana St., 7 th Floor, Honolulu, HI 96813<br />

Fax: (808) 586-7486 Phone: (808) 586-7478<br />

54


UNIVERSITY OF HAWAII JOHN A. BURNS SCHOOL OF MEDICINE<br />

THIRD-YEAR CLERKSHIP IN INTERNAL MEDICINE<br />

STUDENT EVALUATION FORM<br />

Ratings Definitions<br />

EXCEPTIONAL:<br />

The student consistently demonstrates medical knowledge, clinical skills or pr<strong>of</strong>essionalism at a level<br />

higher than expected for a third year student. In other words, the student consistently “goes the extra mile”<br />

and performs “above and beyond” in the item rated.<br />

Comments are mandatory for this rating.<br />

VERY COMPETENT:<br />

The student on occasion demonstrates medical knowledge, clinical skills or pr<strong>of</strong>essionalism at a level<br />

higher than expected for a third year student. In other words, the student on occasion, but not consistently,<br />

“goes the extra mile” and performs “above and beyond” in the item rated. The student should continue to<br />

strive for Exceptional ratings.<br />

COMPETENT:<br />

The student demonstrates medical knowledge, clinical skills or pr<strong>of</strong>essionalism at the level expected for a<br />

third year student. Note that the <strong>clerkship</strong> expects that most students will receive Competent ratings for the<br />

majority <strong>of</strong> items. Some students in the past have incorrectly interpreted Competent ratings as punitive.<br />

On the contrary, this rating merely indicates that the student performed at the expected level for that<br />

particular item. The student should continue to strive for Very Competent and Exceptional ratings.<br />

MARGINAL:<br />

The student demonstrates medical knowledge, clinical skills or pr<strong>of</strong>essionalism slightly below the level<br />

expected for a third year student. An item rated Marginal is an area <strong>of</strong> weakness that has been identified.<br />

The student will be expected to work on this area <strong>of</strong> weakness and demonstrate improvement to the level<br />

expected for a third year student. A Marginal rating probably precludes an Honors grade.<br />

Comments are mandatory for this rating.<br />

UNSATISFACTORY:<br />

The student demonstrates medical knowledge, clinical skills or pr<strong>of</strong>essionalism below the level expected<br />

for a third year student. An item rated Unsatisfactory is an area <strong>of</strong> weakness that has been identified.<br />

Formal remediation may be required. An Unsatisfactory rating precludes an Honors grade.<br />

Comments are mandatory for this rating.<br />

55


OTHER FORMS<br />

56


U.H. John A. Burns School <strong>of</strong> <strong>Medicine</strong><br />

Third-Year Clerkship in Internal <strong>Medicine</strong><br />

MS3-RESIDENT<br />

INPATIENT MEDICINE AGREEMENT FORM<br />

(Revised 8/11/12)<br />

Name <strong>of</strong> MS3 _______________________________________________<br />

1. The upper level (UL) resident (Level 2 or 3) is responsible for the third-year medical<br />

student’s inpatient medicine experience. At the beginning and throughout the student’s<br />

inpatient experience, the UL should clarify what is expected <strong>of</strong> the student.<br />

2. The earliest time that the student is permitted to arrive at the hospital is:<br />

o 4:00 a.m. at Kuakini<br />

o 5:00 a.m. at Queen’s<br />

o 5:00 a.m. at Tripler<br />

The earliest time that the student is permitted to see patients is:<br />

o 4:30 a.m. at Kuakini<br />

o 5:30 a.m. at Queen’s<br />

o 5:30 a.m. at Tripler<br />

3. The student must take every call with his/her team throughout the inpatient rotation.<br />

Call at Kuakini:<br />

The student must take every call (q4) with his/her team throughout the inpatient rotation.<br />

Overnight Call - 1 required, preferably a Fri night<br />

If the student is unable to take overnight call on a Fri night, he/she must talk to the<br />

CMR and the Hospital Site Coordinator about taking overnight call on a Sat instead.<br />

Call – q4 days until 10 p.m. at the latest<br />

(May leave earlier depending on admissions and UL approval.)<br />

Non-Call Days – The student will assist his/her team until after the team signs out.<br />

Call at Queen’s:<br />

The student must take every call (q4) with his/her team throughout the inpatient rotation.<br />

Overnight Call - none<br />

Long Call – q4 days until 10 p.m. at the latest<br />

(May leave earlier depending on admissions and UL approval.)<br />

Non-Call Days –The student will assist his/her team with admitting patients and may<br />

leave after team is done with work, with UL approval.<br />

Call at Tripler:<br />

The student must take every call (q4) with his/her team throughout the inpatient rotation.<br />

Overnight Call - none<br />

Long Call: 0600 – 1900 q4 days<br />

Short Call: 0600 – 1600 all other days - The student will assist his/her team until sign<br />

out.<br />

4. The UL is responsible for assigning patients to the student (see Training Problems List).<br />

The student will admit 1 - 2 patients per call.<br />

The student should actively follow an average <strong>of</strong> 2 patients at all times (maximum 5).<br />

57


5. The student must interview and examine patients on his/her own. The student may<br />

observe the Intern and/or UL obtain the history and physical, but this observation does not<br />

qualify as the student’s history and physical.<br />

6. The student must pre-round and write daily Progress Notes on all his/her assigned<br />

patients before the Intern and/or UL write their notes. The UL should review the<br />

Progress Notes with the student, give constructive feedback and countersign the note.<br />

7. The UL is responsible for insuring proper supervision <strong>of</strong> the following parts <strong>of</strong> the<br />

physical exam performed by the student: female breast exams, pelvic exams, rectal<br />

exams and prostate exams. The supervision must be provided by a physician (such as<br />

Interns, ULs, Chief Residents or Attendings).<br />

8. The UL is responsible for insuring proper supervision <strong>of</strong> any procedure performed<br />

by the student. The supervision must be provided by a physician (such as Interns, ULs,<br />

Chief Residents or Attendings) who is certified or has expertise to competently perform<br />

the procedure in question. Note that there are no required procedures for students.<br />

9. The UL should assist the student in preparing case presentations at hospital rounds<br />

or conferences at a level that is appropriate for the student’s training. Whenever a<br />

student’s patient will be presented, the student is expected to be the one presenting the<br />

patient (unless it is an ICU patient at Kuakini – then it is up to the CMR and HSC).<br />

10. The student must have one (1) day <strong>of</strong>f per week, either a Saturday or a Sunday.<br />

At Kuakini, the day <strong>of</strong>f should be on Sunday - unless the student has call on Sunday, in<br />

which case the student will take Saturday <strong>of</strong>f.<br />

11. The student must inform the UL <strong>of</strong> the student’s activities and whereabouts at all<br />

times. Specifically, the student must notify the UL whenever leaving the hospital,<br />

including leaving to attend required 3 rd year or <strong>clerkship</strong> activities or to study, and should<br />

discuss if or when he/she needs to return.<br />

12. When the UL is absent or <strong>of</strong>f, the Intern should assume the UL’s role and<br />

responsibilities, including all those listed above, in supervising the student.<br />

13. The Intern and UL should discuss medical student issues and problems with the<br />

Chief Resident and/or Hospital Site Coordinator as soon as possible.<br />

1 st Upper Level Resident:<br />

Name _______________________ Signature ______________________ Date _________<br />

(Required by end <strong>of</strong> MS3’s 1 st week)<br />

2nd Upper Level Resident:<br />

Name _______________________ Signature ______________________ Date_________<br />

(Required by end <strong>of</strong> UL’s 1 st week)<br />

58


Third-Year Clerkship in Internal <strong>Medicine</strong><br />

INPATIENT WORK HOURS LOG<br />

Student _____________________________________<br />

Inpatient site (circle) KMC QMC TAMC<br />

Week ___ beginning ____/_____/2012<br />

On Call?<br />

Other Activities?<br />

# New Patients<br />

(admissions, transfers)<br />

# Old Patients<br />

TOTAL # PATIENTS<br />

For KMC,<br />

# ICU Patients<br />

Time In<br />

Time Out<br />

TOTAL # HOURS<br />

MONDAY TUESDAY WEDNESDAY THURSDAY FRIDAY SATURDAY SUNDAY<br />

TOTAL # HOURS FOR THE WEEK: _______<br />

o<br />

o<br />

o<br />

o<br />

o<br />

o<br />

o<br />

o<br />

o<br />

o<br />

o<br />

o<br />

Please log your hours daily – otherwise it’s difficult to remember.<br />

You should log only the hours that you are "working" which includes patient care and required 3 rd year, hospital and <strong>clerkship</strong> activities such as Colloquia, rounds, conferences,<br />

PBL Tutorials, Bedside Clinical Skills, Chief Rounds, CV PE, EBM, EKG, HIV <strong>Medicine</strong>, METS Sim Session, Neuro, etc.<br />

It's ok to include meals in the middle <strong>of</strong> your “work day” (as long as it's not a 1 hr lunch!) – it’s too much trouble to clock out for lunch and then clock in afterwards.<br />

You should not include meals at the hospital before/after your “work day” or reading at the hospital before/after your “work day.”<br />

You should not include writing your Comprehensive Write-ups and LIs – even if done at the hospital - since that is "home" work.<br />

You should work no more than 80 hours per week, averaged over the course <strong>of</strong> the entire <strong>clerkship</strong>.<br />

You should have 1 day <strong>of</strong>f per week, usually a Sat or Sun.<br />

You should follow the holiday schedule observed by your site, since this varies by site.<br />

On Call? If relevant, please specify Short, Long, or Overnight.<br />

Other Activities? Please specify <strong>of</strong>f-campus activities. Ex: Colloquia, HIV <strong>Medicine</strong>, METS Sim Session, etc.<br />

# Patients is the # <strong>of</strong> patients (counted at the end <strong>of</strong> each day) that you are actively following - that is, pre-rounding on, writing notes on and presenting.<br />

Ask your UL to sign the following Monday _____________________________________________Then turn completed Log in to your CMR/Hospital Site Coordinator.<br />

59


PHYSICIAN’S ORDERS<br />

DATE: / / TIME:<br />

IMPORTANT<br />

INSTRUCTIONS<br />

► 1. All orders must be written in metric system<br />

2. Write with ball point pen only<br />

SIGNATURE<br />

DATE: / / TIME:<br />

SIGNATURE<br />

DATE: / / TIME:<br />

SIGNATURE<br />

60


APPENDICES<br />

61


U.H. John A. Burns School <strong>of</strong> <strong>Medicine</strong><br />

Third-Year Clerkship in Internal <strong>Medicine</strong><br />

TRAINING PROBLEMS LIST<br />

You are required to see at least one patient with each <strong>of</strong> the 33 Training Problems during the course <strong>of</strong> this <strong>clerkship</strong>.<br />

This is the minimum requirement. Your goal, however, should be to see at least one inpatient patient and one<br />

ambulatory patient with each <strong>of</strong> the Training Problems. The Training Problem does not have to be the patient’s Chief<br />

Complaint. In fact, a patient may present with many Training Problems.<br />

Of course, the more patients you see with these Training Problems, the more you will learn.<br />

Keep track <strong>of</strong> your patient encounters in the chart below.<br />

Indicate which encounters involved Direct Patient Care (D) and which encounters did not involve direct patient care,<br />

referred to as “Special” (S), as in your PDA Patient Log.<br />

It is your responsibility to insure that you have seen patients with each <strong>of</strong> the 33 Training Problems by the end <strong>of</strong> the<br />

<strong>clerkship</strong>. You are advised to see your Upper Level Resident, Chief Medical Resident and/or Hospital Site Coordinator<br />

(when you are on inpatient medicine) or Ambulatory Preceptor (when you are on ambulatory medicine) as soon as<br />

possible to assist you in finding appropriate patients.<br />

Please refer to the Student Handbook for the Specific Learning Objectives for each Training Problem.<br />

Your study <strong>of</strong> Internal <strong>Medicine</strong> in this <strong>clerkship</strong> should be guided by these Training Problems and their Specific Learning<br />

Objectives.<br />

Inpatient<br />

Ambulatory<br />

-- -- D S 1. Healthy Patient:<br />

Health promotion, disease prevention and screening<br />

(i.e. annual or routine physical exam)<br />

Patients with a symptom, sign or<br />

lab abnormality (14):<br />

D S D S 2. Abdominal pain<br />

D S D S 3. Altered mental status<br />

D S D S 4. Anemia<br />

D S D S 5. Back pain<br />

D S D S 6. Chest pain<br />

D S D S 7. Cough<br />

D S D S 8. Dyspnea<br />

D S D S 9. Dysuria<br />

D S D S 10. Fever<br />

D S D S 11. Fluid, electrolyte & acid-base disorders<br />

D S D S 12. GI bleeding<br />

D S D S 13. Knee pain<br />

D S D S 14. Rash<br />

D S D S 15. Upper respiratory complaints<br />

62


TRAINING PROBLEMS LIST (continued)<br />

Inpatient<br />

Ambulatory<br />

Patients with a known condition<br />

(18):<br />

D S D S 16. Acute MI<br />

D S D S 17. Acute renal failure & Chronic kidney<br />

disease<br />

D S D S 18. Common cancers<br />

D S D S 19. COPD & Obstructive airways disease<br />

D S D S 20. Diabetes mellitus<br />

D S D S 21. Dyslipidemias<br />

D S D S 22. Heart failure<br />

D S D S 23. HIV infection<br />

D S D S 24. Hypertension<br />

D S D S 25. Liver disease<br />

D S D S 26. Major depression<br />

D S D S 27. Nosocomial infections<br />

D S D S 28. Obesity<br />

D S D S 29. Pneumonia<br />

D S D S 30. Rheumatologic problems<br />

D S D S 31. Smoking cessation<br />

D S D S 32. Substance abuse<br />

D S D S 33. Venous thromboembolism<br />

Complete the Inpatient half <strong>of</strong> this 2 page chart by your last day <strong>of</strong> inpatient medicine, including your Hospital Site<br />

Coordinator’s signature, and then turn in to Julieta Rajlevsky in the Dept <strong>of</strong> <strong>Medicine</strong> at UHT 7 th fl.<br />

Complete the Ambulatory half <strong>of</strong> this 2 page chart by your last day <strong>of</strong> ambulatory medicine, including your<br />

Ambulatory Preceptor’s signature, and then turn in to Julieta Rajlevsky in the Dept <strong>of</strong> <strong>Medicine</strong> at UHT 7 th fl.<br />

Student_____________________________<br />

Name<br />

____________________________ Date____________<br />

Signature<br />

Hospital Site Coordinator____________________ _____________________________ Date____________<br />

Name<br />

Signature<br />

Ambulatory Preceptor______________________<br />

Name<br />

_____________________________ Date ____________<br />

Signature<br />

63


TRAINING PROBLEM #1: THE HEALTHY PATIENT: HEALTH<br />

PROMOTION, DISEASE PREVENTION, AND SCREENING<br />

RATIONALE:<br />

The growing appreciation for the contributions <strong>of</strong> screening, prevention, and health promotion to<br />

health outcomes necessitates that basic clinical education incorporate advances made in this<br />

area. Especially important are those interventions that relate to prevention <strong>of</strong> cardiovascular<br />

disease, the early detection and treatment <strong>of</strong> potentially curable cancers, and to optimizing care<br />

for chronic diseases.<br />

PREREQUISITES:<br />

Prior knowledge, skills and attitudes acquired during the pre-<strong>clerkship</strong> experience should include:<br />

Ability to perform a complete medical history and physical exam.<br />

Ability to obtain a patient history that includes a family history and an assessment <strong>of</strong> risk<br />

factors.<br />

Knowledge <strong>of</strong> the warning signs <strong>of</strong> common cancers.<br />

Knowledge <strong>of</strong> basic criteria and principles <strong>of</strong> health screening.<br />

Knowledge <strong>of</strong> clinical epidemiologic concepts as they pertain to estimation <strong>of</strong> health risk and<br />

quantitative rationale for screening.<br />

SPECIFIC LEARNING OBJECTIVES:<br />

A. KNOWLEDGE: Students should be able to define, describe, and discuss:<br />

1. The epidemiology and definitions <strong>of</strong> hypertension, its contribution to cardiovascular risk, the<br />

impact <strong>of</strong> treatment on risk, and current. Recommendations for screening. (MK)<br />

2. The epidemiology <strong>of</strong> hyperlipidemia, its contribution to cardiovascular risk, the reliability <strong>of</strong><br />

testing modalities, the impact <strong>of</strong> treatment on cardiovascular risk, and current<br />

recommendations for screening. (MK)<br />

3. The epidemiology <strong>of</strong> common cancers, including:<br />

Breast cancer, including the efficacy <strong>of</strong> available screening modalities, impact <strong>of</strong> early<br />

treatment on survival, and current recommendations for screening. (MK)<br />

Common skin cancers, including the warning signs <strong>of</strong> melanoma and basal and squamous<br />

cell carcinoma. (MK)<br />

Cervical cancer, including the utility <strong>of</strong> the Pap smear, impact <strong>of</strong> early treatment on<br />

outcome, and current recommendations for screening.(MK)<br />

Colorectal cancer, including the utility <strong>of</strong> available screening methodologies, the impact <strong>of</strong><br />

early treatment on outcome, and current screening recommendations. (MK)<br />

Prostate cancer, including the utility <strong>of</strong> available screening modalities, impact <strong>of</strong> early<br />

treatment on outcome, and current screening recommendations. (MK)<br />

4. The risks, benefits, methods, and recommendations for immunizing adults against hepatitis B,<br />

influenza, pneumococcal infection, tetanus/diphtheria, and mumps/measles/rubella. (MK)<br />

5. Safe sexual practices and risks, benefits, and efficacy <strong>of</strong> common methods <strong>of</strong> contraception.<br />

(MK)<br />

6. Efficacy <strong>of</strong> seat belt use and proper belt application. (MK)<br />

7. Efficacy <strong>of</strong> exercise and weight loss in prevention <strong>of</strong> cardiovascular disease and recommended<br />

exercise programs. (MK)<br />

8. The clinical presentations <strong>of</strong> substance abuse and basic approaches to prevention and<br />

treatment. (MK)<br />

64


9. The impact <strong>of</strong> smoking on cardiovascular and cancer risk and basic approaches to smoking<br />

cessation. (MK)<br />

10. Daily caloric, fat, carbohydrate, protein, mineral, and vitamin requirements; adequacy <strong>of</strong> diets<br />

in providing such requirements; evidence <strong>of</strong> need for supplements (e.g. calcium, antioxidants).<br />

(MK)<br />

11. The functional status assessment in the geriatric patient and its impact on assuring the best<br />

possible functional state. (MK)<br />

12. Common environmental and occupational hazards. (MK)<br />

13. Controversies and differences that exist in the recommendations for preventive measures and<br />

screening. (MK)<br />

B. SKILLS: Students should demonstrate specific skills including:<br />

1. History-taking skills: Students should be able to obtain, document, and present an ageappropriate<br />

medical history, including:<br />

Dietary intake <strong>of</strong> fats and cholesterol. (PC, CS)<br />

Exercise and activity levels. (PC, CS)<br />

Substance use and its effects, including tobacco, alcohol, and elicit drugs. (PC, CS)<br />

Psychosocial stresses and environmental risks. (PC, CS)<br />

Specific cancer risks (e.g. family history, exposures, warning symptoms, preventive efforts).<br />

(PC, CS)<br />

Any high-risk sexual practices. (PC, CS)<br />

Immunization status appropriate for adults, including:<br />

o Diphtheria/tetanus for all adults. (PC, CS)<br />

o Influenza vaccine and pneumococcal vaccine for the elderly and those with underlying<br />

chronic disease. (PC, CS)<br />

o Rubella for sero-negative women <strong>of</strong> child-bearing age. (PC, CS)<br />

o Hepatitis B vaccine for medical personnel and other at-risk populations. (PC, CS)<br />

2. Physical exam skills: Students should be able to perform a physical exam with features<br />

depending on age/sex/race and medical history <strong>of</strong> an individual, including:<br />

Screening skin examination for signs <strong>of</strong> malignancy. (PC)<br />

Screening breast examination for a dominant nodule and secondary signs <strong>of</strong> malignancy.<br />

(PC)<br />

Participation in obtaining a Pap smear. (PC)<br />

Screening rectal examination that includes palpation <strong>of</strong> the prostate gland, identification <strong>of</strong><br />

any nodules, and performance <strong>of</strong> a stool test for occult blood. (PC)<br />

Performance <strong>of</strong> a functional status examination in the geriatric patient (PC)<br />

3. Differential diagnosis: Students should be able to generate a prioritized differential diagnosis<br />

using specific history, physical exam, and laboratory findings identified during the screening<br />

examination (PC, MK)<br />

4. Laboratory interpretation: Students should be able to recommend and interpret laboratory<br />

tests for screening purposes, including consideration <strong>of</strong> test cost and performance<br />

characteristics as well as patient preferences. Laboratory and other tests may include, when<br />

appropriate:<br />

Complete blood count. (PC, MK)<br />

Fasting lipid panel. (PC, MK)<br />

Fasting blood glucose. (PC, MK)<br />

Urinalysis. (PC, MK)<br />

65


Stool test for occult blood. (PC, MK)<br />

Prostate specific antigen. (PC, MK)<br />

Students should be able to define the indications for and interpret (with consultation) results<br />

<strong>of</strong>:<br />

Mammography. (PC, MK)<br />

Colonoscopy. (PC, MK)<br />

Pap smear. (PC, MK)<br />

Bone densitometry. (PC, MK)<br />

5. Communication skills: Students should be able to:<br />

Communicate results <strong>of</strong> the evaluation and counsel for disease prevention. (PC, CS)<br />

Elicit questions from the patient and his or her family about the plan. (PC, CS)<br />

6. Basic and advanced procedural skills: Students should be able to:<br />

Perform a urinalysis (dipstick and microscopic). (PC)<br />

Stool occult blood testing. (PC)<br />

Calculate a BMI. (PC)<br />

Perform a functional status examination for elderly patients. (PC)<br />

Administer intramuscular injection <strong>of</strong> a vaccine. (PC)<br />

Participate in obtaining a Pap smear. (PC)<br />

7. Management skills: Students should be able to develop an appropriate evaluation and<br />

treatment plan for healthy patients, including:<br />

Designing an appropriate work-up for any abnormalities noted on the screening exam. (PC,<br />

MK)<br />

Teaching breast self-examinations. (PC, CS)<br />

Counseling for:<br />

o Safe sexual practices. (PC, CS)<br />

o Seatbelt use. (PC, CS)<br />

o Healthy diet. (PC, CS)<br />

o Weight loss. (PC, CS)<br />

o Practical exercise program appropriate to the patient's age, and current physical<br />

condition. (PC, CS)<br />

o Stress management. (PC, CS)<br />

o Alcohol abstinence. (PC, CS)<br />

o Smoking cessation. (PC, CS)<br />

o Cancer screening. (PC, CS)<br />

o Limiting risks <strong>of</strong> occupational and environmental hazards. (PC, CS)<br />

Accessing and utilizing appropriate information systems and resources to help delineate<br />

issues related to healthy patients. (PC, PLI)<br />

Using a cost-effective approach based for screening. (PC, SBP)<br />

Incorporating patient preferences. (PC, P)<br />

Engaging the patient as an active participant in his/her health care. (PC,P)<br />

C. ATTITUDES AND PROFESSIONAL BEHAVIORS: Students should be able to:<br />

1. Recognize the importance <strong>of</strong> regularly screening all patients followed and <strong>of</strong> teaching all<br />

patients about preventive measures. (PC, P)<br />

2. Appreciate the necessity <strong>of</strong> keeping detailed records <strong>of</strong> screening and health maintenance<br />

measures. (PC, P)<br />

3. Understand that physicians and health care delivery organizations are frequently judged by<br />

their ability to deliver the highest quality screening and preventive measures. (PLI, P, SBP)<br />

66


4. Recognize the importance <strong>of</strong> addressing community sources <strong>of</strong> health risk. (PC, P)<br />

5. Respond appropriately to patients who are nonadherent preventive measures. (CS, P)<br />

6. Respect the patient’s right to refuse preventive measures and screening. (P)<br />

7. Demonstrate commitment to using risk-benefit, cost-benefit, and evidence-based<br />

considerations in the selection <strong>of</strong> screening tests. (PLI, P)<br />

8. Demonstrate ongoing commitment to self-directed learning regarding prevention and<br />

screening. (PLI, P)<br />

9. Recognize the importance and demonstrate a commitment to the utilization <strong>of</strong> other healthcare<br />

pr<strong>of</strong>essions in preventative medicine. (P, SBP)<br />

D. RESOURCES:<br />

484.<br />

USPSTF Recommendation: Screening for Breast Cancer<br />

www.ahcpr.gov/clinic/3rduspstf/breastcancer/brcanrr.htm<br />

USPSTF Recommendation: Screening for Cervical Cancer<br />

www.ahrq.gov/clinic/3rduspstf/cervcan/cervcanrr.htm<br />

USPSTF Recommendations Statement: Counseling to prevent tobacco use and tobaccocaused<br />

disease<br />

www.ahrq.gov/clinic/3rduspstf/tobacccoun/tobcounrs.htm<br />

Screening for Prostate Cancer. American College <strong>of</strong> Physicians. Ann Int Med 1997; 126: 480-<br />

Summary <strong>of</strong> Recommendations for Adult Immunization. Immunization Action Coalition Bulletin.<br />

Adapted from the recommendations <strong>of</strong> the Advisory Committee on Immunization Practices<br />

(ACIP), August 2005<br />

www.immunize.org/acip<br />

67


TRAINING PROBLEM #2: ABDOMINAL PAIN<br />

RATIONALE:<br />

Abdominal pain is a common symptom that can be attributed to a wide variety <strong>of</strong> acute and<br />

chronic disease processes, many <strong>of</strong> which may represent serious medical problems. Mastery <strong>of</strong><br />

the approach to patients with abdominal pain is important to third year medical students.<br />

PREREQUISITES:<br />

Prior knowledge, skills, and attitudes acquired during the pre-<strong>clerkship</strong> experience should include:<br />

Ability to perform a complete medical history and physical exam.<br />

Ability to communicate with patients <strong>of</strong> diverse backgrounds.<br />

Knowledge <strong>of</strong> gastrointestinal and gynecologic anatomy, physiology, and pathophysiology.<br />

SPECIFIC LEARNING OBJECTIVES:<br />

A. KNOWLEDGE: Students should be able to define, describe, and discuss:<br />

1. Three principal types <strong>of</strong> abdominal pain:<br />

Visceral pain: (MK)<br />

o Poorly localized but site roughly corresponds to dermatome that innervates the affected<br />

organ.<br />

o Characteristics may vary (dull, cramping, burning).<br />

o Frequently accompanied by secondary autonomic effects (nausea, vomiting, pallor,<br />

diaphoresis, restlessness).<br />

o Patient moves around in an attempt to alleviate discomfort.<br />

Somatoparietal or peritoneal pain: (MK)<br />

o More localized and more intense than visceral pain.<br />

o Arises from peritoneal irritation.<br />

o Aggravated by movement (patient attempts to lie still).<br />

Referred pain: (MK)<br />

o Usually well localized but felt in areas remote to affected organ.<br />

o May be felt in skin or in deeper tissues.<br />

o Results from convergence <strong>of</strong> visceral afferent neurons with somatic neurons from<br />

different anatomic regions.<br />

2. Relative likelihood <strong>of</strong> the common causes <strong>of</strong> abdominal pain based on the pain pattern and the<br />

quadrant in which the pain is located. (MK)<br />

3. Diagnostic discrimination between common causes <strong>of</strong> abdominal pain based on history,<br />

physical exam, laboratory testing, and imaging procedures. (MK)<br />

4. Symptoms and signs indicative <strong>of</strong> an acute/surgical abdomen. (MK)<br />

5. The influence <strong>of</strong> age, gender, menopausal status, and immunocompetency on the prevalence<br />

<strong>of</strong> different disease processes that may result in abdominal pain. (MK)<br />

B. SKILLS: Students should be able to demonstrate specific skills, including:<br />

1. History-taking skills: Students should be able to obtain, document, and present an<br />

appropriately complete medical history that differentiates among etiologies <strong>of</strong> disease,<br />

including:<br />

• Chronology. (PC, CS)<br />

68


• Location. (PC, CS)<br />

• Radiation. (PC, CS)<br />

• Character. (PC, CS)<br />

• Intensity. (PC, CS)<br />

• Duration. (PC, CS)<br />

• Aggravating or alleviating factors. (PC, CS)<br />

• Associated symptoms. (PC, CS)<br />

• Pertinent information about previous abdominal or pelvic surgeries, chronic medical<br />

conditions, sexual activity, medications, and family history. (PC, CS)<br />

2. Physical exam skills: Students should be able to perform a focused physical exam in<br />

patients who present with abdominal pain in order to:<br />

Establish a preliminary diagnosis <strong>of</strong> the cause. (PC)<br />

Assess the severity <strong>of</strong> the patient’s presenting symptoms and signs(PC)<br />

Determine the urgency <strong>of</strong> implementing diagnostic and treatment<br />

plans. (PC)<br />

The initial physical examination <strong>of</strong> the patient should include:<br />

A general assessment <strong>of</strong> the patient’s appearance, position, and degree <strong>of</strong> discomfort.<br />

(PC)<br />

Measurement <strong>of</strong> vital signs, including temperature, pulse, blood pressure, and, when<br />

indicated, orthostatic blood pressure and pulse. (PC)<br />

Correct order and technique for examining the abdomen. (PC)<br />

Inspection <strong>of</strong> the abdomen for surgical scars, distension, asymmetry or cutaneous<br />

abnormalities (dilated veins, ecchymoses, etc.). (PC)<br />

Auscultation <strong>of</strong> the abdomen for abnormal bowel sounds, bruits. (PC)<br />

Percussion <strong>of</strong> the abdomen for detection <strong>of</strong> hepatomegaly, splenomegaly, abdominal<br />

masses, or the presence <strong>of</strong> ascites. (PC)<br />

Palpation <strong>of</strong> the abdomen for areas <strong>of</strong> tenderness, signs <strong>of</strong> peritoneal inflammation,<br />

hepatomegaly, splenomegaly, abnormal masses, pulsations, or hernias. (PC)<br />

Performance <strong>of</strong> rectal and pelvic exams (under supervision). (PC)<br />

3. Differential diagnosis: Students should be able to generate a prioritized differential<br />

diagnosis <strong>of</strong> the most important and likely causes <strong>of</strong> a patient’s abdominal pain and<br />

recognize specific history, physical exam, and laboratory findings that distinguish<br />

between the following diagnoses or conditions:<br />

Appendicitis. (PC, MK)<br />

Cholecystitis (biliary colic). (PC, MK)<br />

Pancreatitis. (PC, MK)<br />

Diverticulitis. (PC, MK)<br />

Peptic ulcer disease including perforation. (PC, MK)<br />

Gastroenteritis. (PC, MK)<br />

Hepatitis. (PC, MK)<br />

Irritable bowel syndrome. (PC, MK)<br />

Small bowel obstruction. (PC, MK)<br />

Acute mesenteric ischemia. (MK, PC)<br />

Inflammatory bowel disease. (PC, MK)<br />

Ruptured abdominal aortic aneurysm. (PC, MK)<br />

Ureteral stones (renal colic). (PC, MK)<br />

Pelvic inflammatory disease. (PC, MK)<br />

69


Ruptured ectopic pregnancy. (PC, MK)<br />

Abdominal wall pain. (PC, MK)<br />

Referred pain. (PC, MK)<br />

4. Laboratory interpretation: Students should be able to interpret specific diagnostic tests<br />

and procedures that are commonly ordered to evaluate patients who present with<br />

abdominal pain. Test interpretation should take into account:<br />

Important differential diagnostic considerations including potential diagnostic<br />

emergencies. (PC, MK)<br />

Pre-test and post-test likelihood <strong>of</strong> disease (probabilistic reasoning). (PC, MK)<br />

Performance characteristics <strong>of</strong> individual tests (sensitivity, specificity, positive and<br />

negative predictive value, likelihood ratios). (PC, MK)<br />

Laboratory and diagnostic tests should include, when appropriate:<br />

• CBC with differential. (PC, MK)<br />

• UA. (PC, MK)<br />

• Pregnancy test. (PC, MK)<br />

• Stool for occult blood. (PC, MK)<br />

• Hepatic function panel. (PC, MK)<br />

• Amylase and lipase. (PC, MK)<br />

• Abdominal obstructive series. (PC, MK)<br />

Students should be able to define the indications for, and interpret (with<br />

consultation) the results <strong>of</strong>:<br />

Abdominal ultrasound. (PC, MK)<br />

Abdominal CT scan. (PC, MK)<br />

Paracentesis fluid studies. (PC, MK)<br />

Upper gastrointestinal endoscopy. (PC, MK)<br />

Sigmoidoscopy/colonoscopy. (PC, MK)<br />

Barium contrast studies. (PC, MK)<br />

Radionuclide scan <strong>of</strong> the hepatobiliary system. (PC, MK)<br />

5. Communication skills: Students should be able to:<br />

Communicate the diagnosis, treatment plan, and subsequent follow-up to the patient<br />

and his or her family. (PC, CS)<br />

Elicit questions from the patient and his or her family about the management plan. (PC,<br />

CS)<br />

Communicate in lay terms the indications, risk/benefits, and expected outcomes<br />

essential to obtaining informed consent for diagnostic and therapeutic procedures<br />

commonly used to evaluate and treat patients who present with abdominal pain. (PC,<br />

CS)<br />

6. Basic and advanced procedural skills: Students should be able to:<br />

Insert a nasogastric tube. (PC)<br />

Perform stool occult blood testing. (PC)<br />

Assist in performing a paracentesis after explaining the procedure to the patient. (PC,<br />

CS)<br />

7. Management skills: Students should be able to develop an appropriate<br />

evaluation and treatment plan for patients that includes:<br />

Recognizing the role <strong>of</strong> narcotic analgesics and empiric antibiotics in treating selected<br />

patients who present with acute abdominal pain. (PC, MK)<br />

Determining when to consult a gastroenterologist or a surgeon. (PC, SBP)<br />

Involving a surgeon as soon as possible when a patient is identified as having an acute<br />

abdomen. (PC, SBP)<br />

70


Selecting various tests and procedures commonly used to diagnose patients who<br />

present with symptoms <strong>of</strong> abdominal pain. ( PC, MK)<br />

Recommending basic initial management plans for the various causes <strong>of</strong> abdominal<br />

pain listed in the differential diagnosis. (PC, MK)<br />

Considering the potential value <strong>of</strong> addressing psychosocial issues in the management<br />

<strong>of</strong> chronic abdominal pain. (PC, MK)<br />

Accessing and utilizing appropriate information systems and resources to help<br />

delineate issues related to abdominal pain. (PC, PLI)<br />

Using a cost-effective approach based on the differential diagnosis. (PC, SBP)<br />

Incorporating patient preferences. (PC, P)<br />

C. ATTITUDES AND PROFESSIONAL BEHAVIORS: Students should be able to:<br />

1. Demonstrate commitment to using risk-benefit, cost-benefit, and evidence-based<br />

considerations in the selection <strong>of</strong> diagnostic and therapeutic interventions for abdominal<br />

pain. (PLI, P)<br />

2. Recognize the importance <strong>of</strong> patient needs and preferences when selecting among<br />

diagnostic and therapeutic options for abdominal pain. (P)<br />

3. Recognize the importance and demonstrate a commitment to the utilization <strong>of</strong> other<br />

healthcare pr<strong>of</strong>essions in the treatment <strong>of</strong> abdominal pain. (P, SBP)<br />

D. REFERENCES:<br />

Silen W. Cope’s Early Diagnosis <strong>of</strong> the Acute Abdomen. 20 th ed. New York: Oxford<br />

<strong>University</strong> Press; 2000.<br />

Wagner JM, McKinney WP, Carpenter JL. The rational clinical exam. Does this patient<br />

have appendicitis? JAMA. 1996;276:1589-94.<br />

Lederle F, Simel D. The rational clinical exam. Does this patient have an abdominal<br />

aortic aneurysm? JAMA. 1999;281:77-82.<br />

Trowbridge RL, Rutkowski NK, Shojania KG. The rational clinical exam. Does this<br />

patient have acute cholecystitis? JAMA. 2003;289: 80-86.<br />

71


TRAINING PROBLEM #3: ALTERED MENTAL STATUS<br />

RATIONALE:<br />

The diagnosis and management <strong>of</strong> altered mental status requires a working knowledge <strong>of</strong> all areas <strong>of</strong><br />

internal medicine, so varied are the etiologies and corresponding treatment strategies. Internists must<br />

master an approach to the problem as they are <strong>of</strong>ten the first physicians to see such patients.<br />

PREREQUISITES:<br />

Prior knowledge, skills, and attitudes acquired during the pre-clinical experience should include:<br />

Ability to perform a complete medical history and physical.<br />

Ability to communicate with patients <strong>of</strong> diverse backgrounds.<br />

Basic course work in physiology, pathophysiology, and neuroanatomy.<br />

SPECIFIC LEARNING OBJECTIVES:<br />

A. KNOWLEDGE: Students should be able to define, describe, and discuss:<br />

1. The differentiation <strong>of</strong> delirium, dementia, and depression. (MK)<br />

2. The pathophysiology, symptoms, and signs <strong>of</strong> the most common and most serious causes <strong>of</strong><br />

altered mental status, including:<br />

Metabolic causes (e.g. hyper/hyponatremia, hyper/hypoglycemia, hypercalcemia,<br />

hyper/hypothyroidism, hypoxia/hypercapnea, B12 deficiency, hepatic encephalopathy,<br />

uremic encephalopathy, drug/alcohol intoxication/withdrawal, and Wernicke’s<br />

encephalopathy). (MK)<br />

Structural lesions (e.g. primary or metastatic tumor, intracranial hemorrhage, subdural<br />

hematoma). (MK)<br />

Vascular (e.g. cerebrovascular accident, transient ischemic attack, cerebral vasculitis).<br />

(MK)<br />

Infectious etiologies (e.g. encephalitis, meningitis, urosepsis, endocarditis, pneumonia,<br />

cellulites). (MK)<br />

Seizures/ post-ictal state. (MK)<br />

Hypertensive encephalopathy. (MK)<br />

Low perfusion states (e.g. arrhythmias, MI, shock, acute blood loss, severe dehydration).<br />

(MK)<br />

Miscellaneous causes (e.g. fecal impaction, postoperative state, sleep deprivation, urinary<br />

retention). (MK)<br />

3. The importance <strong>of</strong> thoroughly reviewing prescription medications over-thecounter drugs, and<br />

supplements and inquiring about substance abuse. (MK)<br />

4. The risk factors for developing altered mental status, including:<br />

Dementia. (MK)<br />

Advanced age. (MK)<br />

Substance abuse. (MK)<br />

Comorbid physical problems such as sleep deprivation, immobility, dehydration, pain, and<br />

sensory impairment. (MK)<br />

ICU admission. (MK)<br />

5. The diagnostic evaluation <strong>of</strong> altered mental status. (MK)<br />

6. Indications, contraindications, and complications <strong>of</strong> lumbar puncture. (MK)<br />

72


7. Principles <strong>of</strong> management <strong>of</strong> the common causes <strong>of</strong> altered mental status. (MK)<br />

8. Nonpharmacologic measures to reduce agitation and aggression, including:<br />

Avoiding the use <strong>of</strong> physical restraints whenever possible. (MK)<br />

Using reorientation techniques. (MK)<br />

Assuring the patient has their devices to correct sensory deficits. (MK)<br />

Promoting normal sleep and day/night awareness. (MK)<br />

Preventing dehydration and electrolyte disturbances. (MK)<br />

Avoiding medications which may worsen delirium whenever possible<br />

(e.g. anticholinergics, benzodiazepines, etc.). (MK)<br />

9. The risks <strong>of</strong> using physical restraints. (MK)<br />

10. The risk and benefits <strong>of</strong> using low-dose high potency antipsychotics for delirium associated<br />

agitation and aggression. (MK)<br />

B. SKILLS: Students should be able to demonstrate specific skills, including:<br />

1 History-taking skills: Students should be able to obtain, document, and present an ageappropriate<br />

medical history that differentiates among etiologies <strong>of</strong> altered mental status including<br />

eliciting appropriate information from patients and their families regarding the onset, progression,<br />

associated symptoms, and level <strong>of</strong> physical and mental disability. (PC, CS)<br />

2. Physical exam skills: Students should be able to perform a physical exam to establish the<br />

diagnosis and severity <strong>of</strong> disease, including:<br />

Complete neurologic examination. (PC)<br />

Mental status examination. (PC)<br />

Fundoscopic examination. (PC)<br />

3. Differential diagnosis: Students should be able to generate a prioritized differential diagnosis<br />

recognizing specific history and physical exam findings that suggest a specific etiology for<br />

altered mental status. (PC, MK)<br />

4. Laboratory interpretation: Students should be able to recommend when to order diagnostic<br />

and laboratory tests and be able to interpret them, both prior to and after initiating treatment<br />

based on the differential diagnosis, including consideration <strong>of</strong> test cost and performance<br />

characteristics as well as patient preferences. Laboratory and diagnostic tests should include,<br />

when appropriate:<br />

CBC with differential. (PC, MK)<br />

Electrolytes, BUN/Cr, GLC, hepatic function panel, Ca. (PC, MK)<br />

ABG. (PC, MK)<br />

Toxicology screen. (PC, MK)<br />

VDRL. (PC, MK)<br />

Vitamin B12 and thiamine measurements. (PC, MK)<br />

Thyroid function tests. (PC, MK)<br />

Urinalysis and urine culture. (PC, MK)<br />

Blood cultures. (PC, MK)<br />

Cerebrospinal fluid analysis (color, opening pressure, chemistries, cell<br />

counts, staining, cultures, cytology, cryptococcal antigen, VDRL). (PC, MK)<br />

Students should be able to define the indications for and interpret (with consultation)<br />

the results <strong>of</strong>:<br />

Cranial CT. (PC, MK)<br />

Cranial MRI. (PC, MK)<br />

Electroencephalogram. (PC, MK)<br />

73


5. Communication skills: Students should be able to:<br />

Communicate the diagnosis, treatment plan, and subsequent follow-up to the patient and<br />

his or her family. (PC, CS)<br />

Elicit questions from the patient and his or her family about the management plan. (PC, CS)<br />

When the patient is unable to communicate, obtain a history from a collateral source such<br />

as a family member or other health care proxy. (PC, CS)<br />

6. Basic and advanced procedural skills: Students should be able to:<br />

Obtain an ABG. (PC)<br />

Assist in performing a lumbar puncture after explaining the procedure to the patient. (PC,<br />

CS)<br />

7. Management skills: Students should able to develop an appropriate evaluation and<br />

treatment plan for patients that includes:<br />

Recognizing that altered mental status in a older inpatient is a medical<br />

emergency and requires that the patient be evaluated immediately. (PC, MK)<br />

Writing appropriate fluid and replacement orders for patients with common electrolyte and<br />

metabolic disturbances. (PC, MK)<br />

Writing appropriate antibiotic orders for the treatment <strong>of</strong> common infectious etiologies. (PC,<br />

MK)<br />

Ordering appropriate nonpharmacologic and pharmacologic interventions for patients with<br />

acute altered mental status with accompanying agitation and aggression. (PC, MK)<br />

Determining when to obtain consultation from a neurologist or neurosurgeon. (PC, SBP)<br />

Utilizing hospital and community resources for patients with permanent or disabling<br />

conditions to help assist their transfer back to the community or rehabilitation facility. (PC,<br />

SBP)<br />

Using a cost-effective approach based on the differential diagnosis. (PC, SBP)<br />

Accessing and utilizing appropriate information systems and resources to help delineate<br />

issues related to altered mental status. (PC, PLI)<br />

Incorporating patient preferences. (PC, P)<br />

C. ATTITUDES AND PROFESSIONAL BEHAVIORS: Students should be able to:<br />

1. Appreciate the family’s concern and at times despair arising from a loved one’s<br />

development <strong>of</strong> altered mental status. (CS, P)<br />

2. Appreciate the patient’s distress and emotional response to that may accompany<br />

circumstances <strong>of</strong> altered mental status. (CS,P)<br />

3. Demonstrate commitment to using risk-benefit, cost-benefit, and evidence-based<br />

considerations in the selection diagnostic and therapeutic interventions for altered mental<br />

status. (PLI, P)<br />

4. Recognize the importance <strong>of</strong> patient preferences when selecting among diagnostic<br />

and therapeutic options for altered mental status. (P)<br />

5. Demonstrate ongoing commitment to self-directed learning regarding altered mental<br />

status. (PLI, P)<br />

6. Appreciate the impact altered mental status has on a patient’s quality <strong>of</strong> life, wellbeing,<br />

ability to work, and the family. (P)<br />

7. Recognize the importance <strong>of</strong> and demonstrate a commitment to the utilization <strong>of</strong><br />

other healthcare pr<strong>of</strong>essionals in the diagnosis and treatment <strong>of</strong> altered mental status. (P,<br />

SBP)<br />

D. REFERENCES:<br />

74


Ropper AH. (2005). Acute confusional states and coma. In Kasper DL, Braunwald EB,<br />

Fauci AS, Hauser SL, Longo DL, Jameson JL, eds. Harrison’s Principles <strong>of</strong> Internal <strong>Medicine</strong>.<br />

16 th ed. New York, NY: McGraw-Hill; 2005:1624-31.<br />

Gleason OC. Delirium. Am Fam Physician. 2003;67:1027-34.<br />

Brown TM, Boyle MF. Delirium. BMJ. 2002;325:644-7.<br />

Meagher DJ. Delirium: optimizing management. BMJ. 2001;322:144-9.<br />

75


TRAINING PROBLEM #4: ANEMIA<br />

RATIONALE:<br />

Anemia is a common finding, <strong>of</strong>ten identified incidentally in asymptomatic patients. It can be a<br />

manifestation <strong>of</strong> a serious underlying disease. Distinguishing among the many disorders that cause<br />

anemia, not all <strong>of</strong> which require treatment, is an important training problem for third year medical<br />

students.<br />

PREREQUISITES:<br />

Prior knowledge, skills and attitudes acquired during the pre-<strong>clerkship</strong> experience should<br />

include:<br />

Ability to perform a complete medical history and physical exam.<br />

Ability to communicate with patients <strong>of</strong> diverse backgrounds.<br />

Knowledge <strong>of</strong> pathogenesis and pathophysiology <strong>of</strong> anemia.<br />

Knowledge <strong>of</strong> the basic biochemistry and pathophysiology <strong>of</strong> the blood and bone marrow.<br />

Knowledge <strong>of</strong> the pharmacology <strong>of</strong> medications that can cause anemia as well as those used<br />

to treat it.<br />

SPECIFIC LEARNING OBJECTIVES:<br />

A. KNOWLEDGE: Students should be able to define, describe, and discuss:<br />

1. Classification <strong>of</strong> anemia based on red cell size:<br />

Microcytic:<br />

o Iron deficiency. (MK)<br />

o Thalassemic disorders. (MK)<br />

o Sideroblastic anemia. (MK)<br />

Normocytic:<br />

o Acute blood loss. (MK)<br />

o Hemolysis. (MK)<br />

o Anemia <strong>of</strong> chronic disease (e.g. infection, inflammation, malignancy). (MK)<br />

o Chronic renal insufficiency/erythropoietin deficiency. (MK)<br />

o Bone marrow suppression (e.g. bone marrow invasion, aplastic anemia).<br />

o Hypothyroidism. (MK)<br />

o Testosterone deficiency. (MK)<br />

o Early presentation <strong>of</strong> microcytic or macrocytic anemia (e.g. early iron deficiency<br />

anemia). (MK)<br />

o Combined presentation <strong>of</strong> microcytic and macrocytic anemias. (MK)<br />

Macrocytic:<br />

o Ethanol abuse. (MK)<br />

o B12 deficiency. (MK)<br />

o Folate deficiency. (MK)<br />

o Drug-induced. (MK)<br />

o Reticulcytosis. (MK)<br />

o Liver disease. (MK)<br />

o Myelodysplastic syndromes. (MK)<br />

o Hypothyroidism. (MK)<br />

10. Morphological characteristics, pathophysiology, and relative prevalence <strong>of</strong> each <strong>of</strong> the causes<br />

<strong>of</strong> anemia. (MK)<br />

76


11. The meaning and utility <strong>of</strong> various components <strong>of</strong> the hemogram (e.g. hemoglobin, hematocrit,<br />

mean corpuscular volume, and random distribution width). (MK)<br />

12. The classification <strong>of</strong> anemia into hypoproliferative and hyperproliferative categories and the<br />

utility <strong>of</strong> the reticulocyte count/index. (MK)<br />

13. The potential usefulness <strong>of</strong> the white blood cell count and red blood cell count when attempting<br />

to determine the cause <strong>of</strong> anemia. (MK)<br />

14. The diagnostic utility <strong>of</strong> the various tests for iron deficiency (e.g. serum iron, total iron binding<br />

capacity, transferrin saturation, ferritin). (MK)<br />

15. The genetic basis <strong>of</strong> some forms <strong>of</strong> anemia. (MK)<br />

16. Indications, contraindications, and complications <strong>of</strong> blood transfusion. (MK)<br />

B. SKILLS: Students should be able to demonstrate specific skills, including:<br />

1. History-taking skills: Students should be able to obtain, document, and present an ageappropriate<br />

medical history, that differentiates among etiologies <strong>of</strong> disease, including:<br />

Constitutional and systemic symptoms (e.g. fatigue, weight loss). (PC, CS)<br />

History <strong>of</strong> gastrointestinal bleeding or risk factors for it. (PC, CS)<br />

Abdominal pain. (PC, CS)<br />

Prior history <strong>of</strong> anemia or other blood diseases. (PC, CS)<br />

Medications. (PC, CS)<br />

Diet. (PC, CS)<br />

Alcohol use. (PC, CS)<br />

Menstrual history. (PC, CS)<br />

Family history <strong>of</strong> anemia or other blood diseases. (PC, CS)<br />

2. Physical exam skills: Students should be able to perform a physical exam to establish the<br />

diagnosis and severity <strong>of</strong> disease, including:<br />

Pallor (e.g. palms, conjunctiva, nail beds). (PC)<br />

Mouth (e.g. glossitis, cheilosis). (PC)<br />

Hyperdynamic precordium, systolic flow murmur. (PC)<br />

Lymph nodes. (PC)<br />

spleen. (PC)<br />

Obtain stool for occult blood testing. (PC)<br />

Nervous system. (PC)<br />

3. Differential diagnosis: Students should be able to generate a list <strong>of</strong> the most important and<br />

most common causes <strong>of</strong> anemia, recognizing specific history, physical exam, and laboratory<br />

findings that suggest a specific etiology. (PC, MK)<br />

4. Laboratory interpretation: Students should be able to recommend when to order diagnostic<br />

and laboratory tests and be able to interpret them, both prior to and after initiating treatment<br />

based on the differential diagnosis including consideration <strong>of</strong> test cost and performance<br />

characteristics as well as patient preferences. Laboratory and diagnostic tests should include,<br />

when appropriate:<br />

Hemoglobin and hematocrit. (PC, MK)<br />

Red cell indices (e.g. mean corpuscular volume and random distribution width). (PC, MK)<br />

White blood cell and platelet count. (PC, MK)<br />

Reticulocyte count. (PC, MK)<br />

Iron studies (serum iron, TIBC, ferritin, transferrin). (PC, MK)<br />

Serum B12 and folate. (PC, MK)<br />

Haptoglobin. (PC, MK)<br />

77


Lactic dehydrogenase. (LDH) (PC, MK)<br />

Hemoglobin electrophoresis. (PC, MK)<br />

Blood smear. (PC, MK)<br />

Students should be able to define the indications for and interpret (with consultation) results<br />

<strong>of</strong>:<br />

Bone marrow biopsy. (PC, MK)<br />

5. Communication skills: Students should be able to:<br />

Communicate the diagnosis, treatment plan, and subsequent follow-up to patients. (PC,<br />

CS)<br />

Elicit questions from the patient about the management plan. (PC, CS)<br />

Counsel with regard to (a) possible causes, (b) appropriate further evaluation to establish<br />

the diagnosis <strong>of</strong> an underlying disease, and (c) the impact on the family (genetic<br />

counseling). (PC, CS)<br />

6. Basic procedural skills: Students should be able to perform and interpret:<br />

Stool occult blood testing. (PC)<br />

7. Management skills: Students should be able to develop an appropriate evaluation and<br />

treatment plan for patients that includes:<br />

Evaluating for underlying disease processes, given that anemia is not a disease per se, but<br />

rather a common finding that requires further delineation in order to identify the underlying<br />

cause. (PC, MK)<br />

Prescribing indicated replacement therapy, including iron, vitamin B12, and folic acid. (PC,<br />

MK)<br />

Determining when to obtain consultation from a hematologist. (PC, SBP)<br />

Using a cost-effective approach based on the differential diagnosis. (PC, SBP)<br />

Accessing and utilizing appropriate information systems and resources to help delineate<br />

issues related to anemia. (PC, PLI)<br />

Incorporating patient preferences. (PC, P)<br />

C. ATTITUDES AND PROFESSIONAL BEHAVIORS: Students should be able to:<br />

1 Demonstrate commitment to using risk-benefit, cost-benefit, and evidence-based<br />

considerations in the selection <strong>of</strong> diagnostic and therapeutic interventions for anemia. (PLI, P)<br />

2 Respond appropriately to patients who are non-adherent to treatment for anemia. (CS, P)<br />

3 Demonstrate ongoing commitment to self-directed learning regarding anemia. (PLI, P)<br />

Appreciate the impact anemia has on a patient’s quality <strong>of</strong> life, well-being, ability to work, and<br />

the family. (P)<br />

4 Recognize the importance <strong>of</strong> and demonstrate a commitment to the utilization <strong>of</strong> other<br />

healthcare pr<strong>of</strong>essions in the treatment <strong>of</strong> anemia. (P, SBP)<br />

D. REFERENCES:<br />

Sheth TN. Choudhry NK. Bowes M. Detsky AS. The relation <strong>of</strong> conjunctival pallor to the<br />

presence <strong>of</strong> anemia. J Gen Intern Med. 1997;12:102-6.<br />

Guyatt, G H. Oxman, A D. Ali, M. Willan, A. McIlroy, W. Patterson, C. Laboratory<br />

diagnosis <strong>of</strong> iron-deficiency anemia: an overview. J Gen Intern Med. 1992;7:145-53.<br />

Kis AM. Carnes M. Detecting iron deficiency in anemic patients with concomitant<br />

medical problems. J Gen Intern Med. 1998; 13:455-61.<br />

Bain BJ. Diagnosis from the blood smear. N Engl Journal Med. 2005;353:498-507.<br />

Weiss G. Goodnough LT. Anemia <strong>of</strong> chronic disease. N Engl J Med. 2005;352:1011-23.<br />

78


H<strong>of</strong>fbrand V. Provan D. ABC <strong>of</strong> clinical haematology: macrocytic anaemias. BMJ.<br />

1997;314:430-3.<br />

79


TRAINING PROBLEM #5: BACK PAIN<br />

RATIONALE:<br />

Back pain is one <strong>of</strong> the most commonly encountered problems in the outpatient, primary care internal<br />

medicine setting. It has an important differential diagnosis, and the initial decision-making must be<br />

made on the basis <strong>of</strong> clinical findings. As such, it is an excellent training condition for teaching<br />

decision-making based on careful collection and interpretation <strong>of</strong> basic clinical data. There is<br />

emerging data on test utility, especially as regards expensive spinal imaging, which facilitates<br />

teaching rational, cost-effective test ordering. Moreover, its requirement for skillful management,<br />

patient education, and support facilitate the teaching <strong>of</strong> these competencies.<br />

PREREQUISITES:<br />

Prior knowledge, skills, and attitudes acquired during the pre-clinical experience should include:<br />

Ability to perform a complete medical history and physical.<br />

Ability to communicate with patients <strong>of</strong> diverse backgrounds.<br />

Anatomy and physiology <strong>of</strong> bony, s<strong>of</strong>t tissue, vascular, and <strong>of</strong> the spine.<br />

Pathogenesis and pathophysiology <strong>of</strong> muscular strain, osteoarthritis, spinal stenosis,<br />

osteoporosis, disc degeneration, and spinal metastases.<br />

Pharmacology <strong>of</strong> non-narcotic and narcotic analgesics, nonsteroidal anti-inflammatory drugs,<br />

muscle “relaxants.”<br />

SPECIFIC LEARNING OBJECTIVES:<br />

A. KNOWLEDGE: Students should be able to define, describe, and discuss:<br />

1. The symptoms, signs, and typical clinical course <strong>of</strong> the various causes <strong>of</strong> back pain including:<br />

Ligamentous/muscle strain (nonspecific musculoskeletal back pain). (MK)<br />

Degenerative arthritis (spondylosis). (MK)<br />

Disc herniation. (MK)<br />

Spinal stenosis. (MK)<br />

Vertebral compression fracture. (MK)<br />

Traumatic fracture. (MK)<br />

Sacroileitis. (MK)<br />

Spinal metastases. (MK)<br />

Spinal epidural abscess. (MK)<br />

Cauda equina syndrome. (MK)<br />

2. The role <strong>of</strong> diagnostic studies in the evaluation <strong>of</strong> the back pain there indications,<br />

limitations, cost:<br />

Plain radiography. (MK)<br />

CT. (MK)<br />

MRI. (MK)<br />

Myelogram. (MK)<br />

Electrodiagnosis (i.e. electromyography and nerve conduction studies). (MK)<br />

Bone densitometry. (MK)<br />

3. Response to therapy <strong>of</strong> the various etiologies, with understanding <strong>of</strong> the roles <strong>of</strong>:<br />

80


Bed rest. (MK)<br />

Exercise. (MK)<br />

Analgesia. (MK)<br />

NSAIDs. (MK)<br />

Heat/ice. (MK)<br />

Ultrasound. (MK)<br />

Spinal manipulation. (MK)<br />

Surgical interventions. (MK)<br />

4. Risk factor for and means <strong>of</strong> limiting disability and chronicity. (MK)<br />

5. Fear avoidance behaviors. (MK)<br />

6. Pain related behaviors with regard to chronic narcotic use. (MK)<br />

B. SKILLS: Students should be able to demonstrate specific skills including:<br />

1. History-taking skills: Students should be able to obtain, document, and present an<br />

age-appropriate medical history, that differentiates among etiologies <strong>of</strong> disease,<br />

including:<br />

Cancer history. (PC, CS)<br />

Weight loss. (PC, CS)<br />

Fever. (PC, CS)<br />

Recent infection. (PC, CS)<br />

Intravenous drug use. (PC, CS)<br />

Steroid use. (PC, CS)<br />

Trauma. (PC, CS)<br />

Rapidly progressive focal numbness and/or weakness. (PC, CS)<br />

Bowel/bladder dysfunction. (PC, CS)<br />

Saddle anesthesia. (PC, CS)<br />

Symptoms <strong>of</strong> systemic rheumatologic conditions. (PC, CS)<br />

Anatomic abnormalities (e.g. kyphosis, scoliosis). (PC, CS)<br />

2. Physical exam skills: Students should be able to perform a physical exam to establish the<br />

diagnosis and severity <strong>of</strong> disease, including:<br />

Examination <strong>of</strong> the spine. (PC)<br />

Neurologic examination <strong>of</strong> the lower extremities. (PC)<br />

Straight leg raising test. (PC)<br />

Testing for saddle anesthesia. (PC)<br />

Assessment <strong>of</strong> rectal tone. (PC)<br />

3. Differential diagnosis: Students should be able to generate a prioritized differential diagnosis<br />

recognizing specific history and physical exam findings that suggest a specific etiology for back<br />

pain (PC, MK)<br />

4. Laboratory interpretation: Students should be able to recommend when to order diagnostic<br />

and laboratory tests and be able to interpret them, both prior to and after initiating treatment<br />

based on the differential diagnosis, including consideration <strong>of</strong> test cost and performance<br />

characteristics as well as patient preferences.<br />

Laboratory and diagnostic tests should include, when appropriate:<br />

ESR. (PC, MK)<br />

CBC. (PC, MK)<br />

Serum Alk Phos. (PC, MK)<br />

Students should be able to define the indications for and interpret (with consultation) the<br />

81


esults <strong>of</strong>:<br />

Plain spinal radiography. (PC, MK)<br />

Spinal CT. (PC, MK)<br />

Spinal MRI. (PC, MK)<br />

Radionuclide bone scan. (PC, MK)<br />

Bone densitometry. (PC, MK)<br />

Electrodiagnostic tests. (PC, MK)<br />

5. Communication skills: Students should be able to:<br />

Communicate the diagnosis, treatment plan, and subsequent follow-up to patients. (PC,<br />

CS)<br />

Explain the importance <strong>of</strong> active participation in the treatment plan. (PC, CS)<br />

Elicit questions from the patient and their family about the management plan.<br />

(PC, CS)<br />

6. Management skills: Students should able to develop an appropriate evaluation and<br />

treatment plan for patients that includes:<br />

Patient education about the typical course <strong>of</strong> back pain. (PC, MK)<br />

Methods to prevent the development <strong>of</strong> chronic back pain. (PC, MK)<br />

Proper use <strong>of</strong> analgesics, NSAIDs, muscle relaxants, and local heat/ice. (PC, MK)<br />

Teaching back hygiene measures, exercises, and proper lifting and standing ergonomics.<br />

(PC, MK)<br />

Counseling patients about lifestyle modifications including weight loss. (PC, MK)<br />

The potential role <strong>of</strong> chiropractic, acupuncture, and massage (PC, MK)<br />

Determining when to obtain consultation from an appropriate back pain specialist. (PC,<br />

SBP)<br />

Using a cost-effective approach based on the differential diagnosis. (PC, SBP)<br />

Accessing and utilizing appropriate information systems and resources to help delineate<br />

issues related to back pain. (PC, PLI)<br />

Incorporating patient preferences. (PC, P)<br />

C. ATTITUDES AND PROFESSIONAL BEHAVIORS: Students should be able to:<br />

1 Demonstrate commitment to using risk-benefit, cost-benefit, and evidence-based<br />

considerations in the selection diagnostic and therapeutic interventions for back pain. (PLI, P)<br />

2 Recognize the importance <strong>of</strong> patient preferences when selecting among diagnostic and<br />

therapeutic options for back pain. (P)<br />

3 Appreciate the importance <strong>of</strong> active patient involvement in the treatment <strong>of</strong> back pain. (P)<br />

4 respond appropriately to patients who are nonadherent to treatment for back pain. (CS, P)<br />

5 respond appropriately to patients with chronic back pain (P)<br />

6 Demonstrate ongoing commitment to self-directed learning regarding back pain. (PLI, P)<br />

7 Appreciate the impact back pain has on a patient’s quality <strong>of</strong> life, well-being, ability to work,<br />

and the family. (P)<br />

8 Recognize the importance <strong>of</strong> and demonstrate a commitment to the utilization <strong>of</strong> other<br />

healthcare pr<strong>of</strong>essionals in the treatment <strong>of</strong> back pain. (P, SBP)<br />

D. REFERENCES:<br />

Carragee EJ. Persistent low back pain. N Engl J Med. 2005;352:1891-8.<br />

Deyo RA, Weinstein JN. Low back pain. N Engl J Med. 2001;344:363-70.<br />

Atlas SJ, Deyo RA. Evaluating and managing acute low back pain in the primary care<br />

82


setting. J Gen Intern Med. 2001;16:120-31.<br />

Deyo RA. Diagnostic evaluation <strong>of</strong> LBP: reaching a specific diagnosis is <strong>of</strong>ten<br />

impossible. Arch Intern Med. 2002;162:1444-7; discussion 1447-8.<br />

83


TRAINING PROBLEM #6: CHEST PAIN<br />

RATIONALE:<br />

Chest pain is a common and important presenting symptom for a variety <strong>of</strong> disorders, some <strong>of</strong> which<br />

may be life-threatening emergencies. The ability to distinguish chest pain caused by an acute<br />

coronary syndrome (unstable angina or acute myocardial infarction) from other cardiac,<br />

gastrointestinal, pulmonary, musculoskeletal or psychogenic etiologies is an important training<br />

problem for third-year medical students.<br />

PREREQUISITES:<br />

Prior knowledge, skills and attitudes acquired during the pre-<strong>clerkship</strong> experience should include:<br />

Ability to perform a complete medical history and physical exam.<br />

Ability to communicate appropriately with patients <strong>of</strong> diverse backgrounds, including the elderly<br />

patient.<br />

Knowledge <strong>of</strong> the anatomy <strong>of</strong> the heart, chest and abdomen.<br />

Understanding <strong>of</strong> the epidemiology <strong>of</strong> heart disease.<br />

Knowledge <strong>of</strong> the pathogenesis and pathophysiology <strong>of</strong> cardiovascular disease.<br />

Knowledge <strong>of</strong> the pharmacology <strong>of</strong> cardiovascular drugs.<br />

Ability to perform a cardiovascular risk assessment and understand issues related to primary<br />

and secondary prevention <strong>of</strong> cardiovascular disease.<br />

Ability to understand the impact <strong>of</strong> illness on individuals and their families.<br />

SPECIFIC LEARNING OBJECTIVES:<br />

A. KNOWLEDGE: Students should be able to define, describe and discuss:<br />

1 Symptoms and signs <strong>of</strong> chest pain that may be due to an acute coronary syndrome such as<br />

unstable angina or acute myocardial infarction. (MK)<br />

2 Symptoms and signs <strong>of</strong> chest pain that are characteristic <strong>of</strong> angina pectoris. (MK)<br />

3 Symptoms and signs <strong>of</strong> chest pain due to other cardiac causes such as:<br />

Atypical or variant angina (coronary vasospasm, Prinzmetal angina). (MK)<br />

Cocaine-induced chest pain. (MK)<br />

Pericarditis. (MK)<br />

Aortic dissection. (MK)<br />

Valvular heart disease (aortic stenosis, mitral valve prolapse). (MK)<br />

Non-ischemic cardiomyopathy. (MK)<br />

Syndrome X. (MK)<br />

4. Symptoms and signs <strong>of</strong> chest pain due to gastrointestinal disorders such as:<br />

Esophageal disease (GERD, esophagitis, esophageal dysmotility). (MK)<br />

Biliary disease (cholecystitis, cholangitis). (MK)<br />

Peptic ulcer disease. (MK)<br />

Pancreatitis. (MK)<br />

5. Symptoms and signs <strong>of</strong> chest pain due to pulmonary disorders such as:<br />

Pneumonia. (MK)<br />

Spontaneous pneumothorax. (MK)<br />

Pleurisy. (MK)<br />

Pulmonary embolism. (MK)<br />

84


Pulmonary hypertension/cor pulmonale. (MK)<br />

6. Symptoms and signs <strong>of</strong> chest pain due to musculoskeletal causes such as:<br />

Costochondritis. (MK)<br />

Rib fracture. (MK)<br />

My<strong>of</strong>ascial pain syndromes. (MK)<br />

Muscular strain. (MK)<br />

Herpes zoster. (MK)<br />

7. Symptoms and signs <strong>of</strong> chest pain due to psychogenic causes such as:<br />

Panic disorders. (MK)<br />

Hyperventilation. (MK)<br />

Somat<strong>of</strong>orm disorders. (MK)<br />

8. Factors that may be responsible for provoking or exacerbating symptoms <strong>of</strong> ischemic chest<br />

pain by:<br />

Increasing myocardial oxygen demand.<br />

o Tachycardia or tachyarrhythmia. (MK)<br />

o Hypertension. (MK)<br />

o Increased wall stress (aortic stenosis, cardiomyopathy). (MK)<br />

o Hyperthyroidism. (MK)<br />

Decreasing myocardial oxygen supply.<br />

o Anemia. (MK)<br />

o Hypoxemia. (MK)<br />

9. Risk factors for the development <strong>of</strong> coronary heart disease:<br />

Age and gender. (MK)<br />

Family history <strong>of</strong> sudden death or premature CAD. (MK)<br />

Personal history <strong>of</strong> peripheral vascular or cerebrovascular disease. (MK)<br />

Smoking. (MK)<br />

Lipid abnormalities (includes dietary history <strong>of</strong> saturated fat and cholesterol). (MK)<br />

Diabetes mellitus. (MK)<br />

Hypertension. (MK)<br />

Obesity. (MK)<br />

Sedentary lifestyle. (MK)<br />

Cocaine use. (MK)<br />

Estrogen use. (MK)<br />

Chronic inflammation. (MK)<br />

10. Physiologic basis and/or scientific evidence supporting each type <strong>of</strong> treatment, intervention or<br />

procedure commonly used in the management <strong>of</strong> patients who present with chest pain. (MK)<br />

11. Role <strong>of</strong> a critical pathway or practice guideline in delivering high quality, cost-effective care for<br />

patients presenting with symptoms <strong>of</strong> chest pain in the outpatient clinic, emergency room or<br />

hospital. (MK, PC, SBP)<br />

B. SKILLS: Students should be able to demonstrate specific skills, including:<br />

1. History-taking skills: Students should be able to obtain, document, and present an<br />

appropriately complete medical history that differentiates among the common etiologies <strong>of</strong><br />

chest pain.<br />

85


The initial medical history should allow students to categorize the patients’ symptoms as<br />

angina pectoris, atypical angina or non-cardiac chest pain. (PC, CS)<br />

Specifically, the medical history <strong>of</strong> a patient with chest pain should contain information<br />

about those clinical characteristics that are typical <strong>of</strong> angina pectoris:<br />

o Substernal location. (PC, CS)<br />

o Precipitated by exertion. (PC, CS)<br />

o Relieved by rest or nitroglycerin. (PC, CS)<br />

o Onset, duration, severity, radiation, presence or absence <strong>of</strong> associated symptoms (such<br />

as dyspnea, diaphoresis or lightheadedness). (PC, CS)<br />

The history <strong>of</strong> a patient with chest pain should also contain information about:<br />

o Risk factors for coronary heart disease. (PC, CS)<br />

o Previous history <strong>of</strong> ischemic heart disease or valvular heart disease (rheumatic fever,<br />

cardiac murmurs). (PC, CS)<br />

o Previous history <strong>of</strong> peripheral vascular disease or cerebrovascular disease. (PC, CS)<br />

Students should be able to use the medical history to assess the<br />

functional status <strong>of</strong> patients who present with ischemic chest pain. (PC, CS)<br />

2. Physical exam skills: Students should be able to perform a focused physical exam that<br />

includes the following elements:<br />

Accurate measurement <strong>of</strong> arterial blood pressure and recognition <strong>of</strong> the typical blood<br />

pressure findings that occur in patients with aortic stenosis, aortic insufficiency, and pulsus<br />

paradoxus. (PC)<br />

Assessment <strong>of</strong> major arterial pulses for abnormalities, including bruits. (PC)<br />

Assessment <strong>of</strong> the neck veins for jugular venous distention and, when necessary,<br />

evaluation for abdominal jugular reflux. (PC)<br />

Assessment <strong>of</strong> the conjunctiva and optic fundus. (PC)<br />

Assessment <strong>of</strong> the extremities to ascertain skin condition, including color, temperature and<br />

the presence <strong>of</strong> edema, xanthomas, cyanosis and clubbing. (PC)<br />

Assessment <strong>of</strong> the lungs for crackles, rhonchi, rubs and decreased breath sounds. (PC)<br />

Inspection and palpation <strong>of</strong> the anterior chest to identify right and left sided heaves, lifts,<br />

and thrills. (PC)<br />

Auscultation <strong>of</strong> the heart to determine rhythm, intensity <strong>of</strong> heart sounds, splitting <strong>of</strong> S2 and<br />

the presence <strong>of</strong> rubs, gallops (S3, S4, summation) or extra heart sounds (e.g. clicks). (PC)<br />

Auscultation <strong>of</strong> the heart to detect the presence <strong>of</strong> heart murmurs. When a heart murmur is<br />

present, students should be able to:<br />

o Identify timing (systolic vs. diastolic, holosystolic vs. ejection). (PC)<br />

o Describe pitch, location and pattern <strong>of</strong> radiation. (PC)<br />

o Gauge significance (innocent vs. pathologic, sclerosis vs. stenosis). (PC)<br />

Assessment <strong>of</strong> the abdomen to determine the presence <strong>of</strong> epigastric or right<br />

upper quadrant tenderness, hepatomegaly, abnormal pulsations or bruits. (PC)<br />

3. Differential diagnosis: Students should be able to generate a prioritized differential diagnosis<br />

and recognize specific history, physical exam, and laboratory findings that suggest a diagnosis<br />

<strong>of</strong> myocardial ischemia rather than a non-ischemic cause <strong>of</strong> chest pain (GI, pulmonary,<br />

musculoskeletal, psychogenic or undetermined). (PC, MK)<br />

4. Laboratory interpretation: Students should be able to recommend when to order diagnostic<br />

and laboratory tests and be able to interpret them, both prior to and after initiating treatment<br />

based on the differential diagnosis, including consideration <strong>of</strong> test cost and performance<br />

characteristics as well as patient preferences. Test interpretation should take into account:<br />

Important differential diagnostic considerations including the “must not miss” diagnoses.<br />

(PC, MK)<br />

86


Pre-test and post-test likelihood <strong>of</strong> disease (probabilistic reasoning). (PC, MK)<br />

Performance characteristics <strong>of</strong> individual tests (sensitivity, specificity,<br />

positive and negative predictive value, likelihood ratios). (PC, MK)<br />

Laboratory and diagnostic tests should include, when appropriate:<br />

Cardiac biomarkers indicative <strong>of</strong> myocardial necrosis. (PC, MK)<br />

12-lead ECG. (PC, MK)<br />

Chest radiograph. (PC, MK)<br />

ABG. (PC, MK)<br />

Students should be able to define the indications for, and interpret (with consultation) the<br />

results <strong>of</strong> the following diagnostic tests and procedures:<br />

Echocardiogram (transthoracic and transesophageal). (PC, MK)<br />

Exercise stress test. (PC, MK)<br />

Stress thallium (myocardial perfusion scan). (PC, MK)<br />

Dobutamine stress echocardiography. (PC, MK)<br />

Coronary angiography. (PC, MK)<br />

Electron beam CT scan (for coronary calcification). (PC, MK)<br />

Ventilation/perfusion lung (V/Q) scan. (PC, MK)<br />

Pulmonary embolism protocol CT scan. (PC, MK)<br />

Pulmonary angiography. (PC, MK)<br />

5. Communication skills: Students should be able to:<br />

Communicate the diagnosis, prognosis and treatment plan to patients and their families.<br />

(PC, CS)<br />

As appropriate for age and gender, educate patients about risk factors for cardiovascular<br />

disease. (PC, CS)<br />

Counsel patients or facilitate the provision <strong>of</strong> counseling related to:<br />

o<br />

Smoking cessation. (PC, CS)<br />

o<br />

Reduction <strong>of</strong> dietary saturated fats and cholesterol. (PC, CS)<br />

o<br />

Restriction <strong>of</strong> dietary sodium intake. (PC, CS)<br />

o<br />

Weight reduction. (PC, CS)<br />

o<br />

Increased physical activity. (PC, CS)<br />

6. Basic procedural skills: Students should be able to:<br />

• Perform a 12-lead ECG. (PC)<br />

7. Management skills: Students should be able to develop an appropriate evaluation and<br />

treatment plan for patients that includes:<br />

Identification <strong>of</strong> the indications, contraindications, mechanisms <strong>of</strong> action, adverse reactions,<br />

significant interactions, and relative costs <strong>of</strong> the following medications:<br />

o Anti-platelet agents (aspirin, clopidogrel). (PC, MK)<br />

o Nitroglycerin and long-acting nitrates. (PC, MK)<br />

o Beta-blockers. (PC, MK)<br />

o Angiotensin-converting enzyme inhibitors. (PC, MK)<br />

o Calcium channel blockers. (PC, MK)<br />

o Antithrombotic therapy (heparin, warfarin). (PC, MK)<br />

o Glycoprotein IIb/IIIa inhibitors. (PC, MK)<br />

o Lipid-lowering agents. (PC, MK)<br />

Identification <strong>of</strong> the indications, contraindications, complications, long-term outcomes and<br />

relative costs associated with the following treatment modalities for ischemic heart disease:<br />

o Thrombolytic therapy. (PC, MK)<br />

o Percutaneous coronary intervention (with or without stenting). (PC, MK)<br />

o Coronary artery bypass graft surgery (CABG). (PC, MK)<br />

87


Determining when to consult a cardiologist or other subspecialist in the management <strong>of</strong><br />

patients with chest pain. (PC, SBP)<br />

Description <strong>of</strong> how the diagnosis and treatment <strong>of</strong> chest pain in special populations may<br />

differ (e.g. very elderly, associated co-morbidities). (PC, MK)<br />

Accessing and utilizing appropriate information systems and resources to help delineate<br />

issues related to chest pain. (PC, PLI)<br />

Incorporating patient preferences. (PC, P)<br />

C. ATTITUDES AND PROFESSIONAL BEHAVIORS: Students should be able to:<br />

1. Understand the emotional impact <strong>of</strong> a diagnosis <strong>of</strong> coronary artery disease and its potential<br />

effect on lifestyle (work performance, sexual functioning, etc). (PC, P)<br />

2. Respond appropriately to patient who are nonadherent to lifestyle modifications.<br />

(CS, P)<br />

3. Recognize the importance <strong>of</strong> early detection and modification <strong>of</strong> risk factors that may<br />

contribute to the development <strong>of</strong> atherosclerosis. (PC, P)<br />

4. Demonstrate commitment to using risk-benefit, cost-benefit, and evidence-based<br />

considerations in the selection <strong>of</strong> diagnostic and therapeutic interventions for chest pain. (PLI,<br />

P)<br />

5. Demonstrate ongoing commitment to self-directed learning regarding chest pain. (PLI, P)<br />

6. Recognize the importance and demonstrate a commitment to the utilization <strong>of</strong> other healthcare<br />

pr<strong>of</strong>essions in the treatment <strong>of</strong> chest pain. (P, SBP)<br />

D. REFERENCES:<br />

ACC/AHA 2002 guideline update for the management <strong>of</strong> patients with chronic stable<br />

angina--summary article: a report <strong>of</strong> the American College <strong>of</strong> Cardiology/American<br />

Heart Association Task Force on practice guidelines (Committee on the Management<br />

<strong>of</strong> Patients With Chronic Stable Angina). J Am Coll Cardiol. 2003;41:159-68.<br />

http://www.acc.org/clinical/guidelines/stable/stable_clean.pdf<br />

Panju AA, Hemmeigarn BR, Guyatt GH, Simel DL. Is this patient having a myocardial<br />

infarction? JAMA. 1998;280:1256-63.<br />

Klompas M. Does this patient have an acute thoracic aortic dissection? JAMA.<br />

2002;287:2262-72.<br />

TRAINING PROBLEM #7: COUGH<br />

RATIONALE:<br />

Cough is one <strong>of</strong> the most common symptoms with which a patient will present in the outpatient<br />

setting. There are several common etiologies for cough <strong>of</strong> which a third year medical student<br />

should be aware, as well as more clinically concerning etiologies. A proper understanding <strong>of</strong> the<br />

pathophysiology, diagnosis, and treatment <strong>of</strong> cough is an important learning objective.<br />

PREREQUISITES:<br />

Prior knowledge, skills, and attitudes acquired during the pre-<strong>clerkship</strong> experience should include:<br />

Ability to perform a complete medical history and physical exam.<br />

88


Ability to communicate with patients <strong>of</strong> diverse backgrounds.<br />

Knowledge <strong>of</strong> respiratory anatomy, physiology and pathophysiology.<br />

SPECIFIC LEARNING OBJECTIVES:<br />

A. KNOWLEDGE: Students should be able to define, describe and discuss:<br />

1. The criteria used to classify a cough (e.g. acute vs. chronic, productive vs. non-productive).<br />

(MK)<br />

2. Symptoms, signs, pathophysiology, differential diagnosis, and typical clinical course <strong>of</strong> the<br />

most common causes cough:<br />

Acute cough:<br />

o<br />

Viral tracheitis. (MK)<br />

o<br />

o<br />

Acute bronchitis. (MK)<br />

Pneumonia. (MK)<br />

Chronic cough:<br />

o Gastroesophageal reflux. (MK)<br />

o Post-nasal drip. (MK)<br />

o Asthma/reactive airways disease. (MK)<br />

o Angiotensin converting enzyme inhibitors. (MK)<br />

o Post-infectious. (MK)<br />

o Infectious (pertussis, tuberculosis). (MK)<br />

o Chronic bronchitis. (MK)<br />

o Bronchiectasis. (MK)<br />

o Pleural effusion. (MK)<br />

o Lung cancer. (MK)<br />

o Congestive heart failure. (MK)<br />

B. SKILLS: Students should be able to demonstrate specific skills, including:<br />

1. History-taking skills: Students should be able to obtain, document, and present an ageappropriate<br />

medical history that differentiates among the etiologies <strong>of</strong> disease, including:<br />

Onset. (PC, CS)<br />

Duration. (PC, CS)<br />

Exacerbating/relieving factors. (PC, CS)<br />

Associated symptoms (fever, chills, weight loss). (PC, CS)<br />

Presence or absence <strong>of</strong> hemoptysis. (PC, CS)<br />

Tobacco history. (PC, CS)<br />

Relevant past medical history. (PC, CS)<br />

2. Physical exam skills: Students should be able to perform a physical exam to establish the<br />

diagnosis and severity <strong>of</strong> disease, including:<br />

Accurately determining respiratory rate and level <strong>of</strong> respiratory distress. (PC)<br />

Recognizing the pharyngeal signs <strong>of</strong> post nasal drip. (PC)<br />

Identifying rales, rhonchi, and wheezes. (PC)<br />

Recognizing signs <strong>of</strong> pulmonary consolidation. (PC)<br />

3. Differential diagnosis: Students should be able to generate a prioritorized differential<br />

diagnosis recognizing history, physical exam, and laboratory findings that suggest a specific<br />

etiology <strong>of</strong> cough. (PC, MK)<br />

4. Laboratory interpretations: Students should be able to recommend when to order diagnostic<br />

89


and laboratory tests and be able to interpret them, both prior to and after initiating treatment<br />

based on the differential diagnosis, including consideration <strong>of</strong> test cost and performance<br />

characteristics as well as patient preferences. Laboratory and diagnostic tests should include,<br />

when appropriate:<br />

Chest radiograph. (PC, MK)<br />

Pleural fluid cell count and chemistries. (PC, MK)<br />

PFTs. (PC, MK)<br />

Sputum Gram stain and sputum acid-fast stain. (PC, MK)<br />

Sputum culture and sensitivities. (PC, MK)<br />

Students should be able to define the indications for and interpret (with consultation) results<br />

<strong>of</strong>:<br />

Barium swallow. (PC, MK)<br />

Upper endoscopy. (PC, MK)<br />

Sputum cytology. (PC, MK)<br />

Chest CT scan. (PC, MK)<br />

5. Communication skills: Students should be able to:<br />

Counsel and educate patients about environmental contributors to their disease,<br />

pneumococcal and influenza immunizations, and smoking cessation. (PC, CS)<br />

Communicate the diagnosis, prognosis, and treatment plan, and subsequent follow-up to<br />

the patient and his or her family. (PC, CS)<br />

Elicit input and questions from the patient and his or her family about the management<br />

plan. (PC, CS)<br />

6. Management skills: Students should be able to develop an appropriate evaluation and<br />

treatment plan for patients that includes:<br />

Describing the indications, contraindications, mechanisms <strong>of</strong> action, adverse reactions,<br />

significant interactions, and relative costs <strong>of</strong> the various treatments, interventions, or<br />

procedures commonly used to diagnose and treat patients who present with symptoms <strong>of</strong><br />

cough. (PC, MK, SBP)<br />

Determining when to obtain consultation from a pulmonologist, allergist, otolaryngologist, or<br />

gastroenterologist. (PC, SBP)<br />

Using a cost-effective approach based on the differential diagnosis. (PC, SBP)<br />

Accessing and utilizing appropriate information systems and resources to help delineate<br />

issues related to patients with chronic cough. (PC, PLI)<br />

Incorporating patient needs and preferences. (PC, P)<br />

C. ATTITUDES AND PROFESSIONAL BEHAVIORS: Students should be able to:<br />

1. Demonstrate commitment to using risk-benefit, cost-benefit, and evidence-based<br />

considerations in the selection <strong>of</strong> diagnostic and therapeutic interventions for cough. (PLI, P)<br />

2. Respond appropriately to patients who are non-adherent to treatment for cough and smoking<br />

cessation. (CS, P)<br />

3. Demonstrate ongoing commitment to self-directed learning regarding diagnosis and<br />

management <strong>of</strong> cough. (PLI, P)<br />

4. Appreciate the impact that an acute or chronic cough has on a patient’s quality <strong>of</strong> life, wellbeing,<br />

ability to work, and the family. (P)<br />

5. Recognize the importance and demonstrate a commitment to the utilization <strong>of</strong> other healthcare<br />

pr<strong>of</strong>essions in the treatment <strong>of</strong> cough. (P, SBP)<br />

D. REFERENCES:<br />

90


Currie GP, Gray RD, McKay J. Chronic cough. BMJ. 2003;326:261.<br />

Irwin RS, Madison JM. The persistently troublesome cough. Am J Respir Crit Care Med.<br />

2002;165:1469-74.<br />

Jones HC, Chang SI. Clinical Inquires. What is the best approach to the evaluation and<br />

treatment <strong>of</strong> chronic cough? J Fam Pract. 2001;50:748-9.<br />

Irwin RS, Madison MJ. Primary care: the diagnosis and treatment <strong>of</strong> cough. N Engl J Med.<br />

2000;343:1715-21.<br />

Irwin RS, Boulet LP, Cloutier MM, et al. Managing cough as a defense mechanism and as a<br />

symptom. Consensus panel report <strong>of</strong> the American College <strong>of</strong> Chest Physicians. Chest.<br />

1998;114:133S-181S.<br />

91


TRAINING PROBLEM #8: DYSPNEA<br />

RATIONALE:<br />

Shortness <strong>of</strong> breath or dyspnea is one <strong>of</strong> the most common patient complaints encountered in<br />

internal medicine. It has a very large number <strong>of</strong> etiologic possibilities— some benign but many<br />

potentially life-threatening. Because <strong>of</strong> the latter, a systematic approach to dyspnea is crucial.<br />

PREREQUISITES:<br />

Prior knowledge, skills, and attitudes acquired during the pre-clinical experience should include:<br />

Ability to perform a complete medical history and physical.<br />

Ability to communicate with patients <strong>of</strong> diverse backgrounds.<br />

Anatomy, physiology, and pathophysiology <strong>of</strong> the pulmonary, cardiac, neurologic, and<br />

musculoskeletal systems.<br />

Physiology <strong>of</strong> acid-base homeostasis.<br />

SPECIFIC LEARNING OBJECTIVES:<br />

A. KNOWLEDGE: Students should be able to define, describe, and discuss:<br />

1. Major organ systems/pathologic states causing dyspnea and their<br />

pathophysiology, including:<br />

Cardiac. (MK)<br />

Pulmonary. (MK)<br />

Anemia/hypovolemia. (MK)<br />

Acid-base disorders and other metabolic derangements (MK)<br />

Neuromuscular weakness. (MK)<br />

Central neurologic derangements. (MK)<br />

2. The symptoms, signs, and laboratory values associated with respiratory failure and ventilatory<br />

failure. (MK)<br />

3. The alveolar-arterial oxygen gradient and the pathophysiologic states that can alter it. (MK)<br />

4. The potential risks <strong>of</strong> relying too heavily on pulse oximetry as the sole indicator <strong>of</strong> arterial<br />

oxygen content. (MK)<br />

5. The common causes <strong>of</strong> acute dyspnea, their pathophysiology, symptoms, and signs, including:<br />

Pulmonary edema. (MK)<br />

Pulmonary embolism. (MK)<br />

Pneumonia. (MK)<br />

Acute exacerbation <strong>of</strong> COPD. (MK)<br />

Asthma. (MK)<br />

Cardiac ischemia. (MK)<br />

Pneumothorax. (MK)<br />

Anxiety. (MK)<br />

6. The common causes <strong>of</strong> chronic dyspnea their pathophysiology, symptoms, and signs,<br />

including:<br />

Congestive heart failure. (MK)<br />

COPD. (MK)<br />

Pulmonary parenchymal disease. (MK)<br />

Pulmonary vascular disease. (MK)<br />

92


Anemia. (MK)<br />

Neuromuscular weakness. (MK)<br />

7. Basic treatment options for the common causes <strong>of</strong> acute and chronic dyspnea. (MK)<br />

8. The utility <strong>of</strong> supplemental oxygen therapy and the potential dangers <strong>of</strong> overly aggressive<br />

oxygen supplementation in some pathophysiologic states. (MK)<br />

B. SKILLS: Students should be able to demonstrate specific skills, including:<br />

1. History-taking skills: Students should be able to obtain, document, and present an<br />

age-appropriate medical history, that differentiates among etiologies <strong>of</strong> disease,<br />

including:<br />

Quantity, quality, severity, duration, ameliorating/exacerbating factors <strong>of</strong> the dyspnea. (PC,<br />

CS)<br />

Associated symptoms such as fevers, chills, sweats, orthopnea, paroxysmal nocturnal<br />

dyspnea, wheezing, edema, chest pain, cough, sputum production, hemoptysis,<br />

palpitations, nausea, anxiety, dizziness, orthostasis, weakness. (PC, CS)<br />

History <strong>of</strong> pulmonary, cardiac, neuromuscular/neurologic, renal, hepatic, and coagulopathic<br />

disorders. (PC, CS)<br />

Risk factors for deep vein thrombosis/pulmonary embolism. (PC, CS)<br />

Ingestion <strong>of</strong> drugs and toxic substances, administration <strong>of</strong> IV fluids. (PC, CS<br />

Smoking and environmental exposures. (PC, CS)<br />

2. Physical exam skills: Students should be able to perform a physical exam to establish the<br />

diagnosis and severity <strong>of</strong> disease, including:<br />

Accurately determining respiratory rate and level <strong>of</strong> respiratory distress. (PC)<br />

Assessing the use <strong>of</strong> accessory muscles for breathing. (PC)<br />

Accurately measuring pulsus paradox. (PC)<br />

Identifying bronchial breath sounds, rales, rhonchi, wheezes, and subcutaneous<br />

emphysema. (PC)<br />

Identifying signs <strong>of</strong> pulmonary consolidation and hyperresonance. (PC)<br />

Identifying signs <strong>of</strong> pleural effusion. (PC)<br />

Identifying signs <strong>of</strong> elevated central venous pressure. (PC)<br />

Identifying signs <strong>of</strong> hypovolemia. (PC)<br />

Identifying S3 gallop, edema, and pallor. (PC)<br />

Identifying signs <strong>of</strong> deep vein thrombosis. (PC)<br />

3. Differential diagnosis: Students should be able to generate a prioritized differential diagnosis<br />

recognizing specific history and physical exam findings that suggest a specific etiology <strong>of</strong><br />

dyspnea (PC, MK)<br />

4. Laboratory interpretation: Students should be able to recommend when to order diagnostic<br />

and laboratory tests and be able to interpret them, both prior to and after initiating treatment<br />

based on the differential diagnosis, including consideration <strong>of</strong> test cost and performance<br />

characteristics as well as patient preferences. Laboratory and diagnostic tests should include,<br />

when appropriate:<br />

CBC. (PC, MK)<br />

Electrolytes, BUN/Cr, GLC. (PC, MK)<br />

Pulse oximitry. (PC, MK)<br />

ABG. (PC, MK)<br />

Chest radiograph. (PC, MK)<br />

12-lead ECG. (PC, MK)<br />

Pulmonary function tests. (PC, MK)<br />

93


Students should be able to define the indications for and interpret (with consultation) the<br />

results <strong>of</strong>:<br />

Ventilation perfusion scintigraphy. (PC, MK)<br />

Chest CT. (PC, MK)<br />

Venous Doppler studies. (PC, MK)<br />

Cardiac stress test. (PC, MK)<br />

Echocardiography. (PC, MK)<br />

5. Communication skills: Students should be able to:<br />

Communicate the diagnosis, treatment plan, and subsequent follow-up to the patient and<br />

his or her family. (PC, CS)<br />

Elicit questions from the patient and his or her family about the management plan. (PC, CS)<br />

Counsel and educate patients about environmental contributors to their disease. (PC, CS)<br />

Counsel patients nonjudgmentally about smoking cessation. (PC, CS)<br />

6. Basic and advanced procedural skills: Students should be able to:<br />

Obtain an ABG. (PC)<br />

7. Management skills: Students should able to develop an appropriate evaluation and treatment<br />

plan for patients that includes:<br />

A rapid triage approach to the acutely dyspneic patient. (PC, MK)<br />

An appropriate assessment <strong>of</strong> the patient’s oxygenation status. (PC, MK)<br />

Appropriate oxygen supplementation as indicated. (PC, MK)<br />

Management plans for pulmonary edema/congestive heart failure, pneumonia, COPD,<br />

asthma, pulmonary embolism, cardiac ischemia, hypovolemia, anemia, and pneumothorax.<br />

(PC, MK)<br />

Determining when to obtain consultation from an appropriate specialist. (PC, SBP)<br />

Using a cost-effective approach based on the differential diagnosis. (PC, SBP)<br />

Accessing and utilizing appropriate information systems and resources to help delineate<br />

issues related to dyspnea. (PC, PLI)<br />

Incorporating patient preferences. (PC, P)<br />

C. ATTITUDES AND PROFESSIONAL BEHAVIORS: Students should be able to:<br />

1. Demonstrate commitment to using risk-benefit, cost-benefit, and evidence-based<br />

considerations in the selection diagnostic and therapeutic interventions for dyspnea. (PLI, P)<br />

2. Recognize the importance <strong>of</strong> patient preferences when selecting among diagnostic and<br />

therapeutic options for dyspnea. (P)<br />

3. Demonstrate ongoing commitment to self-directed learning regarding dyspnea. (PLI, P)<br />

4. Appreciate the impact dyspnea has/have on a patient’s quality <strong>of</strong> life, wellbeing, ability to work,<br />

and the family. (P)<br />

5. Recognize the importance <strong>of</strong> and demonstrate a commitment to the utilization<br />

<strong>of</strong> other healthcare pr<strong>of</strong>essionals in the diagnosis and treatment <strong>of</strong> dyspnea. (P, SBP)<br />

6. Show understanding for the difficulties patients face with smoking cessation. (P)<br />

D. REFERENCES:<br />

Ingram RH and Braunwald E. (2005). Dyspnea and pulmonary edema. In Kasper DL,<br />

Braunwald EB, Fauci AS, Hauser SL, Longo DL, Jameson JL, eds. Harrison’s Principles<br />

94


<strong>of</strong> Internal <strong>Medicine</strong>. 16 th ed. New York, NY: McGraw-Hill; 2005:201-5.<br />

Zoorob RJ. Campbell JS. Acute dyspnea in the <strong>of</strong>fice. Am Fam Physician.<br />

2003;68:1803-10.<br />

Fedullo PF. Tapson VF. Clinical practice. The evaluation <strong>of</strong> suspected pulmonary<br />

embolism. N Engl J Med. 2003;349:1247-56.<br />

Manning HL. Schwartzstein RM. Pathophysiology <strong>of</strong> dyspnea. N Engl J Med.<br />

1995;333:1547-53.<br />

95


TRAINING PROBLEM #9: DYSURIA<br />

RATIONALE:<br />

Dysuria is a very common presentation in the outpatient setting. Given the amount <strong>of</strong> health care<br />

dollars that are spent on antibiotic treatment <strong>of</strong> urinary tract infections as well as the emergence <strong>of</strong><br />

resistance, it is important for third year medical students to have a working knowledge <strong>of</strong> how to<br />

approach the patient with this complaint, and how to differentiate patients with cystitis from other<br />

common causes <strong>of</strong> dysuria.<br />

PREREQUISITES:<br />

Prior knowledge, skills, and attitudes acquired during the pre-<strong>clerkship</strong> experience should include:<br />

Ability to perform a complete medical history and physical exam.<br />

Ability to communicate with patients <strong>of</strong> diverse backgrounds.<br />

Knowledge <strong>of</strong> genitourinary anatomy, physiology and pathophysiology.<br />

SPECIFIC LEARNING OBJECTIVES:<br />

A. KNOWLEDGE: Students should be able to define, describe, and discuss:<br />

1. Presenting signs and symptoms <strong>of</strong> the common causes <strong>of</strong> dysuria, including:<br />

Cystitis. (MK)<br />

Urethritis, gonococcal and non-gonococcal (e.g. chlamydia, trichomonas, HSV). (MK)<br />

Pyelonephritis. (MK)<br />

Acute and chronic prostatitis. (MK)<br />

Epididymitis. (MK)<br />

Vaginitis (yeast, bacterial vaginosis, trichomonas, atrophic, irritant). (MK)<br />

Interstitial cystitis. (MK)<br />

2. Symptoms and signs <strong>of</strong> pyelonephritis and how to distinguish an upper from a lower UTI. (MK)<br />

3. Common bacteria that cause UTI. (MK)<br />

4. Aspects <strong>of</strong> pathogenesis that affect UTI, including gender, sexual activity, diabetes, anatomic<br />

anomalies, instrumentation, and use <strong>of</strong> an indwelling catheter. (MK)<br />

5. Indications for pursuing further work up for patients with UTI. (MK)<br />

B. SKILLS: Students should be able to demonstrate specific skills, including:<br />

1. History-taking skills: Students should be able to obtain, document, and present an ageappropriate<br />

history that differentiates among etiologies <strong>of</strong> dysuria, including:<br />

Timing, frequency, severity, and location <strong>of</strong> dysuria. (PC, CS)<br />

Fever, chills, sweats. (PC, CS)<br />

Frequency, urgency, hesitancy, incomplete voiding. (PC, CS)<br />

Back, abdominal, and groin pain. (PC, CS)<br />

History <strong>of</strong> nephrolithiasis. (PC, CS)<br />

Hematuria. (PC, CS)<br />

Vaginal or penile discharge. (PC, CS)<br />

Penile skin lesions. (PC, CS)<br />

Sexual activity. (PC, CS)<br />

History <strong>of</strong> sexual transmitted diseases. (PC, CS)<br />

96


Dyspareunia. (PC, CS)<br />

Scrotal, testicular, and perineal pain. (PC, CS)<br />

Use <strong>of</strong> topical hygiene products. (PC, CS)<br />

2. Physical exam skills: Students should be able to perform a physical exam to establish the<br />

diagnosis and severity <strong>of</strong> disease, including:<br />

Percussion and palpation <strong>of</strong> the bladder to accurately recognize distention and tenderness.<br />

(PC)<br />

Palpation over the kidneys to elicit flank tenderness. (PC)<br />

Palpation <strong>of</strong> the abdomen to elicit tenderness. (PC)<br />

Palpation and massage <strong>of</strong> the male prostate to obtain discharge. (PC)<br />

Accurate recognition <strong>of</strong> perineal or vaginal atrophy and inflammation. (PC)<br />

Techniques <strong>of</strong> the pelvic examination to assess for causes <strong>of</strong> vaginitis. (PC)<br />

3. Differential diagnosis: Students should be able to generate a differential diagnosis<br />

recognizing specific history, physical exam, and laboratory findings that suggest a specific<br />

etiology <strong>of</strong> dysuria. (PC, MK)<br />

4. Laboratory interpretation: Students should be able to recommend when to order diagnostic<br />

and laboratory tests and be able to interpret them, both prior to and after initiating treatment<br />

based on the differential diagnosis, including consideration <strong>of</strong> test cost and performance<br />

characteristics as well as patient preferences.<br />

Laboratory and diagnostic tests should include, when appropriate:<br />

Urinalysis interpretation including cells and casts, urine dipstick and Gram stain when<br />

appropriate. (PC, MK)<br />

Urine culture. (PC, MK)<br />

Gram stain and culture <strong>of</strong> urethral or cervical discharge. (PC, MK)<br />

KOH stain and normal saline wet prep <strong>of</strong> vaginal discharge. (PC, MK)<br />

Urinary or cervical PCR to test for gonorrhea and Chlamydia. (PC, MK)<br />

KUB radiograph. (PC, MK)<br />

5. Communication skills: Students should be able to:<br />

Communicate the diagnosis, treatment plan, and subsequent follow-up to the patient and<br />

his or her family. (PC, CS)<br />

Elicit input and questions from the patient and his or her family about the management<br />

plan. (PC, CS)<br />

Counsel patients about safe sexual activity. (PC, CS)<br />

Explain the risk <strong>of</strong> recurrent UTI and counsel regarding preventative measures. (PC, CS)<br />

6. Management skills: Students should be able to develop an appropriate evaluation and<br />

treatment plan for patients that includes:<br />

Selecting appropriate empiric antibiotic therapy for cystitis, pyelonephritis or urethritis prior<br />

to culture results. (PC, MK)<br />

Counseling patients on symptomatic therapies for acute cystitis. (PC, MK)<br />

Selecting the appropriate duration <strong>of</strong> therapy for cystitis and pyelonephritis. (PC, MK)<br />

Evaluating and managing patients with recurrent urinary tract infections including<br />

prophylaxis. (PC, MK)<br />

Choosing appropriate treatment for vaginitis depending on results <strong>of</strong> evaluation. (PC, MK)<br />

Understanding the treatment <strong>of</strong> prostatitis based on probable organisms and age. (PC, MK)<br />

Determining when to obtain consultation from a urologist or gynecologist. (PC, MK)<br />

Using a cost-effective approach based on the differential diagnosis. (PC, SBP)<br />

Accessing and utilizing appropriate information systems and resources to help delineate<br />

issues related to dysuria. (PC, PLI)<br />

97


Incorporating patient preferences. (PC, P)<br />

C. ATTITUDES AND PROFESSIONAL BEHAVIORS: Students should be able to:<br />

1. Demonstrate commitment to using risk-benefit, cost-benefit, and evidence-based<br />

considerations in the selection <strong>of</strong> diagnostic and therapeutic interventions for dysuria. (PLI, P)<br />

2. Recognize the importance <strong>of</strong> patient needs and preferences when selecting among diagnostic<br />

and therapeutic options for dysuria. (P)<br />

3. Demonstrate ongoing commitment to self-directed learning regarding dysuria. (PLI, P)<br />

4. Recognize the importance <strong>of</strong> and demonstrate a commitment to the utilization <strong>of</strong> other<br />

healthcare pr<strong>of</strong>essionals in the treatment <strong>of</strong> dysuria. (P, SBP)<br />

D. REFERENCES:<br />

32.<br />

Bremnor JD, Sadovsky R. Evaluation <strong>of</strong> dysuria in adults. Am Fam Physician.<br />

2002;65:1589-96.<br />

Bent S, Nallamothu BK, Simel DL, et al. Does this woman have an acute<br />

uncomplicated urinary tract infection? JAMA. 2002;287:2701-10.<br />

Owen MK, Clenney TL. Management <strong>of</strong> vaginitis. Am Fam Physician. 2004;70:2125-<br />

Fihn SD. Clinical practice. Acute uncomplicated urinary tract infection in<br />

women. N Engl J Med. 2003;349:259-66.<br />

98


TRAINING PROBLEM #10: FEVER<br />

RATIONALE:<br />

Because fever can have many infectious or noninfectious causes, patients with fever should be<br />

stratified by host susceptibility factors and evaluated in a systematic manner. A rational approach to<br />

patients with fever will help clinicians recognize presentations that need immediate attention, limit<br />

unnecessary diagnostic testing in less seriously ill patients, and help inform therapeutic decision<br />

making.<br />

PREREQUISITES:<br />

Prior knowledge, skills, and attitudes acquired during the pre-<strong>clerkship</strong> experience should include:<br />

Ability to perform a complete medical history and physical exam.<br />

Ability to communicate with patients <strong>of</strong> diverse backgrounds.<br />

Physiology and pathophysiology <strong>of</strong> thermoregulation and the immune response.<br />

Pharmacology <strong>of</strong> antipyretics.<br />

SPECIFIC LEARNING OBJECTIVES:<br />

A. KNOWLEDGE: Students should be able to define, describe, and discuss:<br />

1. Physiology <strong>of</strong> the acute febrile response, including the:<br />

Beneficial and detrimental effects <strong>of</strong> fever upon the host. (MK)<br />

The differences in clinical manifestations between immunocompetent and<br />

immunocompromised patients. (MK)<br />

2. Risk factors and co-morbidities that are important in determining the host response to infection<br />

(e.g. neutropenia, asplenia, cirrhosis, alcoholism, diabetes, corticosteroid use, malnutrition, T<br />

cell dysfunction) (MK)<br />

3. Etiology <strong>of</strong> fever in special populations, including patients with a history <strong>of</strong>:<br />

Neutropenia due to cancer-related myelosuppression. (MK)<br />

HIV disease. (MK)<br />

Intravenous drug abuse. (MK)<br />

Recent international travel or immigration. (MK)<br />

Concomitant skin rash and lymphadenopathy. (MK)<br />

4. Pathophysiology and clinical presentation <strong>of</strong> patients with sepsis syndromes. (MK)<br />

5. Common causes <strong>of</strong> prolonged fever without apparent source, including:<br />

FUO in a normal host. (MK)<br />

Nosocomial FUO. (MK)<br />

Neutropenic FUO. (MK)<br />

FUO associated with HIV disease. (MK)<br />

B. SKILLS: Students should be able to demonstrate specific skills, including:<br />

1. History-taking skills: Students should be able to obtain, document, and present an ageappropriate<br />

medical history that differentiates among etiologies <strong>of</strong> disease, including:<br />

Chronology, duration and pattern <strong>of</strong> fever. (PC, CS)<br />

Associated symptoms. (PC, CS)<br />

Immune status and baseline co-morbidities. (PC, CS)<br />

Immunization status. (PC, CS)<br />

99


Relevant history <strong>of</strong> exposures. (PC, CS)<br />

Occupational, travel, family, and sexual history. (PC, CS)<br />

Medication history, including use <strong>of</strong> over-the-counter and illicit drugs. (PC, CS)<br />

3. Physical exam skills: Students should be able to perform a complete physical exam to<br />

determine the severity <strong>of</strong> disease and establish a preliminary hypothesis about the cause <strong>of</strong><br />

fever. (PC)<br />

4. Differential diagnosis: Students should be able to generate a prioritized differential diagnosis<br />

recognizing specific history and physical exam findings that suggest a specific etiology:<br />

Infection. (PC, MK)<br />

Rheumatologic disease/inflammatory disorder. (PC, MK)<br />

Malignancy. (PC, MK)<br />

Drug reaction. (PC, MK)<br />

4. Laboratory interpretation: Students should be able to recommend when to order diagnostic<br />

and laboratory tests and be able to interpret them, both prior to and after initiating treatment<br />

based on the differential diagnosis, including consideration <strong>of</strong> test cost and performance<br />

characteristics as well as patient preferences. Laboratory and diagnostic tests should include,<br />

when appropriate:<br />

CBC with differential. (PC, MK)<br />

UA with exam <strong>of</strong> urinary sediment. (PC, MK)<br />

Chest radiography. (PC, MK)<br />

Blood cultures. (PC, MK)<br />

Urine cultures. (PC, MK)<br />

Sputum Gram stain and cultures. (PC, MK)<br />

Sputum AFB stain and culture. (PC, MK)<br />

ESR and/or specific rheumatologic tests. (PC, MK)<br />

PPD. (PC, MK)<br />

Cerebrospinal fluid analysis (color, opening pressure, chemistries, cell<br />

counts, staining, cultures, cytology, cryptococcal antigen, VDRL). (PC, MK)<br />

Chemistries, Gram stain, and culture <strong>of</strong> abnormal fluid collections<br />

(e.g. pleural effusion, ascites, abscesses). (PC, MK)<br />

Stool culture <strong>of</strong> enteric pathogens. (PC, MK)<br />

Stool Clostridium difficile toxin assay. (PC, MK)<br />

Stains and cultures from the throat, urethra, anus, cervix, vagina. (PC, MK)<br />

HIV ELISA and western blot. (PC, MK)<br />

Students should be able to define the indications for and interpret (with consultation) the<br />

results <strong>of</strong>:<br />

CT imaging. (PC, MK)<br />

Echocardiography. (PC, MK)<br />

Tissue biopsy. (PC, MK)<br />

5. Communication skills: Students should be able to:<br />

Communicate the diagnosis, treatment plan, and subsequent follow-up patients. (PC, CS)<br />

Elicit questions from the patient and their family about the management plan. (PC, CS)<br />

6. Basic and advanced procedural skills: Students should be able to:<br />

Obtain blood, wound, and throat cultures. (PC)<br />

Place and interpret a PPD. (PC)<br />

7. Management skills: Students should able to develop an appropriate evaluation and<br />

treatment plan for patients that includes:<br />

100


Developing an appropriate evaluation plan for patients with fever including ordering and<br />

interpreting appropriate laboratory and radiographic studies. (PC)<br />

Assessing the severity <strong>of</strong> presentation based on the history, host factors, physical exam<br />

and laboratory results and recognizing presentations that need immediate attention. (PC)<br />

Developing an appropriate treatment plan for patients with fever including the selection <strong>of</strong><br />

an initial, empiric treatment regimen for neutropenic patients with fever and/or patients with<br />

life threatening sepsis. (PC)<br />

Determining when to obtain consultation from an appropriate specialist. (PC, SBP)<br />

Using a cost-effective approach based on the differential diagnosis. (PC, SBP)<br />

Accessing and utilizing appropriate information systems and resources to help delineate<br />

issues related to fever. (PC, PLI)<br />

Incorporating patient preferences. (PC, P)<br />

C. ATTITUDES AND PROFESSIONAL BEHAVIORS: Students should be able to:<br />

3. Demonstrate commitment to using risk-benefit, cost-benefit, and evidence-based<br />

considerations in the selection diagnostic and therapeutic interventions for fever. (P, PLI)<br />

4. Appreciate the impact fever has on a patient’s quality <strong>of</strong> life, well-being, ability to work, and<br />

family; recognize the emotional impact <strong>of</strong> differential diagnosis. (P)<br />

5. Recognize the importance <strong>of</strong> and demonstrate a commitment to the utilization <strong>of</strong> other<br />

healthcare pr<strong>of</strong>essions in the diagnosis and treatment <strong>of</strong> fever. (P, SBP)<br />

D. REFERENCES:<br />

Dinarello CA, Gelfand JA. (2005). Fever and hyperthermia. In Kasper DL, Braunwald<br />

EB, Fauci AS, Hauser SL, Longo DL, Jameson JL, eds. Harrison’s Principles <strong>of</strong> Internal<br />

<strong>Medicine</strong>. 16 th ed. New York, NY: McGraw-Hill; 2005:104-8.<br />

Kaye ET, Kaye KM. (2005). Fever and rash. In Kasper DL, Braunwald EB, Fauci AS,<br />

Hauser SL, Longo DL, Jameson JL, eds. Harrison’s Principles <strong>of</strong> Internal <strong>Medicine</strong>. 16 th<br />

ed. New York, NY: McGraw-Hill; 2005:108-16.<br />

Gelfand JA, Callahan MV. (2005). Fever <strong>of</strong> unknown origin. In Kasper DL, Braunwald<br />

EB, Fauci AS, Hauser SL, Longo DL, Jameson JL, eds. Harrison’s Principles <strong>of</strong> Internal<br />

<strong>Medicine</strong>. 16 th ed. New York, NY: McGraw-Hill; 2005:116-21.<br />

Roth AR, Basello GM. Approach to the adult patient with fever <strong>of</strong> unknown origin. Am<br />

Fam Physician. 2003;68:2223-8.<br />

101


TRAINING PROBLEM #11: FLUID, ELECTROLYTE AND<br />

ACID-BASE DISORDERS<br />

RATIONALE:<br />

Many disease processes can cause serious disturbances in the fluid, electrolyte and acid-base<br />

status <strong>of</strong> patients. Clinicians must be prepared to identify and correct these disturbances as<br />

efficiently as possible, thus making it an important training problem for third year medical students.<br />

PREREQUISITES:<br />

Prior knowledge, skills and attitudes acquired during the pre-<strong>clerkship</strong> experience should include:<br />

Ability to perform a complete medical history and physical exam.<br />

Ability to communicate with patients <strong>of</strong> diverse backgrounds.<br />

Knowledge <strong>of</strong> pathogenesis and pathophysiology <strong>of</strong> fluid, electrolyte and acid-base disorders.<br />

Knowledge <strong>of</strong> medications that can cause alterations in fluid and electrolyte status as well as<br />

disturbance <strong>of</strong> acid-base status.<br />

SPECIFIC LEARNING OBJECTIVES:<br />

A. KNOWLEDGE: Students should be able to define, describe and discuss:<br />

1. The pathophysiology <strong>of</strong>:<br />

Hypo- and hypervolemia. (MK)<br />

Hypo- and hypernatremia. (MK)<br />

Hypo- and hyperkalemia. (MK)<br />

Hypo- and hypercalcemia. (MK)<br />

Simple and mixed acid-base disorders. (MK)<br />

Hypo- and hyperphosphatemia. (MK)<br />

Hypo- and hypermagnesemia. (MK)<br />

Respiratory acidosis and alkalosis. (MK)<br />

Metabolic acidosis and alkalosis. (MK)<br />

2. Presenting symptoms and signs <strong>of</strong> the above disorders. (MK)<br />

3. The importance <strong>of</strong> total body water and its distribution. (MK)<br />

4. The differential diagnosis <strong>of</strong> hypo- and hypernatremia in the setting <strong>of</strong> volume depletion,<br />

euvolemia, and hypervolemia. (MK)<br />

5. How to distinguish hyponatremia from pseudohyponatremia. (MK)<br />

6. How to identify spurious hyperkalemia or acidosis-related hyperkalemia. (MK)<br />

7. Risks <strong>of</strong> too rapid or delayed therapy for hyponatremia. (MK)<br />

8. The most common causes <strong>of</strong> respiratory acidosis, respiratory alkalosis, metabolic acidosis and<br />

metabolic alkalosis. (MK)<br />

9. How to calculate the anion gap and explain its relevance to determining the cause <strong>of</strong> a<br />

metabolic acidosis. (MK)<br />

10. Changes in total body water distribution that occur with aging. (MK)<br />

11. How altered mental status can contribute to electrolyte disorders. (MK)<br />

12. Tests to use in the evaluation <strong>of</strong> fluid, electrolyte, and acid-base disorders. (MK)<br />

13. Indications for obtaining an ABG. (MK)<br />

102


14. The types <strong>of</strong> fluid preparations to use in the treatment <strong>of</strong> fluid and electrolyte disorders. (MK)<br />

B. SKILLS: Students should demonstrate specific skills, including:<br />

1. History-taking skills: Students should be able to obtain, document, and present an ageappropriate<br />

medical history that differentiates among etiologies <strong>of</strong> disease, including:<br />

Eliciting appropriate information from patients with volume overload, including recent weight<br />

gain, edema or ascites, symptoms <strong>of</strong> heart failure, dietary sodium intake, changes in<br />

medications, noncompliance and intravenous fluid regimens. (PC, CS)<br />

Eliciting appropriate information from patients with volume depletion, including recent<br />

weight loss, thirst, gastrointestinal losses, urinary losses, oral intake, insensible losses, and<br />

intravenous fluid regimens. (PC, CS)<br />

Eliciting appropriate information from patients with electrolyte problems, including use <strong>of</strong><br />

diuretics and other medications, gastrointestinal losses, and history <strong>of</strong> relevant medical<br />

conditions (e.g., heart failure, liver disease, renal disease, pulmonary disease, central<br />

nervous system disease, and malignancy). (PC, CS)<br />

2. Physical exam skills: Students should be able to perform a physical exam to establish the<br />

diagnosis and severity <strong>of</strong> disease, including:<br />

Measurement <strong>of</strong> orthostatic vital signs. (PC)<br />

Identification <strong>of</strong> signs <strong>of</strong> volume overload including peripheral edema, pulmonary edema,<br />

ascites, edema. (PC)<br />

Identification <strong>of</strong> signs <strong>of</strong> volume depletion including tachycardia, orthostatic hypotension,<br />

dry mucous membranes, poor skin turgor. (PC)<br />

Identification <strong>of</strong> signs <strong>of</strong> sodium disorders including lethargy, weakness, encephalopathy,<br />

delirium, seizures. (PC)<br />

Identification <strong>of</strong> signs <strong>of</strong> potassium disorders including weakness, fatigue, constipation,<br />

ileus, cramping, tetany, hypo- or hyperreflexia. (PC)<br />

Identification <strong>of</strong> signs <strong>of</strong> calcium disorders including cramping, tetany, Chvostek’s and<br />

Trousseau’s sign, seizures, anorexia, constipation, polyuria, hypo- or hyperreflexia, stupor,<br />

coma. (PC)<br />

3. Differential diagnosis: Students should be able to generate a prioritized differential<br />

diagnosis recognizing specific history, physical exam, and laboratory findings that<br />

distinguish between:<br />

Hypo- and hypervolemia. (PC, MK)<br />

Hypo- and hypernatremia. (PC, MK)<br />

Hypo- and hyperkalemia. (PC, MK)<br />

Hypo- and hypercalcemia. (PC, MK)<br />

Hypo- and hyperphosphatemia. (PC, MK)<br />

Hypo- and hypermagnesemia. (PC, MK)<br />

Respiratory acidosis and alkalosis. (PC, MK)<br />

Metabolic acidosis and alkalosis. (PC, MK)<br />

4. Laboratory interpretation: Students should be able to recommend when to order diagnostic<br />

and laboratory tests and be able to interpret them, both prior to and after initiating treatment<br />

based on the differential diagnosis, including consideration <strong>of</strong> test cost and performance<br />

characteristics as well as patient preferences. Laboratory and diagnostic tests should include,<br />

when appropriate:<br />

Serum electrolytes, BUN/Cr. (PC, MK)<br />

Anion gap. (PC, MK)<br />

103


ABG. (PC, MK)<br />

Serum and urine osmolality. (PC, MK)<br />

Urinary sodium. (PC, MK)<br />

Fractional excretion <strong>of</strong> sodium. (PC, MK)<br />

ECG findings in hyper- and hypokalemia. (PC, MK)<br />

5. Communication skills: Students should be able to:<br />

Explain to a patient and his or her family why intravenous fluids are needed. (PC, CS)<br />

Communicate the diagnosis, treatment plan, and subsequent follow-up to the patient and<br />

his or her family. (PC, CS)<br />

Elicit input and questions from the patient and their family about the management plan.<br />

(PC, CS)<br />

6. Basic and advanced procedural skills: Students should be able to:<br />

Insert a peripheral intravenous catheter. (PC)<br />

Obtain an ABG. (PC)<br />

Assist in the insertion <strong>of</strong> a central venous catheter. (PC)<br />

7. Management skills: Students should be able to develop an appropriate evaluation and<br />

treatment plan for patients that includes:<br />

Writing appropriate fluid orders for the treatment <strong>of</strong> hypo- and hypervolemia, hypo- and<br />

hypernatremia, hypo- and hyperkalemia, hypo- and hypercalcemia. (PC, MK)<br />

Writing appropriate orders for replacing sodium, potassium, calcium, phosphates, and<br />

magnesium. (PC, MK)<br />

Writing appropriate orders for correcting hyperkalemia, hypercalcemia, hyperphosphatemia<br />

and hypermagnesemia. (PC, MK)<br />

Calculating the water deficit that needs to be corrected to treat hypernatremia. (PC, MK)<br />

Identifying indications for administration <strong>of</strong> bicarbonate. (PC, MK)<br />

Determining when to obtain consultation from a nephrologist. (PC, SBP)<br />

Using a cost-effective approach based on the differential diagnosis. (PC, SBP)<br />

Accessing and utilizing appropriate information systems and resources to help delineate<br />

issues related to fluid, electrolyte, and acid-base disorders. (PC SBP)<br />

Incorporating patient preferences. (PC, P)<br />

C. ATTITUDES AND PROFESSIONAL BEHAVIORS: Students should be able to:<br />

1. Demonstrate commitment to using risk-benefit, cost-benefit, and evidence-based<br />

considerations in the selection <strong>of</strong> diagnostic and therapeutic interventions for problems related<br />

to fluid, electrolyte and acid-base disorders. (PLI, P)<br />

3. Demonstrate ongoing commitment to self-directed learning regarding fluid, electrolyte and<br />

acid-based disorders. (PLI, P)<br />

4. Recognize the importance and demonstrate a commitment to the utilization <strong>of</strong> other healthcare<br />

pr<strong>of</strong>essions in the treatment <strong>of</strong> problems related to fluid, electrolyte and acid-base disorders.<br />

(P, SBP)<br />

D. REFERENCES:<br />

Singer GG, Brenner BM. Fluid and Electrolyte Disturbances. In Kasper DL, Braunwald<br />

EB, Fauci AS, Hauser SL, Longo DL, Jameson JL, eds. Harrison’s Principles <strong>of</strong> Internal<br />

<strong>Medicine</strong>. 16 th ed. New York, NY: McGraw-Hill; 2005:252-63.<br />

DuBose TD. (2005). Acidosis and Alkalosis. In Kasper DL, Braunwald EB, Fauci AS,<br />

104


Hauser SL, Longo DL, Jameson JL, eds. Harrison’s Principles <strong>of</strong> Internal <strong>Medicine</strong>. 16 th<br />

ed. New York, NY: McGraw-Hill; 2005:263-71.<br />

Preston RA. Acid-Base, Fluids, and Electrolytes Made Ridiculously Simple. Miami, FL:<br />

MedMaster Incorporated; 2002.<br />

Adrogue HJ. Madias NE. Hyponatremia. N Engl J Med. 2000;342:1581-9.<br />

Adrogue HJ. Madias NE. Hypernatremia. N Engl J Med. 2000;342:1493-9.<br />

Stewart AF. Clinical practice: Hypercalcemia associated with cancer. N Engl J Med.<br />

2005;352:373-9.<br />

Carroll MF. Schade DS. A practical approach to hypercalcemia. Am Fam Physician.<br />

2003;67:1959-66.<br />

Gennari FJ. Hypokalemia. N Engl J Med. 1998;339:451-8.<br />

105


TRAINING PROBLEM #12: GASTROINTESTINAL BLEEDING<br />

RATIONALE:<br />

Gastrointestinal bleeding is a common disorder which can be life-threatening if not properly<br />

diagnosed and treated. Knowledge <strong>of</strong> etiology, risk factors, approach, and management is<br />

integral to internal medicine training.<br />

Prerequisites:<br />

Prior knowledge, skills, and attitudes acquired during the pre-<strong>clerkship</strong> experience should include:<br />

Ability to perform a complete medical history and physical exam.<br />

Ability to communicate with patients <strong>of</strong> diverse backgrounds.<br />

Anatomy, physiology, and pathophysiology <strong>of</strong> the gastrointestinal tract.<br />

Pharmacology <strong>of</strong> non-steroidal anti-inflammatory medication (a major contributing factor in<br />

etiology <strong>of</strong> gastrointestinal bleeding) as well as proton pump inhibitors and other agents used<br />

in the acute setting for treatment <strong>of</strong> gastrointestinal bleeding.<br />

SPECIFIC LEARNING OBJECTIVES:<br />

A. KNOWLEDGE: Students should be able to define, describe, and discuss:<br />

1. The common causes for and symptoms <strong>of</strong> upper and lower gastrointestinal blood loss,<br />

including:<br />

Esophagitis/esophageal erosions. (MK)<br />

Mallory Weiss tear. (MK)<br />

Peptic and duodenal ulcer disease. (MK)<br />

Esophageal/gastric varices. (MK)<br />

Erosive gastritis. (MK)<br />

Arteriovenous malformations. (MK)<br />

Gastrointestinal tumors, benign and malignant. (MK)<br />

Diverticulosis. (MK)<br />

Ischemic colitis. (MK)<br />

Hemorrhoids. (MK)<br />

Anal fissures. (MK)<br />

2. The distinguishing features <strong>of</strong> upper versus lower GI bleeding (MK)<br />

3. The indications for inpatient versus outpatient evaluation and treatment (MK)<br />

4. The principles <strong>of</strong> stabilization and treatment <strong>of</strong> acute massive GI blood loss. (MK)<br />

5. The role <strong>of</strong> contributing factors in GI bleeding such as H. pylori infection; NSAIDs, alcohol,<br />

cigarette use, coagulopathies; and chronic liver disease. (MK)<br />

B. SKILLS: Students should demonstrate specific skills, including:<br />

1. History-taking skills: Students should be able to obtain, document, and present an age<br />

appropriate history that differentiates among etiologies <strong>of</strong> disease, including:<br />

Features that distinguish upper from lower GI bleeding. (PC, CS)<br />

Quantification <strong>of</strong> degree <strong>of</strong> blood loss. (PC, CS)<br />

Chronology and duration <strong>of</strong> bleeding. (PC, CS)<br />

Associated symptoms. (PC, CS)<br />

106


Relevant past medical history. (PC, CS)<br />

Medication history, including use <strong>of</strong> tobacco and alcohol. (PC, CS)<br />

2. Physical exam skills: Students should be able to perform a physical examination to<br />

establish the diagnosis and severity <strong>of</strong> disease, including:<br />

Postural blood pressure and pulse. (PC, MK)<br />

Abdominal palpation for organomegaly, masses, and tenderness. (PC, MK)<br />

Search for stigmata <strong>of</strong> chronic liver disease. (PC, CS)<br />

Anal and rectal examination. (PC, CS)<br />

3. Differential diagnosis: Students should be able to generate a differential diagnosis<br />

recognizing specific history and physical examination findings that suggest a specific etiology<br />

for GI bleeding. (PC, MK)<br />

4. Laboratory interpretation: Students should be able to recommend when to order<br />

diagnostic and laboratory tests and be able to interpret them, both prior to and after initiating<br />

treatment based on the differential diagnosis, including consideration <strong>of</strong> test cost and<br />

performance characteristics as well as patient preferences.<br />

Laboratory and diagnostic tests should include, when appropriate:<br />

Stool and gastric fluid tests for occult blood. (MK, PC)<br />

CBC. (MK, PC)<br />

PT/PTT. (MK, PC)<br />

Hepatic function panel. (MK, PC)<br />

Tests for Helicobacter pylori. (MK, PC)<br />

Students should be able to define the indications for and interpret (with consultation) results<br />

<strong>of</strong>:<br />

Esophagogastroduodenoscopy (EGD). (MK, PC)<br />

Colonoscopy. (MK, PC)<br />

Barium studies <strong>of</strong> the gastrointestinal tract. (MK, PC)<br />

5. Communication skills: Students should be able to:<br />

Communicate the diagnosis, treatment plan, and subsequent follow-up to patients. (PC,<br />

CS)<br />

Elicit questions from the patient and his or her family about the management plan. (PC, CS)<br />

6. Basic and advanced procedural skills: Students should be able to:<br />

Start an IV line using a large bore (i.e. 18 gauge) needle. (MK, PC)<br />

Perform a stool or emesis occult blood testing. (MK, PC)<br />

7. Management skills: Students should be able to develop an appropriate evaluation and<br />

treatment plan for patients that includes:<br />

Establishing adequate venous access. (PC, MK)<br />

Administering crystalloid fluid resuscitation. (PC, MK)<br />

Ordering blood and blood product transfusion. (PC, MK)<br />

Determining when to obtain consultation from a gastroenterologist or a general surgeon.<br />

(PC, MK)<br />

Using a cost-effective approach based on the differential diagnosis. (PC, SBP)<br />

Accessing and utilizing appropriate information systems and resources to help delineate<br />

issues related to gastrointestinal bleeding. (PC, PLI)<br />

Incorporating patient preferences. (PC,P)<br />

Outlining long-term management when appropriate (e.g. Helicobacter pylori eradication,<br />

antacid, H-2 blocker or proton pump inhibitor therapy, smoking /alcohol cessation, NSAID<br />

107


estriction, and dietary modification. (MK, CS)<br />

C. ATTITUDES AND PROFESSIONAL BEHAVIORS: Students should be able to:<br />

1. Demonstrate commitment to using risk-benefit, cost-benefit, and evidence-based<br />

considerations in the selection <strong>of</strong> diagnostic and therapeutic interventions for gastrointestinal<br />

bleeding. (PLI, P)<br />

2. Respond appropriately to patients who are nonadherent to treatment for gastrointestinal<br />

bleeding. (CS, P)<br />

3. Demonstrate ongoing commitment to self-directed learning regarding gastrointestinal bleeding.<br />

(PLI, P)<br />

4. Appreciate the impact gastrointestinal bleeding has on a patient’s quality <strong>of</strong> life, well-being,<br />

ability to work, and the family. (P)<br />

5. Recognize the importance and demonstrate a commitment to the utilization <strong>of</strong><br />

6. other health care pr<strong>of</strong>essions in the treatment <strong>of</strong> gastrointestinal bleeding. (P, SBP)<br />

D. RESOURCES:<br />

Fallah, MA, Prakash, C, Edmundowicz, S. Acute gastrointestinal bleeding. Med Clin<br />

North Am. 2000;84:1183-208.<br />

Laine L. (2005). Gastrointestinal bleeding. In Kasper DL, Braunwald EB, Fauci AS,<br />

Hauser SL, Longo DL, Jameson JL, eds. Harrison’s Principles <strong>of</strong> Internal <strong>Medicine</strong>. 16 th ed.<br />

New York, NY: McGraw-Hill; 2005:235-8.<br />

Del Valle J. (2005). Peptic ulcer disease and related disorders. In Kasper DL,<br />

Braunwald EB, Fauci AS, Hauser SL, Longo DL, Jameson JL, eds. Harrison’s Principles <strong>of</strong><br />

Internal <strong>Medicine</strong>. 16 th ed. New York, NY: McGraw-Hill; 2005:1746-62.<br />

Gearhart SL, Bulkley G. (2005). Common diseases <strong>of</strong> the colon and anorectum and<br />

mesenteric vascular insufficiency. In Kasper DL, Braunwald EB, Fauci AS, Hauser SL, Longo<br />

DL, Jameson JL, eds. Harrison’s Principles <strong>of</strong> Internal <strong>Medicine</strong>. 16 th ed. New York, NY:<br />

McGraw-Hill; 2005:1795-803.<br />

Mitchell SH, Schaefer DC. A new view <strong>of</strong> occult and obscure gastrointestinal<br />

bleeding. Am Fam Physician. 2004;69:875-81.<br />

108


TRAINING PROBLEM #13: KNEE PAIN<br />

RATIONALE:<br />

Musculoskeletal complaints are some <strong>of</strong> the most common problems for which patients seek medical<br />

attention, and the knee is the single most common joint pain. Many <strong>of</strong> these problems can be<br />

effectively tackled in the primary care setting without need for consultation. The principles presented<br />

in this training problem can be readily applied to other joint pains.<br />

PREREQUISITES:<br />

Prior knowledge, skills, and attitudes acquired during the pre-<strong>clerkship</strong> experience should include:<br />

Ability to perform a complete medical history and physical exam.<br />

Ability to communicate with patients <strong>of</strong> diverse backgrounds.<br />

Anatomy and physiology <strong>of</strong> the musculoskeletal system.<br />

Pharmacology <strong>of</strong> acetaminophen, nonsteroidal anti-inflammatory drugs (NSAIDs), topical<br />

medications (capsaicin and lidocaine) and glucocorticoids.<br />

Basic bone radiograph interpretation.<br />

SPECIFIC LEARNING OBJECTIVES:<br />

A. KNOWLEDGE: Students should be able to define, describe, and discuss:<br />

11. A systematic approach to joint pain based on an understanding <strong>of</strong> pathophysiology to classify<br />

potential causes. (MK)<br />

12. The effect <strong>of</strong> the time course <strong>of</strong> symptoms on the potential causes <strong>of</strong> joint pain (acute vs.<br />

subacute vs. chronic). (MK)<br />

13. The difference between and pathophysiology <strong>of</strong> arthralgia vs. arthritis and mechanical vs.<br />

inflammatory joint pain. (MK)<br />

14. The distinguishing features <strong>of</strong> intra-articular and periarticular complaints (joint pain vs. bursitis<br />

and tendonitis). (MK)<br />

15. The effect <strong>of</strong> the features <strong>of</strong> joint involvement on the potential causes <strong>of</strong> joint pain<br />

(monoarticular vs. oligoarticular vs. polyarticular, symmetric vs. asymmetric, axial and/or<br />

appendicular, small vs. large joints, additive vs. migratory vs. intermittent). (MK)<br />

16. Indications for performing an arthrocentesis and the results <strong>of</strong> synovial fluid analysis. (MK)<br />

17. The utility <strong>of</strong> describing the relative location <strong>of</strong> knee pain (anterior, medial, lateral, posterior).<br />

(MK)<br />

18. The relative frequency <strong>of</strong> the various causes <strong>of</strong> knee pain. (MK)<br />

19. The differential diagnosis, pathophysiology, and typical presentations <strong>of</strong> the common intraarticular<br />

causes <strong>of</strong> knee pain:<br />

Osteoarthritis. (MK)<br />

Inflammatory arthropathies. (MK)<br />

Crystalline arthropathies. (MK)<br />

Septic arthritis. (MK)<br />

Patell<strong>of</strong>emoral pain syndrome. (MK)<br />

Cruciate ligament tear. (MK)<br />

Meniscal damage. (MK)<br />

10. The differential diagnosis, pathophysiology, and typical presentations <strong>of</strong> the common<br />

periarticular causes <strong>of</strong> knee pain:<br />

109


Collateral ligament sprain/tear. (MK)<br />

Ileotibial band syndrome. (MK)<br />

Prepatellar bursitis. (MK)<br />

Popliteal (Baker) cyst. (MK)<br />

11. Basic symptomatic treatment for knee pain, including:<br />

Relative rest. (MK)<br />

Ice/heat. (MK)<br />

Compression. (MK)<br />

Elevation. (MK)<br />

Acetaminophen. (MK)<br />

Nonsteroidal anti-inflammatory drugs. (MK)<br />

Glucosamine and chondroitin sulfate. (MK)<br />

Physical therapy. (MK)<br />

Assistive devices. (MK)<br />

Topical “analgesics.” (MK)<br />

Corticosteroid injection. (MK)<br />

12. Indications for and efficacy <strong>of</strong> intra-articular corticosteroid injections. (MK)<br />

B. SKILLS: Students should be able to demonstrate specific skills, including:<br />

1. History-taking skills: Students should be able to obtain, document, and present an ageappropriate<br />

medical history that differentiates among etiologies <strong>of</strong> disease, including:<br />

Delineation <strong>of</strong> the specific features <strong>of</strong> the pain. (PC, CS)<br />

Presence <strong>of</strong> stiffness, swelling, warmth, redness. (PC, CS)<br />

Symptoms <strong>of</strong> instability, locking, clicking/popping, and weakness. (PC, CS)<br />

History <strong>of</strong> trauma, new activities, repetitive motion. (PC, CS)<br />

Impact on the patient’s ability to carry out activities <strong>of</strong> daily living. (PC, CS)<br />

2. Physical exam skills: Students should be able to perform a physical exam to establish the<br />

diagnosis and severity <strong>of</strong> disease, including:<br />

Examination <strong>of</strong> the knee, including:<br />

o Inspection. (PC)<br />

o Palpation. (PC)<br />

o Range <strong>of</strong> motion. (PC)<br />

o Gait assessment. (PC)<br />

o Evaluation for effusion. (PC)<br />

o Assessment <strong>of</strong> ligamentous and cartilaginous stability. (PC)<br />

3 Differential diagnosis: Students should be able to generate a prioritized differential diagnosis<br />

recognizing specific history and physical exam findings that suggest a specific etiology for<br />

knee pain. (PC, MK)<br />

4. Laboratory interpretation: Students should be able to recommend when to order diagnostic<br />

and laboratory tests and be able to interpret them, both prior to and after initiating treatment<br />

based on the differential diagnosis, including consideration <strong>of</strong> test cost and performance<br />

characteristics as well as patient preferences.<br />

Laboratory and diagnostic tests should include, when appropriate:<br />

Synovial fluid analysis. (PC, MK)<br />

Students should be able to define the indications for and interpret (with consultation) the<br />

results <strong>of</strong>:<br />

Plain radiographs <strong>of</strong> the knee. (PC, MK)<br />

CT and MRI <strong>of</strong> the knee. (PC, MK)<br />

110


5. Communication skills: Students should be able to:<br />

Communicate the diagnosis, treatment plan, and subsequent follow-up to patients. (PC,<br />

CS)<br />

Elicit questions from the patient and his or her family about the management plan. (PC, CS)<br />

6. Basic and advanced procedure skills: Students should be able to:<br />

Assist in the performance <strong>of</strong> an arthrocentesis and intra-articular corticosteroid injection.<br />

(PC)<br />

7. Management skills: Students should able to develop an appropriate evaluation and treatment<br />

plan for patients that includes:<br />

Determining when to perform an arthrocentesis. (PC, MK)<br />

Prescribing simple, nonmedicinal symptomatic measures such as rest, ice/heat,<br />

compression, and elevation. (PC, MK)<br />

Prescribing physical therapy and assistive devices (PC, MK)<br />

Prescribing exercise. (PC, MK)<br />

Counseling patients regarding weight loss. (PC, MK)<br />

Prescribing non-narcotic analgesics and anti-inflammatory agents. (PC, MK)<br />

Determining when to prescribe narcotic analgesics. (PC, MK)<br />

Determining when to prescribe intra-articular corticosteroid injection. (PC, MK)<br />

Determining when to obtain consultation from an orthopedic surgeon and rheumatologist.<br />

(PC, MK)<br />

Using a cost-effective approach based on the differential diagnosis. (PC, SBP)<br />

Accessing and utilizing appropriate information systems and resources to help delineate<br />

issues related to knee pain. (PC, PLI)<br />

Incorporating patient preferences. (PC, P)<br />

C. ATTITUDES AND PROFESSIONAL BEHAVIORS: Students should be able to:<br />

1. Demonstrate commitment to using risk-benefit, cost-benefit, and evidence-based<br />

considerations in the selection diagnostic and therapeutic interventions for knee pain. (PLI, P)<br />

2. Recognize the importance <strong>of</strong> patient preferences when selecting among diagnostic<br />

and therapeutic options for knee pain. (P)<br />

3. Respond appropriately to patients who are nonadherent to treatment for knee pain.<br />

(CS, P)<br />

4. Appreciate the impact chronic knee pain has on a patient’s quality <strong>of</strong> life,<br />

psychological well-being, ability to work, and the family. (P)<br />

5. Recognize the importance <strong>of</strong> and demonstrate a commitment to the utilization <strong>of</strong><br />

other health care pr<strong>of</strong>essions in the treatment <strong>of</strong> knee pain. (P, SBP)<br />

6. Appreciate the difficulty patients with limited mobility have in achieving weight loss.<br />

(P)<br />

7. Demonstrate an appropriate attitude in managing patients with chronic pain. (P)<br />

D. REFERENCES:<br />

Cush JJ, Lipsky PE. (2005). Approach to articular and musculoskeletal disorders. In<br />

Kasper DL, Braunwald EB, Fauci AS, Hauser SL, Longo DL, Jameson JL, eds.<br />

111


Harrison’s Principles <strong>of</strong> Internal <strong>Medicine</strong>. 16 th ed. New York, NY: McGraw-Hill;<br />

2005:2029-36.<br />

Brandt KD. (2005). Osteoarthritis. In Kasper DL, Braunwald EB, Fauci AS, Hauser SL,<br />

Longo DL, Jameson JL, eds. Harrison’s Principles <strong>of</strong> Internal <strong>Medicine</strong>. 16 th ed. New<br />

York, NY: McGraw-Hill; 2005:2036-45.<br />

Principles <strong>of</strong> diagnosis and management: Pattern recognition in arthritis. In Klippel JH,<br />

Dieppe PA, Ferri FF, eds. Primary Care Rheumatology. St. Louis, MO: Mosby; 1999:11-<br />

7.<br />

Regional pain and monoarticular disorders: Pain in the knee. (1999). In Klippel JH,<br />

Dieppe PA, Ferri FF, eds. Primary Care Rheumatology. St. Louis, MO: Mosby; 1999:99-<br />

107.<br />

DeHaven KE. Knee and lower leg. In Greene WB, ed. Essentials <strong>of</strong> Musculoskeletal<br />

Care. 2 nd ed. Rosemont, IL: American Academy <strong>of</strong> Orthopedic Surgeons; 2001:341-405.<br />

112


TRAINING PROBLEM #14: RASH<br />

RATIONALE:<br />

Rash is an extremely common complaint. It may be the manifestation <strong>of</strong> a primary cutaneous<br />

disorder or secondary to a systemic condition. Internists see many patients with both and,<br />

therefore, must be acquainted with the diagnosis and management.<br />

PREREQUISITES:<br />

Prior knowledge, skills, and attitudes acquired during the pre-<strong>clerkship</strong> experience should include:<br />

Ability to perform a complete medical history and physical exam.<br />

Ability to communicate with patients <strong>of</strong> diverse backgrounds.<br />

Anatomy, physiology, and pathophysiology <strong>of</strong> the skin.<br />

Pharmacology <strong>of</strong> glucocorticoids, antifungals, antibiotics, benzoyl peroxide, salicylic acid, and<br />

retinoids and derivatives.<br />

SPECIFIC LEARNING OBJECTIVES:<br />

A. KNOWLEDGE: Students should be able to define, describe, and discuss:<br />

3. The standard nomenclature used to describe rashes (macule, patch, papule, nodule, plaque,<br />

vesicle, pustule, bulla, cyst, wheal, telangiectasia, petechia, purpura, erosion, ulcer). (MK)<br />

4. The morphologic features used to describe potentially malignant skin lesions (Asymmetry,<br />

Border, Color, Diameter, Dynamic i.e. changing, Elevation, and Enlargement, “ABCDE”). (MK)<br />

5. The significance <strong>of</strong> focal, organ-based, and constitutional signs and symptoms in the context <strong>of</strong><br />

a rash (e.g. rash and fever, rash and arthritis, rash and renal failure). (MK)<br />

4. The differential diagnosis, pathophysiology, and typical presentations <strong>of</strong> the common causes <strong>of</strong><br />

eczematous dermatoses:<br />

Atopic dermatitis. (MK)<br />

Contact dermatitis. (MK)<br />

Stasis dermatitis. (MK)<br />

Seborrheic dermatitis. (MK)<br />

5. The differential diagnosis, pathophysiology, and typical presentations <strong>of</strong> the common causes <strong>of</strong><br />

maculopapular eruptions:<br />

viral exanthems. (MK)<br />

bacterial exanthems. (MK)<br />

erythema multiforme. (MK)<br />

6. The differential diagnosis, pathophysiology, and typical presentations <strong>of</strong> the common causes <strong>of</strong><br />

papulosquamous dermatoses:<br />

Psoriasis. (MK)<br />

Pityriasis rosea. (MK)<br />

7. The differential diagnosis, pathophysiology, and typical presentations <strong>of</strong> the common causes <strong>of</strong><br />

cutaneous infections:<br />

Impetigo. (MK)<br />

Cellulitis. (MK)<br />

Folliculitis. (MK)<br />

Dermatophytosis (tinea corporis, tinea capitis, tinea cruris, tinea pedis, onychomycosis).<br />

(MK)<br />

113


Tinea versicolor. (MK)<br />

Candidiasis. (MK)<br />

Condylomata. (MK)<br />

Herpes zoster. (MK)<br />

8. The prevention <strong>of</strong> community acquisition <strong>of</strong> Methicillin-resistant Staphylococcus aureus<br />

(MRSA), including good hygiene practices:<br />

Keeping hands clean by washing thoroughly with soap and water or using an alcohol-based<br />

sanitizer. (MK)<br />

Keeping cuts and scrapes clean and covered with a bandage until healed. (MK)<br />

Avoiding contact with other people’s wounds or bandages. (MK)<br />

Avoiding sharing personal items such as towels and razors. (MK)<br />

9. The differential diagnosis, pathophysiology, and typical presentations <strong>of</strong> the common causes <strong>of</strong><br />

pustular diseases:<br />

Acne. (MK)<br />

Rosacea. (MK)<br />

10. The differential diagnosis, pathophysiology, and typical presentations <strong>of</strong> the common causes <strong>of</strong><br />

cutaneous ulcers.<br />

Venous insufficiency. (MK)<br />

Peripheral arterial disease. (MK)<br />

Neuropathic. (MK)<br />

11. The significance <strong>of</strong> palpable purpura and other cutaneous findings <strong>of</strong> vasculitis. (MK)<br />

12. The differential diagnosis, pathophysiology, and typical presentations <strong>of</strong> the common causes <strong>of</strong><br />

urticaria and angioedema. (MK)<br />

13. The differential diagnosis, pathophysiology, and typical presentations <strong>of</strong> drug eruptions. (MK)<br />

14. The differential diagnosis, pathophysiology, and typical presentations <strong>of</strong> the common causes <strong>of</strong><br />

benign neoplasms and hyperplasias:<br />

Seborrheic keratosis. (MK)<br />

Epidermoid cyst. (MK)<br />

15. The differential diagnosis, pathophysiology, and typical presentations <strong>of</strong> the common causes <strong>of</strong><br />

premalignant lesions and malignancies:<br />

Actinic keratosis. (MK)<br />

Basal cell carcinoma. (MK)<br />

Squamous cell carcinoma. (MK)<br />

Malignant melanoma. (MK)<br />

16. The differential diagnosis, pathophysiology, and typical presentations <strong>of</strong> the cutaneous<br />

manifestations <strong>of</strong> sexually transmitted diseases.<br />

Syphilis. (MK)<br />

Disseminated gonorrhea infection. (MK)<br />

Human papilloma virus. (MK)<br />

Herpes simplex virus. (MK)<br />

17. The differential diagnosis, pathophysiology, and typical presentations <strong>of</strong> the cutaneous<br />

manifestations <strong>of</strong> internal/systemic diseases. (MK)<br />

18. The general indications for skin biopsy. (MK)<br />

B. SKILLS: Students should be able to demonstrate specific skills, including:<br />

1. History-taking skills: Students should be able to obtain, document, and present an ageappropriate<br />

medical history that differentiates among etiologies <strong>of</strong> disease, including:<br />

Evolution (site <strong>of</strong> onset, manner <strong>of</strong> spread, duration). (PC, CS)<br />

114


Symptoms associated with the rash (pruritis, pain, photosensitivity, malaise, fever,<br />

arthralgias). (PC, CS)<br />

Past medical history <strong>of</strong> systemic diseases known to have cutaneous manifestation. (PC,<br />

CS)<br />

Sexual history. (PC, CS)<br />

Medication usage and allergies. (PC, CS)<br />

Skin care product usage. (PC, CS)<br />

Chemical skin exposure. (PC, CS)<br />

Sun exposure. (PC, CS)<br />

Travel history. (PC, CS)<br />

2. Physical exam skills: Students should be able to perform a physical exam to establish the<br />

diagnosis and severity <strong>of</strong> disease including:<br />

Description <strong>of</strong> the type <strong>of</strong> primary skin lesion (macule, patch, papule, nodule, plaque,<br />

vesicle, pustule, bulla, cyst, wheal, telangiectasia, petechia, purpura, erosion, ulcer). (PC)<br />

Description <strong>of</strong> the shape, margination, color, arrangement, and distribution <strong>of</strong> the individual<br />

lesions. (PC)<br />

Describe potentially malignant lesions in terms <strong>of</strong> Asymmetry, Border, Color, Diameter,<br />

Elevation, and Enlargement (“ABCDE”). (PC)<br />

Presence <strong>of</strong> exudates: dry (crust) or wet (weeping) exudates. (PC)<br />

Presence <strong>of</strong> scale or lichenification. (PC)<br />

Palpation <strong>of</strong> lesions for consistency, alteration <strong>of</strong> temperature, mobility, and tenderness.<br />

(PC)<br />

3. Differential diagnosis: Students should be able to generate a prioritized differential<br />

diagnosis recognizing specific history and physical exam findings that suggest a specific<br />

etiology for a rash. (PC, MK)<br />

4. Laboratory interpretation: Students should be able to recommend when to order<br />

diagnostic and laboratory tests and be able to interpret them, both prior to and after initiating<br />

treatment based on the differential diagnosis, including consideration <strong>of</strong> test cost and<br />

performance characteristics as well as patient preferences.<br />

Laboratory and diagnostic tests should include, when appropriate:<br />

KOH preparation. (PC, MK)<br />

CBC with differential. (PC, MK)<br />

RPR and VDRL. (PC, MK)<br />

Bacterial culture. (PC, MK)<br />

Students should be able to define the indications for and interpret (with consultation) the<br />

significance <strong>of</strong> the results <strong>of</strong>:<br />

• Skin biopsy. (PC, MK)<br />

5. Communication skills: Students should be able to:<br />

Explain the dangers <strong>of</strong> excess sun exposure. (PC, CS)<br />

Communicate the diagnosis, treatment plan, and subsequent follow-up to the patient and<br />

his or her family. (PC, CS)<br />

Elicit questions from the patient and his or her family about the management plan. (PC, CS)<br />

Counsel patients regarding the prevention <strong>of</strong> community acquisition <strong>of</strong> MRSA. (PC, CS)<br />

6. Basic and advanced procedural skills: Students should be able to:<br />

Perform a skin scraping and KOH preparation. (PC)<br />

7. Management skills: Students should able to develop an appropriate evaluation and<br />

treatment plan for patients that includes:<br />

Determining when to perform a skin scraping and KOH preparation. (MK, PC)<br />

Determining when to obtain tests appropriate for the diagnosis <strong>of</strong> systemic medical<br />

115


conditions suspected as the cause <strong>of</strong> rash. (MK, PC)<br />

Prescribing a simple hypoallergenic skin care regimen. (MK, PC)<br />

Prescribing appropriate moisturizing/emollient treatment. (MK, PC)<br />

Discussing the importance <strong>of</strong> and prescribing sunscreen use. (PC, MK, CS)<br />

Prescribing appropriate treatment for eczematous dermatoses, mild psoriasis, common<br />

cutaneous skin infections, acne, rosacea, venous stasis dermatitis and ulcers, and common<br />

drug eruptions. (PC, MK)<br />

Determining when to obtain a consultation from a dermatologist. (PC)<br />

Using a cost-effective approach based on the differential diagnosis. (PC, SBP)<br />

Accessing and utilizing appropriate information systems and resources to help delineate<br />

issues related to common dermatologic complaints. (PC, PLI)<br />

Incorporating patient preferences. (PC, P)<br />

C. ATTITUDES AND PROFESSIONAL BEHAVIORS: Students should be able to:<br />

3. Demonstrate commitment to using risk-benefit, cost-benefit, and evidence-based<br />

considerations in the selection diagnostic and therapeutic interventions for rashes. (PLI, P)<br />

4. Recognize the importance <strong>of</strong> patient preferences when selecting among diagnostic and<br />

therapeutic options for rashes. (P)<br />

5. Appreciate the impact rashes have on a patient’s quality <strong>of</strong> life, well-being, ability to work,<br />

and the family. (P)<br />

D. REFERENCES:<br />

Drage LA. Life-threatening rashes: dermatologic signs <strong>of</strong> four infectious diseases. Mayo<br />

Clin Proc. 199;74:68-72.<br />

Williams HC. Clinical practice. Atopic dermatitis. N Engl J Med. 2005;352:2314-24.<br />

Abbasi NR. Shaw HM. Rigel DS. Friedman RJ. McCarthy WH. Osman I. Kopf AW.<br />

Polsky D. Early diagnosis <strong>of</strong> cutaneous melanoma: revisiting the ABCD criteria. JAMA.<br />

2004;292:2771-6.<br />

James WD. Clinical practice. Acne. N Engl J Med. 2005;352:1463-72.<br />

Wolff K, Johnson AJ, Suurmond R. Fitzpatrick’s Color Atlas & Synopsis <strong>of</strong> Clinical<br />

Dermatology. 4 th ed. New York, NY; 2005.<br />

American Academy <strong>of</strong> Dermatology<br />

Medical Student Core Curriculum<br />

www.aad.org/pr<strong>of</strong>essionals/Residents/MedStudCoreCurr/MedStudCoreC urr.htm<br />

Community-Associated MRSA<br />

Division <strong>of</strong> Healthcare Quality Promotion<br />

National Center for Infectious Diseases<br />

Centers for Disease Control and Prevention<br />

U.S. <strong>Department</strong> <strong>of</strong> Health and Human Services<br />

www.cdc.gov/ncidod/dhqp/ar_mrsa_ca.html<br />

116


TRAINING PROBLEM #15: UPPER RESPIRATORY COMPLAINTS<br />

RATIONALE:<br />

Upper respiratory tract infections (URIs) are some <strong>of</strong> the most common problems for which patients<br />

seek medical attention. Many patients inappropriately receive antibiotic therapy for these mostly<br />

viral infections.<br />

PREREQUISITES:<br />

Prior knowledge, skills, and attitudes acquired during the pre-<strong>clerkship</strong> experience should include:<br />

Ability to perform a complete medical history and physical exam.<br />

Ability to communicate with patients <strong>of</strong> diverse backgrounds.<br />

Anatomy and physiology <strong>of</strong> the upper airway, Eustachian tubes, and sinuses.<br />

Anatomy and physiology <strong>of</strong> the respiratory system.<br />

Pathogenesis and pathophysiology <strong>of</strong> upper respiratory tract diseases.<br />

Microbial pathogens associated with upper respiratory tract infections.<br />

Pharmacology <strong>of</strong> antibiotics.<br />

SPECIFIC LEARNING OBJECTIVES:<br />

A. KNOWLEDGE: Students should be able to define, describe, and discuss:<br />

1. A rational approach to the common URIs: nasal congestion, rhinorrhea, facial pain/tenderness,<br />

cough, sputum production, sore throat, and ear pain. (MK)<br />

2. Common constitutional symptoms that accompany URIs: generalized weakness, fatigue,<br />

malaise, headache, mild myalgias, and modest fever. (MK)<br />

3. The microbiology <strong>of</strong> URIs, highlighting the relative frequencies <strong>of</strong> viral and bacterial etiologies.<br />

(MK)<br />

4. The most common microbiologic agents that cause the common URIs. (MK)<br />

5. The pathophysiology and typical clinical presentation <strong>of</strong> the common URIs:<br />

Common cold. (MK)<br />

Acute bronchitis. (MK)<br />

Pharyngitis. (MK)<br />

Acute sinusitis. (MK)<br />

Otitis media. (MK)<br />

6. The pathophysiologic similarities between the common cold and acute sinusitis. (MK)<br />

7. The clinical features and microbiology <strong>of</strong> acute compared to chronic sinusitis. (MK)<br />

8. The pathophysiology and symptomatology <strong>of</strong> allergic rhinitis and the clinical features that may<br />

help differentiate it from the common cold and acute sinusitis. (MK)<br />

9. The clinical features that may help differentiate the common URIs from influenza. (MK)<br />

10. The pathophysiology and clinical features <strong>of</strong> acute compared to chronic bronchitis. (MK)<br />

11. The pathophysiology and clinical features <strong>of</strong> acute bronchitis compared to pneumonia. (MK)<br />

12. The pathophysiology and clinical features <strong>of</strong> otitis media and Eustachian tube malfunction.<br />

(MK)<br />

13. The signs and symptoms that may help distinguish viral from bacterial pharyngitis. (MK)<br />

14. Symptomatic treatment for URIs and the major side effects/contraindications for these<br />

treatments, including:<br />

Decongestants. (MK)<br />

Non-selective antihistamines. (MK)<br />

117


Mucolytics. (MK)<br />

Cough suppressants. (MK)<br />

Pain relievers/fever reducers. (MK)<br />

15. The general role <strong>of</strong> antibiotics in the treatment <strong>of</strong> URIs and specific evidence-based indications<br />

for them. (MK)<br />

16. The basic elements <strong>of</strong> the treatment <strong>of</strong> allergic rhinitis. (MK)<br />

17. The use <strong>of</strong> antiviral agents in the prophylaxis and treatment <strong>of</strong> influenza. (MK)<br />

B. SKILLS: Students should be able to demonstrate specific skills, including:<br />

1. History-taking skills: Students should be able to obtain, document, and present an ageappropriate<br />

medical history, that differentiates among etiologies <strong>of</strong> disease, including:<br />

The predominant symptom (nasal congestion/rhinorrhea, purulent nasal discharge with<br />

facial pain/tenderness, sore throat, cough with or without sputum, sore throat or ear pain).<br />

(PC, CS)<br />

Constitutional symptoms. (PC, CS)<br />

Symptoms <strong>of</strong> potential pneumonia. (PC, CS)<br />

History <strong>of</strong> or symptoms <strong>of</strong> serious cardiopulmonary diseases (e.g. asthma, chronic<br />

obstructive pulmonary disease, congestive heart failure) that may alter the treatment plan.<br />

(PC, CS)<br />

2. Physical exam skills: Students should be able to perform a physical exam to establish the<br />

diagnosis and severity <strong>of</strong> disease, including:<br />

Examination <strong>of</strong> the nasal cavity, pharynx, and sinuses. (PC)<br />

Otoscopic examination. (PC)<br />

Evaluation <strong>of</strong> the head and neck for lymphadenopathy. (PC)<br />

Auscultation <strong>of</strong> the lungs to distinguish pulmonary consolidation, pleural effusion,<br />

pulmonary congestion, and chronic obstructive pulmonary disease. (PC)<br />

3. Differential diagnosis: Students should be able to generate a prioritized differential diagnosis<br />

recognizing specific history and physical exam findings that suggest a specific etiology <strong>of</strong><br />

upper respiratory complaints:<br />

Common cold. (PC, MK)<br />

Acute sinusitis. (PC, MK)<br />

Chronic sinusitis. (PC, MK)<br />

Allergic rhinitis. (PC, MK)<br />

Pharyngitis. (PC, MK)<br />

Otitis media. (PC, MK)<br />

Otitis externa. (PC, MK)<br />

Acute bronchitis. (PC, MK)<br />

Chronic bronchitis. (PC, MK)<br />

Influenza. (PC, MK)<br />

Pneumonia. (PC, MK)<br />

Infectious mononucleosis. (PC, MK)<br />

4. Laboratory interpretation: Students should be able to recommend when to order diagnostic<br />

and laboratory tests and be able to interpret them, both prior to and after initiating treatment<br />

based on the differential diagnosis, including consideration <strong>of</strong> test cost and performance<br />

characteristics as well as patient preferences Laboratory and diagnostic tests should include,<br />

when appropriate:<br />

CBC with differential. (PC)<br />

Rapid strep test. (PC)<br />

118


Throat culture. (PC)<br />

Chest radiograph. (PC)<br />

PFTs. (PC)<br />

Monospot/heterophile antibody. (PC)<br />

5. Communication skills: Students should be able to:<br />

Communicate the diagnosis, treatment plan, and subsequent follow-up to the patient and<br />

his or her family. (CS)<br />

Elicit questions from the patient and his or her family about the management plan. (CS)<br />

Explain the microbiologic origin <strong>of</strong> most URIs and why antibiotics are generally ineffective.<br />

(CS)<br />

Explain the importance <strong>of</strong> antimicrobial resistance. (CS)<br />

6. Basic and advanced procedure skills:<br />

• Throat culture. (PC)<br />

7. Management skills: Students should able to develop an appropriate evaluation and<br />

treatment plan for patients that includes:<br />

Determining when to obtain a chest radiograph. (PC, MK)<br />

Determining when to prescribe antibiotics. (PC, MK)<br />

Selecting the most appropriate antibiotic for acute bacterial sinusitis, streptococcal<br />

pharyngitis, and bacterial otitis media. (PC, MK)<br />

Prescribing symptomatic treatments. (PC, MK)<br />

Determining when to obtain consultation from an allergist, otolaryngologist, or<br />

pulmonologist. (PC, SBP)<br />

Using a cost-effective approach based on the differential diagnosis. (PC, SBP)<br />

Accessing and utilizing appropriate information systems and resources to help delineate<br />

issues related to URIs. (PC, PLI)<br />

Incorporating patient preferences. (PC, P)<br />

C. ATTITUDES AND PROFESSIONAL BEHAVIORS: Students should be able to:<br />

1 Demonstrate commitment to using risk-benefit, cost-benefit, and evidence-based<br />

considerations in the selection diagnostic and therapeutic interventions for common URI complaints.<br />

(P, PLI)<br />

2 Appreciate the impact common URI complaints have on a patient’s quality <strong>of</strong> life, well-being,<br />

ability to work, and the family. (P)<br />

3 Discuss the patient’s perspective regarding the use <strong>of</strong> antibiotics for URIs. (CS, P)<br />

4 Discuss the role physicians play in the over-prescribing <strong>of</strong> antibiotics for URIs. (P)<br />

5 Discuss the importance <strong>of</strong> antimicrobial resistance from the point <strong>of</strong> view <strong>of</strong> the individual and<br />

society at large. (P)<br />

D. REFERENCES:<br />

Arroll B, Kenealy T. Antibiotics for the common cold and acute purulent rhinitis.<br />

Cochrane Database Syst Rev. 2005;(3):CD000247.<br />

Bisno AL. Acute pharyngitis. N Engl J Med. 2001;344:205-11.<br />

Del Mar CB, Glasziou PP, Spinks AB. Antibiotics for sore throat. Cochrane Database<br />

Syst Rev. 2004;(2):CD000023.<br />

Metlay JP, Kapoor WN, Fine MJ. Does this patient have community-acquired<br />

pneumonia? Diagnosing pneumonia by history and physical examination. JAMA.<br />

1997;278:1440-5.<br />

119


Smucny J, Fahey T, Becker L, et al. Antibiotics for acute bronchitis. Cochrane<br />

Database Syst Rev. 2004;(4):CD000245.<br />

Snow V, Mottur-Pilson C, Gonzales R, et al. Principles <strong>of</strong> appropriate antibiotic use for<br />

treatment <strong>of</strong> acute bronchitis in adults. Ann Intern Med. 2001;134:518-20.<br />

Guidelines for the Control <strong>of</strong> Pertussis Outbreaks<br />

National Immunization Program<br />

Centers for Disease Control and Prevention<br />

U.S. <strong>Department</strong> <strong>of</strong> Health and Human Services<br />

www.cdc.gov/nip/publications/pertussis/guide.htm<br />

Get Smart. Know When Antibiotics Work<br />

National Campaign for Appropriate Antibiotic Use<br />

Division <strong>of</strong> Bacterial and Mycotic Diseases<br />

National Center for Infectious Diseases<br />

Centers for Disease Control and Prevention<br />

U.S. <strong>Department</strong> <strong>of</strong> Health and Human Services<br />

www.cdc.gov/drugresistance/community/<br />

120


TRAINING PROBLEM #16: ACUTE MYOCARDIAL INFARCTION<br />

RATIONALE:<br />

Cardiovascular disease is the number one killer <strong>of</strong> Americans. Many associated risk factors are<br />

quite modifiable. Proper urgent management <strong>of</strong> acute myocardial infarctions significantly reduces<br />

mortality.<br />

PREREQUISITES:<br />

Prior knowledge, skills, and attitudes acquired during the pre-clinical experience should include:<br />

Ability to perform a complete medical history and physical.<br />

Ability to communicate with patients <strong>of</strong> diverse backgrounds.<br />

Anatomy and physiology <strong>of</strong> the heart and coronary vessels.<br />

Risk factors for and pathogenesis/pathophysiology <strong>of</strong> atherosclerosis.<br />

Pharmacology <strong>of</strong> aspirin, morphine, nitroglycerine, heparin, antiplatelet agents, thrombolytic<br />

agents, beta-blockers, angiotensin converting enzyme inhibitors (ACE-I), angiotensin II<br />

receptor blockers (ARB), and HMG-CoA reductase inhibitors.<br />

SPECIFIC LEARNING OBJECTIVES:<br />

A. KNOWLEDGE: Students should be able to define, describe, and discuss:<br />

1. The primary and secondary prevention <strong>of</strong> ischemic heart disease through the reduction <strong>of</strong><br />

cardiovascular risk factors (e.g. controlling hypertension and dyslipidemia, aggressive diabetes<br />

management, avoiding tobacco, and aspirin prophylaxis). (MK)<br />

2. The basic principles <strong>of</strong> the role <strong>of</strong> genetics in CAD. (MK)<br />

3. Pathogenesis, signs, and symptoms <strong>of</strong> the acute coronary syndromes:<br />

Unstable angina. (MK)<br />

Non-ST-elevation myocardial infarction (NSTEMI). (MK)<br />

ST-elevation myocardial infarction (STEMI). (MK)<br />

4. Atypical presentations <strong>of</strong> cardiac ischemia/infraction. (MK)<br />

5. The typical clinical course <strong>of</strong> the acute coronary syndromes. (MK)<br />

6. ECG findings and macromolecular markers (myoglobin, CK-MB, Troponin-I, Troponin-T) <strong>of</strong><br />

acute ischemia/MI. (MK)<br />

7. The utility <strong>of</strong> echocardiography in acute MI. (MK)<br />

8. The importance <strong>of</strong> monitoring for and immediate treatment <strong>of</strong> ventricular fibrillation in acute MI.<br />

(MK)<br />

9. Therapeutic options for acute MI and how they may differ for NSTEMI and STEMI, including:<br />

Aspirin. (MK)<br />

Morphine. (MK)<br />

Nitroglycerine. (MK)<br />

Oxygen. (MK)<br />

Heparin. (MK)<br />

Antiplatelet agents (glycoprotein IIb/IIIa inhibitors). (MK)<br />

Beta-blockers. (MK)<br />

ACE-I/ARB. (MK)<br />

HMG-CoA reductase inhibitors. (MK)<br />

Thrombolytic agents. (MK)<br />

121


Emergent cardiac catheterization with percutaneous coronary intervention. (MK)<br />

10. Pathogenesis, signs, and symptoms <strong>of</strong> the complications <strong>of</strong> acute MI, including arrhythmias,<br />

reduced ventricular function, cardiogenic shock, pericarditis, papillary muscle<br />

dysfunction/rupture, acute valvular dysfunction, and cardiac free wall rupture. (MK)<br />

11. The general approach to the evaluation and treatment <strong>of</strong> ventricular tachycardia and fibrillation.<br />

(MK)<br />

12. The importance <strong>of</strong> post-MI risk stratification, including the burden <strong>of</strong> residual coronary disease<br />

and assessment <strong>of</strong> left ventricular function. (MK)<br />

13. Basic principles <strong>of</strong> cardiac rehabilitation. (MK)<br />

14. Indications for coronary artery bypass grafting (CABG). (MK)<br />

15. The Centers for Medicare & Medicaid Services (CMS) and the Joint Commission on the<br />

Accreditation <strong>of</strong> Healthcare Organizations (JCAHO) quality measures for acute MI treatment.<br />

(MK, PLI, SBP)<br />

B. SKILLS: Students should be able to demonstrate specific skills, including:<br />

1. History-taking skills: Students should be able to obtain, document, and present an ageappropriate<br />

medical history that differentiates among etiologies <strong>of</strong> disease, including:<br />

Cardiac risk factors. (PC, CS)<br />

Location, duration, intensity, exacerbating/ameliorating factors, radiation <strong>of</strong> chest pain. (PC,<br />

CS)<br />

Symptoms associated with chest pain (e.g. nausea, emesis, dyspnea, diaphoresis,<br />

palpitations, dizziness, syncope, heartburn belching, etc.). (PC, CS)<br />

2. Physical exam skills: Students should be able to perform a physical exam to establish the<br />

diagnosis and severity <strong>of</strong> disease including:<br />

Recognition <strong>of</strong> dyspnea and anxiety. (PC)<br />

Accurate measurement <strong>of</strong> vital signs. (PC)<br />

Examination <strong>of</strong> the heart and vascular system. (PC)<br />

Examination <strong>of</strong> the lungs. (PC)<br />

3. Differential diagnosis: Students should be able to generate a prioritized differential<br />

diagnosis recognizing specific history and physical exam findings that suggest a specific<br />

etiology <strong>of</strong> chest pain:<br />

Stable angina. (PC, MK)<br />

Coronary vasospasm. (PC, MK)<br />

Unstable angina. (PC, MK)<br />

Acute MI. (PC, MK)<br />

Pericarditis. (PC, MK)<br />

Aortic dissection. (PC, MK)<br />

Pulmonary embolism. (PC, MK)<br />

Other noncardiac causes <strong>of</strong> chest pain. (PC, MK)<br />

4. Laboratory interpretation: Students should be able to recommend when to order diagnostic<br />

and laboratory tests and be able to interpret them, both prior to and after initiating treatment<br />

based on the differential diagnosis, including consideration <strong>of</strong> test cost and performance<br />

characteristics as well as patient preferences<br />

Laboratory and diagnostic tests should include, when appropriate:<br />

ECG. (PC, MK)<br />

Chest radiograph. (PC, MK)<br />

Macromolecular markers (myoglobin, CK-MB, Troponin-I, Troponin- T). (PC, MK)<br />

Students should be able to define the indications for and interpret (with consultation) the<br />

122


esults <strong>of</strong>:<br />

Echocardiogram. (PC, MK)<br />

Cardiac stress testing. (PC, MK)<br />

Coronary angiography. (PC, MK)<br />

5. Communication skills: Students should be able to:<br />

Communicate the diagnosis, treatment plan, and subsequent follow-up to patients. (PC,<br />

CS)<br />

Elicit questions from the patient and his or her family about the diagnostic and management<br />

plan. (PC, CS)<br />

Educate patients about modifying cardiac risk factors. (PC, CS)<br />

6. Management skills: Students should able to develop an appropriate evaluation and<br />

treatment plan for patients that includes:<br />

Medical management <strong>of</strong> acute MI. (PC, MK)<br />

CCU monitoring. (PC, MK)<br />

Indications for and complications <strong>of</strong> thrombolytic therapy, cardiac catheterization with<br />

percutaneous coronary intervention, and CABG. (PC, MK)<br />

Proper pre-discharge risk stratification. (PC, MK)<br />

Secondary risk factor modification. (PC, MK)<br />

Determining when to obtain consultation from a cardiologist and cardiothoracic surgeon.<br />

(PC, SBP)<br />

Using a cost-effective approach based on the differential diagnosis. (PC, SBP)<br />

Accessing and utilizing appropriate information systems and resources to help delineate<br />

issues related to acute MI. (PC, PLI)<br />

Incorporating patient preferences. (PC, P)<br />

C. ATTITUDES AND PROFESSIONAL BEHAVIORS: Students should be able to:<br />

1. Demonstrate a commitment to meeting national quality standards for the care <strong>of</strong><br />

patient with acute MI. (P, PLI, SBP)<br />

2. Demonstrate commitment to using risk-benefit, cost-benefit, and evidence-based<br />

considerations in the selection diagnostic and therapeutic interventions for acute MI. (PLI, P)<br />

3. Recognize the importance <strong>of</strong> patient preferences when selecting among diagnostic<br />

and therapeutic options for acute MI. (P)<br />

4. Demonstrate ongoing commitment to self-directed learning regarding acute MI. (PLI,<br />

P)<br />

5. Appreciate the impact acute MI has on a patient’s quality <strong>of</strong> life, well-being, ability to<br />

work, and the family. (P)<br />

6. Recognize the importance <strong>of</strong> and demonstrate a commitment to the utilization <strong>of</strong><br />

other healthcare pr<strong>of</strong>essionals in the treatment <strong>of</strong> acute MI. (P, SBP)<br />

D. REFERENCES:<br />

ACC/AHA 2002 guideline update for the management <strong>of</strong> patients with chronic stable<br />

angina--summary article: a report <strong>of</strong> the American College <strong>of</strong> Cardiology/American<br />

Heart Association Task Force on practice guidelines (Committee on the Management<br />

<strong>of</strong> Patients with Chronic Stable Angina). J Am Coll Cardiol 2003; 41:159-68.<br />

www.acc.org/clinical/topic/topic.htm#guidelines<br />

ACC/AHA 2002 guideline update for the management <strong>of</strong> patients with unstable<br />

angina and non-ST-segment elevation myocardial infarction-summary article: a report<br />

123


<strong>of</strong> the American College <strong>of</strong> Cardiology/American Heart Association task force on<br />

practice guidelines (Committee on the Management <strong>of</strong> Patients With Unstable<br />

Angina). J Am Coll Cardiol. 2002;40:1366-74.<br />

www.acc.org/clinical/topic/topic.htm#guidelines<br />

ACC/AHA guidelines for the management <strong>of</strong> patients with ST-elevation myocardial<br />

infarction--executive summary. A report <strong>of</strong> the American College <strong>of</strong><br />

Cardiology/American Heart Association Task Force on Practice Guidelines (Writing<br />

Committee to revise the 1999 guidelines for the management <strong>of</strong> patients with acute<br />

myocardial infarction). J Am Coll Cardiol. 2004;44:671-719.<br />

Erratum in: J Am Coll Cardiol. 2005;45(8):1376.<br />

www.acc.org/clinical/topic/topic.htm#guidelines<br />

AHA/ACC Scientific Statement: AHA/ACC guidelines for preventing heart attack and<br />

death in patients with atherosclerotic cardiovascular disease: 2001 update: A<br />

statement for healthcare pr<strong>of</strong>essionals from the American Heart Association and the<br />

American College <strong>of</strong> Cardiology. Circulation. 2001;25:1577-9.<br />

www.acc.org/clinical/topic/topic.htm#guidelines<br />

124


TRAINING PROBLEM #17: ACUTE RENAL FAILURE AND<br />

CHRONIC KIDNEY DISEASE<br />

RATIONALE:<br />

Renal disease is a common problem in internal medicine and may manifest with symptoms<br />

referable to the kidney as well as other systems. Patients who go on to end-stage renal disease<br />

have high morbidity and mortality, despite advances in dialysis treatment. Thus, an understanding<br />

<strong>of</strong> chronic kidney disease is useful to all physicians. A rational approach to patients with suspected<br />

or known acute renal failure allows students and clinicians to quickly assess the etiology and initiate<br />

treatment without unnecessary delay in an effort to prevent the development <strong>of</strong> chronic kidney<br />

disease.<br />

PREREQUISITES:<br />

Prior knowledge, skills and attitudes acquired during the pre-<strong>clerkship</strong> experience should include:<br />

Ability to perform a complete medical history and physical exam.<br />

Ability to communicate with patients <strong>of</strong> diverse backgrounds.<br />

Knowledge <strong>of</strong> pathogenesis and pathophysiology <strong>of</strong> acute renal failure and the development <strong>of</strong><br />

chronic kidney disease.<br />

Understanding <strong>of</strong> drugs that can have adverse effect on renal function.<br />

SPECIFIC LEARNING OBJECTIVES:<br />

A. KNOWLEDGE: Students should be able to define, describe and discuss:<br />

1. The distinction between the three major pathophysiologic etiologies for acute renal failure<br />

(ARF):<br />

Decreased renal perfusion (prerenal). (MK)<br />

Intrinsic renal disease (renal). (MK)<br />

Acute renal obstruction (postrenal). (MK)<br />

2. The pathophysiology <strong>of</strong> the major etiologies <strong>of</strong> “prerenal” ARF, including:<br />

Hypovolemia. (MK)<br />

Decreased cardiac output. (MK)<br />

Systemic vasodilation. (MK)<br />

Renal vasoconstriction. (MK)<br />

3. The pathophysiology <strong>of</strong> the major etiologies <strong>of</strong> intrinsic “renal” ARF, including:<br />

Vascular lesions. (MK)<br />

Glomerular lesions. (MK)<br />

interstitial nephritis. (MK)<br />

Intra-tubule deposition/obstruction. (MK)<br />

Acute tubular necrosis (ATN). (MK)<br />

4. The pathophysiology <strong>of</strong> the major etiologies <strong>of</strong> “postrenal” ARF, including:<br />

Urethral (e.g. tumors, calculi, clot, sloughed papillae, retroperitoneal fibrosis,<br />

lymphadenopathy). (MK)<br />

Bladder neck (e.g. tumors, calculi, prostatic hypertrophy or carcinoma, neurogenic). (MK)<br />

Urethral (e.g. stricture, tumors, obstructed indwelling catheters). (MK)<br />

125


5. The pathophysiology and clinical findings <strong>of</strong> uremia. (MK)<br />

6. The natural history, initial evaluation and treatment, and complications <strong>of</strong> ARF. (MK)<br />

7. The most common etiologies <strong>of</strong> chronic kidney disease (CKD):<br />

DM. (MK)<br />

Hypertension. (MK)<br />

Glomerulonephritis. (MK)<br />

Polycystic kidney disease. (MK)<br />

Autoimmune diseases (e.g. systemic lupus erythematosus). (MK)<br />

The staging scheme for CKD. (MK)<br />

8. The significance for proteinuria in CKD. (MK)<br />

9. The use <strong>of</strong> ACE-Is and ARBs in the management <strong>of</strong> CKD. (MK)<br />

10. The importance <strong>of</strong> secondary hyperparathyroidism in CKD. (MK)<br />

11. The pathophysiology <strong>of</strong> anemia in CKD. (MK)<br />

12. The value <strong>of</strong> glycemic and hypertension control in limiting the progression <strong>of</strong> CKD.<br />

(MK)<br />

13. The value <strong>of</strong> CAD risk factor modification in patients with CKD, particularly those<br />

treated with dialysis. (MK)<br />

14. The basic principles <strong>of</strong> renal replacement therapy (e.g., hemodialysis and peritoneal<br />

dialysis) as well as the complications. (MK)<br />

B. SKILLS: Students should be able to demonstrate specific skills, including:<br />

1 History-taking skills: Students should be able to obtain, document, and present an ageappropriate<br />

history that distinguishes among the three major reasons for ARF (pre-renal, renal,<br />

post-renal), including the predisposing conditions, nephrotoxic drugs or agents, and systemic<br />

disease and the major causes <strong>of</strong> CKD. (PC, CS)<br />

2. Physical exam skills: Students should be able to perform a physical examination to establish<br />

the diagnosis and severity <strong>of</strong> disease, including:<br />

The determination <strong>of</strong> a patient’s volume status through estimation <strong>of</strong> the central venous<br />

pressure using the height <strong>of</strong> jugular venous distention and measurement <strong>of</strong> pulse and blood<br />

pressure in the lying/standing position. (PC)<br />

Palpation and percussion <strong>of</strong> the bladder to recognize bladder distention. (PC)<br />

Palpation <strong>of</strong> the prostate. (PC)<br />

Determination <strong>of</strong> the presence <strong>of</strong> pulmonary edema, peripheral edema, ascites, and signs<br />

<strong>of</strong> heart failure. (PC)<br />

Findings consistent with uremia. (PC)<br />

Examination for evidence <strong>of</strong> systemic disease, including but not limited to: skin, joints, and<br />

nails. (PC)<br />

3. Differential diagnosis: Students should be able to generate a differential diagnosis for a<br />

patient with ARF or CKD recognizing specific history, physical exam, and laboratory findings<br />

that suggest a specific etiology. (PC, MK)<br />

4. Laboratory interpretation: Students should be able to recommend when to order diagnostic<br />

and laboratory tests and be able to interpret them, both prior to and after initiating treatment<br />

based on the differential diagnosis, including consideration <strong>of</strong> test cost and performance<br />

characteristics as well as patient preferences. Laboratory and diagnostic tests should include,<br />

when appropriate:<br />

Serum electrolytes, BUN/Cr, calcium, phosphorus. (PC, MK)<br />

Urine sodium. (PC, MK)<br />

Serum and urine osmolality. (PC, MK)<br />

126


Anion gap. (PC, MK)<br />

ABG (PC, MK)<br />

Serum BUN to Cr ratio. (PC, MK)<br />

CBC, ferritin. (PC, MK)<br />

Performing and interpreting a urinalysis, including microscopic examination for casts, red<br />

blood cells, white blood cells, and crystals. (PC, MK)<br />

Calculating fractional excretion <strong>of</strong> sodium and appreciate its usefulness in distinguishing<br />

between pre-renal and intrinsic renal disease. (PC, MK)<br />

Calculating creatinine clearance using the Cockcr<strong>of</strong>t-Gault or MDRD (“modification <strong>of</strong> diet<br />

in renal disease study”) equations. (PC, MK)<br />

Serum parathyroid hormone level. (PC, MK)<br />

ECG findings in hyperkalemia. (PC, MK)<br />

Students should be able to define the indications for and interpret (with consultation) results<br />

<strong>of</strong>:<br />

Renal ultrasonography. (PC, MK)<br />

5. Communication skills: Students should be able to:<br />

Communicate the diagnosis, treatment plan, and subsequent follow-up to the patient and<br />

his or her family. (PC, CS)<br />

Elicit questions from the patient and his or her family about the management plan. (PC, CS)<br />

Counsel patients regarding a renal diet. (PC, CS)<br />

6. Basic and advanced procedure skills: Students should be able to:<br />

Insert a peripheral intravenous catheter. (PC)<br />

Place a urinary catheter. (PC)<br />

Obtain an ABG. (PC)<br />

7. Management skills: Students should be able to develop an appropriate evaluation and<br />

treatment plan for patients, including:<br />

Designing an appropriate management plan for initial management <strong>of</strong> ARF, including<br />

volume management, dietary recommendations, drug dosage alterations, electrolyte<br />

monitoring, and indications for dialysis. (PC, MK)<br />

Developing a management plan to effectively treat HTN and DM. (PC, MK)<br />

Recommending treatment with phosphate binders, calcium replacement, and vitamin D<br />

replacement. (PC, MK)<br />

Recommending treatment for dyslipidemia. (PC, MK)<br />

Recommending treatment for anemia secondary to CKD. (PC, MK)<br />

Recommending acute treatment for hyperkalemia. (PC, MK)<br />

Determining when to obtain consultation from a nephrologist. (PC, MK)<br />

Using a cost-effective approach based on the differential diagnosis. (PC, SBP)<br />

Accessing and utilizing appropriate information systems and resources to help delineate<br />

issues related to renal failure. (PC, PLI)<br />

Incorporating patient preferences. (PC, P)<br />

C. ATTITUDES AND PROFESSIONAL BEHAVIORS: Students should be able to:<br />

3. Demonstrate commitment to using risk-benefit, cost-benefit, and evidence-based considerations in<br />

the selection <strong>of</strong> diagnostic and therapeutic interventions for ARF and CKD. (PLI, P)<br />

4. Respond appropriately to patients who are nonadherent to treatment for renal failure. (CS, P)<br />

5. Demonstrate ongoing commitment to self-directed learning regarding renal failure. (PLI, P)<br />

4. Appreciate the impact renal failure has on a patient’s quality <strong>of</strong> life, wellbeing, ability to work,<br />

127


and the family. (P)<br />

5. Recognize the importance and demonstrate a commitment to the utilization <strong>of</strong> other healthcare<br />

pr<strong>of</strong>essions in the treatment <strong>of</strong> renal failure. (P, SBP)<br />

D. REFERENCES:<br />

11.<br />

Lamiere N, Van Breson W, Vanholder R. Acute renal failure. Lancet. 2005;365:417-30.<br />

Singri N, Ahya SN, Levin ML. Acute renal failure. JAMA. 2003;289:747-51.<br />

Schrier RW, Wang W, Poole B, Mitra A. Acute renal failure: definitions, diagnosis,<br />

pathogenesis, and therapy. J Clin Invest. 2004;114:5-14.<br />

Esson ML, Schrier RW. Diagnosis and treatment <strong>of</strong> acute tubular necrosis. Ann Intern<br />

Med. 2002;137:744-52.<br />

Levey AS. Clinical practice. Nondiabetic kidney disease. N Engl J Med. 2002;347:1505-<br />

Palmer BF. Renal dysfunction complicating the treatment <strong>of</strong> hypertension. N Engl J<br />

Med. 2002;347:1256-61.<br />

128


TRAINING PROBLEM #18: COMMON CANCERS<br />

RATIONALE:<br />

A skillful initial workup for suspected cancer is an essential part <strong>of</strong> effective primary care practice.<br />

Developing a logical and practical diagnostic approach to the more common cancers (e.g. skin,<br />

colorectal, lung, breast, cervical, and prostate) is an excellent means <strong>of</strong> honing basic history-taking,<br />

physical examination, and communication skills and learning how to use diagnostic studies in a cost<br />

effective manner. Encountering patients in whom cancer is a diagnostic possibility will stimulate<br />

learning <strong>of</strong> the important clinical presentations and natural histories <strong>of</strong> these life-threatening<br />

conditions. Focusing on cancer diagnosis helps to concentrate the student’s learning and avoids<br />

premature immersion in the <strong>of</strong>ten very technical and specialized issues <strong>of</strong> cancer treatment.<br />

PREREQUISITES:<br />

Prior knowledge, skills, and attitudes acquired during the pre-<strong>clerkship</strong> experience should include:<br />

Ability to perform a complete medical history and physical exam.<br />

Ability to communicate with patients <strong>of</strong> diverse backgrounds.<br />

Anatomy, physiology, and pathophysiology <strong>of</strong> common cancers.<br />

Basic knowledge <strong>of</strong> the common symptoms and signs <strong>of</strong> the most common cancers.<br />

Knowledge <strong>of</strong> basic concepts <strong>of</strong> clinical epidemiology pertinent to test selection and<br />

interpretation (e.g. sensitivity, specificity, positive predictive value, negative predictive value).<br />

SPECIFIC LEARNING OBJECTIVES:<br />

A. KNOWLEDGE: Students should be able to define, describe, and discuss:<br />

3. Primary prevention measures for common cancers. (MK)<br />

4. Current screening recommendations for skin, colorectal, lung, breast, cervical, and<br />

prostate cancer. (MK)<br />

3. Principle clinical presentations, clinical courses, complications, and causes <strong>of</strong> death for the<br />

most common cancers (e.g. skin, colorectal, lung, breast, cervical, and prostate). (MK)<br />

4. Basic methods <strong>of</strong> initial evaluation, including the sensitivity and specificity <strong>of</strong> basic diagnostic<br />

studies and indication for their use, including:<br />

Indications for skin biopsy in a patient with a suspicious skin lesion. (MK)<br />

Indications for colonoscopy in individuals a risk for colon cancer. (MK)<br />

Indications for breast biopsy in a patient with a breast nodule or abnormal screening<br />

mammogram. (MK) Indications for a lymph node biopsy in a patent with suspicious<br />

lymphadenopathy. (MK)<br />

Initial cost-effective workups for: isolated pleural effusion, pulmonary nodule, liver nodule,<br />

prostate nodule, elevated prostate-specific antigen, testicular nodule, stool test positive for<br />

occult blood, abnormal<br />

Pap smear, and other findings suggestive <strong>of</strong> gastrointestinal and urogenital cancers.<br />

(MK)<br />

5. Genetic considerations <strong>of</strong> selected cancers (e.g. hereditary nonpolyposis colon cancer, familial<br />

adenomatous polyposis, BRCA1/BRCA2, HER2, Philadelphia chromosome/BRC-ABL). (MK)<br />

6. The role <strong>of</strong> human papilloma virus in cervical cancer. (MK)<br />

7. The similarities and differences between curative and palliative cancer care. (MK)<br />

8. The principles <strong>of</strong> palliative care and hospice care. (MK)<br />

9. Symptoms sometimes seen during end-<strong>of</strong>-life care and the basic principles <strong>of</strong> their<br />

129


management (e.g., pain, dyspnea, nausea and vomiting, anorexia, fatigue, depression,<br />

delirium, constipation). (MK)<br />

B. SKILLS: Students should be able to demonstrate specific skills, including:<br />

1. History-taking skills: Students should be able to obtain, document, and present an<br />

age-appropriate medical history, that differentiates among etiologies <strong>of</strong> disease,<br />

including:<br />

Unintentional weight loss, fever, bone pain. (PC, CS)<br />

Sun exposure history, abnormal skin lesions. (PC, CS)<br />

Blood in the stool, alterations in bowel movements, abdominal pain, abdominal mass. (PC,<br />

CS)<br />

Smoking, cough, hemoptysis, chest pain, dyspnea. (PC, CS)<br />

Breast nodules and secondary signs <strong>of</strong> breast cancer. (PC, CS)<br />

Abnormal vaginal bleeding. (PC, CS)<br />

Abnormal urinary symptoms. (PC, CS)<br />

Lymphadenopathy. (PC, CS)<br />

2. Physical exam skills: Students should be able to perform a physical exam to establish the<br />

diagnosis and severity <strong>of</strong> disease, including:<br />

Skin examination. (PC)<br />

Digital rectal examination. (PC)<br />

Breast examination. (PC)<br />

Lymph node examination. (PC)<br />

Male genital examination and prostate examination. (PC)<br />

Pelvic examination and Pap smear. (PC)<br />

3. Differential diagnosis: Students should be able to generate a prioritized differential<br />

diagnosis recognizing specific history and physical exam findings that suggest a specific<br />

etiology for:<br />

Unintentional weight loss. (PC, MK)<br />

Fever. (PC, MK)<br />

Abnormal skin lesions. (PC, MK)<br />

Occult blood positive stool. (PC, MK)<br />

Colorectal mass. (PC, MK)<br />

Chronic cough, hemoptysis, pulmonary nodule, and pleural effusion. (PC, MK)<br />

Breast mass. (PC, MK)<br />

Abnormal Pap smear. (PC, MK)<br />

Abdominal or pelvic mass. (PC, MK)<br />

Prostate nodule and elevated prostate specific antigen. (PC, MK)<br />

Lymphadenopathy. (PC, MK)<br />

5. Laboratory interpretation: Students should be able to recommend when to order<br />

diagnostic and laboratory tests and be able to interpret them, both prior to and after<br />

initiating treatment based on the differential diagnosis, including consideration <strong>of</strong> test<br />

cost and performance characteristics as well as patient preferences.<br />

Laboratory and diagnostic tests should include, when appropriate:<br />

CBC. (PC)<br />

Electrolytes, BUN/Cr, Ca, hepatic function panel. (PC)<br />

Stool occult blood testing. (PC)<br />

PSA. (PC)<br />

130


Students should be able to define the indications for and interpret (with consultation) the<br />

significance <strong>of</strong> the results <strong>of</strong>:<br />

Skin biopsy. (PC)<br />

Mammogram. (PC)<br />

Breast biopsy. (PC)<br />

Colon/rectal biopsy. (PC)<br />

Lung biopsy. (PC)<br />

Pap smear. (PC)<br />

Prostate biopsy. (PC)<br />

Lymph node biopsy. (PC)<br />

5. Communication skills: Students should be able to:<br />

Communicate the diagnostic plan and subsequent follow-up to patients. (PC, CS)<br />

Elicit questions from the patient and his or her family about the management plan. (PC, CS)<br />

With guidance and direct supervision, participate in breaking bad news to patients. (PC,<br />

CS)<br />

With guidance and direct supervision, participate in discussing basic issues regarding<br />

advance directives with the patient and his or her family. (PC, CS)<br />

With guidance and direct supervision participate in discussing basic end-<strong>of</strong>-life issues with<br />

the patient and his or her family. (PC, CS)<br />

6. Basic and advanced procedure skills: Students should be able to:<br />

Cervical Pap smear. (PC)<br />

Stool occult blood testing. (PC)<br />

7. Management skills: Students should able to develop an appropriate evaluation and<br />

treatment plan for patients that includes:<br />

Initial work-up <strong>of</strong> the symptom, sign, or abnormal laboratory value suspected to be due to<br />

cancer. (PC)<br />

Provision <strong>of</strong> support and information for the patient. (PC)<br />

Coordination <strong>of</strong> care for workup. (PC, SBP)<br />

Determining when to obtain consultation from appropriate specialists. (PC, SBP)<br />

A cost-effective approach based on the differential diagnosis. (PC, SBP)<br />

Accessing and utilizing appropriate information systems and resources to help delineate<br />

issues related to common cancers. (PC, PLI)<br />

Incorporating patient needs and preferences. (PC, P)<br />

Appropriately assessing and treating pain when necessary with nonnarctoic and narcotic<br />

analgesics. (PC)<br />

Anticipating and treating narcotic side effects if necessary. (PC)<br />

Adjusting the therapeutic plan when goals <strong>of</strong> care change (e.g., a shift toward palliative<br />

care). (PC)<br />

Alleviation <strong>of</strong> symptoms sometimes seen during end <strong>of</strong> life care (e.g., pain, dyspnea,<br />

nausea and vomiting, anorexia, fatigue, depression, delirium, constipation). (PC)<br />

Utilizing supportive care or hospice service when appropriate. (PC, SBP)<br />

C. ATTITUDES AND PROFESSIONAL BEHAVIORS: Students should be able to:<br />

1. Appreciate the uncertainty and fear patients experience when cancer is a significant diagnostic<br />

possibility. (P)<br />

2. Respect the patient’s right to refuse cancer screening. (P)<br />

3. Demonstrate commitment to using risk-benefit, cost-benefit, and evidence-based<br />

131


considerations in the selection diagnostic and therapeutic interventions for common cancers. (PLI, P)<br />

4. Recognize the importance <strong>of</strong> patient preferences when selecting among diagnostic options for<br />

common cancers. (P)<br />

5. Demonstrate ongoing commitment to self-directed learning regarding common cancers. (PLI,<br />

P)<br />

6. Appreciate the impact common cancers have on a patient’s quality <strong>of</strong> life, well-being, ability to<br />

work, and the family. (P)<br />

7. Recognize the importance <strong>of</strong> and demonstrate a commitment to the utilization <strong>of</strong> other<br />

healthcare pr<strong>of</strong>essions in the workup and treatment <strong>of</strong> common cancers. (P, SBP)<br />

D. REFERENCES:<br />

National Cancer Institute<br />

National Institutes <strong>of</strong> Health<br />

www.cancer.gov<br />

Guide to Clinical Preventive Services<br />

U.S. Preventative Services Task Force (USPSTF)<br />

Agency for Healthcare Research and Quality<br />

U.S. <strong>Department</strong> <strong>of</strong> Health and Human Services<br />

www.ahrq.gov/clinic/cps3dix.htm#cancer<br />

American Cancer Society<br />

www.cancer.org<br />

NCCN Clinical Practice Guidelines in Oncology National Comprehensive<br />

Cancer Network<br />

www.nccn.org/pr<strong>of</strong>essionals/physician_gls/default.asp<br />

132


TRAINING PROBLEM #19: COPD/OBSTRUCTIVE AIRWAYS<br />

DISEASE<br />

RATIONALE:<br />

The chronic obstructive pulmonary diseases (chronic bronchitis and emphysema) are important<br />

causes <strong>of</strong> morbidity and mortality and are a major cause <strong>of</strong> total disability, second only to coronary<br />

artery disease. Cigarette smoking plays a major role in the progression <strong>of</strong> the disease, with survival<br />

rates lower among patients who continue to smoke cigarettes. The severity and debilitation <strong>of</strong> these<br />

disorders make them an important training problem for all third year medical students. The number<br />

<strong>of</strong> new cases <strong>of</strong> asthma is dramatically increasing. Most cases with appropriate treatment can have<br />

minimal symptoms.<br />

PREREQUISITES:<br />

Prior knowledge, skills, and attitudes acquired during the pre-clinical experience should include:<br />

Ability to perform a complete medical history and physical.<br />

Ability to communicate with patients <strong>of</strong> diverse backgrounds.<br />

Normal structure and function <strong>of</strong> the heart and lungs and how these are altered in respiratory<br />

system diseases.<br />

Pathogenesis and pathophysiology <strong>of</strong> pulmonary diseases.<br />

Pharmacology <strong>of</strong> bronchodilators, corticosteroids, and antibiotics.<br />

SPECIFIC LEARNING OBJECTIVES:<br />

A. KNOWLEDGE: Students should be able to define, describe, and discuss:<br />

1. The epidemiology, risk factors, symptoms, signs, and typical clinical course <strong>of</strong> the common<br />

forms <strong>of</strong> COPD, including chronic bronchitis and emphysema. (MK)<br />

2. Common causes <strong>of</strong> acute exacerbations <strong>of</strong> COPD (AECOPD), including:<br />

Acute infectious bronchitis. (MK)<br />

Pneumonia. (MK)<br />

Pulmonary edema. (MK)<br />

Poor air quality (e.g. ozone, pollutants, tobacco smoke). (MK)<br />

Occupational exposures. (MK)<br />

Medical noncompliance. (MK)<br />

3. The etiology, pathogenesis, evaluation, and management <strong>of</strong> hypoxemia and hypercapnia.<br />

(MK)<br />

4. The genetics and role <strong>of</strong> alpha-1 antitrypsin deficiency in some patients with emphysema. (MK)<br />

5. The epidemiology, risk factors, symptoms, signs, and typical clinical course <strong>of</strong> asthma. (MK)<br />

6. Allergic and non-allergic factors that may precipitate bronchospasm and exacerbate asthma,<br />

including:<br />

Grass and tree pollen. (MK)<br />

Animal dander. (MK)<br />

Cockroaches. (MK)<br />

Dust mites. (MK)<br />

Allergic rhinitis/post-nasal drip. (MK)<br />

133


Acute/chronic infectious sinusitis. (MK)<br />

Acute infectious bronchitis. (MK)<br />

Pneumonia. (MK)<br />

Pulmonary edema. (MK)<br />

Exercise. (MK)<br />

Anxiety/stress. (MK)<br />

Poor air quality (e.g. ozone, pollutants, tobacco smoke). (MK)<br />

Occupational exposures. (MK)<br />

Medical noncompliance. (MK)<br />

7. Therapies for COPD and asthma, including:<br />

Beta-agonist bronchodilators. (MK)<br />

Anticholinergic bronchodilators. (MK)<br />

Leukotriene inhibitors. (MK)<br />

Mast cell stabilizers. (MK)<br />

Theophylline. (MK)<br />

Inhaled corticosteroids. (MK)<br />

Systemic corticosteroids. (MK)<br />

Antimicrobial agents. (MK)<br />

Supplemental oxygen. (MK)<br />

Immunotherapy. (MK)<br />

8. The indications for and the efficacy <strong>of</strong> influenza and pneumococcal vaccines.<br />

(MK)<br />

B. SKILLS: Students should be able to demonstrate specific skills, including:<br />

1. History-taking skills: Students should be able to obtain, document, and present an ageappropriate<br />

medical history, that differentiates among etiologies <strong>of</strong> disease including:<br />

Existence, duration, and severity <strong>of</strong> dyspnea, orthopnea, paroxysmal nocturnal dyspnea,<br />

cough, sputum production, wheezing, fever, chills, sweats, chest pain, hemoptysis. (PC,<br />

CS)<br />

Smoking history and passive exposure to tobacco smoke. (PC, CS)<br />

Occupational history. (PC, CS)<br />

Family history <strong>of</strong> pulmonary problems. (PC, CS)<br />

2. Physical exam skills: Students should be able to perform a physical exam to establish the<br />

diagnosis and severity <strong>of</strong> disease including:<br />

Accurately determining respiratory rate and level <strong>of</strong> respiratory distress. (PC)<br />

Assessing the use <strong>of</strong> accessory muscles for breathing. (PC)<br />

Identifying bronchial breath sounds, rales, rhonchi, and wheezes. (PC)<br />

Identifying signs <strong>of</strong> pulmonary consolidation, pleural effusion, and pneumothorax. (PC)<br />

Identifying the signs <strong>of</strong> pulmonary hyperresonance/hyperexpansion. (PC)<br />

3. Differential diagnosis: Students should be able to generate a prioritized differential diagnosis<br />

recognizing specific history and physical exam findings that suggest a diagnosis <strong>of</strong> chronic<br />

bronchitis, emphysema, asthma, or other conditions with similar findings.<br />

4. Laboratory interpretation: Students should be able to recommend when to order diagnostic<br />

and laboratory tests and be able to interpret them, both prior to and after initiating treatment<br />

based on the differential diagnosis, including consideration <strong>of</strong> test cost and performance<br />

characteristics as well as patient preferences.<br />

Laboratory and diagnostic tests should include, when appropriate:<br />

134


Pulse oximitry. (PC, MK)<br />

ABG. (PC, MK)<br />

Chest radiograph. (PC, MK)<br />

Pulmonary function tests. (PC, MK)<br />

5. Communication skills: Students should be able to:<br />

Communicate the diagnosis, treatment plan, and subsequent follow-up to the patient and<br />

his or her family. (PC, CS)<br />

Elicit questions from the patient and his or her family about the management plan. (PC, CS)<br />

Counsel patients about smoking cessation. (PC, CS)<br />

Counsel patients about the performance <strong>of</strong> home peak flow monitoring. (PC, CS)<br />

Counsel patients about environmental controls. (PC, CS)<br />

Encourage asthma patients to be involved in their own disease management and counsel<br />

them about an “asthma action plan.” (PC, CS)<br />

6. Basic and advanced procedure skills: Students should be able to:<br />

Obtain an ABG. (PC)<br />

7. Management skills: Students should able to develop an appropriate evaluation and treatment<br />

plan for patients that includes:<br />

The use <strong>of</strong> bronchodilators and inhaled corticosteroids. (PC, MK)<br />

The key components <strong>of</strong> the care <strong>of</strong> patients admitted with acute exacerbations <strong>of</strong> COPD<br />

and asthma. (PC, MK)<br />

Using systemic corticosteroids appropriately. (PC, MK)<br />

Judicious use <strong>of</strong> antimicrobial agents. (PC, MK)<br />

The principles <strong>of</strong> oxygen therapy. (PC, MK)<br />

Determining when to obtain consultation from a pulmonologist or allergist/immunologist.<br />

(PC, SBP)<br />

Smoking cessation strategies. (PC)<br />

Using a cost-effective approach based on the differential diagnosis. (PC, SBP)<br />

Accessing and utilizing appropriate information systems and resources to help delineate<br />

issues related to COPD and asthma. (PC, PLI)<br />

Incorporating patient preferences. (PC, P)<br />

C. ATTITUDES AND PROFESSIONAL BEHAVIORS: Students should be able to:<br />

1 Demonstrate commitment to using risk-benefit, cost-benefit, and evidence-based<br />

considerations in the selection diagnostic and therapeutic interventions for COPD and asthma.<br />

(PLI, P)<br />

2 Recognize the importance <strong>of</strong> patient preferences when selecting among diagnostic and<br />

therapeutic options for COPD and asthma (P)<br />

3 Respond appropriately to patients who are nonadherent to treatment for COPD and asthma.<br />

(CS, P)<br />

4 Appreciate the impact <strong>of</strong> working, living, and environmental conditions on the development and<br />

progression <strong>of</strong> respiratory tract disease; demonstrate understanding that patients are <strong>of</strong>ten<br />

unable to change these factors on their own. (P)<br />

5 Demonstrate ongoing commitment to self-directed learning regarding COPD and asthma. (PLI,<br />

P)<br />

6 Appreciate the impact COPD and asthma have on a patient’s quality <strong>of</strong> life, well-being, ability<br />

to work, and the family. (P)<br />

7 Recognize the importance <strong>of</strong> and demonstrate a commitment to the utilization <strong>of</strong> other<br />

healthcare pr<strong>of</strong>essionals in the diagnosis and treatment <strong>of</strong> COPD and asthma. (P, SBP)<br />

135


8 Appreciate the importance <strong>of</strong> antimicrobial resistance. (P)<br />

9. Show understanding for the difficulties patients face with smoking cessation. (P)<br />

D. REFERENCES:<br />

National Heart Lung and Blood Institute/World Health Organization Global Initiative for<br />

Chronic Obstructive Lung Disease Diagnosis, management, and prevention <strong>of</strong> chronic<br />

obstructive pulmonary disease<br />

www.goldcopd.com<br />

National Institutes <strong>of</strong> Health, National Heart, Lung, and Blood Institute National Asthma<br />

Education and Prevention Program Practical Guide for the Diagnosis and Management<br />

<strong>of</strong> Asthma NIH Publication 97-4074. Bethesda, MD. 2003.<br />

http://www.nhlbi.nih.gov/health/indexpro.htm<br />

Celli BR, MacNee W. Standards for the diagnosis and treatment <strong>of</strong> patients with<br />

COPD: a summary <strong>of</strong> the ATS/ERS position paper. Eur Respir J 2004;23:932-46.<br />

136


TRAINING PROBLEM #20: DIABETES MELLITUS<br />

RATIONALE:<br />

Diabetes mellitus is an increasingly prevalent illness in the United States. It is estimated that five to<br />

nine percent <strong>of</strong> American adults are diabetic with the illness appearing at earlier ages in some<br />

populations. It is a leading cause <strong>of</strong> disability and death. Over 130 billion health care dollars are<br />

spent on diabetes annually. All internists must identify those at risk and institute appropriate<br />

management to ameliorate the potentially fatal complications <strong>of</strong> this illness.<br />

PREREQUISITES:<br />

Prior knowledge, skills, and attitudes acquired during the pre-clinical experience should include:<br />

Ability to perform a complete medical history and physical.<br />

Ability to communicate with patients <strong>of</strong> diverse backgrounds.<br />

Pathogenesis and pathophysiology <strong>of</strong> type I and II diabetes mellitus, diabetic ketoacidosis,<br />

nonketotic hyperglycemia.<br />

Effects <strong>of</strong> insulin on glucose and fat metabolism.<br />

Pharmacology <strong>of</strong> insulin, sulfonylureas, metformin, thiazolidinediones, and glucose absorption<br />

inhibitors.<br />

SPECIFIC LEARNING OBJECTIVES:<br />

A. KNOWLEDGE: Students should be able to define, describe, and discuss:<br />

1. Diagnostic criteria for impaired fasting glucose and impaired glucose tolerance. (MK)<br />

2. Diagnostic criteria for type I and type II diabetes mellitus, based on a history, physical<br />

examination, and laboratory testing. (MK)<br />

3. Pathophysiology, risk factors, and epidemiology <strong>of</strong> type I and type II diabetes mellitus. (MK)<br />

4. The basic principles <strong>of</strong> the role <strong>of</strong> genetics in diabetes mellitus. (MK)<br />

5. Presenting symptoms and signs <strong>of</strong> type I and type II diabetes mellitus. (MK)<br />

6. Presenting symptoms and signs <strong>of</strong> diabetic ketoacidosis (DKA) and nonketotic hyperglycemic<br />

(NKH). (MK)<br />

7. Pathophysiology for the abnormal laboratory values in DKA and NKH including plasma<br />

sodium, potassium, and bicarbonate. (MK)<br />

8. Precipitants <strong>of</strong> DKA and NKH. (MK)<br />

9. Major causes <strong>of</strong> morbidity and mortality in diabetes mellitus (coronary artery disease,<br />

peripheral vascular disease, hypoglycemia, DKA, NKH coma, retinopathy, neuropathy—<br />

peripheral and autonomic, nephropathy, foot disorders, infections). (MK)<br />

10. Laboratory tests needed to screen, diagnose, and follow diabetic patients including: glucose,<br />

electrolytes, blood urea nitrogen/creatinine, fasting lipid pr<strong>of</strong>ile, HgA1c, urine<br />

microalbumin/creatinine ratio, urine dipstick for protein. (MK)<br />

11. Non-pharmacologic and pharmacologic (drugs and side effects) treatment <strong>of</strong> diabetes mellitus<br />

to maintain acceptable levels <strong>of</strong> glycemic control, prevent target organ disease, and other<br />

associated complications. (MK)<br />

12. The specific components <strong>of</strong> the American Diabetes Association (ADA) dietary recommendations<br />

for type I and type II diabetes mellitus. (MK)<br />

13. Basic management <strong>of</strong> diabetic ketoacidosis and nonketotic hyperglycemic states, including the<br />

similarities and differences in fluid and electrolyte replacement. (MK)<br />

14. Basic management <strong>of</strong> blood glucoses in the hospitalized patient. (MK)<br />

137


15. The Somogyi effect and the Dawn phenomenon and the implications <strong>of</strong> each in diabetes<br />

pharmacologic management. (MK)<br />

16. The fundamental aspects <strong>of</strong> the American Diabetes Association (ADA) clinical practice<br />

recommendations and how they encourage high quality diabetes care. (MK, PLI, SBP)<br />

17. Basic management <strong>of</strong> hypertension and hyperlipidemia in the diabetic patient. (MK)<br />

B. SKILLS: Students should be able to demonstrate specific skills including:<br />

1. History-taking skills: Students should be able to obtain, document, and present an<br />

age-appropriate medical history, that differentiates among etiologies <strong>of</strong> disease,<br />

including:<br />

Weight changes. (PC, CS)<br />

Hypo- or hyperglycemic symptoms. (PC, CS)<br />

Medication history (adherence, side effects, other medications). (PC, CS)<br />

Home glucose monitoring results. (PC, CS)<br />

Target organ disease complications (cardiovascular, foot, gastrointestinal, infectious,<br />

neurological, sexual, skin, urinary, or vision symptoms). (PC, CS)<br />

Diet history (total caloric intake, intake <strong>of</strong> sugar-containing foods, intake <strong>of</strong> saturated fat and<br />

cholesterol, physical activity level, timing <strong>of</strong> meals). (PC, CS)<br />

2. Physical exam skills: Students should be able to perform a physical exam to establish the<br />

diagnosis and severity <strong>of</strong> disease, including:<br />

Skin examination for diabetic dermopathy, furuncles/carbuncles, candidiasis, necrobiosis<br />

lipoidica diabeticorum, dermatophytosis, and acanthosis nigricans. (PC)<br />

Fundoscopic exam. (PC)<br />

Arterial pulses. (PC)<br />

Peripheral nerves (e.g. mon<strong>of</strong>ilament testing). (PC)<br />

Examination <strong>of</strong> the feet for corns, calluses, and ulcerations. (PC)<br />

In patients with DKA or NKH evaluate for mental status alterations, Kussmaul’s<br />

respirations, fruity breath, and signs <strong>of</strong> volume depletion. (PC)<br />

3. Differential diagnosis: Students should be able to generate a prioritized differential diagnosis<br />

recognizing specific history and physical exam findings that suggest a specific etiology for:<br />

Hyperglycemia. (PC, MK)<br />

Hypoglycemia. (PC, MK)<br />

Anion gap acidosis. (PC, MK)<br />

Ketosis. (PC, MK)<br />

Hyperosmolality. (PC, MK)<br />

4. Laboratory interpretation: Students should be able to recommend when to order diagnostic<br />

and laboratory tests and be able to interpret them, both prior to and after initiating treatment<br />

based on the differential diagnosis, including consideration <strong>of</strong> test cost and performance<br />

characteristics as well as patient preferences Laboratory and diagnostic tests should include,<br />

when appropriate:<br />

Fasting serum GLC. (PC, MK)<br />

Electrolytes, BUN/Cr. (PC, MK)<br />

Serum and urine ketones. (PC, MK)<br />

Serum and urine osmolality. (PC, MK)<br />

HbA1c. (PC, MK)<br />

Fasting lipid pr<strong>of</strong>ile. (PC, MK)<br />

UA. (PC, MK)<br />

Urine microalbumin/creatinine ratio. (PC, MK)<br />

138


24-hour urine for protein and creatinine clearance. (PC, MK)<br />

5. Communication skills: Students should be able to:<br />

Communicate the diagnosis, treatment plan, and subsequent follow-up to patients. (PC,<br />

CS)<br />

Elicit questions from the patient and their family about the management plan. (PC, CS)<br />

Counsel patients appropriately on dietary measures, exercise, medication adherence,<br />

proper foot care, and prevention <strong>of</strong> other target organ disease. (PC, CS)<br />

6. Basic and advanced procedural skills: Students should be able to:<br />

Finger-stick capillary blood glucose determination. (PC)<br />

Obtain an ABG. (PC)<br />

7. Management skills: Students should able to develop an appropriate evaluation and<br />

treatment plan for patients that includes:<br />

Writing appropriate fluid and insulin orders and outline critical steps for the treatment <strong>of</strong><br />

DKA and DKH. (PC, MK)<br />

Counseling patients regarding basic features <strong>of</strong> ADA diabetic diet recommendations. (PC,<br />

CS)<br />

Instructing patients in home blood glucose monitoring. (PC, CS)<br />

Counseling patients on behavior changes (smoking cessation, medication adherence, poor<br />

glycemic control, obesity, hypertension, dyslipidemia, and infection) to avoid the<br />

complications <strong>of</strong> diabetes. (PC, CS)<br />

Counseling patients regarding basic foot care. (PC, CS)<br />

Determining when to institute diet therapy, oral hypoglycemic agents, and insulin therapy.<br />

(PC, MK)<br />

Calculating an appropriate insulin dose for a diabetic patient. (PC, MK)<br />

Using community resources (ADA, hospital and community-based education programs) to<br />

aid the patient in understanding and managing his or her illness. (PC, SBP)<br />

Determining when to obtain consultation from an endocrinologist, nephrologist,<br />

ophthalmologist, podiatrist, and dietician. (PC, SBP)<br />

Accessing and utilizing appropriate information systems and resources to help delineate<br />

issues related to diabetes mellitus. (PC, PLI)<br />

Incorporating patient preferences. (PC)<br />

C. ATTITUDES AND PROFESSIONAL BEHAVIORS: Students should be able to:<br />

1. Demonstrate a commitment to meeting ADA clinical practice<br />

recommendations to insure quality diabetes care. (PLI, P, SBP)<br />

2. Demonstrate commitment to using risk-benefit, cost-benefit, and evidence-based considerations in<br />

the selection diagnostic and therapeutic interventions for diabetes mellitus. (PLI, P)<br />

3. Recognize the importance <strong>of</strong> patient preferences when selecting among diagnostic and<br />

therapeutic options for diabetes mellitus. (P)<br />

4. Respond appropriately to patients who are nonadherent to treatment for diabetes mellitus. (CS, P)<br />

5. Demonstrate ongoing commitment to self-directed learning regarding diabetes mellitus. (PLI, P)<br />

6. Appreciate the impact diabetes mellitus has on a patient’s quality <strong>of</strong> life, wellbeing, ability to work,<br />

and the family. (P)<br />

7. Recognize the importance <strong>of</strong> and demonstrate a commitment to the utilization <strong>of</strong> other healthcare<br />

pr<strong>of</strong>essionals in the treatment <strong>of</strong> diabetes mellitus. (P, SBP)<br />

D. REFERENCES:<br />

139


The Diabetes Control and Complication Trail Research Group. The effect <strong>of</strong> intensive<br />

treatment <strong>of</strong> diabetes on the development and progression <strong>of</strong> long-term complications in<br />

insulin-dependent diabetes mellitus. N Engl JMed. 1993;329:977-86.<br />

UK Prospective Diabetes Study (UKPDS) Group. Intensive blood-glucose control with<br />

sulphonylureas or insulin compared with conventional treatment and risk <strong>of</strong><br />

complications in patients with type 2 diabetes (UKPDS 33). Lancet. 1998;352:837-53.<br />

Clinical Practice Recommendations American Diabetes Association<br />

www.diabetes.org/for-health-pr<strong>of</strong>essionals-and-scientists/cpr.jsp<br />

Kitabchi AE. Wall BM. Management <strong>of</strong> diabetic ketoacidosis. Am Fam Physician.<br />

1999;60:455-64.<br />

140


TRAINING PROBLEM #21: DYSLIPIDEMIA<br />

RATIONALE:<br />

Dyslipidemia is a common, important, and treatable cardiovascular risk factor. Its pathophysiology is<br />

increasingly understood, diagnostic tests are readily available, and treatment modalities range from<br />

diet and exercise to a multitude <strong>of</strong> pharmacotherapies. Competency in the evaluation and<br />

management <strong>of</strong> this problem helps develop skills in rational test selection, patient education, and<br />

design <strong>of</strong> cost-effective treatment strategies. It also draws attention to the importance <strong>of</strong> community<br />

health education and nutrition.<br />

PREREQUISITES:<br />

Prior knowledge, skills, and attitudes acquired during the pre-clinical experience should include:<br />

Ability to perform a complete medical history and physical.<br />

Ability to communicate with patients <strong>of</strong> diverse backgrounds.<br />

Anatomy and physiology <strong>of</strong> the vascular system.<br />

Basic cholesterol and lipoprotein metabolism.<br />

Pathogenesis and pathophysiology <strong>of</strong> atherosclerosis.<br />

Pharmacology <strong>of</strong> bile acid sequestrants (resins), nicotinic acid, fibric acid derivatives, HMG-<br />

CoA reductase inhibitors (statins), and cholesterol absorption inhibitors (ezetimibe).<br />

SPECIFIC LEARNING OBJECTIVES:<br />

A. KNOWLEDGE: Students should be able to define, describe, and discuss:<br />

1 The contribution <strong>of</strong> lipoproteins to atherogenesis and CAD risk, including the importance <strong>of</strong><br />

elevations in total cholesterol, LDL cholesterol, ratio <strong>of</strong> total to HDL cholesterol, and<br />

Lipoprotein a. (MK)<br />

2 The classification and etiologies <strong>of</strong> primary dyslipidemias. (MK)<br />

3 Etiologies and underlying pathophysiology <strong>of</strong> secondary dyslipidemias. (MK)<br />

4. The basic principles <strong>of</strong> the role <strong>of</strong> genetics in dyslipidemia, particularly familial combined<br />

hyperlipidemia. (MK)<br />

5. Screening recommendations for dyslipidemias in American adults. (MK)<br />

6. The importance <strong>of</strong> identifying and treating asymptomatic patients at high risk for CAD as<br />

aggressively as those with symptomatic disease. (MK)<br />

7. The available diagnostic studies and their use, particularly determinations <strong>of</strong> HDL, LDL, and total<br />

cholesterol, as well as the need to test for other cardiovascular risk factors. (MK)<br />

8. The current National Cholesterol Education Program (NCEP, ATP III) guidelines for risk factor<br />

assessment, diagnosis and management <strong>of</strong> dyslipidemias, including goal LDL cholesterol, goal<br />

non-HDL cholesterol, and the concept <strong>of</strong> coronary artery disease equivalent based on risk<br />

factors for coronary artery disease. (MK, PLI, SBP)<br />

9. Basic management <strong>of</strong> the common dyslipidemias, including diet, fiber, exercise, and<br />

risk/benefits/cost <strong>of</strong> drug therapy (statins, fibrates, ezetimide, nicotinic acid, resins). (MK)<br />

10. Diagnosis and implications <strong>of</strong> the “metabolic syndrome.” (MK)<br />

B. SKILLS: Students should be able to demonstrate specific skills including:<br />

1. History-taking skills: Students should be able to obtain, document, and present an age-<br />

141


appropriate medical history, that differentiates among etiologies <strong>of</strong> disease including:<br />

Prior patient or family history <strong>of</strong> dyslipidemia. (PC, CS)<br />

Other coronary risk factors. (PC, CS)<br />

Family history <strong>of</strong> early cardiovascular disease. (PC, CS)<br />

Dietary fat, saturated fat, fiber, cholesterol, and refined carbohydrate intake. (PC, CS)<br />

Exercise habits. (PC, CS)<br />

Alcohol use. (PC, CS)<br />

Past history <strong>of</strong> established CAD, cerebral vascular disease, and other vascular disease.<br />

(PC, CS)<br />

Presence <strong>of</strong> symptoms <strong>of</strong> angina and peripheral vascular disease. (PC, CS)<br />

History <strong>of</strong> renal, hepatic, or myopathic disease. (PC, CS)<br />

2. Physical exam skills: Students should be able to perform a physical exam to establish the<br />

diagnosis and severity <strong>of</strong> disease, including:<br />

Blood pressure elevation. (PC)<br />

Xanthomata. (PC)<br />

Atherosclerotic fundoscopic changes. (PC)<br />

Carotid or femoral bruits. (PC, CS)<br />

Diminished peripheral pulses. (PC)<br />

3. Differential diagnosis: Students should be able to generate a prioritized differential diagnosis<br />

recognizing specific history and physical exam findings that suggest primary or secondary<br />

causes <strong>of</strong> dyslipidemia. (PC, CS)<br />

4. Laboratory interpretation: Students should be able to recommend when to order diagnostic<br />

and laboratory tests and be able to interpret them, both prior to and after initiating treatment<br />

based on the differential diagnosis, including consideration <strong>of</strong> test cost and performance<br />

characteristics as well as patient preferences.<br />

Laboratory and diagnostic tests should include, when appropriate:<br />

Fasting lipid pr<strong>of</strong>ile. (PC, MK)<br />

TSH (PC, MK)<br />

Fasting GLC, electrolytes, BUN/Cr. (PC, MK)<br />

Hepatic function panel. (PC, MK)<br />

CK. (PC, MK)<br />

5. Communication skills: Students should be able to:<br />

Communicate the diagnosis, treatment plan, and subsequent follow-up to patients. (PC,<br />

CS)<br />

Elicit questions from the patient and his or her family about the management plan. (PC, CS)<br />

Counsel patients about dietary measures to reduce cholesterol and saturated fats. (PC,<br />

CS)<br />

Counsel patients about ways to increase exercise. (PC, CS)<br />

Counsel patients about other modifiable cardiovascular risk factors. (PC, CS)<br />

6. Management skills: Students should able to develop an appropriate evaluation and<br />

treatment plan for patients that includes:<br />

An individual treatment plan that follows the NCEP ATP III guidelines. (PC, MK)<br />

Lifestyle modification (diet, exercise). (PC, MK)<br />

Appropriate pharmacologic interventions, including bile acid sequestrants (resins), nicotinic<br />

acid, fibric acid derivatives, HMG-CoA reductase inhibitors (statins), and cholesterol<br />

absorption inhibitors (ezetimibe). (PC, MK)<br />

Monitoring for adherence and side effects due to pharmacologic management. (PC, MK)<br />

142


Laboratory response to therapy. (PC, MK)<br />

Identifying barriers that prevent patients from adhering to recommended dietary, exercise,<br />

and pharmacologic plans. (PC, MK)<br />

Determining when to obtain consultation from an endocrinologist, or dietician. (PC, SBP)<br />

Using a cost-effective approach based on the differential diagnosis. (PC, SBP)<br />

Accessing and utilizing appropriate information systems and resources to help delineate<br />

issues related to dyslipidemia. (PC, PLI)<br />

Incorporating patient preferences. (PC)<br />

C. ATTITUDES AND PROFESSIONAL BEHAVIORS: Students should be able to:<br />

1 Demonstrate a commitment to meeting NCEP ATP III guidelines to insure quality care <strong>of</strong><br />

patients with dyslipidemia. (PLI, P, SBP)<br />

2 Appreciate the importance <strong>of</strong> encouraging patients to assume responsibility for modifying their<br />

diet and increasing their exercise level. (P, CS)<br />

3 Appreciate the difficulties and frustrations that patients and health care providers face with<br />

recommended dietary changes. (P)<br />

4 Demonstrate commitment to using risk-benefit, cost-benefit, and evidence-based<br />

considerations in the selection diagnostic and therapeutic interventions for dyslipidemia. (PLI,<br />

P)<br />

5 Recognize the importance <strong>of</strong> patient preferences when selecting among diagnostic and<br />

therapeutic options for dyslipidemia. (P)<br />

6 Respond appropriately to patients who are nonadherent to treatment for dyslipidemia. (CS, P)<br />

7 Demonstrate ongoing commitment to self-directed learning regarding dyslipidemia. (PLI, P)<br />

8 Recognize the importance <strong>of</strong> and demonstrate a commitment to the utilization <strong>of</strong> other<br />

healthcare pr<strong>of</strong>essionals in the treatment <strong>of</strong> dyslipidemia. (P, SBP)<br />

D. REFERENCES:<br />

National Institutes <strong>of</strong> Health, National Heart, Lung, and Blood Institute National<br />

Cholesterol Education Program Clinical Practice Guidelines for Cholesterol<br />

Management in Adults (ATP III)<br />

www.nhlbi.nih.gov/about/ncep<br />

Grundy SM, Cleeman JI, Merz CN, et al; National Heart, Lung, and Blood Institute;<br />

American College <strong>of</strong> Cardiology Foundation; American Heart Association. Implications<br />

<strong>of</strong> recent clinical trials for the National Cholesterol Education Program Adult Treatment<br />

Panel III guidelines. Circulation. 2004;110:227-39.<br />

Ashen MD, Blumenthal RS. Clinical practice. Low HDL cholesterol levels. N Engl J<br />

Med. 2005;353:1252-60.<br />

Hansson GK. Inflammation, atherosclerosis, and coronary artery disease. N Engl J<br />

Med. 2005;352:1685-95.<br />

143


TRAINING PROBLEM #22: HEART FAILURE<br />

RATIONALE:<br />

Chronic heart failure (HF) is one <strong>of</strong> the most common cardiac problems encountered in clinical<br />

practice. Identification and correction <strong>of</strong> treatable underlying causes, control <strong>of</strong> precipitating factors<br />

and judicious use <strong>of</strong> multi-drug regimens for individuals with HF are important issues for third-year<br />

medical students.<br />

PREREQUISITES:<br />

Prior knowledge, skills and attitudes acquired during the pre-<strong>clerkship</strong> years should include:<br />

Knowledge <strong>of</strong> the structure and function <strong>of</strong> the heart and lungs.<br />

Understanding <strong>of</strong> the epidemiology <strong>of</strong> heart disease.<br />

Knowledge <strong>of</strong> the atherogenesis and pathophysiology <strong>of</strong> cardiovascular disease.<br />

Knowledge <strong>of</strong> the pharmacology <strong>of</strong> cardiovascular drugs.<br />

Ability to communicate appropriately with all types <strong>of</strong> patients including the elderly and those<br />

with diverse backgrounds.<br />

Ability to perform a complete medical history and physical exam.<br />

Ability to perform a cardiovascular risk assessment and understand issues related to primary<br />

and secondary prevention <strong>of</strong> cardiovascular disease.<br />

Ability to understand the impact <strong>of</strong> illness on individuals and their families and, when<br />

appropriate, to address issues related to end-<strong>of</strong>-life care.<br />

SPECIFIC LEARNING OBJECTIVES:<br />

A. KNOWLEDGE: Students should be able to define, describe, and discuss:<br />

1. Types <strong>of</strong> processes and most common disease entities that cause HF (i.e. ischemic, valvular,<br />

hypertrophic, infiltrative, inflammatory, etc.). (MK)<br />

2. The basic role <strong>of</strong> genetics in certain forms <strong>of</strong> cardiomyopathy. (MK)<br />

3. Staging system for heart failure:<br />

Stage A: high risk for HF but no structural heart disease is present. (MK)<br />

Stage B: structural heart disease is present but never any symptoms. (MK)<br />

Stage C: past or current symptoms associated with structural heart disease. (MK)<br />

Stage D: end-stage disease with requirements for specialized treatment. (MK)<br />

4. Types <strong>of</strong> processes that cause systolic vs. diastolic dysfunction. (MK)<br />

5. Symptoms and signs <strong>of</strong> left-sided vs. right-sided heart failure. (MK)<br />

6. Compensatory mechanisms <strong>of</strong> heart failure including cardiac remodeling and activation <strong>of</strong><br />

endogenous neurohormonal systems. (MK)<br />

7. Factors leading to symptomatic exacerbation <strong>of</strong> HF, including ischemia, arrhythmias,<br />

hypoxemia, anemia, fever, hypertension, thyroid disorders, non-compliance with medications<br />

and dietary restrictions and use <strong>of</strong> nonsteroidal anti-inflammatory drugs. (MK)<br />

8. Importance <strong>of</strong> age, gender and ethnicity on the prevalence and<br />

prognosis <strong>of</strong> HF. (MK)<br />

9. Physiological basis and scientific evidence supporting each type<br />

<strong>of</strong> treatment, intervention, or procedure commonly used in the management <strong>of</strong> patients who<br />

present with HF. (MK)<br />

10. The general approach to the evaluation and treatment <strong>of</strong> atrial<br />

144


fibrillation (MK)<br />

11. Role <strong>of</strong> critical pathways or practice guidelines in delivering<br />

high-quality, cost effective care for patients presenting with new or recurrent heart failure. (PC,<br />

SBP)<br />

12. The Centers for Medicare & Medicaid Services (CMS) and the<br />

Joint Commission on the Accreditation <strong>of</strong> Healthcare Organizations (JCAHO) quality measures<br />

for HF treatment. (MK, PLI, SBP)<br />

B. SKILLS: Students should be able to demonstrate specific skills, including:<br />

1. History-taking skills: Students should be able to obtain, document, and present an<br />

age-appropriate medical history, including:<br />

Differentiating between various etiologies <strong>of</strong> heart failure (answers the question: Why is the<br />

patient in heart failure?). (PC, CS)<br />

Identifying clinical factors responsible for symptomatic exacerbation (answers the question:<br />

Why is the patient worse now?). (PC, CS)<br />

Exercise intolerance (fatigue, dyspnea on exertion). (PC, CS)<br />

Fluid retention (peripheral edema, dyspnea). (PC, CS)<br />

Changes in sleep pattern (orthopnea, paroxysmal nocturnal dyspnea [PND], nocturia). (PC,<br />

CS)<br />

Assessing the functional capacity <strong>of</strong> patients with HF (walking distance, New York Heart<br />

classification). (PC, CS)<br />

Cardiac risk factors. (PC, CS)<br />

2. Physical exam skills: Students should be able to perform a focused physical exam to<br />

help establish the diagnosis <strong>of</strong> HF and estimate its severity:<br />

Measurement <strong>of</strong> vital signs including weight and respiratory rate/pattern. (PC)<br />

Accurate measurement <strong>of</strong> arterial blood pressure and recognition <strong>of</strong> the typical blood<br />

pressure findings that occur in patients with aortic stenosis, aortic insufficiency and pulsus<br />

paradoxus. (PC)<br />

Assessment <strong>of</strong> major arterial pulses for abnormalities, including bruits. (PC)<br />

Assessment <strong>of</strong> the neck veins for jugular venous distention and, when necessary,<br />

evaluation for abdominal jugular reflux. (PC)<br />

Assessment <strong>of</strong> the conjunctiva and optic fundus. (PC)<br />

Assessment <strong>of</strong> the extremities to ascertain for skin conditions, including color, temperature<br />

and the presence <strong>of</strong> edema, cyanosis or clubbing. (PC)<br />

Assessment <strong>of</strong> the lungs for crackles, rhonchi and decreased breath sounds. (PC)<br />

Inspection and palpation <strong>of</strong> the anterior chest to identify right and left sided heaves, lifts and<br />

thrills. (PC)<br />

Auscultation <strong>of</strong> the heart to determine rhythm, intensity <strong>of</strong> heart sounds, splitting <strong>of</strong> S2 and<br />

the presence <strong>of</strong> rubs, gallops (S3, S4, summation) or extra heart sounds (e.g. clicks). (PC)<br />

Auscultation <strong>of</strong> the heart to detect the presence <strong>of</strong> heart murmurs; when a murmur is<br />

present, students should be able to:<br />

o Identify timing (systolic vs. diastolic, holosystolic vs. ejection). (PC)<br />

o Describe pitch, location and pattern <strong>of</strong> radiation. (PC)<br />

o Gauge significance (innocent vs. pathologic, sclerosis vs. stenosis). (PC)<br />

Assessment <strong>of</strong> the abdomen to determine the presence <strong>of</strong> hepatomegaly,<br />

ascites, abnormal pulsations and bruits. (PC)<br />

3. Differential diagnosis: Students should be able to generate a prioritized differential<br />

145


diagnosis and recognize specific history, physical exam and/or laboratory findings that:<br />

Help support or refute a clinical diagnosis <strong>of</strong> heart failure. (PC, MK)<br />

Distinguish between various underlying etiologies <strong>of</strong> HF, including disease processes that<br />

primarily affect:<br />

o Pericardium (constrictive pericarditis, pericardial tamponade). (PC, MK)<br />

o Endocardium (valvular [congenital, acquired], endocarditis). (PC, MK)<br />

o Myocardium (hypertrophic, restrictive, congestive). (PC, MK)<br />

4. Laboratory interpretation: Students should be able interpret specific diagnostic tests and<br />

procedures that are commonly ordered to evaluate patients who present with heart failure.<br />

Test interpretation should take into account: Laboratory and diagnostic tests should include,<br />

when appropriate:<br />

12-lead ECG. (PC, MK)<br />

Chest radiograph. (PC, MK)<br />

B-type natriuretic peptide. (PC, MK)<br />

Students should be able to define the indications for, and interpret (with consultation) the<br />

results <strong>of</strong> the following diagnostic tests and procedures:<br />

Echocardiography. (PC, MK)<br />

Treadmill and nuclear exercise testing. (PC, MK)<br />

Radionuclide ventriculogram. (PC, MK)<br />

Cardiac. (PC, MK)<br />

Coronary angiography. (PC, MK)<br />

5. Communication skills: Students should be able to:<br />

Communicate the diagnosis, prognosis and treatment plan to the patient and his or her<br />

family. (PC, CS)<br />

Elicit questions from the patient and his or her family about the management plan. (PC, CS)<br />

Educate patients about cardiovascular risk factors. (PC, CS)<br />

Council patients regarding a sodium-restricted diet. (PC, CS)<br />

Address palliative care and end-<strong>of</strong>-life issues with patients who have<br />

intractable symptoms associated with end-stage heart failure. (PC, CS, P)<br />

6. Basic and advanced procedural skills: students should be able to:<br />

Perform a 12-lead ECG. (PC)<br />

Obtain an ABG. (PC)<br />

7. Management skills: Students should be able to develop an appropriate evaluation and<br />

treatment plan for patients that includes:<br />

Recognize the importance <strong>of</strong> early detection and treatment <strong>of</strong> risk factors that may lead to<br />

the development <strong>of</strong> heart failure. (PC)<br />

Identifying the indications, contraindications, mechanisms <strong>of</strong> action, adverse reactions,<br />

significant interactions, and relative costs <strong>of</strong> the following treatments/interventions:<br />

o Non-pharmacological management. (PC, MK)<br />

-Sodium restriction. (PC, MK)<br />

-Physical activity and limitations. (PC, MK)<br />

o Pharmacological management (recommended for routine use). (PC, MK)<br />

-Diuretics. (PC, MK)<br />

-ACE-I/ARB. (PC, MK)<br />

-Beta-blockers. (PC, MK)<br />

-Aldosterone antagonists (spironolactone, eplerenone). (PC, MK)<br />

-digoxin. (PC, MK)<br />

o Interventions considered for use in selected patients. (PC, MK)<br />

-Hydralazine and isosorbide dinitrate. (PC,MK)<br />

146


-Angoitensin receptor blockers. (PC, MK)<br />

-Calcium channel blockers. (PC, MK)<br />

-Anti-arrhythmic agents. (PC, MK)<br />

-Anticoagulants/anti a thrombotic agents. (PC, MK)<br />

o other modalities(PC, MK)<br />

-Coronary revascularization. (PC, MK)<br />

-Synchronized biventricular pacing. (PC, MK)<br />

-Implantable cardiac defibrillators. (PC, MK)<br />

Developing a timely and appropriate evaluation and treatment plan for patients with heart<br />

failure due to diastolic dysfunction, including:<br />

o Control <strong>of</strong> physiologic factors (blood pressure, heart rate). (PC, MK)<br />

o Reduction in central blood volume by judicious use <strong>of</strong> diuretics. (PC, MK)<br />

o Alleviation <strong>of</strong> myocardial ischemia. (PC, MK)<br />

o Use <strong>of</strong> calcium channel blockers. (PC, MK)<br />

Describing use <strong>of</strong> other agents and interventions that may be useful in treating patients with<br />

refractory, end-stage heart failure:<br />

o Intravenous vasodilators. (PC, MK)<br />

o Intravenous positive inotropic agents. (PC, MK)<br />

o Infusion <strong>of</strong> B-type natriuretic peptide (nesiritide). (PC, MK)<br />

o Ventricular assist devices. (PC, MK)<br />

o Heart transplantation. (PC, MK)<br />

Defining and describing how the diagnosis and treatment <strong>of</strong> HF in special<br />

populations may differ (e.g. very elderly, associated co-morbidities). (PC, MK)<br />

Demonstrating how critical pathways or practice guidelines in ambulatory or hospitalized<br />

patients with HF can be used to guide diagnostic test ordering and medical decision<br />

making. (PC, PLI, SBP)<br />

Determining when to consult a cardiologist. (PC, SBP)<br />

Identifying when palliative care may be appropriate for patients with refractory symptoms<br />

associated with end-stage disease. (PC)<br />

Using a cost-effective approach based on the differential diagnosis. (PC, SBP)<br />

Accessing and utilizing appropriate information systems and resources to help delineate<br />

issues related to HF. (PC, PLI)<br />

Incorporating patient preferences. (PC)<br />

C. ATTITUDES AND PROFESSIONAL BEHAVIORS: Students should be able to:<br />

1. Demonstrate commitment to using risk-benefit, cost-benefit, and evidence-based<br />

considerations in the selection diagnostic and therapeutic interventions for HF. (PLI, P)<br />

2. Recognize the significant morbidity and mortality associated with HF (P)<br />

3. Recognize the impact <strong>of</strong> lifestyle limitations caused by HF. (P)<br />

4. Respond appropriately to patients who are non-adherent to treatment for HF. (CS, P)<br />

5. Demonstrate ongoing commitment to self-directed learning regarding heart failure. (PLI, P)<br />

6. Recognize the importance and demonstrate a commitment to the utilization <strong>of</strong> other healthcare<br />

pr<strong>of</strong>essions in the treatment <strong>of</strong> heart failure. (P, SBP)<br />

7. Appreciate the importance <strong>of</strong> and demonstrate a commitment to meeting national health care<br />

quality measures for the treatment <strong>of</strong> HF. (PLI, P, SBP)<br />

147


D. REFERENCES:<br />

ACC/AHA 2005 guideline update for the diagnosis and management <strong>of</strong> chronic<br />

heart failure in the adult: a report <strong>of</strong> the American College <strong>of</strong> Cardiology/American<br />

Heart Association Task Force on Practice Guidelines (Writing Committee to<br />

Update the 2001 Guidelines for the Evaluation and Management <strong>of</strong> Heart Failure).<br />

J Am Coll Cardiol. 2005;46:1116-43.<br />

http://www.acc.org/clinical/guidelines/failure/update/index.pdf<br />

Cook DJ, Simel DL. Does this patient have abnormal central venous pressure? JAMA.<br />

1996; 275: 630-634.<br />

Wang CS. FitzGerald JM. Schulzer M. Mak E. Ayas NT. Does this dyspneic patient in<br />

the emergency department have congestive heart failure? JAMA. 2005;294:1944-56.<br />

Doust JA. Glasziou PP. Pietrzak E. Dobson AJ. A systematic review <strong>of</strong> the diagnostic<br />

accuracy <strong>of</strong> natriuretic peptides for heart failure. Arch Intern Med. 2004;164:1978-84.<br />

Yan AT. Yan RT. Liu PP. Narrative review: pharmacotherapy for chronic heart failure:<br />

evidence from recent clinical trials. Ann Intern Med. 2005;142(2):132-45.<br />

Aurigemma GP. Gaasch WH. Clinical practice. Diastolic heart failure. N Engl J Med.<br />

2004;351:1097-105.<br />

148


TRAINING PROBLEM #23: HIV INFECTION<br />

RATIONALE:<br />

HIV infection and AIDS represent one <strong>of</strong> the most difficult challenges in clinical medicine today. An<br />

HIV specialist (usually an infectious diseases physician) cares for the vast majority <strong>of</strong> patients with<br />

HIV infection and AIDS. Given that there is no proven cure, this remains an important training<br />

problem for third year medical students. The enormous and continuously evolving complexities <strong>of</strong><br />

antiretroviral treatment are generally beyond the level <strong>of</strong> the third year medical student and for that<br />

matter most general internists. Rather, an approach to HIV infection, AIDS, and its most common<br />

and serious complications are stressed.<br />

PREREQUISITES:<br />

Prior knowledge, skills, and attitudes acquired during the pre-clinical experience should include:<br />

Ability to perform a complete medical history and physical.<br />

Ability to communicate with patients <strong>of</strong> diverse backgrounds.<br />

Knowledge <strong>of</strong> the worldwide epidemiology, biology, and immunology <strong>of</strong> HIV.<br />

Microbiology <strong>of</strong> common opportunistic organisms.<br />

Pharmacology <strong>of</strong> antimicrobial agents and antiretrovirals.<br />

Understanding <strong>of</strong> universal precautions.<br />

SPECIFIC LEARNING OBJECTIVES:<br />

A. KNOWLEDGE: Students should be able to define, describe, and discuss:<br />

1. Symptoms and signs <strong>of</strong> acute HIV seroconversion. (MK)<br />

2. CDC AIDS case definition. (MK)<br />

3. Specific tests for HIV (e.g. HIV ELISA, confirmatory western blot, quantitative PCR) and their<br />

operating characteristics. (MK)<br />

4. Relationship <strong>of</strong> CD4 lymphocyte count to opportunistic infections as well as relationship between<br />

CD4 lymphocyte count and viral load to overall disease progression. (MK)<br />

5. The basic principles <strong>of</strong> highly active antiretroviral therapy (HAART), including the different classes<br />

<strong>of</strong> antiviral medications and their use, as well as common side effects and drug-drug<br />

interactions. (MK)<br />

6. Basics <strong>of</strong> post-exposure prophylaxis. (MK)<br />

7. The marked importance <strong>of</strong> antiretroviral medication adherence and the potential consequences <strong>of</strong><br />

erratic or poor adherence. (MK)<br />

8. Vaccination recommendation for patients infected with HIV. (MK)<br />

9. Indications for and utility and risks <strong>of</strong> prophylaxis <strong>of</strong> HIV-related opportunistic infections. (MK)<br />

10. Pathogenesis, symptoms, signs, typical clinical course, and management <strong>of</strong> HIV-related<br />

opportunistic infections with a recognition <strong>of</strong> which are most common:<br />

Pneumocystis jiroveci. (MK)<br />

Candidiasis (oral, esophageal, vaginal). (MK)<br />

Cryptococcus ne<strong>of</strong>ormans. (MK)<br />

Cryptosporidium parvum. (MK)<br />

Cytomegalovirus infection (gastrointestinal, neurologic, retinal). (MK)<br />

Varicella-zoster virus. (MK)<br />

Isospora belli. (MK)<br />

149


Microsporidiosis. (MK)<br />

Mycobacterium avium complex. (MK)<br />

Mycobacterium tuberculosis. (MK)<br />

Toxoplasma gondii. (MK)<br />

11. Symptoms and signs <strong>of</strong> the following HIV-related malignancies:<br />

Kaposi’s sarcoma. (MK)<br />

Non-Hodgkin’s lymphoma. (MK)<br />

Cervical carcinoma. (MK)<br />

12. Common skin and oral manifestations <strong>of</strong> HIV infection and AIDS:<br />

Molluscum contagiosum. (MK)<br />

Cryptococcus ne<strong>of</strong>ormans. (MK)<br />

Viral warts. (MK)<br />

Lipodystrophy. (MK)<br />

Herpes zoster. (MK)<br />

Seborrhoeic dermatitis. (MK)<br />

Buccal candidiasis. (MK)<br />

Oral hairy leukoplakia. (MK)<br />

13. “Safe sex” practices (MK)<br />

14. The importance <strong>of</strong> proper ongoing dental care. (MK)<br />

B. SKILLS: Students should be able to demonstrate specific skills, including:<br />

1. History-taking skills: Students should be able to obtain, document, and present an ageappropriate<br />

medical history that differentiates among etiologies <strong>of</strong> disease, including:<br />

HIV infection risk factors. (PC, CS)<br />

Sexual contacts. (PC, CS)<br />

Parenteral exposure to infected blood by needle sharing or transfusion. (PC, CS)<br />

Occupational exposures. (PC, CS)<br />

Other sexually transmitted diseases. (PC, CS)<br />

Tuberculosis exposure. (PC, CS)<br />

Prior HIV serology results, CD4 lymphocyte count and viral load. (PC, CS)<br />

Prior HIV-related opportunistic infections. (PC, CS)<br />

Current/prior antiretroviral medications and their side effects. (PC, CS)<br />

Fever, sweats, anorexia, unintentional weight loss, rash/skin lesions, lymphadenopathy.<br />

(PC, CS)<br />

Cough, sputum production, dyspnea, chest pain. (PC, CS)<br />

Headache, altered mental status, psychiatric complaints. (PC, CS)<br />

Odynophagia, dysphagia. (PC, CS)<br />

Vaginal discharge, history <strong>of</strong> cervical dysplasia or neoplasia. (PC, CS)<br />

Diarrhea. (PC, CS)<br />

Visual changes. (PC, CS)<br />

A dietary history to determine caloric intake. (PC, CS)<br />

2. Physical exam skills: Students should be able to perform a physical exam to establish the<br />

diagnosis and severity <strong>of</strong> disease, including:<br />

General appearance regarding atrophy/wasting/cachexia. (PC)<br />

Complete neurologic examination. (PC)<br />

150


Mental status examination. (PC)<br />

Fundoscopic examination. (PC)<br />

Lymph node examination. (PC)<br />

Skin and oral examination. (PC)<br />

Pelvic and male genital examination. (PC)<br />

3. Differential diagnosis: Students should be able to generate a prioritized differential diagnosis<br />

recognizing specific history and physical exam findings that suggest a specific etiology in an<br />

potentially or known HIV-infected patient for the following:<br />

Fever. (PC, MK)<br />

Unintentional weight loss/wasting/cachexia. (PC, MK)<br />

Lymphadenopathy. (PC, MK)<br />

Rash and skin lesions. (PC, MK)<br />

Cough, sputum production, dyspnea, abnormal chest radiography. (PC, MK)<br />

Diarrhea, odynophagia, dysphagia. (PC, MK)<br />

Altered mental status and psychiatric changes. (PC, MK)<br />

Headache. (PC, MK)<br />

Oral lesions. (PC, MK)<br />

Visual/retinal abnormalities. (PC, MK)<br />

4. Laboratory interpretation: Students should be able to recommend when to order diagnostic<br />

and laboratory tests and be able to interpret them, both prior to and after initiating treatment<br />

based on the differential diagnosis, including consideration <strong>of</strong> test cost and performance<br />

characteristics as well as patient preferences.<br />

Laboratory and diagnostic tests should include, when appropriate:<br />

Specific tests for HIV (e.g. HIV ELISA, confirmatory western blot, quantitative PCR). (PC,<br />

MK)<br />

CD4 lymphocyte count. (PC, MK)<br />

CBC with differential. (PC, MK)<br />

Sputum staining and cultures. (PC, MK)<br />

Blood cultures. (PC, MK)<br />

Cerebrospinal fluid analysis (color, opening pressure, chemistries, cell counts, staining,<br />

cultures, cytology, cryptococcal antigen, VDRL,<br />

Ebstein Barr virus, cytomegalovirus, toxoplasmosis, JC virus). (PC, MK)<br />

Stool for ova and parasites, cryptosporium, isospora, microsporidia, cytomegalovirus<br />

antigen. (PC, MK)<br />

Chest radiograph. (PC, MK)<br />

Students should be able to define the indications for and interpret (with consultation) the<br />

results <strong>of</strong>:<br />

Chest CT. (PC, MK)<br />

Cranial CT. (PC, MK)<br />

Cranial MRI. (PC, MK)<br />

5. Communication skills: Students should be able to:<br />

Communicate the diagnosis, treatment plan, and subsequent follow-up to the patient and<br />

his or her family. (PC, CS)<br />

Elicit input and questions from the patient and his or her family about the management<br />

plan. (PC, CS)<br />

counsel and educate patients about HIV exposure prevention (PC, CS)<br />

Counsel an exposed patient about seroconversion rates and, in appropriate situations, the<br />

availability <strong>of</strong> post-exposure prophylaxis. (PC, CS)<br />

151


Counsel and educate patients about complications <strong>of</strong> HIV drug therapy, drug-drug<br />

interactions, and the marked importance <strong>of</strong> adherence. (PC, CS)<br />

6. Basic and advanced procedural skills: Students should be able to:<br />

Obtain blood cultures. (PC)<br />

Obtain an ABG. (PC)<br />

Place and interpret a PPD. (PC)<br />

Assist in performing a lumbar puncture after explaining the procedure to the patient. (PC)<br />

7. Management skills: Students should able to develop an appropriate evaluation and<br />

treatment plan for patients that includes:<br />

Ordering appropriate laboratory tests. (PC, MK)<br />

Advising patients regarding HIV transmission prevention. (PC, MK)<br />

Insuring antiretroviral adherence. (PC, MK)<br />

Following parameters <strong>of</strong> disease progression/activity. (e.g. CD4 lymphocyte count, viral<br />

load). (PC, MK)<br />

Monitoring for the development <strong>of</strong> side effects from antiretroviral treatment and drug-drug<br />

interactions. (PC, MK)<br />

Insuring the administration <strong>of</strong> appropriate vaccinations. (PC, MK)<br />

Assessing PPD status. (PC, MK)<br />

Prescribing and monitoring appropriate opportunist infection prophylaxis. (PC, MK)<br />

Ordering nutritional supplements to manage and prevent malnutrition. (PC, MK)<br />

Assisting in the procurement <strong>of</strong> proper and ongoing dental care. (PC, MK)<br />

Identifying and recommending community health care resources available for the care <strong>of</strong><br />

AIDS patients. (PC, SBP)<br />

Determining when to obtain consultation from an infectious diseases specialist. (PC, SBP)<br />

Using a cost-effective approach based on the differential diagnosis. (PC, SBP)<br />

Accessing and utilizing appropriate information systems and resources to help delineate<br />

issues related to HIV infection and AIDS. (PC, PLI)<br />

Incorporating patient need and preferences. (PC)<br />

C. ATTITUDES AND PROFESSIONAL BEHAVIORS: Students should be able to:<br />

1. Appreciate the bioethical, social, and legal issues concerning patient confidentiality <strong>of</strong> HIV<br />

infection. (PC, CS)<br />

2. Demonstrate a nonjudgmental attitude regarding the mode <strong>of</strong> HIV acquisition. (P)<br />

3 Appreciate the sometimes severe social stigma <strong>of</strong> HIV infection and AIDS. (P)<br />

4 Show respect <strong>of</strong> “alternative lifestyles.” (P)<br />

5 Demonstrate commitment to using risk-benefit, cost-benefit, and evidence-based<br />

considerations in the selection <strong>of</strong> diagnostic and therapeutic interventions for HIV infection and<br />

AIDS. (PLI, P)<br />

6 Recognize the importance <strong>of</strong> patient needs and preferences when selecting among diagnostic<br />

and therapeutic options for patients with HIV infection or AIDS. (P)<br />

7 Respond appropriately to patients who are nonadherent to antiretroviral treatment. (CS, P)<br />

8 Demonstrate ongoing commitment to self-directed learning regarding HIV infection and AIDS.<br />

(PLI, P)<br />

9 Appreciate the impact HIV infection and AIDS have on a patient’s quality <strong>of</strong> life, well-being,<br />

ability to work, and the family. (P)<br />

10 Recognize the importance <strong>of</strong> and demonstrate a commitment to the utilization <strong>of</strong> other health<br />

care pr<strong>of</strong>essionals in the diagnosis and treatment <strong>of</strong> HIV infection and AIDS. (P, SBP)<br />

152


D. REFERENCES:<br />

Guidelines for preventing Opportunistic Infections Among HIV-Infected Persons-2002.<br />

Recommendations <strong>of</strong> the U.S. Public Health Service and the Infectious Diseases Society <strong>of</strong><br />

America. MMWR 51(RR-08);1-60. Centers for Disease Control and Prevention<br />

U.S. <strong>Department</strong> <strong>of</strong> Health and Human Services<br />

www.cdc.gov/mmwr/PDF/RR/RR5108.pdf<br />

Guidelines for Using Antiretroviral Agents Among HIV-Infected Adults and Adolescents-<br />

2002. Recommendations <strong>of</strong> the Panel on Clinical Practices for Treatment <strong>of</strong> HIV. MMWR<br />

51(RR-07);1-64.<br />

Centers for Disease Control and Prevention<br />

U.S. <strong>Department</strong> <strong>of</strong> Health and Human Services<br />

www.cdc.gov/mmwr/PDF/RR/RR5107<br />

Aberg JA, Gallant JE, Anderson J, Oleske JM, Libman H, Currier JS, Stone VE, Kaplan,<br />

JE. Primary care guideline for the management <strong>of</strong> persons infected with human<br />

immunodeficiency virus: recommendation <strong>of</strong> the HIV <strong>Medicine</strong> Association <strong>of</strong> the Infectious<br />

Diseases Society <strong>of</strong> America. Clin Infect Dis. 2004;39:609-29.<br />

www.journals.uchicago.edu/CID/journal/issues/v39n5/34135/34135.web.p df<br />

www.guideline.gov/summary/summary.aspx?view_id=1&doc_id=5625<br />

AIDSinfo National Institutes <strong>of</strong> Health<br />

U.S. <strong>Department</strong> <strong>of</strong> Health and Human Services<br />

www.aidsinfo.nih.gov<br />

Divisions <strong>of</strong> HIV/AIDS Prevention National Center for HIV, STD and TB Prevention Centers<br />

for Disease Control and Prevention U.S. <strong>Department</strong> <strong>of</strong> Health and Human Services<br />

www.cdc.gov/hiv/dhap.htm<br />

Hammer SM. Management <strong>of</strong> newly diagnosed HIV infection. N Engl J Med.<br />

2005;353:1702-10.<br />

153


TRAINING PROBLEM #24: HYPERTENSION<br />

RATIONALE:<br />

As many as 50 million Americans have elevated blood pressure (systolic pressure 140 mmHg or<br />

greater and/or diastolic blood pressure 90 mmHg or greater) or are taking antihypertensive<br />

medication. Nonfatal and fatal cardiovascular disease (CVD)— including coronary heart disease<br />

(CHD), peripheral vascular disease, stroke and renal disease—all increase progressively with higher<br />

levels <strong>of</strong> both systolic (SBP) and diastolic (DBP) blood pressure levels. These relationships are<br />

strong, continuous, independent, predictive and etiologically significant, and indicate that reduction <strong>of</strong><br />

blood pressure reduces these risks.<br />

PREREQUISITES:<br />

Prior knowledge, skills and attitudes acquired during the pre-clinical years should include:<br />

Ability to perform a complete medical history and physical exam.<br />

Ability to communicate with patients <strong>of</strong> diverse backgrounds.<br />

Knowledge <strong>of</strong> the pathogenesis and pathophysiology <strong>of</strong> hypertension.<br />

Knowledge <strong>of</strong> the epidemiology and risk factors for hypertension.<br />

Understanding <strong>of</strong> the pharmacologic management <strong>of</strong> acute and chronic hypertension.<br />

Understanding the behavioral issues by sex, race, culture, and age that relate to the<br />

management and treatment <strong>of</strong> hypertension.<br />

SPECIFIC LEARNING OBJECTIVES:<br />

A. KNOWLEDGE: Students should be able to define, describe and discuss:<br />

1. The etiologies and relative prevalence <strong>of</strong> primary and secondary hypertension. (MK)<br />

2. The basic principles <strong>of</strong> the role <strong>of</strong> genetics in hypertension. (MK)<br />

3. The definition <strong>of</strong> hypertensive urgency and emergency, citing examples <strong>of</strong> both. (MK)<br />

4. The difference between essential (primary) and secondary hypertension. (MK)<br />

5. Symptoms and signs <strong>of</strong> the following disorders associated with secondary hypertension:<br />

Renovascular hypertension. (MK)<br />

Renal failure. (MK)<br />

Polycystic kidney disease. (MK)<br />

Cushing’s disease or syndrome. (MK)<br />

Hyperaldosteronism. (MK)<br />

Hyperthyroidism. (MK)<br />

Hypercalcemia. (MK)<br />

Medication, alcohol, and illicit drug use. (MK)<br />

Coarctation <strong>of</strong> the aorta. (MK)<br />

Sleep apnea. (MK)<br />

6. The manifestations <strong>of</strong> target-organ disease due to hypertension. (MK)<br />

7. Classification <strong>of</strong> blood pressure (SBP and DBP for all age 18 or older). (MK)<br />

8. Basic approaches to the pharmacological management <strong>of</strong> acute and chronic hypertension,<br />

including the physiologic basis and scientific evidence supporting these approaches, and<br />

causes for lack <strong>of</strong> responsiveness to therapy. (MK)<br />

9. Prevention strategies for reducing hypertension (including lifestyle factors, such as dietary<br />

intake <strong>of</strong> sodium, weight, and exercise level), and explain the physiologic basis and/or<br />

154


scientific evidence supporting each strategy. (MK)<br />

10. Steps in management <strong>of</strong> patients with a hypertensive emergency. (MK)<br />

11. Factors that contribute to non-adherence with antihypertensive medications. (MK)<br />

B. SKILLS: Students should demonstrate specific skills including:<br />

1. History-taking skills: Students should be able to obtain, document, and present an ageappropriate<br />

medical history that differentiates among etiologies <strong>of</strong> disease, including:<br />

Duration and levels <strong>of</strong> elevated blood pressure. (PC, CS)<br />

History <strong>of</strong> symptoms <strong>of</strong> cardiovascular, cerebrovascular, peripheral vascular or renal<br />

disease; diabetes; dyslipidemia; or gout. (PC, CS)<br />

History <strong>of</strong> symptoms suggesting secondary hypertension. (PC, CS)<br />

History <strong>of</strong> weight gain, leisure-time physical activities, and smoking or other tobacco use.<br />

(PC, CS)<br />

Family history <strong>of</strong> high blood pressure, premature CHD, stroke, CVD, diabetes mellitus and<br />

dyslipidemia. (PC, CS)<br />

Psychosocial and environmental factors that may elevate blood pressure (family situation,<br />

employment status, working conditions, education level). (PC, CS)<br />

Dietary assessment, including sodium intake and intake <strong>of</strong> saturated fat and cholesterol.<br />

(PC, CS)<br />

Results and side effects <strong>of</strong> previous antihypertensive therapy. (PC, CS)<br />

Use <strong>of</strong> commonly prescribed, over-the -counter, and illicit medications that may raise blood<br />

pressure or interfere with the effectiveness <strong>of</strong> antihypertensive medications. (PC, CS)<br />

Alcohol intake. (PC, CS)<br />

2. Physical exam skills: Students should be able to perform a physical exam to establish the<br />

diagnosis and severity <strong>of</strong> disease, including:<br />

Blood pressure measurements to detect and confirm the presence <strong>of</strong> high blood pressure.<br />

(PC)<br />

Examination <strong>of</strong> the fundus for arteriolar narrowing, arteriovenous nicking, hemorrhages,<br />

exudates, or papilledema. (PC)<br />

Neck for carotid bruits, distended veins, or an enlarged thyroid gland. ((PC)<br />

Heart for increased rate, increased size, precordial heave, clicks, murmurs, arrhythmias,<br />

and third (S3) and fourth (S4) sounds. (PC)<br />

Abdomen for bruits, enlarged kidneys, masses, and abnormal aortic pulsation. (PC)<br />

Extremities for diminished, delayed, or absent peripheral arterial pulsations, bruits, and<br />

edema. (PC)<br />

Peripheral pulses specifically femoral arterial pulses. (PC)<br />

Body habitus, looking for changes associated with secondary hypertension. (PC)<br />

Peripheral and central nervous system for ischemic changes. (PC)<br />

3. Differential diagnosis: Students should be able to generate a prioritized differential<br />

diagnosis recognizing specific history, physical exam, and laboratory findings that suggest a<br />

specific etiology <strong>of</strong> hypertension. (PC, MK)<br />

4. Laboratory interpretation: Students should be able to recommend and interpret<br />

diagnostic and laboratory tests, both prior to and after initiating treatment based on the<br />

differential diagnosis, including consideration <strong>of</strong> test cost and performance characteristics as<br />

well as patient preferences. Laboratory and diagnostic tests should include, when appropriate:<br />

UA. (PC, MK)<br />

CBC. (PC, MK)<br />

Blood glucose (fasting if possible). (PC, MK)<br />

155


Electrolytes, BUN/Cr. (PC, MK)<br />

Uric acid. (PC, MK)<br />

Fasting lipid pr<strong>of</strong>ile. (PC, MK)<br />

ECG. (PC, MK)<br />

5. Communication skills: Students should be able to:<br />

Communicate the diagnosis, treatment plan and prognosis <strong>of</strong> the disease to the patient and<br />

his or her family, taking into account the patient’s knowledge <strong>of</strong> hypertension and his or her<br />

preferences regarding treatment options. (PC, CS)<br />

Elicit questions from the patient and his or her family about the management plan. (PC, CS)<br />

Educate patients about hypertension risk factors, taking into account:<br />

o Demographics. (PC, CS)<br />

o Concomitant diseases and therapies. (PC, CS)<br />

o Quality <strong>of</strong> life. (PC, CS)<br />

o Physiologic and biochemical measurements. (PC, CS)<br />

o Economic considerations. (PC, CS)<br />

6. Basic and advanced procedural skills: Students should be able to perform:<br />

UA (dipstick and microscopic). (PC)<br />

12-lead ECG. (PC)<br />

7. Management skills: Students should be able to develop an appropriate evaluation and<br />

treatment plan for patients that includes:<br />

Treating acute and chronic hypertension. (PC, MK)<br />

Treating primary (essential) hypertension versus secondary hypertension. (PC, MK)<br />

Using a cost-effective approach based on the differential diagnosis. (PC, SBP)<br />

Prescribing preventative strategies to diminish hypertension, including:<br />

o Weight reduction. (PC, MK)<br />

o Moderation <strong>of</strong> alcohol intake. (PC, MK)<br />

o Regular physical activity. (PC, MK)<br />

o Reduction <strong>of</strong> sodium intake. (PC, MK)<br />

o Increase in potassium intake. (PC, MK)<br />

o Smoking cessation. (PC, MK)<br />

Accessing and utilizing appropriate information systems and resources to help delineate<br />

issues related to hypertension. (PC, PLI)<br />

Incorporating patient preferences. (PC)<br />

C. ATTITUDES AND PROFESSIONAL BEHAVIORS: Students should be able to:<br />

1. Appreciate the importance <strong>of</strong> patient preferences and adherence with management plans for<br />

those with hypertension. (P)<br />

2. Recognize the responsibility <strong>of</strong> the physician with regard to non-adherence. (P)<br />

3. Respond appropriately to patients who are non-adherent to treatment for hypertension. (CS, P)<br />

4. Appreciate how preventative strategies may diminish need for medications. (P)<br />

5. Appreciate the importance <strong>of</strong> side effects <strong>of</strong> medications and their impact on quality <strong>of</strong> life and<br />

adherence (including those side effects to which the geriatric population may be more prone)<br />

and demonstrate a commitment to limiting the whenever possible. (P)<br />

6. Demonstrate commitment to using risk-benefit, cost-benefit, and evidence-based<br />

considerations in the selection <strong>of</strong> diagnostic and therapeutic interventions for hypertension.<br />

(PLI, P)<br />

7. Demonstrate ongoing commitment to self-directed learning regarding hypertension. (PLI, P)<br />

8. Appreciate the impact hypertension has on a patient’s quality <strong>of</strong> life, wellbeing, ability to work,<br />

156


and the family. (P)<br />

9. Recognize the importance and demonstrate a commitment to the utilization <strong>of</strong> other healthcare<br />

pr<strong>of</strong>essions in the treatment <strong>of</strong> hypertension. (P, SBP)<br />

D. REFERENCES:<br />

Chobanian, AV, Bakris, GL, Black, HR, et al. The seventh report <strong>of</strong> the joint national<br />

committee on prevention, detection, evaluation, and treatment <strong>of</strong> high blood pressure:<br />

the JNC 7 report. JAMA. 2003; 289:2560-72.<br />

www.nhlbi.nih.gov/guidelines/hypertension/jncintro.htm<br />

Major cardiovascular events in hypertensive patients randomized to doxazosin vs<br />

chlorthalidone: the antihypertensive and lipid-lowering treatment to prevent<br />

heart attack trial (ALLHAT). ALLHAT Collaborative Research Group. JAMA.<br />

2000;283:1967-75.<br />

ALLHAT Officers and Coordinators for the ALLHAT Collaborative Research Group. The<br />

Antihypertensive and Lipid-Lowering Treatment to Prevent Heart Attack Trial. Major<br />

outcomes in high-risk hypertensive patients randomized to angiotensin-converting<br />

enzyme inhibitor or calcium channel blocker vs diuretic: The Antihypertensive and Lipid-<br />

Lowering Treatment to Prevent Heart Attack Trial (ALLHAT). JAMA. 2002;288:2981-97.<br />

Davis BR, Furberg CD, Wright JT Jr, Cutler JA, Whelton P. ALLHAT Collaborative<br />

Research Group. ALLHAT: setting the record straight. Ann Intern Med. 2004;141:39-<br />

46.<br />

Eisenberg MJ, Brox A, Bestawros AN. Calcium channel blockers: an update. Am J<br />

Med. 2004;116:35-43.<br />

Wing LM, Reid CM, Ryan P, et al. Second Australian National Blood Pressure Study<br />

Group. A comparison <strong>of</strong> outcomes with angiotensin-converting--enzyme inhibitors and<br />

diuretics for hypertension in the elderly. N Engl J Med. 2003;348:583-92.<br />

Agodoa LY, Appel L, Bakris GL, et al. African American Study <strong>of</strong> Kidney Disease and<br />

Hypertension (AASK) Study Group. Effect <strong>of</strong> ramipril vs amlodipine on renal outcomes<br />

in hypertensive nephrosclerosis: a randomized controlled trial. JAMA. 2001;285:2719-<br />

28.<br />

Brenner BM, Cooper ME, de Zeeuw D, et al. RENAAL Study Investigators. Effects <strong>of</strong><br />

losartan on renal and cardiovascular outcomes in patients with type 2 diabetes and<br />

nephropathy. N Engl J Med. 2001;345:861-9.<br />

Lewis EJ, Hunsicker LG, Clarke WR, et al. Collaborative Study Group. Renoprotective<br />

effect <strong>of</strong> the angiotensin-receptor antagonist irbesartan in patients with nephropathy due<br />

to type 2 diabetes. N Engl J Med. 2001;345:851-60.<br />

Tuomilehto J, Rastenyte D, Birkenhager WH, et al. Effects <strong>of</strong> calcium-channel blockade<br />

in older patients with diabetes and systolic hypertension. Systolic Hypertension in<br />

Europe Trial Investigators. N Engl J Med. 1999;340:677-84.<br />

Anonymous. Tight blood pressure control and risk <strong>of</strong> macrovascular and microvascular<br />

complications in type 2 diabetes: UKPDS 38. UK Prospective Diabetes Study Group.<br />

BMJ. 1998;317:703-13.<br />

Yusuf S, Sleight P, Pogue J, Bosch J, Davies R, Dagenais G. Effects <strong>of</strong> an angiotensinconverting-enzyme<br />

inhibitor, ramipril, on cardiovascular events in high-risk patients. The<br />

Heart Outcomes Prevention Evaluation Study Investigators. N Engl J Med.<br />

2000;342:145-53.<br />

Dahl<strong>of</strong> B, Devereux RB, Kjeldsen SE, et al. LIFE Study Group. Cardiovascular<br />

morbidity and mortality in the Losartan Intervention For Endpoint reduction in<br />

hypertension study (LIFE): a randomised trial against atenolol. Lancet. 2002;359:995-<br />

157


1003.<br />

158


TRAINING PROBLEM #25: LIVER DISEASE<br />

RATIONALE:<br />

The causes <strong>of</strong> hepatobiliary disease are many and can be quite overwhelming to the internal<br />

medicine clerk. A thorough understanding <strong>of</strong> a systematic approach to hyperbilirubinemia/jaundice is<br />

by far preferable to random knowledge <strong>of</strong> highly specific etiologies. The liver responds pathologically<br />

to injury in characteristic ways and knowledge <strong>of</strong> these patterns can also be very useful in differential<br />

diagnosis. Several etiologies <strong>of</strong> liver disease such as acute/chronic viral hepatitis and alcoholinduced<br />

liver disease are sufficiently common as to require specific attention. In addition, many liver<br />

diseases can result in cirrhosis and its complications and, therefore, understanding this end-stage<br />

development is important.<br />

PREREQUISITES:<br />

Prior knowledge, skills, and attitudes acquired during the pre-clinical experience should include:<br />

Ability to perform a complete medical history and physical.<br />

Ability to communicate with patients <strong>of</strong> diverse backgrounds.<br />

Anatomy, physiology, and pathophysiology <strong>of</strong> the hepatobiliary system.<br />

SPECIFIC LEARNING OBJECTIVES:<br />

A. KNOWLEDGE: Students should be able to define, describe, and discuss:<br />

1. The biochemical/physiologic/mechanistic approach to hyperbilirubinemia, including:<br />

Increased production. (MK)<br />

Decreased hepatocyte uptake. (MK)<br />

Decreased conjugation. (MK)<br />

Decreased excretion from the hepatocyte. (MK)<br />

Decreased small duct transport (intrahepatic cholestasis). (MK)<br />

Decreased large duct transport (extrahepatic cholestasis, obstructive jaundice). (MK)<br />

2. The biochemistry and common causes <strong>of</strong> unconjugated and conjugated hyperbilirubinemia.<br />

(MK)<br />

3. The use <strong>of</strong> serum markers <strong>of</strong> liver injury (e.g. AST, ALT, GGT, Alk Phos) and function (e.g.<br />

bilirubin, ALB, PT/INR) in the diagnostic evaluation <strong>of</strong> hepatobiliary disease. (MK)<br />

4. The clinical significance <strong>of</strong> asymptomatic, isolated elevation <strong>of</strong> AST, ALT, GGT, and/or Alk<br />

Phos. (MK)<br />

5. The common pathologic patterns <strong>of</strong> liver disease and their common causes, including:<br />

Steatosis (fatty liver). (MK)<br />

Hepatitis. (MK)<br />

Cirrhosis. (MK)<br />

Infiltrative. (MK)<br />

Intrahepatic cholestasis. (MK)<br />

Extrahepatic cholestasis (obstructive jaundice). (MK)<br />

6. The epidemiology, symptoms, signs, typical clinical course, and prevention <strong>of</strong> viral hepatitis.<br />

(MK)<br />

7. The distinctions between acute and chronic hepatitis. (MK)<br />

8. The indications for and efficacy <strong>of</strong> hepatitis A and B vaccinations. (MK)<br />

9. The common causes and clinical significance <strong>of</strong> hepatic steatosis and steatohepatis. (MK)<br />

159


10. The epidemiology, symptoms, signs, and typical clinical course <strong>of</strong> autoimmune liver diseases<br />

such as autoimmune hepatitis, primary biliary cirrhosis, and primary sclerosing cholangitis.<br />

(MK)<br />

11. The epidemiology, symptoms, signs, and typical clinical course <strong>of</strong> cirrhosis. (MK)<br />

12. The pathophysiologic manifestations, symptoms, signs, and complications <strong>of</strong> alcohol-induced<br />

liver disease. (MK)<br />

13. The symptoms, signs, and complications <strong>of</strong> portal hypertension. (MK)<br />

14. The pathophysiology and common causes <strong>of</strong> ascites. (MK)<br />

15. The pathophysiologic manifestations, symptoms, and signs <strong>of</strong> spontaneous bacterial<br />

peritonitis. (MK)<br />

16. The basic pathophysiology, symptoms, signs, typical clinical course, and precipitants <strong>of</strong><br />

hepatic encephalopathy. (MK)<br />

17. The basic pathophysiology, symptoms, signs, and typical clinical course <strong>of</strong> the hepatorenal<br />

syndrome. (MK)<br />

18. The analysis <strong>of</strong> ascitic fluid and its use in the diagnostic evaluation <strong>of</strong> liver disease. (MK)<br />

19. Common causes <strong>of</strong> drug-induced liver injury. (MK)<br />

20. Genetic considerations in liver disease (i.e. hemochromatosis, Wilson’s disease, alpha-1<br />

antitrypsin deficiency, Gilbert’s syndrome). (MK)<br />

21. The epidemiology, pathophysiology, symptoms, signs, and typical clinical course <strong>of</strong><br />

cholelithiasis and cholecystitis. (MK)<br />

22. The clinical syndrome <strong>of</strong> “ascending cholangitis” including its common causes and typical<br />

clinical course. (MK)<br />

23. The indications for and risks <strong>of</strong> paracentesis and liver biopsy. (MK)<br />

24. The indications for and utility <strong>of</strong> hepatobiliary imaging studies, including:<br />

Ultrasound. (MK)<br />

Nuclear medicine studies. (MK)<br />

CT. (MK)<br />

MRI. (MK)<br />

Magnetic resonance cholangiopancreatography (MRCP). (MK)<br />

Endoscopic retrograde cholangiopancreatography (ERCP). (MK)<br />

B. SKILLS: Students should be able to demonstrate specific skills, including:<br />

1. History-taking skills: Students should be able to obtain, document, and present an<br />

age-appropriate medical history, that differentiates among etiologies <strong>of</strong> disease,<br />

including:<br />

Jaundice, discolored urine, pruritis, light-colored stool, unintentional weight loss, fever,<br />

nausea, emesis, diarrhea, altered mental status, abdominal pain, increased abdominal<br />

girth, edema, rectal bleeding, hematemesis. (PC, CS)<br />

DM. (PC, CS)<br />

Alcohol use. (PC, CS)<br />

Prescription, over-the-counter, and illicit drug use. (PC, CS)<br />

Transfusions and other sources <strong>of</strong> potential blood-born pathogen exposure. (PC, CS)<br />

Consumption <strong>of</strong> uncooked shellfish and other food items potentially contaminated with fecal<br />

matter. (PC, CS)<br />

Sexual history. (PC, CS)<br />

Vaccination history. (PC, CS)<br />

Family history <strong>of</strong> liver diseases. (PC, CS)<br />

2. Physical exam skills: Students should be able to perform a physical exam to establish the<br />

160


diagnosis and severity <strong>of</strong> disease, including:<br />

Jaundice. (PC)<br />

Complete abdominal examination including findings consistent with ascites (e.g. bulging<br />

flanks, shifting dullness, fluid wave). (PC)<br />

Findings compatible with chronic alcohol use and portal hypertension (e.g. palmar<br />

erythema, spider angiomas, gynecomastia, testicular atrophy, Dupuytren’s contracture,<br />

muscle wasting, splenomegaly, ascites, edema, caput medusa, hemorrhoids). (PC)<br />

Findings compatible with hepatic (portosystemic) encephalopathy (e.g. disturbances <strong>of</strong><br />

consciousness and behavior, fluctuating neurologic signs, asterixis). (PC)<br />

3. Differential diagnosis: Students should be able to generate a prioritized differential diagnosis<br />

recognizing specific history and physical exam findings that suggest a specific etiology <strong>of</strong> liver<br />

disease. (PC, MK)<br />

4. Laboratory interpretation: Students should be able to recommend when to order diagnostic<br />

and laboratory tests and be able to interpret them, both prior to and after initiating treatment<br />

based on the differential diagnosis, including consideration <strong>of</strong> test cost and performance<br />

characteristics as well as patient preferences. Laboratory and diagnostic tests should include,<br />

when appropriate:<br />

CBC. (PC, MK)<br />

Electrolytes, BUN/Cr, GLC. (PC, MK)<br />

ALB, TP, total bilirubin, direct bilirubin, PT/INR, AST, ALT, Alk Phos. (PC, MK)<br />

Hepatitis serology. (PC, MK)<br />

Ascitic fluid ALB, amylase, cell counts, staining, cultures, and the<br />

serum-ascites albumin gradient (SAAG). (PC, MK)<br />

Students should be able to define the indications for and interpret (with consultation) the<br />

results <strong>of</strong>:<br />

Ultrasound. (PC, MK)<br />

Nuclear medicine studies. (PC, MK)<br />

CT. (PC, MK)<br />

MRI. (PC, MK)<br />

Magnetic resonance cholangiopancreatography (MRCP). (PC, MK)<br />

Endoscopic retrograde cholangiopancreatography (ERCP). (PC, MK)<br />

5. Communication skills: Students should be able to:<br />

Communicate the diagnosis, treatment plan, and subsequent follow-up to the patient and<br />

his or her family. (PC, CS)<br />

Elicit input and questions from the patient and his or her family about the management<br />

plan. (PC, CS)<br />

Discuss the avoidance <strong>of</strong> known hepatotoxins. (PC, CS)<br />

Counsel patients regarding alcohol abstinence. (PC, CS)<br />

Discuss the importance <strong>of</strong> hepatitis A and B vaccinations for nonimmune patients. (PC, CS)<br />

6. Basic and advanced procedural skills: Students should be able to:<br />

Assist in performing a paracentesis after explaining the procedure to the patient. (PC)<br />

7. Management skills: Students should able to develop an appropriate evaluation and<br />

treatment plan for patients that includes:<br />

The diagnostic evaluation <strong>of</strong> asymptomatic, isolated elevation <strong>of</strong> the transaminases and/or<br />

Alk Phos. (PC, MK)<br />

The diagnostic evaluation <strong>of</strong> patients with jaundice and unconjugated or conjugated<br />

hyperbilirubinemia. (PC, MK)<br />

The basic management <strong>of</strong> steatosis, hepatitis, cirrhosis, intra- and extra hepatic<br />

cholestasis, acute cholecystitis, ascites, portal hypertension, spontaneous bacterial<br />

161


peritonitis, and hepatic encephalopathy. (PC, MK)<br />

Determining when to obtain consultation from a gastroenterologist, hepatologist, or biliary<br />

surgeon. (PC, SBP)<br />

Using a cost-effective approach based on the differential diagnosis. (PC, SBP)<br />

Accessing and utilizing appropriate information systems and resources to help delineate<br />

issues related to liver disease. (PC, PLI)<br />

Incorporating patient preferences. (PC)<br />

C. ATTITUDES AND PROFESSIONAL BEHAVIORS: Students should be able to:<br />

1. Demonstrate commitment to using risk-benefit, cost-benefit, and evidence-based<br />

considerations in the selection <strong>of</strong> diagnostic and therapeutic interventions for liver disease. (PLI, P)<br />

2. Recognize the importance <strong>of</strong> patient needs and preferences when selecting among diagnostic<br />

and therapeutic options for liver disease. (P)<br />

3. Respond appropriately to patients who are nonadherent to treatment for liver disease. (CS, P)<br />

4. Demonstrate ongoing commitment to self-directed learning regarding liver disease. (PLI, P)<br />

5. Appreciate the impact liver disease has on a patient’s quality <strong>of</strong> life, wellbeing, ability to work,<br />

and the family. (P)<br />

6. Recognize the importance <strong>of</strong> and demonstrate a commitment to the utilization <strong>of</strong> other<br />

healthcare pr<strong>of</strong>essionals in the diagnosis and treatment <strong>of</strong> liver disease. (P, SBP)<br />

7. Discuss the public health role physicians play in the prevention <strong>of</strong> viral hepatitis. (P, SBP)<br />

8. Appreciate the difficulties patient face with alcohol abstinence. (P)<br />

D. REFERENCES:<br />

Viral Hepatitis<br />

National Center for Infectious Diseases<br />

Center for Disease Control and Prevention<br />

U.S. <strong>Department</strong> <strong>of</strong> Health and Human Services<br />

www.cdc.gov/ncidod/diseases/hepatits/index.htm<br />

Practice Guidelines<br />

American Association for the Study <strong>of</strong> Liver Diseases<br />

www.aasld.org<br />

National Institute on Alcohol Abuse and Alcoholism<br />

National Institutes <strong>of</strong> Health<br />

U.S. <strong>Department</strong> <strong>of</strong> Health and Human Services<br />

www.niaaa.nih.gov<br />

Pratt DS, Kaplan MM. Evaluation <strong>of</strong> abnormal liver-enzyme results in asymptomatic<br />

patients. N Engl J Med. 2000;342:1266-71.<br />

Angulo P. Nonalcoholic fatty liver disease. N Engl J Med. 2002;346:1221-31.<br />

Trowbridge RL, Rutkowski NK, Shojania. Does this patient have acute cholecystitis?<br />

JAMA. 2003;289:80-6.<br />

Indar AA, Beckingham IJ. Acute cholecystitis. BMJ. 2002;325:639-43.<br />

Krige JE, Bechingham IJ. ABC <strong>of</strong> diseases <strong>of</strong> liver, pancreas, and biliary system.<br />

Portal hypertension-1: varices.<br />

Krige JE, Bechingham IJ. ABC <strong>of</strong> diseases <strong>of</strong> liver, pancreas, and biliary system.<br />

Portal hypertension-2. Ascites, encephalopathy, and other conditions. BMJ.<br />

2001;322:416-8.<br />

162


TRAINING PROBLEM #26: MAJOR DEPRESSION<br />

RATIONALE:<br />

Major depression is a very common problem in adults, resulting in significant morbidity and mortality.<br />

Most <strong>of</strong>ten the primary care provider is the first health care pr<strong>of</strong>essional to see a depressed patient.<br />

Frequently, the initial presentation is associated with somatic complaints that bring the patient to the<br />

physician. Major depression is also a relatively common accompaniment to serious medical<br />

conditions. There is significant evidence that primary care physicians commonly fail to diagnose<br />

major depression. With relatively recent improvements in available treatment, it is even more<br />

important for internists to screen for major depression and to know the common presenting<br />

symptoms. The internist should also be familiar with available therapeutic options and be prepared to<br />

treat selected patients, including those who decline consultation with a mental health pr<strong>of</strong>essional.<br />

PREREQUISITES:<br />

Prior knowledge, skills, and attitudes acquired during the pre-clinical experience should include:<br />

Ability to perform a complete medical history and physical.<br />

Ability to communicate with patients <strong>of</strong> diverse backgrounds.<br />

Neurochemistry <strong>of</strong> major depression.<br />

Pharmacology <strong>of</strong> the major classes <strong>of</strong> antidepressants.<br />

Basic understanding <strong>of</strong> the efficacy <strong>of</strong> psychotherapy, antidepressants, and electroconvulsive<br />

therapy.<br />

SPECIFIC LEARNING OBJECTIVES:<br />

A. KNOWLEDGE: Students should be able to define, describe, and discuss:<br />

1. The epidemiology <strong>of</strong> major depression in the general population and the impact <strong>of</strong> major illness<br />

on the prevalence <strong>of</strong> major depression (e.g. stroke, heart disease, DM, cancer, Parkinson’s<br />

disease, HIV/AIDS). (MK)<br />

2. The impact <strong>of</strong> major depression on the outcome <strong>of</strong> medical illness. (MK)<br />

3. The American Psychiatric Associations’ Diagnostic and Statistical Manual 4 th edition (DSM-IV)<br />

diagnostic criteria for major depression. (MK)<br />

4. Common psychological symptoms and signs <strong>of</strong> major depression (e.g. low mood/affect,<br />

anxiety, irritability/anger, disinterest, anhedonia, decreased libido, guilt, poor self-esteem, poor<br />

concentration, rumination, helplessness, hopelessness, thoughts <strong>of</strong> death and suicide, somatic<br />

complaints). (MK)<br />

5. Common neurovegetative symptoms and signs <strong>of</strong> major depression (e.g. appetite disturbance,<br />

decreased energy, psychomotor retardation or agitation, sleep disturbance). (MK)<br />

6. Common somatic complaints that accompany depressive disorders and the potential for the<br />

occurrence <strong>of</strong> these symptoms without obvious psychological symptoms (e.g. fatigue,<br />

weakness, myalgias, arthralgias, headache, nausea, dyspnea, palpitations, chest<br />

pain/discomfort, lightheadedness/dizziness, bowel movement alterations). (MK)<br />

7. The distinguishing features <strong>of</strong> major depression with psychotic features, bipolar disorder,<br />

dementia, and delirium. (MK)<br />

8. The differential diagnosis <strong>of</strong> major depression, including:<br />

Other psychiatric disorders. (MK)<br />

Drug-induced (e.g. corticosteroids, cimetidine, metoclopramide, clonidine, etc.). (MK)<br />

163


Drug withdrawal (e.g. amphetamine, cocaine). (MK)<br />

Infection (e.g. tertiary syphilis). (MK)<br />

Endocrine/metabolic (e.g. hypo/hyperthyroidism, Cushing’s, Addison’s). (MK)<br />

Collagen vascular diseases (e.g. lupus, fibromyalgia). (MK)<br />

Neurologic (e.g. stoke, multiple sclerosis, Parkinson’s disease, head trauma, complex<br />

partial seizures). (MK)<br />

Nutritional (e.g. B12, folate, niacin, thiamine deficiencies). (MK)<br />

Neoplastic (e.g. pancreatic cancer, disseminated carcinomatosis). (MK)<br />

9. US Preventive Services Task Force (USPSTF) depression screening recommendations. (MK)<br />

10. The risks <strong>of</strong> untreated major depression. (MK)<br />

11. Assessment <strong>of</strong> the risk <strong>of</strong> suicide. (MK)<br />

12. The demographics and risk factors for completed suicide. (MK)<br />

13. The potential link between major depression and substance abuse. (MK)<br />

14. The potential role <strong>of</strong> genetics in depression.(MK)<br />

15. Indications and efficacy <strong>of</strong> the basic therapeutic options for major depression, including:<br />

Psychotherapy (cognitive behavioral therapy or interpersonal psychotherapy). (MK)<br />

Pharmacotherapy. (MK)<br />

Electroconvulsive therapy. (MK)<br />

16. The side effects <strong>of</strong> the major classes <strong>of</strong> antidepressants and common interaction with other<br />

medications. (MK)<br />

B. SKILLS: Students should be able to demonstrate specific skills including:<br />

1. History-taking skills: Students should be able to obtain, document, and present an ageappropriate<br />

medical history, that differentiates among etiologies <strong>of</strong> disease including:<br />

Eliciting the symptoms <strong>of</strong> major depression. (PC, CS)<br />

Determining the presence or absence <strong>of</strong> underlying dementia, anxiety disorders, adverse<br />

drug effects, and grief in any patient suspected <strong>of</strong> having major depression. (PC, CS)<br />

Obtaining a complete drug history (including illicit drugs). (PC, CS)<br />

Identifying chronic diseases that are associated with increased risk <strong>of</strong> major depression.<br />

(PC, CS)<br />

2. Physical exam skills: Students should be able to perform a physical exam to establish the<br />

diagnosis and severity <strong>of</strong> disease, including:<br />

A complete neurologic examination. (PC)<br />

A complete mental status exam. (PC)<br />

5. Differential diagnosis: Students should be able to generate a prioritized differential diagnosis<br />

recognizing specific history and physical exam findings that suggest a specific etiology for<br />

major depression (psychiatric and nonpsychiatric). (PC, MK)<br />

6. Laboratory interpretation: Students should be able to recommend when to order diagnostic<br />

and laboratory tests and be able to interpret them, both prior to and after initiating treatment<br />

based on the differential diagnosis, including consideration <strong>of</strong> test cost and performance<br />

characteristics as well as patient preferences.<br />

Laboratory and diagnostic tests should include, when appropriate:<br />

Blood and urine drug screening. (PC, MK)<br />

Thyroid function tests. (PC, MK)<br />

Serum RPR and VDRL. (PC, MK)<br />

B12, folate, and thiamine levels. (PC, MK)<br />

Students should be able to define the indications for and interpret (with consultation) the<br />

results <strong>of</strong>:<br />

164


Cranial CT. (PC, MK)<br />

Cranial MRI. (PC, MK)<br />

7. Communication skills: Students should be able to:<br />

Communicate the diagnosis, treatment plan, and subsequent follow-up to the patient and<br />

his or her family. (PC, CS)<br />

Elicit input and questions from the patient and his or her family about the management<br />

plan. (PC, CS)<br />

Demonstrate effective listening skills and empathy. (PC, CS)<br />

Advise the patient <strong>of</strong> the delay in therapeutic benefit from antidepressant medications. (PC,<br />

CS)<br />

6. Management skills: Students should able to develop an appropriate evaluation and treatment<br />

plan for patients that includes:<br />

An appreciation <strong>of</strong> the fact that major depression is not generally a “diagnosis <strong>of</strong> exclusion”<br />

and that ruling out all other possible medical causes is typically not necessary. (PC, MK)<br />

Making an accurate diagnosis <strong>of</strong> major depression. (PC, MK)<br />

Assessing for the risk <strong>of</strong> suicide. (PC, MK)<br />

Recommending psychotherapy (cognitive behavioral therapy or interpersonal<br />

psychotherapy). (PC, MK)<br />

Selecting appropriate initial pharmacologic therapy considering efficacy, side effects, and<br />

potential drug-drug interactions. (PC, MK, SBP)<br />

Identifying barriers to major depression treatment. (PC, SBP)<br />

Anticipating potential resistance to seeing a psychiatrist and antidepressant treatment. (PC)<br />

Planning appropriate follow-up. (PC, MK)<br />

Recognizing success or failure <strong>of</strong> initial treatment and making appropriate adjustments.<br />

(PC, MK)<br />

Determining when to obtain consultation from a psychiatrist, psychologist, or other mental<br />

health pr<strong>of</strong>essional. (PC, SBP)<br />

Using a cost-effective approach to treatment. (PC, SBP)<br />

Accessing and utilizing appropriate information systems and resources to help delineate<br />

issues related to major depression. (PC, PLI)<br />

Incorporating patient needs and preferences. (PC)<br />

C. ATTITUDES AND PROFESSIONAL BEHAVIORS: Students should be able to:<br />

1. Recognize major depression as an important and potentially life-threatening disease. (P)<br />

2. Appreciate the social stigma <strong>of</strong> psychiatric diagnoses and the ways non-psychiatric physicians<br />

may inadvertently contribute to this. (P)<br />

3. Appreciated the reluctance <strong>of</strong> some patients to see a psychiatrist. (P)<br />

4. Demonstrate commitment to using risk-benefit, cost-benefit, and evidence-based<br />

considerations in the selection <strong>of</strong> diagnostic and therapeutic interventions for major depression. (PLI,<br />

P)<br />

5. Recognize the importance <strong>of</strong> patient needs and preferences when selecting among diagnostic<br />

and therapeutic options for major depression. (P)<br />

6. Respond appropriately to patients who are nonadherent to treatment for major depression.<br />

(CS, P)<br />

7. Demonstrate ongoing commitment to self-directed learning regarding major depression. (PLI,<br />

P)<br />

8. Appreciate the impact major depression has on a patient’s quality <strong>of</strong> life, wellbeing, ability to<br />

work, and the family. (P)<br />

165


9. Recognize the importance <strong>of</strong> and demonstrate a commitment to the utilization <strong>of</strong> other health<br />

care pr<strong>of</strong>essionals in the diagnosis and treatment <strong>of</strong> major depression. (P, SBP)<br />

D. REFERENCES:<br />

Depression<br />

National Institute <strong>of</strong> Mental Health<br />

National Institutes <strong>of</strong> Health<br />

U.S. <strong>Department</strong> <strong>of</strong> Health and Human Services<br />

www.nimh.nih.gov/publicat//defresssion.cfm<br />

Practice Guidelines<br />

American Psychiatric Association<br />

www.psych.org/psych_pract/treatg/pg/prac_guide.cfm<br />

The National Association on Mental Illness<br />

www.nami.org/Template.cfm?Section=By_Illness&Template=/TaggedPa<br />

ge/TaggedPageDisplay.cfm&TPLID=54&ContentID=26414<br />

Mann JJ. The medical management <strong>of</strong> depression. N Engl J Med. 2005;353:1819-34.<br />

166


TRAINING PROBLEM #27: NOSOCOMIAL INFECTIONS<br />

RATIONALE:<br />

Nosocomial infections have been occurring since the inception <strong>of</strong> the hospital. Despite many<br />

advances the incidence is still roughly five percent <strong>of</strong> all acute care hospitalizations or about two<br />

million cases a year. Nosocomial infections are estimated to approximately double the morbidity and<br />

mortality rates <strong>of</strong> any person admitted to the hospital. Directly attributable deaths can total up to<br />

88,000 per year with the expenditure <strong>of</strong> many millions <strong>of</strong> excess health care dollars. Preventing<br />

nosocomial infections is the responsibility <strong>of</strong> every heath care worker, including physicians, house<br />

<strong>of</strong>ficers, medical students, nurses, technicians, administrators, etc. Also considered here are<br />

occupational exposures for which health care workers are at risk.<br />

PREREQUISITES:<br />

Prior knowledge, skills, and attitudes acquired during the pre-clinical experience should include:<br />

Ability to perform a complete medical history and physical.<br />

Ability to communicate with patients <strong>of</strong> diverse backgrounds.<br />

Basic training in body substance isolation procedures.<br />

Microbiology and pathophysiology <strong>of</strong> the common nosocomial organisms, including<br />

Staphylococcus aureus (methicillin sensitive and resistant), Staphylococcus epidermidis,<br />

Enterococcus species (vancomycin sensitive and resistant), Pseudomonas aeruginosa and<br />

other nosocomial gram-negative bacilli, Clostridium difficile, and Candida species.<br />

The pharmacology <strong>of</strong> antimicrobial agents.<br />

SPECIFIC LEARNING OBJECTIVES:<br />

A. KNOWLEDGE: Students should be able to define, describe, and discuss:<br />

1. The epidemiology and significance <strong>of</strong> nosocomial infections in the United States. (MK)<br />

2. The general clinical risk factors for nosocomial infection. including:<br />

Immunocompromise. (MK)<br />

Immunosuppressive drugs. (MK)<br />

Extremes <strong>of</strong> age. (MK)<br />

Compromise <strong>of</strong> the skin and mucosal surfaces secondary to:<br />

o Drugs. (MK)<br />

o Irradiation. (MK)<br />

o Trauma. (MK)<br />

o Invasive diagnostic and therapeutic procedures. (MK)<br />

o Invasive indwelling devises (e.g. intravenous catheter, bladder catheter, endotracheal<br />

tube, etc.). (MK)<br />

3. The major routes <strong>of</strong> nosocomial infection transmission, including:<br />

Contact. (MK)<br />

Droplet. (MK)<br />

Airborne. (MK)<br />

Common vehicle. (MK)<br />

4. The epidemiology, pathophysiology, microbiology, symptoms, signs, typical clinical course,<br />

and preventive strategies for the most common nosocomial infections, including:<br />

Urinary tract infection. (MK)<br />

167


Pneumonia. (MK)<br />

Surgical site infection. (MK)<br />

Intravascular devised-related bloodstream infections. (MK)<br />

Skin infections. (MK)<br />

Health care associated diarrhea. (MK)<br />

5. Empiric antibiotic therapy for the most common nosocomial infections. (MK)<br />

6. The epidemiology, pathophysiology, microbiology, symptoms, signs, typical clinical course,<br />

and preventive strategies for colonization or infection with the following organisms:<br />

Vancomycin-resistant enterococci. (MK)<br />

Clostridium difficile. (MK)<br />

Methicillin-resistant Staphylococcus aureus. (MRSA) (MK)<br />

Multidrug-resistant Gram-negative bacteria. (MK)<br />

7. The crucial importance <strong>of</strong> judicious antibiotic use. (MK)<br />

8. The effect <strong>of</strong> widespread use <strong>of</strong> broad spectrum anti-microbial agents on endogenous body<br />

flora and the hospital microbial flora. (MK)<br />

9. The types <strong>of</strong> isolation procedures and their indications:<br />

Standard. (MK)<br />

Airborne. (MK)<br />

Contact. (MK)<br />

Droplet. (MK)<br />

10. The Centers for Disease Control and Prevention (CDC) guidelines for hand hygiene. (MK)<br />

11. Preventive strategies for needlestick and sharps injuries intended to reduce the transmission <strong>of</strong><br />

bloodborne pathogens (hepatitis B, hepatitis C, and HIV). (MK)<br />

12. Local hospital post-exposure (i.e. after an eye/mucous membrane splash, needlestick or other<br />

sharps injury) protocols for prompt reporting, evaluation, counseling, treatment, and follow-up.<br />

(MK, SBP)<br />

13. The indications, efficacy, and side effects <strong>of</strong> post-exposure prophylaxis for hepatitis B and<br />

HIV/AIDS. (MK)<br />

14. negative-pressure ventilation isolation for known or suspected tuberculosis patients (MK)<br />

15. National Institute for Occupational Safety and Health (NIOSH) approved personal respiratory<br />

protective equipment (i.e. N95 respirator) use for the prevention <strong>of</strong> transmission <strong>of</strong><br />

Mycobacterium tuberculosis to health care workers. (MK)<br />

B. SKILLS: Students should be able to demonstrate specific skills, including:<br />

3. History-taking skills: Students should be able to obtain, document, and present an ageappropriate<br />

medical history, that differentiates among etiologies <strong>of</strong> disease in the organ systems<br />

likely to be involved with nosocomial infection. (PC, CS)<br />

4. Physical exam skills: Students should be able to perform a physical examination <strong>of</strong> skin,<br />

vascular access sites, lungs, abdomen, wounds, and catheter and drain sites and recognize signs<br />

<strong>of</strong> local or systemic infection (PC)<br />

3. Differential diagnosis: Students should be able to generate a prioritized differential diagnosis<br />

<strong>of</strong> the likely sites and organisms involved, recognizing specific history and physical exam<br />

findings that suggest a specific etiology. (PC, MK)<br />

4. Laboratory interpretation: Students should be able to recommend when to order diagnostic<br />

and laboratory tests and be able to interpret them, both prior to and after initiating treatment<br />

based o the differential diagnosis, including consideration <strong>of</strong> test cost and performance<br />

168


characteristics as well as patient preferences.<br />

Laboratory and diagnostic tests should include, when appropriate:<br />

Urinalysis and culture and sensitivities. (PC, MK)<br />

Sputum Gram stain and culture and sensitivities. (PC, MK)<br />

Chest radiograph. (PC, MK)<br />

Wound cultures and sensitivities. (PC, MK)<br />

Clostridium difficile toxin assay. (PC, MK)<br />

Hepatitis serologies. (PC, MK)<br />

HIV ELISA and western blot. (PC, MK)<br />

Sputum AFB staining and culture. (PC, MK)<br />

5. Communication skills: Students should be able to:<br />

Communicate the diagnosis, treatment plan, and subsequent follow-up to the patient and<br />

his or her family. (PC, CS)<br />

Elicit input and questions from the patient and his or her family about the management<br />

plan. (PC, CS)<br />

Explain the necessity for isolation procedures. (PC, CS)<br />

Counsel patients about the need for judicious antibiotic usage and the potential patientspecific<br />

and public health risks <strong>of</strong> not doing so. (PC, CS)<br />

6. Basic and advanced procedural skills: Students should be able to:<br />

Obtain blood cultures. (PC)<br />

Place and interpret a PPD. (PC)<br />

Demonstrate proper sterile technique for invasive procedures. (PC)<br />

7. Management skills: Students should able to develop an appropriate evaluation and<br />

treatment plan for patients that includes:<br />

Assessing a hospitalized patient who develops a new fever 48 or more hours after<br />

admission. (PC, MK)<br />

Developing a plan for the evaluation and treatment <strong>of</strong> hospital acquired infection. (PC, MK)<br />

Demonstrating appropriate choice <strong>of</strong> antimicrobial drugs which considers mechanisms <strong>of</strong><br />

action, spectrum <strong>of</strong> activity, pharmacokinetics, drug interactions, and adverse reactions.<br />

(PC, MK)<br />

Recognizing when indwelling intravascular and urinary collection devices should be<br />

removed. (PC, MK)<br />

Requesting appropriate isolation measures to protect other patients and health care<br />

workers. (PC, SBP)<br />

Determining when to obtain consultation from an infectious diseases specialist. (PC, SBP)<br />

Contacting hospital infection control experts when appropriate. (SBP)<br />

Using a cost-effective approach based on the differential diagnosis. (PC, SBP)<br />

Accessing and utilizing appropriate information systems and resources to help delineate<br />

issues related to nosocomial infections. (PC, PLI)<br />

Incorporating patient needs and preferences. (PC)<br />

C. ATTITUDES AND PROFESSIONAL BEHAVIORS: Students should be able to:<br />

1. Serve as a role model to all other health care providers by strictly following all infection control<br />

measures including hand hygiene and all isolation procedures. (P, SBP)<br />

2. Appreciate the role physicians play in the inappropriate prescribing <strong>of</strong> antimicrobial agents and<br />

the public health ramifications. (P, SBP)<br />

3. Demonstrate commitment to using risk-benefit, cost-benefit, and evidence-based<br />

considerations in the selection <strong>of</strong> diagnostic and therapeutic interventions for nosocomial infections.<br />

169


(PLI, P)<br />

4. Recognize the importance <strong>of</strong> patient needs and preferences when selecting among diagnostic<br />

and therapeutic options for nosocomial infections. (P)<br />

5. Demonstrate ongoing commitment to self-directed learning regarding nosocomial infections.<br />

(PLI, P)<br />

6. Appreciate the impact nosocomial infections have on a patient’s quality <strong>of</strong> life, well-being,<br />

ability to work, and the family. (P)<br />

7. Recognize the importance <strong>of</strong> and demonstrate a commitment to the utilization <strong>of</strong> other health<br />

care pr<strong>of</strong>essionals in the diagnosis, treatment, and prevention <strong>of</strong> nosocomial infections. (P, SBP)<br />

D. REFERENCES:<br />

Infection Control Guidelines<br />

Division <strong>of</strong> Healthcare Quality Promotion<br />

National Center for Infectious Diseases<br />

Centers for Disease Control and Prevention<br />

U.S. <strong>Department</strong> <strong>of</strong> Health and Human Services<br />

www.cdc.gov/ncidod/hip/default.htm<br />

Vancomycin-Resistant Enterococci Division <strong>of</strong> Healthcare<br />

Quality Promotion National Center for Infectious Diseases<br />

Centers for Disease Control and Prevention<br />

U.S. <strong>Department</strong> <strong>of</strong> Health and Human Services<br />

www.cdc.gov/ncidod/hip/ARESIST/vre.htm<br />

Pautanen SM, Simor AE. Clostridium difficile associated diarrhea in adults. Can Med<br />

Assoc J. 2004;171:51-8.<br />

www.cmaj.ca/cgi/reprint/171/1/51<br />

Healthcare-Associated MRSA Division <strong>of</strong> Healthcare Quality Promotion National Center<br />

for Infectious Diseases Centers for Disease Control and Prevention<br />

U.S. <strong>Department</strong> <strong>of</strong> Health and Human Services<br />

www.cdc.gov/ncidod/dhqp/ar_mrsa.html<br />

170


TRAINING PROBLEM #28: OBESITY<br />

RATIONALE:<br />

Obesity and overweight are recognized as ever growing epidemics in the United States. These<br />

conditions have been correlated with the development <strong>of</strong> medical conditions such as diabetes,<br />

hypertension, heart disease, and osteoarthritis. Mastery <strong>of</strong> the approach to patients who are not at<br />

an ideal body weight is important to general internists because they <strong>of</strong>ten deal with the sequelae <strong>of</strong><br />

the comorbid illnesses.<br />

PREREQUISITES:<br />

Prior knowledge, skills, and attitudes acquired during the pre-<strong>clerkship</strong> experience should include:<br />

Ability to perform a complete medical history and physical exam.<br />

Ability to communicate with patients <strong>of</strong> diverse backgrounds.<br />

Psychology associated with addictive behavior.<br />

Anatomy, physiology, and pathophysiology <strong>of</strong> the gastrointestinal tract and digestion.<br />

Pharmacology <strong>of</strong> the available drugs used to treat obesity.<br />

Nutrition and caloric requirements.<br />

SPECIFIC LEARNING OBJECTIVES:<br />

A. KNOWLEDGE: Students should be able to define, describe, and discuss:<br />

1. The etiology <strong>of</strong> obesity including excessive caloric intake, insufficient energy expenditure<br />

leading to low resting metabolic rate, genetic predisposition, environmental factors affecting weight<br />

gain, psychologic stressors, and lower socioeconomic status. (MK)<br />

2. The definition and classification <strong>of</strong> overweight and obese using BMI. (MK)<br />

3. The health implications that being overweight or obese may have on the patient. (MK)<br />

4. How daily caloric requirements are calculated and the caloric deficit required to achieve a five<br />

to 10 percent weight reduction in six to 12 months. (MK)<br />

5. The principles <strong>of</strong> behavior modification. (MK)<br />

6. How to develop an exercise program and assist the patient in setting goals for weight loss.<br />

(MK)<br />

7. Treatment options, including nonpharmacologic and pharmacologic treatment, behavioral<br />

therapy and surgical intervention. (MK)<br />

B. SKILLS: Students should be able to demonstrate specific skills including:<br />

1. History-taking skills: Students should be able to obtain, document, and present an<br />

age-appropriate medical history, including:<br />

Reviewing the patient’s weight history from childhood. (PC, CS)<br />

Assessing the risk factors for obesity related conditions. (PC, CS)<br />

Assessing the patient’s motivation for losing weight. (PC, CS)<br />

Reviewing the patient’s past experience with losing weight and determining barriers<br />

encountered in prior attempts. (PC, CS)<br />

Reviewing the patient’s activity level and diet. (PC, CS)<br />

Obtaining an assessment <strong>of</strong> tobacco and drug use especially noting if the patient is in the<br />

171


process <strong>of</strong> stopping either. (PC, CS)<br />

Obtaining a family history focusing on weight related issues and comorbid illnesses<br />

associated with obesity. (PC, CS)<br />

Obtaining a focused review <strong>of</strong> systems including signs and symptoms <strong>of</strong> secondary causes<br />

<strong>of</strong> obesity such as Cushing’s syndrome, hypothyroidism, and hypogonadism. (PC, CS)<br />

2. Physical exam skills: Students should be able to perform a physical exam to establish the<br />

diagnosis and severity <strong>of</strong> disease, including:<br />

Calculation <strong>of</strong> degree <strong>of</strong> obesity from the patient’s height and weight by calculating BMI.<br />

(PC)<br />

Noting the presence <strong>of</strong> abdominal obesity based on waist-to-hip circumference. (PC)<br />

Assessing the signs <strong>of</strong> vascular disease including hypertension, carotid bruits, abdominal<br />

aortic size, blood pressure and peripheral pulses. (PC)<br />

Assessing for signs <strong>of</strong> endocrine abnormalities, including: striae, peripheral neuropathy,<br />

depressed tendon reflexes, bruising, and signs <strong>of</strong> dyslipidemia (e.g. xanthomas and<br />

xanthalasma). (PC)<br />

3. Differential diagnosis: Students should be able to generate a prioritized differential diagnosis<br />

recognizing specific history and physical exam findings that suggest a specific etiology <strong>of</strong><br />

primary and secondary obesity. (MK, PC)<br />

4. Laboratory interpretation: Students should be able to recommend when to order diagnostic<br />

and laboratory tests and be able to interpret them, both prior to and after initiating treatment<br />

based on the differential diagnosis, including consideration <strong>of</strong> test cost and performance<br />

characteristics as well as patient preferences.<br />

Laboratory and diagnostic tests should include, when appropriate:<br />

Serum GLC. (PC, MK)<br />

TSH. (PC, MK)<br />

Lipid pr<strong>of</strong>ile. (PC, MK)<br />

HbA1c. (PC, MK)<br />

BUN/Cr. (PC, MK)<br />

Urine microalbumin. (PC, MK)<br />

ECG. (PC, MK)<br />

24-hour urinary cortisol (PC, MK)<br />

5. Communication skills: Students should be able to:<br />

Communicate the diagnosis, treatment plan, and subsequent follow-up to patients. (PC,<br />

CS)<br />

Elicit questions from the patient and his or her family about the management plan. (PC, CS)<br />

Adapt to the patient’s life-style and preferences, with emphasis on the patient’s role in<br />

treatment and maximizing compliance. (PC, CS)<br />

Assist the patient in understanding that attainment <strong>of</strong> ideal body weight may not necessarily<br />

be a realistic goal and that health benefits may be achieved with losses <strong>of</strong> five to 10<br />

percent body weight. (PC, CS)<br />

6. Management skills: Students should able to develop an appropriate evaluation and treatment<br />

plan for patients that includes:<br />

Determining when to obtain consultation from an endocrinologist, dietician, or obesity<br />

management specialist. (PC, SBP)<br />

Developing reasonable weight loss goals with the patient. (PC, MK)<br />

Developing a dietary plan. (PC, MK)<br />

Developing a prescription for physical activity. (PC, MK)<br />

Identifying indications for pharmacotherapy. (PC, MK)<br />

Identifying indications for bariatric surgery. (PC, MK)<br />

172


Accessing and utilizing appropriate information systems and resources to help delineate<br />

issues related to obesity. (PC, PLI)<br />

Incorporating patient preferences in the treatment plan. (PC)<br />

C. ATTITUDES AND PROFESSIONAL BEHAVIORS: Students should be able to:<br />

1. Demonstrate commitment to using risk-benefit, cost-benefit, and evidence-based<br />

considerations in the selection diagnostic and therapeutic interventions for obesity. (PLI, P)<br />

2. Respond appropriately to patients who are nonadherent to treatment for obesity.<br />

(CS, P)<br />

3. Demonstrate ongoing commitment to self-directed learning regarding obesity. (PLI,<br />

P)<br />

4. Appreciate the impact obesity has on a patient’s quality <strong>of</strong> life, well-being, ability to<br />

work, and family. (P)<br />

5. Recognize the importance <strong>of</strong> and demonstrate a commitment to the utilization <strong>of</strong><br />

other healthcare pr<strong>of</strong>essions in the treatment <strong>of</strong> obesity. (P, SBP)<br />

D. REFERENCES:<br />

Overweight and Obesity<br />

National Center for Chronic Disease Prevention and Health Promotion Center for Disease<br />

Control and Prevention<br />

U.S. <strong>Department</strong> <strong>of</strong> Health and Human Services<br />

www.cdc.gov/nccdphp/dnpa/obesity<br />

Aim for a Healthy Weight National Heart, Lung, and Blood Institute Obesity Education<br />

Initiative National Institutes <strong>of</strong> Health<br />

U.S. <strong>Department</strong> <strong>of</strong> Health and Human Services<br />

www.nhlbi.nih.gov/health/public/heart/obesity/lose_wt/index.htm<br />

Screening for Obesity in Adults<br />

Agency for Healthcare Research and Quality<br />

U.S. <strong>Department</strong> <strong>of</strong> Health and Human Services<br />

www.ahrq.gov/clinic/uspstf/uspsobes.htm<br />

Tsai AG, Wadden TA. Systematic review: an evaluation <strong>of</strong> major commercial weight loss<br />

programs in the United States. Ann Intern Med. 2005;152-56-66.<br />

Buchwald H, Avidor Y, Braunwald E, et al. Bariatric surgery: a systematic review and<br />

meta-analysis. JAMA. 2004;292:1724-37.<br />

Li Z, Maglione M, Tu W, et al. Meta-analysis: pharmacologic treatment <strong>of</strong> obesity. Ann<br />

Intern Med. 2005;142:532-46. \<br />

173


TRAINING PROBLEM #29: PNEUMONIA<br />

RATIONALE:<br />

Pneumonia continues to be a major public health issue, a leading reason for hospitalization, and<br />

a significant cause <strong>of</strong> mortality. Not only that, it is an important complication <strong>of</strong> admission for<br />

other causes. Many different specialties encounter pneumonia in the course <strong>of</strong> practice, the<br />

internist most particularly.<br />

PREREQUISITES:<br />

Prior knowledge, skills, and attitudes acquired during the pre-clinical experience should include:<br />

Ability to perform a complete medical history and physical.<br />

Ability to communicate with patients <strong>of</strong> diverse backgrounds.<br />

Anatomy and physiology <strong>of</strong> the pulmonary system.<br />

Pathogenesis and pathophysiology <strong>of</strong> pneumonia.<br />

Microbiology <strong>of</strong> the common pneumonia pathogens.<br />

Pharmacology <strong>of</strong> antimicrobial agents.<br />

SPECIFIC LEARNING OBJECTIVES:<br />

A. KNOWLEDGE: Students should be able to define, describe, and discuss:<br />

1. The epidemiology, pathophysiology, symptoms, signs, and typical clinical course <strong>of</strong><br />

community-acquired, nosocomial, and aspiration pneumonia and pneumonia in the<br />

immunocompromised host. (MK)<br />

2. The conceptualization <strong>of</strong> “typical” and “atypical” pneumonia and its limitations. (MK)<br />

3. Common pneumonia pathogens (viral, bacterial, mycobacterial, and fungal) in<br />

immunocompetent and immunocompromised hosts). (MK)<br />

4. Identify patients who are at risk for impaired immunity. (MK)<br />

5. Indications for hospitalization and ICU admission <strong>of</strong> patient with pneumonia. (MK)<br />

6. The radiographic findings <strong>of</strong> the various types <strong>of</strong> pneumonia. (MK)<br />

7. The antimicrobial treatments (e.g. antiviral, antibacterial, antimycobacterial, and antifungal)<br />

for community-acquired, nosocomial, and aspiration pneumonia, and pneumonia in the<br />

immunocompromised host. (MK)<br />

8. The implications <strong>of</strong> antimicrobial resistance. (MK)<br />

9. The pathogenesis, symptoms, and signs <strong>of</strong> the complications <strong>of</strong> acute bacterial pneumonia<br />

including: bacteremia, sepsis, parapneumonic effusion, empyema, meningitis, and<br />

metastatic microabscesses. (MK)<br />

10. The indications for and complications <strong>of</strong> chest tube placement. (MK)<br />

11. The indications for and efficacy <strong>of</strong> influenza and pneumococcal vaccinations. (MK)<br />

12. The indications and procedures for respiratory isolation. (MK)<br />

13. The Centers for Medicare & Medicaid Services (CMS) and the Joint Commission on the<br />

Accreditation <strong>of</strong> Healthcare Organizations (JCAHO) quality measures for communityacquired<br />

pneumonia treatment. (MK, PLI, SBP)<br />

B. SKILLS: Students should be able to demonstrate specific skills including:<br />

1. History-taking skills: Students should be able to obtain, document, and present an ageappropriate<br />

medical history that differentiates among etiologies <strong>of</strong> disease, including:<br />

174


The presence and quantification <strong>of</strong> fever, chills, sweats, cough, sputum, hemoptysis,<br />

dyspnea, and chest pain. (PC, CS)<br />

Historical features consistent with potential immunocompromise. (PC, CS)<br />

potential tuberculosis exposure (PC, CS)<br />

2. Physical exam skills: Students should be able to perform a physical exam to establish the<br />

diagnosis and severity <strong>of</strong> disease, including:<br />

Accurately determining respiratory rate and level <strong>of</strong> respiratory distress. (PC)<br />

Identifying bronchial breath sounds, rales, rhonchi, and wheezes. (PC)<br />

Identifying signs <strong>of</strong> pulmonary consolidation. (PC)<br />

Identifying signs <strong>of</strong> pleural effusion. (PC)<br />

Identifying signs <strong>of</strong> the complications <strong>of</strong> pneumonia. (PC)<br />

3. Differential diagnosis: Students should be able to generate a prioritized differential<br />

diagnosis recognizing specific history and physical exam findings that suggest a specific<br />

etiology <strong>of</strong> pneumonia and other possible diagnoses, including:<br />

Common cold. (PC, MK)<br />

Acute bronchitis. (PC, MK)<br />

Influenza. (PC, MK)<br />

Acute exacerbation <strong>of</strong> COPD. (PC, MK)<br />

Asthma exacerbation. (PC, MK)<br />

CHF. (PC, MK)<br />

Pulmonary embolism. (PC, MK)<br />

Aspiration. (PC, MK)<br />

4. Laboratory interpretation: Students should be able to recommend when to order diagnostic<br />

and laboratory tests and be able to interpret them, both prior to and after initiating treatment<br />

based on the differential diagnosis, including consideration <strong>of</strong> test cost and performance<br />

characteristics as well as patient preferences. Laboratory and diagnostic tests should include,<br />

when appropriate:<br />

CBC. (PC, MK)<br />

Blood cultures. (PC, MK)<br />

ABG. (PC, MK)<br />

Pleural fluid chemistry, cell counts, staining, and culture. (PC, MK)<br />

Chest radiograph. (PC, MK)<br />

Students should be able to define the indications for and interpret (with consultation)<br />

the results <strong>of</strong>:<br />

Chest CT. (PC, MK)<br />

5. Communication skills: Students should be able to:<br />

Communicate the diagnosis, treatment plan, prognosis, and subsequent follow-up to the<br />

patient and his or her family. (PC, CS)<br />

Elicit questions from the patient and his or her family about the management plan. (PC, CS)<br />

Educate the patient about pneumococcal and influenza immunizations. (PC, CS)<br />

Educate the patient about the importance <strong>of</strong> smoking cessation. (PC, CS)<br />

6. Basic and advanced procedural skills: Students should be able to:<br />

Place and interpret a tuberculin skin test (PPD). (PC)<br />

Obtain blood cultures. (PC)<br />

Obtain an ABG. (PC)<br />

7. Management skills: Students should able to develop an appropriate evaluation and<br />

treatment plan for patients that includes:<br />

175


Selecting an appropriate empiric antibiotic regimen for community-acquired, nosocomial,<br />

immunocompromised-host, and aspiration pneumonia, taking into account pertinent patient<br />

features. (PC, MK)<br />

Adjusting antimicrobial treatment according to the sputum staining and culture results. (PC,<br />

MK)<br />

Recognizing the complications <strong>of</strong> pneumonia. (PC, MK)<br />

Determining when to obtain consultation from a pulmonologist or infectious diseases<br />

specialist. (PC, SBP)<br />

Using a cost-effective approach based on the differential diagnosis. (PC, SBP)<br />

Accessing and utilizing appropriate information systems and resources to help delineate<br />

issues related to pneumonia. (PC, PLI)<br />

Incorporating patient preferences. (PC)<br />

C. ATTITUDES AND PROFESSIONAL BEHAVIORS: Students should be able to:<br />

1. Demonstrate commitment to using risk-benefit, cost-benefit, and evidence-based<br />

considerations in the selection diagnostic and therapeutic interventions for the various types <strong>of</strong><br />

pneumonia. (PLI, P)<br />

2. Recognize the importance <strong>of</strong> patient preferences when selecting among diagnostic and<br />

therapeutic options for pneumonia. (P)<br />

3. Demonstrate ongoing commitment to self-directed learning regarding pneumonia. (PLI, P)<br />

4. Appreciate the impact pneumonia has on a patient’s quality <strong>of</strong> life, well-being, ability to work,<br />

and the family. (P)<br />

5. Recognize the importance <strong>of</strong> and demonstrate a commitment to the utilization <strong>of</strong> other<br />

healthcare pr<strong>of</strong>essionals in the treatment <strong>of</strong> pneumonia. (P, SBP)<br />

6. Appreciate the importance <strong>of</strong> antimicrobial resistance. (P)<br />

7. Appreciate the public health role <strong>of</strong> the physician when treating certain types <strong>of</strong> pneumonia<br />

(e.g. tuberculosis). (P)<br />

8. Appreciate the importance <strong>of</strong> and demonstrate a commitment to meeting national health care<br />

quality measures for the treatment <strong>of</strong> acute MI. (P, SBP, PLI)<br />

D. REFERENCES:<br />

Improving Treatment Decisions for Patients with Community-Acquired Pneumonia Agency<br />

for Healthcare Research and Quality<br />

U.S. <strong>Department</strong> <strong>of</strong> Health and Human Services<br />

www.ahrq.gov/clinic/pneumonia/pneumonria.htm<br />

Ramsdell J, Narsavage GL, Fink JB. Management <strong>of</strong> community-acquired pneumonia<br />

in the home: an American College <strong>of</strong> Chest Physicians clinical position statement.<br />

Chest. 2005;127:1752-63.<br />

www.guideline.gov/summary/summary.aspx?ss=15&doc_id=7325&nbr=4 348<br />

Influenza<br />

Centers for Disease Control and Prevention<br />

U.S. <strong>Department</strong> <strong>of</strong> Health and Human Services<br />

www.cdc.gov/flu/<br />

Prevention <strong>of</strong> Pneumococcal Disease: Recommendations <strong>of</strong> the Advisory<br />

Committee on Immunization Practices (ACIP). MMWR 46(RR-08);1-24. Centers for<br />

Disease Control and Prevention<br />

U.S. <strong>Department</strong> <strong>of</strong> Health and Human Services<br />

176


www.cdc.gov/mmwr/PDF/RR/RR4608.pdf<br />

Prevention and Control <strong>of</strong> Influenza: Recommendations <strong>of</strong> the Advisory Committee<br />

on Immunization Practices (ACIP). MMWR 50(RR-04);1-46. Centers for Disease<br />

Control and Prevention<br />

U.S. <strong>Department</strong> <strong>of</strong> Health and Human Services<br />

www.cdc.gov/mmwr/PDF/RR/RR5004<br />

177


TRAINING PROBLEM #30: RHEUMATOLOGIC PROBLEMS<br />

RATIONALE:<br />

Rheumatologic diseases are an important part <strong>of</strong> the practice <strong>of</strong> internal medicine. This includes<br />

problems referring to specific joints as well as patients with systemic symptoms that are sometimes<br />

difficult to unify into a single diagnosis.<br />

PREREQUISITES:<br />

Prior knowledge, skills, and attitudes acquired during the pre-<strong>clerkship</strong> experience should include:<br />

Ability to perform a complete medical history and physical exam.<br />

Ability to communicate with patients <strong>of</strong> diverse backgrounds.<br />

Anatomy, physiology, and pathophysiology <strong>of</strong> the musculoskeletal system.<br />

Basic course work in immunology.<br />

Pharmacology <strong>of</strong> acetaminophen, nonsteroidal anti-inflammatory drugs (NSAIDs),<br />

glucocorticoids, disease-modifying antirheumatic drugs (DMARDs), drugs use in the treatment<br />

<strong>of</strong> gout.<br />

Basic bone radiograph interpretation.<br />

SPECIFIC LEARNING OBJECTIVES:<br />

A. KNOWLEDGE: Students should be able to define, describe, and discuss:<br />

1. A systematic approach to joint pain based on an understanding <strong>of</strong> pathophysiology to classify<br />

potential causes. (MK)<br />

2. The effect <strong>of</strong> the time course <strong>of</strong> symptoms on the potential causes <strong>of</strong> joint pain (acute vs.<br />

subacute vs. chronic). (MK)<br />

3. The difference between and pathophysiology <strong>of</strong> arthralgia vs. arthritis and mechanical vs.<br />

inflammatory joint pain. (MK)<br />

4. The distinguishing features <strong>of</strong> intra-articular and periarticular complaints (joint pain vs. bursitis<br />

and tendonitis). (MK)<br />

5. The effect <strong>of</strong> the features <strong>of</strong> joint involvement on the potential causes <strong>of</strong> joint pain<br />

(monoarticular vs. oligoarticular vs. polyarticular, symmetric vs. asymmetric, axial and/or<br />

appendicular, small vs. large joints, additive vs. migratory vs. intermittent). (MK)<br />

6. Indications for performing an arthrocentesis and the results <strong>of</strong> synovial fluid analysis. (MK)<br />

7. The pathophysiology and common signs and symptoms <strong>of</strong>:<br />

Osteoarthritis. (MK)<br />

Crystalline arthropathies. (MK)<br />

Septic arthritis. (MK)<br />

8. Indications for and effectiveness <strong>of</strong> intra-articular steroid injections. (MK)<br />

9. Treatment options for gout (e.g. colchicine, NSAIDs, steroids, uricosurics, xanthine oxidase<br />

inhibitors). (MK)<br />

10. The pathophysiology and common signs and symptoms <strong>of</strong> common periarticular disorders:<br />

Sprain/stain. (MK)<br />

Tendonitis. (MK)<br />

Bursitis. (MK)<br />

11. The basic pathophysiology <strong>of</strong> autoimmunity and autoimmune diseases. (MK)<br />

12. The basic role <strong>of</strong> genetics in autoimmune disorders. (MK)<br />

178


13. Typical clinical scenarios when systemic rheumatologic disorders should be considered:<br />

Diffuse aches and pains. (MK)<br />

Generalized weakness/fatigue. (MK)<br />

Myalgias with or without weakness. (MK)<br />

Arthritis with systemic signs (e.g. fever, weight loss). (MK)<br />

Arthritis with disorders <strong>of</strong> other systems (e.g. rash, cardiopulmonary symptoms,<br />

gastrointestinal symptoms, eye disease, renal disease, neurologic symptoms). (MK)<br />

14. The common signs and symptoms <strong>of</strong> and diagnostic approach to:<br />

Rheumatoid arthritis. (MK)<br />

Spondyloarthropathies (reactive arthritis/Reiter’s syndrome, ankylosing spondylitis,<br />

psoriatic arthritis). (MK)<br />

Systemic lupus erythematosus. (MK)<br />

Systemic sclerosis. (MK)<br />

Raynaud’s syndrome/phenomenon. (MK)<br />

Sjögren’s syndrome. (MK)<br />

Temporal arteritis and polymyalgia rheumatica. (MK)<br />

Other systemic vasculitides. (MK)<br />

Polymyositis and dermatomyositis. (MK)<br />

Fibromyalgia. (MK)<br />

B. SKILLS: Students should be able to demonstrate specific skills, including:<br />

1. History-taking skills: Students should be able to obtain, document, and present an ageappropriate<br />

medical history that differentiates among etiologies <strong>of</strong> disease, including:<br />

Eliciting features <strong>of</strong> joint complaints:<br />

o Pain. (PC, CS)<br />

o Stiffness. (PC, CS)<br />

o Location. (PC, CS)<br />

o Mode <strong>of</strong> onset. (PC, CS)<br />

o Duration. (PC, CS)<br />

o Severity. (PC, CS)<br />

o Exacerbating and alleviating factors. (PC, CS)<br />

o Warmth, redness, and tenderness. (PC, CS)<br />

o Associated nonarticular symptoms. (PC, CS)<br />

Determining when in the course <strong>of</strong> acute arthritis it is necessary to obtain a sexual history.<br />

(PC, CS)<br />

Determining the impact <strong>of</strong> rheumatologic complaints on a patient's activities <strong>of</strong> daily living.<br />

(PC, CS)<br />

2. Physical exam skills: Students should be able to perform a physical exam to establish the<br />

diagnosis and severity <strong>of</strong> disease, including:<br />

A systematic examination <strong>of</strong> all joints identifying the following abnormal findings:<br />

o Erythema, warmth, tenderness, and swelling. (PC)<br />

o Effusion. (PC)<br />

o Crepitus. (PC)<br />

o Altered range <strong>of</strong> motion. (PC)<br />

o Ulnar deviation. (PC)<br />

o Synovial thickening. (PC)<br />

o Joint alignment deformities (e.g. varus and valgus). (PC)<br />

o Podagra. (PC)<br />

179


Muscular bulk, strength, and tenderness. (PC)<br />

Examination <strong>of</strong> the skin identifying the following abnormal findings:<br />

o Rheumatoid and tophaceous nodules. (PC)<br />

o Alopecia. (PC)<br />

o Malar rash. (PC)<br />

o Sclerodactyly. (PC)<br />

o Telangiectasias. (PC)<br />

o Raynaud’s phenomenon. (PC)<br />

o Psoriasis. (PC)<br />

o Cutaneous manifestations <strong>of</strong> vasculitis (e.g. palpable purpura). (PC)<br />

3. Differential diagnosis: Students should be able to generate a prioritized differential diagnosis<br />

recognizing specific history and physical exam findings that suggest a specific etiology:<br />

Osteoarthritis. (PC, MK)<br />

Crystalline arthropathies. (PC, MK)<br />

Septic arthritis. (PC, MK)<br />

Rheumatoid arthritis. (PC, MK)<br />

Spondyloarthropathies (reactive arthritis/Reiter’s syndrome, ankylosing spondylitis,<br />

psoriatic arthritis). (PC, MK)<br />

Systemic lupus erythematosus. (PC, MK)<br />

Systemic sclerosis. (PC, MK)<br />

Raynaud’s syndrome/phenomenon. (PC, MK)<br />

Sjörgren’s syndrome. (PC, MK)<br />

Temporal arteritis and polymyalgia rheumatica. (PC, MK)<br />

Other systemic vasculitides. (PC, MK)<br />

Polymyositis and dermatomyositis. (PC, MK)<br />

Fibromyalgia. (PC, MK)<br />

5. Laboratory interpretation: Students should be able to recommend when to order diagnostic and<br />

laboratory tests and be able to interpret them, both priorto and after initiating treatment based<br />

on the differential diagnosis, including consideration <strong>of</strong> test cost and performance<br />

characteristics as well as patient preferences.<br />

Laboratory and diagnostic tests should include, when appropriate:<br />

CBC with differential. (PC, MK)<br />

Synovial fluid analysis (Gram stain, culture, crystal exam, cell count with differential, and<br />

glucose). (PC, MK)<br />

Uric acid. (PC, MK)<br />

ESR. (PC, MK)<br />

Rheumatoid factor (RF). (PC, MK)<br />

Antinuclear antibody test (ANA) and anti-DNA test. (PC, MK)<br />

Students should be able to define the indications for and interpret (with consultation)<br />

the results <strong>of</strong>:<br />

Plain radiographs <strong>of</strong> the shoulder, elbow, wrist, hand, hip, knee, ankle, and foot. (PC, MK)<br />

5. Communication skills: Students should be able to:<br />

Communicate the diagnosis, treatment plan, and subsequent follow-up to patients. (PC,<br />

CS)<br />

Elicit questions from the patient about the management plan. (PC, CS)<br />

6. Basic and advanced procedure skills: Students should be able to:<br />

Assist in the performance <strong>of</strong> an arthrocentesis and intra-articular corticosteroid<br />

injection. (PC)<br />

7. Management skills: Students should able to develop an appropriate evaluation and<br />

180


treatment plan for patients that includes:<br />

Selecting appropriate medications for the relief <strong>of</strong> joint pain. (PC, MK)<br />

Prescribing acute and preventative treatment for crystalline arthropathies. (PC, MK)<br />

Prescribing basic treatment options for septic arthritis. (PC, MK)<br />

Prescribing basic treatment options for systemic rheumatologic conditions. (PC, MK)<br />

Determining when to obtain consultation from a rheumatologist and orthopedic surgeon.<br />

(PC, SBP)<br />

Using a cost-effective approach based on the differential diagnosis. (PC, SBP)<br />

Accessing and utilizing appropriate information systems and resources to help delineate<br />

issues related to rheumatologic problems. (PC, PLI)<br />

Incorporating patient preferences. (PC)<br />

C. ATTITUDES AND PROFESSIONAL BEHAVIORS: Students should be able to:<br />

1. Demonstrate commitment to using risk-benefit, cost-benefit, and evidence-based<br />

considerations in the selection diagnostic and therapeutic interventions for rheumatologic<br />

problems. (PLI, P)<br />

2. Recognize the importance <strong>of</strong> patient preferences when selecting among diagnostic and<br />

therapeutic options for rheumatologic problems. (P)<br />

3. Respond appropriately to patients who are nonadherent to treatment for rheumatologic<br />

problems. (CS, P)<br />

4. Demonstrate ongoing commitment to self-directed learning regarding rheumatologic problems.<br />

(PLI, P)<br />

5. Appreciate the impact rheumatologic problems have on a patient’s quality <strong>of</strong> life, well-being,<br />

ability to work, and the family. (P)<br />

6. Recognize the importance <strong>of</strong> and demonstrate a commitment to the utilization<br />

<strong>of</strong> other healthcare pr<strong>of</strong>essions in the treatment <strong>of</strong> rheumatologic problems. (P, SBP)<br />

D. REFERENCES:<br />

Evaluation <strong>of</strong> the patient – history and physical examination, laboratory assessment,<br />

arthrocentesis and synovial fluid analysis. In Schumacher HR, Klippel JH, Koopman<br />

WJ, eds. Primer on the Rheumatic Diseases. 12 th Ed. Atlanta, GA: Arthritis Foundation;<br />

2001.<br />

Management Guidelines<br />

American College <strong>of</strong> Rheumatology<br />

www.rheumatology.org/publications/guidelines/index.asp<br />

Arthritis Foundation<br />

www.arthritis.org<br />

National Institute <strong>of</strong> Arthritis and Musculoskeletal and Skin Diseases<br />

National Institutes <strong>of</strong> Health<br />

U.S. <strong>Department</strong> <strong>of</strong> Health and Human Services<br />

www.niams.nih.gov<br />

181


TRAINING PROBLEM #31: SMOKING CESSATION<br />

RATIONALE:<br />

Smoking is a major public health issue because it causes or aggravates many serious illnesses.<br />

Effective intervention strategies for chronic smokers have been developed using principals <strong>of</strong><br />

behavioral counseling. These principals are applicable to other risky health behaviors. Health<br />

behavior risk assessment and intervention is now expected <strong>of</strong> physicians as part <strong>of</strong> the<br />

comprehensive care <strong>of</strong> adults. Selecting and performing an appropriate smoking cessation<br />

intervention is an important training problem for the third year medical student.<br />

PREREQUISITES:<br />

Prior knowledge, skills, and attitudes acquired during the pre-<strong>clerkship</strong> experience should include:<br />

Ability to perform a complete medical history and physical exam.<br />

Ability to communicate with patients <strong>of</strong> diverse backgrounds.<br />

Knowledge <strong>of</strong> the anatomy, physiology, and pathophysiology <strong>of</strong> the cardiopulmonary system.<br />

Knowledge <strong>of</strong> the pharmacology <strong>of</strong> addictive drugs.<br />

Knowledge <strong>of</strong> the risks <strong>of</strong> smoking, passive smoke, and smokeless tobacco.<br />

Appreciation <strong>of</strong> the reasons for or against discontinuing smoking.<br />

SPECIFIC LEARNING OBJECTIVES:<br />

A. KNOWLEDGE: Students should be able to define, describe, and discuss:<br />

1. The pharmacologic effects <strong>of</strong> nicotine. (MK)<br />

2. Nicotine withdrawal symptoms. (MK)<br />

3. Intervention strategies physicians can use for those patients willing and not willing to quit. (MK)<br />

4. The stages <strong>of</strong> change, including:<br />

Precontemplation. (MK)<br />

Contemplation. (MK)<br />

Preparation. (MK)<br />

Action. (MK)<br />

Maintenance. (MK)<br />

5. The “five A’s” <strong>of</strong> smoking cessation:<br />

Ask. (MK)<br />

Advise. (MK)<br />

Assess. (MK)<br />

Assist. (MK)<br />

Arrange. (MK)<br />

6. The “five R’s” <strong>of</strong> smoking cessation:<br />

Relevance. (MK)<br />

Risks. (MK)<br />

Rewards. (MK)<br />

Roadblocks. (MK)<br />

Repetition. (MK)<br />

7. The common barriers preventing patients from undertaking smoking cessation. (MK)<br />

8. The principles <strong>of</strong> at least one theory <strong>of</strong> behavior modification. (MK)<br />

9. Common medical diseases associated with chronic smoking and the effects <strong>of</strong> stopping on<br />

182


future risk. (MK)<br />

10. The indications for nicotine replacement therapy, pharmacotherapy (i.e. bupropion) or both.<br />

(MK)<br />

11. The association between smoking cessation and weight gain. (MK)<br />

12. The fact that tobacco dependence is considered a chronic relapsing disorder. (MK)<br />

13. The Centers for Medicare & Medicaid Services (CMS) and the Joint Commission on the<br />

Accreditation <strong>of</strong> Healthcare Organizations (JCAHO) quality measures for smoking cessation<br />

advice (i.e. all smoking patients admitted with pneumonia, HF, or an acute MI are given<br />

smoking cessation advice or counseling during hospital stay). (MK, PLI, SBP)<br />

B. SKILLS: Students should demonstrate specific skills, including:<br />

1. History-taking skills: Students should be able to obtain, document, and present an ageappropriate<br />

medical history, including:<br />

Ask the patient if he or she uses tobacco. (PC, CS)<br />

Determine the length and magnitude <strong>of</strong> tobacco use. (PC, CS)<br />

Ask if the patient is interested in stopping.<br />

Ask about the patient’s past experiences with smoking cessation. (PC, CS)<br />

Ask relevant questions regarding the symptoms <strong>of</strong> diseases associated with long-term<br />

smoking (e.g. CAD, COPD, PVD, CVA, lung cancer). (PC, CS)<br />

2. Physical exam skills: Students should be able to perform a physical exam to establish the<br />

diagnosis and severity <strong>of</strong> disease, including:<br />

Identification <strong>of</strong> nicotine stains. (PC)<br />

Identification <strong>of</strong> lesions with malignant potential on the lips and in the oral cavity. (PC)<br />

Identification <strong>of</strong> chest findings consistent with chronic obstructive lung disease and lung<br />

cancer. (PC)<br />

Examination <strong>of</strong> the heart and vascular system. (PC)<br />

3. Laboratory interpretation: Students should be able to recommend when to order diagnostic<br />

and laboratory tests and be able to interpret them, both prior to and after initiating treatment<br />

based on the differential diagnosis, including consideration <strong>of</strong> test cost and performance<br />

characteristics as well as patient preferences. Laboratory and diagnostic tests should include,<br />

when appropriate:<br />

Complete blood count to detect erythrocytosis. (PC, MK)<br />

Lipid pr<strong>of</strong>ile to aid in cardiovascular stratification. (PC, MK)<br />

4. Communication skills: Students should be able to:<br />

1. Ask every patient if he or she uses tobacco. (PC, CS)<br />

2. Advise every patient who smokes to stop in a nonjudgmental manner. (PC, CS)<br />

3. Assess the patient’s willingness to make attempt to quit. (PC, CS)<br />

4. Assist those who are willing to make a quit attempt through counseling. (PC, CS)<br />

5. Respond positively and non-judgmentally to the patient’s excuses or concerns about<br />

cessation. (PC, CS)<br />

6. Get the patient to commit to a specific action plan that can lead to complete<br />

cessation. (PC, CS)<br />

7. For those unwilling to quit, use <strong>of</strong> “5 R’s” to motivate the patient:<br />

o Relevance. (PC, CS)<br />

o Risks. (PC, CS)<br />

o Rewards. (PC, CS)<br />

183


o Roadblocks. (PC, CS)<br />

o Repetition. (PC, CS)<br />

5. Management skills: Students should be able to develop an appropriate evaluation and<br />

treatment plan for patient, including:<br />

Designing an intervention that matches the stage <strong>of</strong> behavior change demonstrated by the<br />

patient. (PC, CS)<br />

Explaining how to use nicotine patch, nasal spray or inhaler, and/or bupropion therapy.<br />

(PC, CS)<br />

Negotiating a follow-up plan with the patient. (PC, CS)<br />

Encouraging the patient to increase physical activity to lessen weight gain, if medically<br />

appropriate. (PC, CS)<br />

Accessing and utilizing appropriate information systems and resources to help delineate<br />

issues/resources related to aiding smoking cessation. (PC, PLI)<br />

Incorporating patient preferences. (PC)<br />

C. ATTITUDES AND PROFESSIONAL BEHAVIORS: Student should be able to:<br />

1. Demonstrate a commitment to meeting national quality standards for smoking cessation. (P,<br />

PLI, SBP)<br />

2. Maintain a non-judgmental attitude at all times regarding smoking cessation. (P)<br />

3. Demonstrate a commitment to deliver a non-judgmental "stop smoking" message to every<br />

patient who smokes. (P)<br />

4. Promote problem-solving by the patient. (P)<br />

5. Demonstrate commitment to using risk-benefit, cost-benefit, and evidence-based<br />

considerations in the selection <strong>of</strong> diagnostic and therapeutic interventions for smoking<br />

cessation. (PLI, P)<br />

6. Respond appropriately to patients who are non-adherent to treatment for smoking cessation.<br />

(P)<br />

7. Demonstrate ongoing commitment to self-directed learning regarding smoking cessation. (PLI,<br />

P)<br />

8. Appreciate the impact smoking cessation has on a patient’s quality <strong>of</strong> life, well-being, ability to<br />

work, and the famRecognize the importance and demonstrate a commitment to the utilization<br />

<strong>of</strong> other healthcare pr<strong>of</strong>essions in the treatment <strong>of</strong> smoking cessation. (P, SBP)<br />

D. RESOURCES:<br />

Public Health Service<br />

<strong>Department</strong> <strong>of</strong> Health and Human Services<br />

Tobacco Cessation Guideline<br />

www.surgeongeneral.gov/tobacco/default.htm<br />

Silagy C, Lancaster T, Stead L, Mant D, Fowler G. Nicotine replacement therapy for<br />

smoking cessation. Cochrane Database <strong>of</strong> Syst Rev. 2004;(3):CD000146.<br />

Schroeder SA. What to do with a patient who smokes. JAMA. 2005;294:482-7.<br />

Talwar A, Jain M, Vijayan VK. Pharmacotherapy <strong>of</strong> tobacco dependence. Med Clin<br />

North Am. 2004;88:1517-34.<br />

184


TRAINING PROBLEM #32: SUBSTANCE ABUSE<br />

RATIONALE:<br />

Substance abuse is a prevalent problem that intersects with patient care on a variety <strong>of</strong> different<br />

levels and in patients from every socio-economic status. Being able to recognize it, counsel<br />

patients appropriately, and devise an appropriate treatment plan is integral to the practice <strong>of</strong><br />

internal medicine.<br />

PREREQUISITES:<br />

Prior knowledge, skills, and attitudes acquired during the pre-<strong>clerkship</strong> experiences should<br />

include:<br />

Ability to perform a complete medical history and physical exam.<br />

Ability to communicate with patients <strong>of</strong> diverse backgrounds.<br />

Knowledge <strong>of</strong> drug and alcohol metabolism and physiology.<br />

SPECIFIC LEARNING OBJECTIVES:<br />

A. KNOWLEDGE: Students should be able to define, describe, and discuss:<br />

1. Presenting signs and symptoms <strong>of</strong> abuse <strong>of</strong> the following substances:<br />

Alcohol. (MK)<br />

Opioids. (MK)<br />

Cocaine. (MK)<br />

Amphetamines.(MK)<br />

Hallucinogens. (MK)<br />

Barbiturates. (MK)<br />

Marijuana. (MK)<br />

Anabolic steroids. (MK)<br />

Benzodiazepines. (MK)<br />

2. Signs, symptoms, risk factors for, and major causes <strong>of</strong> morbidity and mortality secondary to<br />

alcohol and drug abuse, intoxication, overdose, and withdrawal. (MK)<br />

3, Diagnostic criteria for substance abuse, dependency and addiction. (MK)<br />

4. Questions in the CAGE questionnaire:<br />

Cut down. (MK)<br />

Annoyed/angry. (MK)<br />

Guilty. (MK)<br />

Eye opener. (MK)<br />

5. Health benefits <strong>of</strong> substance abuse cessation. (MK)<br />

6. The potential role <strong>of</strong> genetics in substance abuse vulnerability. (MK)<br />

B. SKILLS: Students should demonstrate specific skills, including:<br />

1. History-taking skills: Students should be able to obtain, document, and present an<br />

age-appropriate medical history, that differentiates among etiologies <strong>of</strong> disease,<br />

including:<br />

Social history that is elicited in a nonjudgmental, supportive manner, using appropriate<br />

questioning (e.g. CAGE questions, etc.). (PC, CS)<br />

185


Use <strong>of</strong> injection drugs and shared needles. (PC, CS)<br />

Relevant medication history. (PC, CS)<br />

Immune status. (PC, CS)<br />

Family history <strong>of</strong> substance abuse. (PC, CS)<br />

Lifestyle factors that will influence patient’s access to illicit substances and interfere with<br />

ability to enable effective treatment. (PC, CS)<br />

Screening for depression and other psychiatric disease. (PC, CS)<br />

2. Physical exam skills: Students should be able to perform a physical exam to establish the<br />

diagnosis and severity <strong>of</strong> disease, including:<br />

Accurate recognition <strong>of</strong> signs that may indicate intoxication or withdrawal (e.g. behavioral or<br />

speech changes, changes in pupil size, conjunctival or nasal injection, tachycardia,<br />

sweating, piloerection, yawning, unsteady gait, etc.). (PC, MK)<br />

Examination <strong>of</strong> the nose for septal perforation as complication <strong>of</strong> cocaine use. (PC, MK)<br />

Examination <strong>of</strong> the skin for track marks or signs <strong>of</strong> needle use. (PC, MK)<br />

Identification <strong>of</strong> stigmata <strong>of</strong> secondary disease states (e.g. cirrhosis – splenomegaly,<br />

gynecomastia, telangiectasias, caput medusa, etc.) (PC, MK)<br />

Assessing for signs <strong>of</strong> endocarditis (e.g., fever, murmur, rash, etc). (PC, MK)<br />

Obtaining full mental status examination. (PC, MK)<br />

3. Differential diagnosis: Students should be able to generate a differential diagnosis<br />

recognizing history, physical exam and/or laboratory findings to determine the diagnosis <strong>of</strong><br />

abuse <strong>of</strong> drugs or alcohol and their sequelae. (PC, MK)<br />

4. Laboratory interpretation: Students should be able to recommend when to order diagnostic<br />

and laboratory tests and be able to interpret them, both prior to and after initiating treatment<br />

based on the differential diagnosis, including consideration <strong>of</strong> test cost and performance<br />

characteristics as well as patient preferences.<br />

Laboratory and diagnostic tests should include, when appropriate:<br />

Blood alcohol level. (PC, MK)<br />

Urine and serum toxicology screens. (PC, MK)<br />

Hepatic function panel. (PC, MK)<br />

Amylase and lipase levels. (PC, MK)<br />

Tests for HIV, hepatitis B and hepatitis C. (PC, MK)<br />

CBC. (PC, MK)<br />

Blood cultures. (PC, MK)<br />

5. Communication skills: Students should be able to:<br />

Communicate the evaluation, treatment plan, and subsequent follow up to the patient and<br />

his or her family in a non-judgmental manner. (PC, CS)<br />

Elicit questions from the patient and his or her family about the disease process and<br />

management plan. (PC, CS)<br />

Counsel patients regarding cessation and available community referral resources. (PC, CS,<br />

SBP)<br />

6. Management skills: Students should be able to develop an appropriate evaluation and<br />

treatment plan that includes:<br />

Assessing the patient’s motivation for achieving sobriety/abstinence. (PC, MK)<br />

Understanding the principles <strong>of</strong> acute management <strong>of</strong> drug/alcohol intoxication and<br />

withdrawal versus long-term treatment planning. (MK, PC)<br />

Using Clinical Institute Withdrawal Assessment for Alcohol. (CIWA-Ar) scale in acute<br />

alcohol withdrawal to prevent seizures or delirium tremens (MK, PC)<br />

Recommending appropriate use <strong>of</strong> benzodiazepines for alcohol withdrawal. (MK, PC)<br />

186


Determining when to obtain consultation from a psychiatrist. (PC, SBP)<br />

Accessing and utilizing appropriate information systems and resources to help delineate<br />

issues related to substance abuse. (PC, PLI)<br />

Incorporating patient preferences and understanding limitations <strong>of</strong> treatment. (PC)<br />

C. ATTITUDES AND PROFESSIONAL BEHAVIORS: Students should be able to:<br />

1. Demonstrate commitment to using risk-benefit, cost-benefit, and evidence-based<br />

considerations in the selection <strong>of</strong> diagnostic and therapeutic interventions for substance abuse. (PLI,<br />

P)<br />

2. Respond appropriately to patients who are non-adherent to treatment for substance abuse.<br />

(CS, P)<br />

3. Demonstrate ongoing commitment to self-directed learning regarding substance abuse. (PLI,<br />

P)<br />

4. Appreciate the impact substance abuse has on a patient’s as well as a family’s quality <strong>of</strong> life,<br />

well-being, and ability to work. (P)<br />

5. Recognize the importance and demonstrate a commitment to the utilization <strong>of</strong> other healthcare<br />

pr<strong>of</strong>essions in the treatment <strong>of</strong> substance abuse. (P, SBP)<br />

D. REFERENCES:<br />

U.S. Preventive Services Task Force. Screening and behavioral counseling<br />

interventions in primary care to reduce alcohol misuse recommendation statement. Ann<br />

Intern Med. 2004;140:554-6.<br />

Kosten TR, O'Connor PG. Management <strong>of</strong> drug and alcohol withdrawal. N Engl J Med.<br />

2003;348:1786-95.<br />

Mersy DJ. Recognition <strong>of</strong> alcohol and substance abuse. Am Fam Physician.<br />

2003;67:1529-32.<br />

187


TRAINING PROBLEM #33: VENOUS THROMBOEMBOLISM<br />

RATIONALE:<br />

Venous thromboembolic disease (DVT and PE) is a very common problem in internal medicine and<br />

one that can have devastating consequences if not appropriately diagnosed and treated. Diagnosis <strong>of</strong><br />

DVT and PE can be especially challenging. Prophylactic measures are very effective but do not<br />

eliminate the risk.<br />

PREREQUISITES:<br />

Prior knowledge, skills and attitudes acquired during the pre-<strong>clerkship</strong> experience should include:<br />

Ability to perform a complete medical history and physical exam (with particular attention to the<br />

cardiac, pulmonary, and venous systems).<br />

Ability to communicate with patients <strong>of</strong> diverse backgrounds.<br />

Knowledge <strong>of</strong> the anatomy, physiology, and pathophysiology <strong>of</strong> the cardiac, pulmonary, and<br />

venous systems.<br />

Physiology and pathophysiology <strong>of</strong> the hemostatic system.<br />

Pharmacology <strong>of</strong> antithrombotic agents.<br />

SPECIFIC LEARNING OBJECTIVES:<br />

A. KNOWLEDGE: Students should be able to define, describe and discuss:<br />

1. Risk factors for developing DVT, including:<br />

Prior history <strong>of</strong> DVT/PE. (MK)<br />

Immobility/hospitalization. (MK)<br />

Increasing age. (MK)<br />

Obesity. (MK)<br />

Trauma. (MK)<br />

Smoking. (MK)<br />

Surgery. (MK)<br />

Cancer. (MK)<br />

Acute MI. (MK)<br />

Stroke and neurologic trauma. (MK)<br />

Coagulopathy. (MK)<br />

Pregnancy. (MK)<br />

Oral estrogens. (MK)<br />

2. Genetic considerations predisposing to venous thrombosis. (MK)<br />

3. The symptoms and signs <strong>of</strong> DVT and PE. (MK)<br />

4. The differential diagnosis <strong>of</strong> DVT including the many causes <strong>of</strong> unilateral leg pain and swelling:<br />

Venous stasis and the postphlebitic syndrome. (MK)<br />

Lymphedema. (MK)<br />

Cellulitis. (MK)<br />

Superficial thrombophlebitis. (MK)<br />

Ruptured popliteal cyst. (MK)<br />

Musculoskeletal injury. (MK)<br />

Arterial occlusive disorders. (MK)<br />

5. The differential diagnosis <strong>of</strong> PE including the many causes <strong>of</strong> chest pain and dyspnea:<br />

188


MI/unstable angina. (MK)<br />

Congestive heart failure. (MK)<br />

Pericarditis. (MK)<br />

Pneumonia/bronchitis/COPD exacerbation. (MK)<br />

Asthma. (MK)<br />

Pulmonary hypertension. (MK)<br />

Pneumothorax. (MK)<br />

Musculoskeletal pain (e.g. rib fracture, costochondritis). (MK)<br />

6. Treatment modalities for DVT/PE, including:<br />

Unfractionated heparin. (MK)<br />

Low-molecular-weight heparin. (MK)<br />

Warfarin. (MK)<br />

Thrombolytics. (MK)<br />

7. The risks, benefits, and indications for inferior vena cava filters. (MK)<br />

8. The long-term sequelae <strong>of</strong> DVT and PE. (MK)<br />

9. Methods <strong>of</strong> DVT/PE prophylaxis, their indications and efficacy, including:<br />

Ambulation. (MK)<br />

Graded compression stockings. (MK)<br />

Pneumatic compression devices. (MK)<br />

Unfractionated heparin. (MK)<br />

Low-molecular-weight heparin. (MK)<br />

Warfarin. (MK)<br />

B. SKILLS: Students should demonstrate specific skills, including:<br />

1. History-taking skills: Students should be able to obtain, document and present an ageappropriate<br />

medical history that suggests the diagnosis <strong>of</strong> DVT or PE, including:<br />

The presence or absence <strong>of</strong> known risk factors. (PC, CS)<br />

Presence or absence <strong>of</strong> leg pain, swelling, warmth, discoloration, and palpable cord. (PC,<br />

CS)<br />

The presence or absence <strong>of</strong> dyspnea, chest pain, palpitations, cough, hemoptysis. (PC,<br />

CS)<br />

2. Physical exam skills: Students should be able to perform a physical examination to<br />

establish the diagnosis and severity <strong>of</strong> disease, including:<br />

Assessment <strong>of</strong> vital signs (i.e. hypotension, tachycardia, tachypnea, fever) and general<br />

appearance (i.e. degree <strong>of</strong> respiratory distress, anxiety). (PC)<br />

Accurate identification <strong>of</strong> leg swelling, erythema, warmth, and tenderness. (PC)<br />

Inspection for signs <strong>of</strong> lower extremity trauma, arthritis, or joint effusion. (PC)<br />

Identification <strong>of</strong> pleural friction rubs, wheezes, rales, rhonchi, and signs <strong>of</strong> pneumothorax.<br />

(PC)<br />

3. Differential diagnosis: Students should be able to generate a differential diagnosis<br />

for a patient suspected <strong>of</strong> having DVT/PE, recognizing specific history, physical examination<br />

and laboratory findings which suggest DVT/PE, including the disease states noted above. (PC,<br />

MK)<br />

4. Laboratory interpretation: Students should be able to recommend when to order diagnostic<br />

and laboratory tests and be able to interpret them, both prior to and after initiating treatment<br />

based on<br />

189


the differential diagnosis, including consideration <strong>of</strong> test cost and performance characteristics<br />

as well as patient preferences.<br />

Laboratory and diagnostic tests should include, where appropriate:<br />

Pulse oximetry. (PC, MK)<br />

12-lead ECG. (PC, MK)<br />

Chest radiograph. (PC, MK)<br />

ABG. (PC, MK)<br />

D-dimer. (PC, MK)<br />

Students should be able to define the indications for and interpret (with consultation)<br />

the results <strong>of</strong>:<br />

Duplex venous ultrasonography. (PC, MK)<br />

Ventilation perfusion (V/Q) scan. (PC, MK)<br />

CT angiography. (PC, MK)<br />

Pulmonary angiography. (PC, MK)<br />

Echocardiography. (PC, MK)<br />

5. Communication skills: Students should be able to:<br />

Communicate the diagnosis, treatment plan, and subsequent follow-up to the patient and<br />

his or her family. (PC, CS)<br />

Elicit questions from the patient and his or her family about the management plan. (PC, CS)<br />

6. Basic and advanced procedural skills: Students should be able to:<br />

Perform a 12-lead ECG. (PC)<br />

Obtain an ABG. (PC)<br />

7. Management skills: Students should be able to develop an appropriate evaluation and<br />

treatment plan for patients that includes:<br />

Outlining the acute and long-term treatment <strong>of</strong> isolated calf vein phlebitis, superficial<br />

thrombophlebitis, DVT, and thromboembolism, including appropriate use and monitoring <strong>of</strong><br />

heparin and warfarin. (MK,<br />

PC)<br />

Understanding the indications for placement <strong>of</strong> inferior vena cava filter, indications and<br />

complications <strong>of</strong> thrombolytic therapy, as well as indications for performing a<br />

hypercoaguability work-up. (PC, MK)<br />

Determining when to obtain consultation from a pulmonologist or interventional radiologist.<br />

(PC, MK)<br />

Using a cost-effective approach based on the differential diagnosis. (PC, SBP)<br />

Accessing and utilizing appropriate information systems and resources to help delineate<br />

issues related to venous thromboembolism. (PC, PLI)<br />

Incorporating patient preferences. (PC)<br />

C. ATTITUDES AND PROFESSIONAL BEHAVIORS: Students should be able to:<br />

1. Demonstrate commitment to using risk-benefit, cost-benefit, and evidence-based<br />

considerations in the selection <strong>of</strong> diagnostic and therapeutic interventions for venous<br />

thromboembolic disease. (PLI, P)<br />

2. Respond appropriately to patients who are non-adherent to treatment for venous<br />

thromboembolic disease. (CS, P)<br />

3. Demonstrate ongoing commitment to self-directed learning regarding venous thromboembolic<br />

disease. (PLI, P)<br />

4. Appreciate the impact venous thromboembolic disease has on a patient’s quality <strong>of</strong> life, well-<br />

190


eing, ability to work, and the family. (P)<br />

5. Recognize the importance and demonstrate a commitment to the utilization <strong>of</strong> other healthcare<br />

pr<strong>of</strong>essions in the treatment <strong>of</strong> venous thromboembolic disease. (P, SBP)<br />

D. REFERENCES:<br />

American College <strong>of</strong> Chest Physicians. The seventh ACCP conference on<br />

antithrombotic and thrombolytic therapy: evidence-based guidelines. Chest. 2004;126<br />

(Number 3 Supplement)<br />

www.chestjournal.org/content/vol126/3_suppl<br />

Spyropoulos AC. Emerging strategies in the prevention <strong>of</strong> venous thromboembolism in<br />

hospitalized medical patients. Chest. 2005;128:958-69.<br />

Stein PD, Hull RD, Patel KC, Olson RE, Ghali WA, Brant R, Biel RK, Bharadia V, Kalra<br />

NK. D-dimer for the exclusion <strong>of</strong> acute venous thrombosis and pulmonary embolism: a<br />

systematic review. Ann Intern Med. 2004;140:589-602.<br />

Bates SM, Ginsberg JS. Clinical practice. Treatment <strong>of</strong> deep-vein thrombosis. N Engl J<br />

Med. 2004;351:268-77.<br />

Fedullo PF. Tapson VF. Clinical practice. The evaluation <strong>of</strong> suspected pulmonary<br />

embolism. N Engl J Med. 2003;349:1247-56.<br />

191


GENERAL CLINICAL CORE COMPETENCIES IN INTERNAL MEDICINE<br />

#1 DIAGNOSTIC DECISION MAKING<br />

RATIONALE:<br />

Physicians are responsible for directing and conducting the diagnostic evaluation <strong>of</strong> a broad range <strong>of</strong><br />

patients, including patients seeking advice on prevention <strong>of</strong> and screening for disease and patients<br />

with acute and chronic illnesses. In a time <strong>of</strong> rapidly proliferating tests, medical students must learn<br />

how to design safe, expeditious, and cost-effective diagnostic evaluations. This requires welldeveloped<br />

diagnostic decision-making skills that incorporate probability-based thinking.<br />

PREREQUISITES:<br />

Prior knowledge, skills, and attitudes acquired during the pre-<strong>clerkship</strong> experience should include:<br />

Required course in pathophysiology.<br />

Required course in clinical epidemiology and biostatistics.<br />

Ability to perform a complete medical history and physical exam.<br />

Ability to communicate with patients <strong>of</strong> diverse backgrounds.<br />

SPECIFIC LEARNING OBJECTIVES:<br />

A. KNOWLEDGE: Students should be able to define, describe, and discuss:<br />

1. Key history and physical examination findings pertinent to the differential diagnosis. (MK)<br />

2. Information resources for determining diagnostic options for patients with common and<br />

uncommon medical problems. (MK, PLI)<br />

3. Key factors to consider when selecting from among diagnostic tests, including pretest<br />

probabilities, performance characteristics <strong>of</strong> tests (sensitivity, specificity, likelihood ratios), cost,<br />

risk, and patient preferences. (MK, P)<br />

4. The basics <strong>of</strong> the potential role <strong>of</strong> genetic information in diagnostic decision making. (MK)<br />

5. Relative cost <strong>of</strong> diagnostic tests. (MK)<br />

6. How critical pathways or practice guidelines can be used to guide diagnostic test ordering.<br />

(MK)<br />

7. The methods <strong>of</strong> deductive reasoning, forward thinking, and pattern recognition in clinical<br />

decision making. (MK)<br />

B. SKILLS: Students should demonstrate specific skills, including:<br />

1. Identifying problems with which a patient presents, appropriately synthesizing these into logical<br />

clinical syndromes. (PC)<br />

2. Identifying which problems are <strong>of</strong> highest priority. (PC)<br />

3. Formulating a differential diagnosis based on the findings from the history and physical<br />

examination. (PC)<br />

4. Using probability-based thinking and pattern recognition to identify the most likely diagnoses.<br />

(PC)<br />

5. Using the differential diagnosis to help guide diagnostic test ordering and sequencing. (PC)<br />

6. Using pretest probabilities and scientific evidence about performance characteristics <strong>of</strong> tests<br />

(sensitivity, specificity, likelihood ratios) to determine post-test probabilities according to the<br />

predictive value paradigm. (PC)<br />

7. Participating in selecting the diagnostic studies with the greatest likelihood <strong>of</strong> providing useful<br />

results at a reasonable cost. (PC)<br />

192


8. Communicating the prioritized differential diagnosis to the patient and his or her family. (CS)<br />

C. ATTITUDES AND PROFESSIONAL BEHAVIORS: Students should be able to:<br />

1. Incorporate the patient’s perspective into diagnostic decision making. (P)<br />

2. Recognize the importance <strong>of</strong> patient preferences when selecting among diagnostic tests. (P)<br />

3. Demonstrate commitment to using risk-benefit, cost-benefit, and evidence-based<br />

considerations in the selection <strong>of</strong> diagnostic tests. (PLI, P)<br />

4. Seek feedback regularly regarding diagnostic decision making and respond appropriately and<br />

productively. (P)<br />

5. Limit the chances <strong>of</strong> false positive/false negative results by demonstrating thoughtful test<br />

selection. (P)<br />

6. Appreciate the element <strong>of</strong> uncertainty in diagnostic testing, including the occurrence and<br />

causes <strong>of</strong> false positive and false negative results. (P)<br />

7. Appreciate the impact uncertainty may have on the patient. (P)<br />

8. Recognize the importance <strong>of</strong> and demonstrate a commitment to the utilization <strong>of</strong> other health<br />

care pr<strong>of</strong>essionals in diagnostic decision making. (P, SBP)<br />

D. REFERENCES:<br />

Mark, DB. Decision-making in clinical medicine. In Kasper DL, Braunwald EB, Fauci AS, Hauser<br />

SL, Longo DL, Jameson JL, eds. Harrison’s Principles <strong>of</strong> Internal <strong>Medicine</strong>. 16th ed. New York, NY:<br />

McGraw-Hill; 2005:6-13.<br />

Strauss SE, Richardson WS, Glasziou P, Haynes RB. Evidence Based <strong>Medicine</strong>: How to Practice<br />

and Teach EBM. 3rd ed. New York, NY: Churchill Livingstone; 2005.<br />

Primer to the Internal <strong>Medicine</strong> Clerkship A Guide Produced by the Clerkship Directors in Internal<br />

<strong>Medicine</strong> Clerkship Directors in Internal <strong>Medicine</strong><br />

www.im.org/CDIM/primer.htm<br />

Ferri FF. Differential diagnosis. In Ferri FF, ed. Practical Guide to the Care <strong>of</strong> the Medical Patient.<br />

6th ed. St. Louis, MO: Mosby; 2004:39-113.<br />

193


#2 CASE PRESENTATION<br />

GENERAL CLINICAL CORE COMPETENCIES<br />

RATIONALE:<br />

Communicating patient care information to colleagues and other health care pr<strong>of</strong>essionals is an<br />

essential skill regardless <strong>of</strong> specialty. Internists have traditionally given special attention to case<br />

presentation skills because <strong>of</strong> the comprehensive nature <strong>of</strong> patient evaluations and the various<br />

settings in which internal medicine is practiced. Students should develop facility with different types<br />

<strong>of</strong> case presentations: written and oral, new patient and follow-up, inpatient and outpatient.<br />

PREREQUISITES:<br />

Prior knowledge, skills, and attitudes acquired during the pre-<strong>clerkship</strong> experience should include:<br />

Ability to perform a complete medical history and physical exam.<br />

Ability to communicate with patients <strong>of</strong> diverse backgrounds.<br />

SPECIFIC LEARNING OBJECTIVES:<br />

A. KNOWLEDGE: Students should be able to define, describe, and discuss:<br />

1. Components <strong>of</strong> comprehensive and abbreviated case presentations (oral and written)<br />

and settings appropriate for each. (MK)<br />

B. SKILLS: Students should be able to demonstrate specific skills, including:<br />

1. Prepare legible, comprehensive, and focused new patient workups that include the<br />

following features as clinically appropriate:<br />

Chief complaint. (PC, CS)<br />

Identifying data. (PC, CS)<br />

Concise history <strong>of</strong> the present illness organized chronologically, with minimal repetition,<br />

omission, or extraneous information, and including pertinent positives and negatives.<br />

(PC, CS)<br />

Past medical history, including relevant details. (PC, CS)<br />

Medications with dosages and frequencies, including herbals, supplements, and overthe-counter<br />

medications. (PC, CS)<br />

Allergies with specific details <strong>of</strong> the reaction. (PC, CS)<br />

Substance use, including tobacco, alcohol, and illicit drugs. (PC, CS)<br />

Family history. (PC, CS)<br />

Social history. (PC, CS)<br />

Review <strong>of</strong> symptoms. (PC, CS)<br />

A comprehensive physical examination with detail pertinent to the patient’s problem.<br />

(PC, CS)<br />

A succinct, prioritized, and where appropriate complete list <strong>of</strong> all problems identified by<br />

the history and physical examination. (PC, CS)<br />

A differential diagnosis (appropriate for the student’s level <strong>of</strong> training) for each problem<br />

that is neither over-inclusive or under-inclusive, addresses all reasonable possibilities,<br />

pays special attention to diagnoses that are potentially the most serious or lifethreatening,<br />

and is supported by the use <strong>of</strong> pertinent positives and negatives. (PC, CS)<br />

194


A diagnostic and treatment plan for each problem (appropriate for the student’s level <strong>of</strong><br />

training). (PC, CS)<br />

2. Orally present a new inpatient’s or outpatient’s case in a manner that includes the<br />

following characteristics:<br />

Logically and chronologically develops the history <strong>of</strong> the present illness and tells the<br />

patient’s “story.” (PC, CS)<br />

Summarizes the pertinent positives and negatives. (PC, CS)<br />

Succinctly presents past medical history, family history, social history, and review <strong>of</strong><br />

symptoms. (PC, CS)<br />

Includes a logical, organized, and prioritized differential diagnosis. (PC, CS)<br />

Includes diagnostic and therapeutic plans. (PC, CS)<br />

Can be made briefer when necessary. (PC, CS)<br />

Is presented as much from memory as possible with minimal reference to memory<br />

aids with the exception <strong>of</strong> highly important dates, diagnostic tests, laboratory values.<br />

(PC, CS)<br />

3. Orally present a follow-up inpatient’s or outpatient’s case in a manner that includes the<br />

following characteristics:<br />

Focused and very concise. (PC, CS)<br />

Problem-based. (PC, CS)<br />

Emphasizes pertinent new findings. (PC, CS)<br />

Includes diagnostic and therapeutic plans. (PC, CS)<br />

Can be made briefer when necessary. (PC, CS)<br />

Is presented as much from memory as possible with minimal reference to memory<br />

aids with the exception <strong>of</strong> highly important dates, diagnostic tests, laboratory values.<br />

(PC, CS)<br />

4. Produce inpatient or outpatient progress notes in a manner that includes the following<br />

characteristics:<br />

Is appropriately titled. (PC, CS)<br />

Includes a brief subjective that addresses new or changed patient symptoms. (PC,<br />

CS)<br />

Provides an accurate and succinct accounting <strong>of</strong> the objective data<br />

(e.g. vital signs, in/outs, telemetry monitoring, focused physical examination,<br />

laboratory results, and diagnostic tests). (PC, CS)<br />

Includes a prioritized problem list with a concise assessment and plan for each. (PC,<br />

CS)<br />

5. Select the mode <strong>of</strong> presentation that is most appropriate to the clinical situation (e.g.<br />

written vs. oral, long vs. short, etc.). (PC, CS)<br />

C. ATTITUDES AND PROFESSIONAL BEHAVIORS: Students should be able to:<br />

1. Demonstrate ongoing commitment to self-directed learning regarding case presentation<br />

skills by regularly seeking feedback on presentations. (PLI, P)<br />

2. Respond appropriately and productively to feedback regarding performance.<br />

(P)<br />

3. Accurately and objectively record and present all data. (P)<br />

195


4. Demonstrate respect for the patient’s privacy when dealing with protected health<br />

information and follow Health Information Portability and Accountability Act (HIPAA)<br />

standards. (P)<br />

D. REFERENCES:<br />

Primer to the Internal <strong>Medicine</strong> Clerkship A Guide Produced by the Clerkship Directors in<br />

Internal <strong>Medicine</strong> Clerkship Directors in Internal <strong>Medicine</strong><br />

www.im.org/CDIM/primer.htm<br />

Sobel RK. MSL – medicine as a second language. N Engl J Med. 2005;352:1945<br />

196


GENERAL CLINICAL CORE COMPETENCIES<br />

#3 HISTORY TAKING AND PHYSICAL EXAMINATION<br />

RATIONALE:<br />

The ability to obtain an accurate medical history and carefully perform a physical examination is<br />

fundamental to providing comprehensive care to adult patients. In particular, the internist must be<br />

thorough and efficient in obtaining a history and performing a physical examination with a wide variety<br />

<strong>of</strong> patients, including healthy adults (both young and old), adults with acute and chronic medical<br />

problems, adults with complex life-threatening diseases, and adults from diverse socioeconomic and<br />

cultural backgrounds. The optimal selection <strong>of</strong> diagnostic tests, choice <strong>of</strong> treatment, and use <strong>of</strong><br />

subspecialists, as well as the physician’s relationship and rapport with patients, all depend on welldeveloped<br />

history-taking and physical diagnosis skills. These skills, which are fundamental to<br />

effective patient care, should be a primary focus <strong>of</strong> the student’s work during the core <strong>clerkship</strong> in<br />

internal medicine.<br />

PREREQUISITES:<br />

Prior knowledge, skills, and attitudes acquired during the pre-<strong>clerkship</strong> experience<br />

should include:<br />

Required pre-clinical courses in physical examination and physician-patient<br />

communication (should include instruction in breast, pelvic, rectal, and male genital exams).<br />

Ability to perform a complete medical history and physical exam on a wide variety <strong>of</strong><br />

patients including adolescents and older adults.<br />

Ability to effectively communicate with patients <strong>of</strong> diverse backgrounds.<br />

Basic skills for obtaining a history related to substance abuse, sexual, occupational,<br />

and mental health.<br />

SPECIFIC LEARNING OBJECTIVES:<br />

A. KNOWLEDGE: Students should be able to define, describe, and discuss:<br />

1. The significant attributes <strong>of</strong> a symptom, including: location and radiation, intensity, quality,<br />

temporal sequence (onset, duration, frequency), alleviating factors, aggravating factors,<br />

setting, associated symptoms, functional impairment, and patient’s interpretation <strong>of</strong> symptom.<br />

(MK)<br />

2. The four methods <strong>of</strong> physical examination (inspection, palpation, percussion, and<br />

auscultation), including where and when to use them, their purposes, and the findings they<br />

elicit. (MK)<br />

3. The physiologic mechanisms that explain key findings in the history and physical exam. (MK)<br />

4. The diagnostic value <strong>of</strong> the history and physical examination. (MK)<br />

B. SKILLS: Students should be able to demonstrate specific skills, including:<br />

1. Using language appropriate for each patient. (PC, CS)<br />

2. Using non-verbal techniques to facilitate communication and pursue relevant inquiry. (PC, CS)<br />

3. Eliciting the patient’s chief complaint as well as a complete list <strong>of</strong> the patient’s concerns.<br />

(PC,CS)<br />

4. Obtaining a patient’s history in a logical, organized, and thorough manner, covering the<br />

following:<br />

197


History <strong>of</strong> present illness. (PC, CS)<br />

past medical history (including usual source <strong>of</strong> and access to health care, childhood and<br />

adult illnesses, injuries, surgical procedures, obstetrical history, psychiatric problems,<br />

sexual history, and hospitalizations). (PC, CS)<br />

Preventive health measures. (PC, CS)<br />

Medications with dosages and frequencies, including herbals, supplements, and overthe-counter<br />

medications. (PC, CS)<br />

Allergies with specific details <strong>of</strong> the reaction. (PC, CS)<br />

Substance use including tobacco, alcohol, and illicit drugs. (PC, CS)<br />

Family history. (PC, CS)<br />

Social history. (PC, CS)<br />

Occupational history. (PC, CS)<br />

Review <strong>of</strong> symptoms. (PC, CS)<br />

5. Obtaining, whenever necessary, supplemental historical information from collateral sources,<br />

such as significant others or previous physicians. (PC, CS)<br />

6. Demonstrating proper hygienic practices whenever examining a patient. (PC)<br />

7. Positioning the patient and self properly for each part <strong>of</strong> the physical examination. (PC)<br />

8. Performing a physical examination for a patient in a logical, organized, respectful, and<br />

thorough manner, including:<br />

The patient’s general appearance. (PC)<br />

Vital signs. (PC)<br />

Pertinent body regions/organ systems. (PC)<br />

When appropriate breast, pelvic, rectal, male genital exams. (PC)<br />

When appropriate fundoscopic exam. (PC)<br />

When appropriate full neurologic exam. (PC)<br />

9. Adapting the scope and focus <strong>of</strong> the history and physical exam appropriately to the medical<br />

situation and the time available. (PC)<br />

10. Being observant <strong>of</strong> the patient’s modesty as much as possible. (PC, P)<br />

C. ATTITUDES AND PROFESSIONAL BEHAVIORS: Students should be able to:<br />

1. Appreciate the essential contribution <strong>of</strong> a pertinent and history and physical examination<br />

to patient care. (P)<br />

2. Demonstrate ongoing commitment to self-directed learning regarding history taking and<br />

physical examination skills. (PLI, P)<br />

3. Seek feedback regularly regarding history and physical examination skills and respond<br />

appropriately and productively. (P)<br />

4. Recognize the importance <strong>of</strong> and demonstrate a commitment to the utilization <strong>of</strong> other<br />

health care pr<strong>of</strong>essions in obtaining a history and physical examination (e.g. interpreter<br />

services, advanced practice nurses, etc.). (P, SBP)<br />

5. Establish a habit <strong>of</strong> updating historical information and repeating important parts <strong>of</strong> the<br />

physical examination during follow-up visits. (P)<br />

6. Demonstrate consideration for the patient’s modesty, feelings, limitations, and<br />

sociocultural background whenever taking a history and performing a physical<br />

examination. (P)<br />

7. Appreciate that some patients will be very anxious about the physical examination,<br />

particularly the breast, pelvic, rectal, and male genital exams. (P)<br />

D. REFERENCES:<br />

198


McGee SR. Evidence-Based Physical Diagnosis. Philadelphia, PA: W. B. Saunders Company;<br />

2001.<br />

Bickley LS, Szilagyi PG. Bates’ Guide to Physical Examination and History Taking. 8th Edition.<br />

Philadelphia, PA: Lipincott Williams and Wilkens; 2002.<br />

The Auscultation Assistant<br />

www.med.ucla.edu/wilkes/intro.html<br />

Criley JM. The Physiologic Origin <strong>of</strong> Heart Sounds and Murmurs: The Unique Interactive Guide<br />

to Cardiac Diagnosis. Philadelphia, PA: Lippincott Williams and Wilkins; 1997.<br />

Heart Sounds and Cardiac Arrhythmias<br />

Medical Multimedia Laboratories<br />

www.blaufuss.org<br />

199


GENERAL CLINICAL CORE COMPETENCIES<br />

#4 COMMUNICATION AND RELATIONSHIPS WITH PATIENTS AND COLLEAGUES<br />

RATIONALE:<br />

The physician-patient relationship forms the core <strong>of</strong> the practice <strong>of</strong> internal medicine. Many<br />

physicians view it as the most satisfying aspect <strong>of</strong> their work. The medical interview and the<br />

relationship between physician and patient are important diagnostic and therapeutic tools. Effective<br />

communication skills are needed for a physician to serve as an effective patient advocate.<br />

Communication skills also are needed to address patient concerns and requests. Pr<strong>of</strong>iciency in<br />

communicating with patients results in increased patient and physician satisfaction, increased<br />

adherence to therapy, and reduced risk <strong>of</strong> malpractice claims. The student on the internal medicine<br />

<strong>clerkship</strong> interacts with a diverse array <strong>of</strong> patients, physicians, and other health team members,<br />

necessitating pr<strong>of</strong>iciency in communication and interpersonal skills. Students also witness how<br />

diversities <strong>of</strong> age, gender, race, culture, socioeconomic class, personality, and intellect require a<br />

sensitive and flexible approach. The result <strong>of</strong> pr<strong>of</strong>iciency in communication and interpersonal skills is<br />

increased satisfaction for both doctor and patient.<br />

PREREQUISITES:<br />

Prior knowledge, skills, and attitudes acquired during the pre-<strong>clerkship</strong> experience should<br />

include:<br />

Required pre-clinical courses in physician-patient communication.<br />

Ability to perform a complete medical history on a wide variety <strong>of</strong> patients, including adolescents<br />

and older adults.<br />

Ability to communicate with patients <strong>of</strong> diverse backgrounds.<br />

Basic skills for obtaining a history related to substance abuse and sexual, occupational, and<br />

mental health.<br />

Basic skills for discussing issues relating to advance directives.<br />

Basic skills for breaking bad news.<br />

SPECIFIC LEARNING OBJECTIVES:<br />

A. KNOWLEDGE: Students should be able to define, describe, and discuss:<br />

1. How patients’ and physicians’ perceptions, preferences, and actions are affected by<br />

cultural and psychosocial factors and how these factors affect the doctor-patient<br />

relationship. (MK, P)<br />

2. The role and contribution <strong>of</strong> each team member to the care <strong>of</strong> the patient. (MK, SBP)<br />

3. The role <strong>of</strong> psychosocial factors in team interactions. (MK)<br />

4. The role <strong>of</strong> the physician as patient advocate. (MK)<br />

5. Strategies for establishing positive patient-doctor relationships. (MK)<br />

6. Patient, physician, and system barriers to successfully negotiated treatment plans and<br />

patient adherence; strategies that may be used to overcome these barriers. (MK, SBP)<br />

6. Useful strategies when a communicating with patients via an interpreter. (MK)<br />

7. Basic techniques for breaking bad news. (MK)<br />

8. Basic tenants <strong>of</strong> genetic counseling. (MK)<br />

B. SKILLS: Students should be able to demonstrate specific skills, including:<br />

200


1. Demonstrating appropriate listening skills, including verbal and non-verbal techniques<br />

(e.g., restating, probing, clarifying, silence, eye contact, posture, touch) to communicate<br />

empathy and help educate the patient. (CS)<br />

2. Demonstrating effective verbal skills including appropriate use <strong>of</strong> open- and closedended<br />

questions, repetition, facilitation, explanation, and interpretation. (CS)<br />

3. Determining the information a patient has independently obtained about his or her<br />

problems. (CS)<br />

4. Identifying patients’ emotional needs. (CS)<br />

5. Respond to empathic opportunities by naming the emotions or feelings expressed. (CS)<br />

6. Eliciting the patient’s point <strong>of</strong> view and concerns about his or her illness and the medical<br />

care he or she is receiving. (CS)<br />

7. Discussing how the health problem affects the patient’s life. (CS)<br />

8. Determining the extent to which a patient wants to be involved in making decisions<br />

about his or her care. (CS)<br />

9. Providing basic information and an explanation <strong>of</strong> the diagnosis, prognosis, and<br />

treatment plan. (CS)<br />

10. Responding to patients’ concerns and expectations. (CS)<br />

11. With guidance and direct supervision, participating in breaking bad news to patients.<br />

(CS)<br />

12. With guidance and direct supervision, participating in discussing basic issues regarding<br />

advance directives with patients and their families. (CS)<br />

13. With guidance and direct supervision, participating in discussing basic end-<strong>of</strong> life issues<br />

with patients and their families. (CS)<br />

14. Assessing patient commitment and adherence to a treatment plan taking into account<br />

personal and economic circumstances. (CS)<br />

15. Working with a variety <strong>of</strong> patients, including multi-problem patients, angry patients,<br />

somatizing patients, and substance abuse patients. (CS)<br />

16. Working as an effective member <strong>of</strong> the patient care team, incorporating skills in interpr<strong>of</strong>essional<br />

communication and collaboration. (CS, SBP)<br />

17. Giving and receiving constructive feedback. (CS)<br />

18. Orally presenting a new inpatient’s or outpatient’s case in a manner that includes the<br />

following characteristics:<br />

Logically and chronologically develops the history <strong>of</strong> the present illness and tells<br />

the patient’s “story.” (PC, CS)<br />

Summarizes the pertinent positives and negatives. (PC, CS)<br />

Succinctly presents past medical history, family history, social history, and review<br />

<strong>of</strong> symptoms. (PC, CS)<br />

includes a logical, organized, and prioritized differential diagnosis (PC, CS)<br />

Includes diagnostic and therapeutic plans. (PC, CS)<br />

Can be made briefer when necessary. (PC, CS)<br />

Is presented as much from memory as possible with minimal reference to<br />

memory aids with the exception <strong>of</strong> highly important dates, diagnostic tests,<br />

laboratory values. (PC, CS)<br />

19. Orally presenting a follow-up inpatient’s or outpatient’s case in a manner that includes<br />

the following characteristics:<br />

Is focused, very concise, and problem-based. (PC, CS)<br />

Emphasizes pertinent new findings. (PC, CS)<br />

Includes diagnostic and therapeutic plans. (PC, CS)<br />

201


Can be made briefer when necessary. (PC, CS)<br />

Is presented as much from memory as possible with minimal reference to<br />

memory aids with the exception <strong>of</strong> highly important dates, diagnostic tests, and<br />

laboratory values. (PC, CS)<br />

20. Demonstrating the ability to make clear and concise presentations about topics<br />

assigned to research. (CS)<br />

21. Demonstrating basic strategies for conflict management and resolution. (CS)<br />

22. Demonstrating basic techniques <strong>of</strong> communication with non-English speaking patient<br />

via an interpreter. (PC, CS)<br />

C. ATTITUDES AND PROFESSIONAL BEHAVIORS: Students should be able to:<br />

1. Demonstrate ongoing commitment to self-directed learning regarding effective doctorpatient<br />

communication skills. (PLI, P)<br />

2. Seek feedback regularly regarding communication skills and respond appropriately and<br />

productively. (P)<br />

3. take into consideration in each case the patient’s psychosocial status (P)<br />

4. Demonstrate respect for patients. (P)<br />

5. Involve the patient actively in his or her health care whenever possible. (P)<br />

6. Demonstrate teamwork and respect toward all members <strong>of</strong> the health care team, as<br />

manifested by reliability, responsibility, honesty, helpfulness, selflessness, and initiative<br />

in working with the team. (SBP, P)<br />

7. Attend to or advocate for the patient’s interests and needs in a manner appropriate to<br />

the student’s role. (P)<br />

8. Maintain confidentiality when dealing with protected health information and follow Health<br />

Information Portability and Accountability Act (HIPAA) guidelines. (P, SBP)<br />

D. REFERENCES:<br />

Clinician-Patient Communication to Enhance Health Outcomes. Institute for Health Care<br />

communication, Inc., West Haven, Connecticut, 1998.<br />

www.healthcarecomm.org<br />

Contemporary Issues in <strong>Medicine</strong>: Communication in <strong>Medicine</strong> Medical School Objectives<br />

Project, October 1999 American Association <strong>of</strong> Medical Colleges<br />

www.aamc.org/meded/msop/msop3.pdf<br />

Kurtz S, Silverman J, Draper J. Teaching and Learning Communication Skills in <strong>Medicine</strong>.<br />

Oxford: Radcliffe Medical Press Ltd; 1998.<br />

Kurtz S, Silverman J, Benson J, Draper J. Marrying content and process in clinical method<br />

teaching: enhancing the Calgary-Cambridge guides. Acad Med. 2003;78:802-9.<br />

Makoul G. The SEGUE Framework for teaching and assessing<br />

communication skills. Patient Educ Couns. 2001;45:23-34.<br />

Makoul G. Essential elements <strong>of</strong> communication in medical encounters: the Kalamazoo<br />

consensus statement. Acad Med. 2001;76:390-3.<br />

von Gunten CF, Ferris FD, Emanuel LL. Ensuring competency in end-<strong>of</strong>-life care:<br />

communication and relational skills. JAMA. 2000;284:3051-7.<br />

202


GENERAL CLINICAL CORE COMPETENCIES<br />

#5 INTERPRETATION OF CLINICAL INFORMATION<br />

RATIONALE:<br />

In the routine course <strong>of</strong> clinical practice, most physicians are required to order and interpret a wide<br />

variety <strong>of</strong> diagnostic tests and procedures. Determining how these test results will influence clinical<br />

decision making and communicating this information to patients in a timely and effective manner are<br />

core clinical skills that third-year medical students should possess.<br />

PREREQUISITES:<br />

Prior knowledge, skills and attitudes acquired during the pre-<strong>clerkship</strong> experience should include:<br />

Introductory course in clinical pathology and laboratory medicine.<br />

Introductory course in epidemiology and biostatistics.<br />

SPECIFIC LEARNING OBJECTIVES:<br />

A. KNOWLEDGE: Students should be able to:<br />

1. Interpret specific diagnostic tests and procedures that are ordered to evaluate patients who<br />

present with common symptoms and diagnoses encountered in the practice <strong>of</strong> internal<br />

medicine. (PC, MK)<br />

2. Take into account:<br />

Important differential diagnostic considerations, including potential diagnostic<br />

emergencies. (PC, MK)<br />

Pre-test and post-test likelihood <strong>of</strong> disease (probabilistic reasoning). (PC, MK)<br />

Performance characteristics <strong>of</strong> individual tests. (sensitivity, specificity, positive and<br />

negative predictive value, likelihood ratios). (PC, MK)<br />

3. Define and describe for the tests and procedures listed:<br />

Indications for testing. (PC, MK)<br />

Range <strong>of</strong> normal variation. (PC, MK)<br />

Critical values that require immediate attention. (PC, MK)<br />

Pathophysiologic implications <strong>of</strong> abnormal results. (PC, MK)<br />

Relative cost. (MK, SBP)<br />

4. Independently interpret the results <strong>of</strong> the following laboratory tests:<br />

CBC with diff and blood smear. (PC, MK)<br />

UA. (PC, MK)<br />

Electrolytes. (PC, MK)<br />

BUN/Cr. (PC, MK)<br />

GLC. (PC, MK)<br />

Hepatic function panel. (PC, MK)<br />

Hepatitis serologies. (PC, MK)<br />

Cardiac biomarkers (e.g. myoglobin, CK-MB, and Troponin I/T). (PC, MK)<br />

Routine coagulation tests (e.g. PT/PTT and INR). (PC, MK)<br />

Thyroid function tests (e.g. T3, T4, and TSH). (PC, MK)<br />

ABG.(PC, MK)<br />

Body fluid cell counts and chemistries. (PC, MK)<br />

5. Independently interpret the results <strong>of</strong> the following diagnostic procedures:<br />

203


12-lead ECG. (PC, MK)<br />

Chest radiograph. (PC, MK)<br />

Plain abdominal films (e.g. obstructive series, KUB). (PC, MK)<br />

Pulmonary function tests. (PC, MK)<br />

6. Describe the basic electrophysiologic events that produce the surface ECG. (MK)<br />

7. Describe how errors in test interpretation can affect clinical outcomes and costs. (PC, MK)<br />

8. Describe the concept <strong>of</strong> a threshold as it relates to testing and treatment decisions. (PC, MK)<br />

9. Describe the basic principles <strong>of</strong> using genetic information in clinical decision making. (PC, MK)<br />

B. SKILLS: Students should be able to demonstrate specific skills, including:<br />

1. Interpreting a blood smear, Gram stain, and UA. (PC)<br />

2. Approaching ECG interpretation in a systematic and logical fashion analyzing the following:<br />

rate, rhythm, P wave morphology, PR interval, QRS width, axis, voltage, QT interval, ST<br />

segment morphology, and T wave morphology. (PC)<br />

3. Recognizing the following on ECG:<br />

Sinus tachycardia, sinus bradycardia, sinus arrhythmia. (PC)<br />

Premature atrial beats, ectopic atrial rhythm/tachycardia, narrow complex supraventricular<br />

tachycardia. (PC)<br />

Multifocal atrial tachycardia, atrial flutter, atrial fibrillation (PC)<br />

First degree, second degree (Mobitz type I and II), and third degree (complete) heart<br />

block. (PC)<br />

Junctional rhythm. (PC)<br />

Premature ventricular beats. (PC)<br />

Typical ventricular tachycardia, ventricular fibrillation. (PC)<br />

Left and right atrial enlargement. (PC)<br />

Left ventricular hypertrophy. (PC)<br />

Left and right bundle branch block, left anterior and posterior fascicular block. (PC)<br />

The characteristic features <strong>of</strong> a properly functioning ventricular or dual chamber<br />

pacemaker. (PC)<br />

The delta wave in Wolf-Parkinson-White Syndrome. (PC)<br />

The classic features <strong>of</strong> myocardial ischemia and infarction and be able to localize the<br />

findings (i.e. inferior, anterior, lateral, posterior, right ventricular) and identify the probable<br />

culprit vessel. (PC)<br />

The classic features <strong>of</strong> pulmonary embolism. (PC)<br />

The characteristic effects <strong>of</strong> hypo- and hyperkalemia. (PC)<br />

4. Approaching chest radiography interpretation in a systematic and logical fashion analyzing the<br />

following: technique (e.g. view, rotation, exposure), visible abdomen, s<strong>of</strong>t tissues and bones <strong>of</strong><br />

the thorax, mediastinum/hila, and lungs. (PC)<br />

5. Recognizing the following on chest radiograph:<br />

Rib fracture. (PC)<br />

Cardiomegaly. (PC)<br />

Lobar pneumonia. (PC)<br />

Pleural effusion. (PC)<br />

Pneumothorax. (PC)<br />

Pulmonary nodule. (PC)<br />

204


Pulmonary edema/“congestive heart failure” (e.g. cardiomegaly, pulmonary vascular<br />

redistribution, Kerley’s B Lines, interstitial/alveolar edema). (PC)<br />

Hilar lymphadenopathy. (PC)<br />

Mediastinal widening. (PC)<br />

6. Recording the results <strong>of</strong> laboratory tests in an organized manner, using flow sheets when<br />

appropriate. (PC)<br />

7. Estimating the pre-test likelihood <strong>of</strong> a disease or condition. (PC, MK)<br />

8. Estimating the post-test probability <strong>of</strong> disease and stating the clinical significance <strong>of</strong> the results<br />

<strong>of</strong> laboratory tests and diagnostic procedures. (PC, MK)<br />

C. ATTITUDES AND PROFESSIONAL BEHAVIORS: Students should be able to:<br />

1. Demonstrate commitment to using risk-benefit, cost-benefit, and evidence-based<br />

considerations in the selection <strong>of</strong> diagnostic and therapeutic interventions for acute MI. (PLI, P,<br />

SBP)<br />

2. Regularly seek feedback regarding interpretation <strong>of</strong> clinical information and respond<br />

appropriately and productively. (P)<br />

3. Recognize the importance <strong>of</strong> patient preferences when selecting among diagnostic testing<br />

options. (P)<br />

4. Demonstrate ongoing commitment to self-directed learning regarding test interpretation. (PLI,<br />

P)<br />

5. Appreciate the implications <strong>of</strong> test results before ordering tests. (P)<br />

6. Appreciate the importance <strong>of</strong> follow-up on all diagnostic tests and procedures and timely<br />

communication <strong>of</strong> information to patients and appropriate team members. (P)<br />

7. Demonstrate a commitment to excellence by personally reviewing radiographs, ECGs, Gram<br />

stains, blood smears, etc. to assess the accuracy and significance <strong>of</strong> the results. (P)<br />

D. REFERENCES:<br />

Jaeschke R, Guyatt G, Sackett DL. Users’ guides to the medical literature.<br />

III. How to use an article about a diagnostic test. A. Are the results <strong>of</strong> the study valid? JAMA.<br />

1994; 271:389-91.<br />

Jaeschke R, Guyatt G, Sackett DL. Users’ guides to the medical literature.<br />

III. How to use an article about a diagnostic test. B. What are the results and will they help me<br />

in caring for my patients? JAMA. 1994; 271: 703-7.<br />

Dubin D. Rapid Interpretation <strong>of</strong> EKG's. 5th ed. Tampa, FL: Cover Publishing Company; 2000.<br />

Goodman LR. Felson’s Principles <strong>of</strong> Chest Roentgenology: A Programmed Text. 2nd ed.<br />

Philadelphia, PA: W. B. Saunders; 1999.<br />

Novelline RA. Squire's Fundamentals <strong>of</strong> Radiology. 6th ed. Cambridge, MA: <strong>University</strong> Press;<br />

2004.<br />

Mark, DB. Decision-making in clinical medicine. In Kasper DL, Braunwald EB, Fauci AS,<br />

Hauser SL, Longo DL, Jameson JL, eds. Harrison’s Principles <strong>of</strong> Internal <strong>Medicine</strong>. 16th ed.<br />

New York: McGraw-Hill; 2005:6-13.<br />

Lab Test Online<br />

www.labtestsonline.org/<br />

RadQuiz: Your Gateway to Radiology Resources<br />

www.radquiz.com<br />

Introduction to Chest Radiology<br />

205


<strong>Department</strong> <strong>of</strong> Radiology<br />

<strong>University</strong> <strong>of</strong> Virginia Health Sciences Center<br />

www.med-ed.virginia.edu/courses/rad/cxr/index.html<br />

Ferri FF. Laboratory values and interpretation <strong>of</strong> results. In Ferri FF, ed. Practical Guide to<br />

the Care <strong>of</strong> the Medical Patient. 6th ed. St. Louis, MO: Mosby; 2004:935-976.<br />

206


#6 THERAPEUTIC DECISION MAKING<br />

GENERAL CLINICAL CORE COMPETENCIES<br />

RATIONALE:<br />

Internists are responsible for directing and coordinating the therapeutic management <strong>of</strong> patients with<br />

a wide variety <strong>of</strong> problems, including critically ill patients with complex medical problems and the<br />

chronically ill. To manage patients effectively, physicians need basic therapeutic decision-making<br />

skills that incorporate both pathophysiologic reasoning and evidence-based knowledge.<br />

PREREQUISITES:<br />

Introductory coursework in clinical epidemiology and biostatistics.<br />

Introductory coursework in physiology and pathology.<br />

Introductory coursework in pharmacology.<br />

SPECIFIC LEARNING OBJECTIVES:<br />

A. KNOWLEDGE: Students should be able to define, describe, and discuss:<br />

1. Information resources for determining medical and surgical treatment options for patients<br />

with common and uncommon medical problems. (MK)<br />

2. Key factors to consider in choosing among treatment options, including risk, cost, evidence<br />

about efficacy, and consistency with pathophysiologic reasoning (MK)<br />

3. How to use critical pathways and clinical practice guidelines to help guide therapeutic<br />

decision making. (MK)<br />

4. Factors that frequently alter the effects <strong>of</strong> medications, including drug interactions and<br />

compliance problems. (MK)<br />

5. Factors to consider in selecting a medication from within a class <strong>of</strong> medications. (MK)<br />

6. Factors to consider in monitoring a patient’s response to treatment, including potential<br />

adverse effects. (MK)<br />

7. Various ways that evidence about clinical effectiveness is presented to clinicians and the<br />

potential biases <strong>of</strong> using absolute or relative risk or number <strong>of</strong> patients needed to treat. (MK)<br />

8. Methods <strong>of</strong> monitoring therapy and how to communicate them in both written and oral form.<br />

(MK)<br />

9. The basics <strong>of</strong> the potential role <strong>of</strong> genetic information in therapeutic decision making. (MK)<br />

B. SKILLS: Students should be able to demonstrate specific skills, including:<br />

1. Formulating an initial therapeutic plan. (PC)<br />

2. Changing the therapeutic plan when goals <strong>of</strong> care change (e.g. a shift toward palliative<br />

care). (PC)<br />

3. Accessing and utilizing, when appropriate, information resources to help develop an<br />

appropriate and timely therapeutic plan. (PC, PLI)<br />

4. Explaining the extent to which the therapeutic plan is based on pathophysiologic reasoning<br />

and scientific evidence <strong>of</strong> effectiveness. (PC)<br />

5. Beginning to estimate the probability that a therapeutic plan will produce the desired<br />

outcome. (PC)<br />

6. Writing prescriptions and inpatient orders safely and accurately. (PC)<br />

7. Counseling patients about how to take their medications and what to expect<br />

when doing so, including beneficial outcomes and potential adverse effects. (PC, CS)<br />

207


8. Monitoring response to therapy. (PC)<br />

9. Recognizing when to seek consultation for additional diagnostic and therapeutic<br />

recommendations. (PC, SBP)<br />

10. Recognizing when to screen for certain conditions based on age and risk factors and what<br />

to do with the results <strong>of</strong> the screening tests. (PC)<br />

C. ATTITUDES AND PROFESSIONAL BEHAVIORS: Students should be able to:<br />

1. Demonstrate commitment to using risk-benefit, cost-benefit, and evidence-based<br />

consideration in the selection <strong>of</strong> therapeutic interventions. (PLI, P)<br />

2. Demonstrate ongoing commitment to self-directed learning regarding therapeutic<br />

interventions. (PLI, P)<br />

3. Seek feedback regularly regarding therapeutic decision making and respond appropriately<br />

and productively. (P)<br />

4. Appreciate the impact therapeutic decisions have on a patient’s quality <strong>of</strong> life (P)<br />

5. Incorporate the patient in therapeutic decision making, explaining the risks and benefits <strong>of</strong><br />

treatment. (CS, P)<br />

6. Respect patients’ autonomy and informed choices, including the right to refuse treatment.<br />

(P)<br />

7. Demonstrate an understanding <strong>of</strong> the importance <strong>of</strong> close follow-up <strong>of</strong> patients under active<br />

care. (P)<br />

8. Recognize the importance <strong>of</strong> and demonstrate a commitment to the utilization <strong>of</strong> other<br />

health care pr<strong>of</strong>essionals in therapeutic decision making. (P, SBP)<br />

D. REFERENCES:<br />

Mark, DB. Decision-making in clinical medicine. In Kasper DL, Braunwald EB, Fauci AS,<br />

Hauser SL, Longo DL, Jameson JL, eds. Harrison’s Principles <strong>of</strong> Internal <strong>Medicine</strong>. 16th ed.<br />

New York: McGraw-Hill; 2005:6-13.<br />

Roden DM. Principles <strong>of</strong> clinical pharmacology. In Kasper DL, Braunwald EB, Fauci AS,<br />

Hauser SL, Longo DL, Jameson JL, eds. Harrison’s Principles <strong>of</strong> Internal <strong>Medicine</strong>. 16th ed.<br />

New York: McGraw-Hill; 2005:13-25.<br />

Users' guides to the medical literature. II. How to use an article about therapy or prevention. A.<br />

Are the results <strong>of</strong> the study valid? Evidence-Based <strong>Medicine</strong> Working Group. JAMA.<br />

1993;270:2598-601.<br />

Users' guides to the medical literature. II. How to use an article about therapy or prevention. B.<br />

What were the results and will they help me in caring for my patients? Evidence-Based<br />

<strong>Medicine</strong> Working Group. JAMA. 1994;271:59-63.<br />

#7 BIOETHICS OF CARE<br />

GENERAL CLINICAL CORE COMPETENCIES<br />

RATIONALE:<br />

A basic understanding <strong>of</strong> ethical principles and their application to patient care is essential for all<br />

physicians. During the internal medicine core <strong>clerkship</strong>, the student can put into practice some <strong>of</strong> the<br />

ethical principles learned in the preclinical years, especially by participating in discussions <strong>of</strong> informed<br />

208


consent and advance directives. Additionally, the student learns to recognize ethical dilemmas and<br />

respect different perceptions <strong>of</strong> health, illness, and health care held by patients <strong>of</strong> various religious<br />

and cultural backgrounds.<br />

PREREQUISITES:<br />

Introductory course on medical ethics providing a basic understanding <strong>of</strong> ethical principles and<br />

fiduciary relationships and their application in clinical medicine:<br />

Autonomy.<br />

Beneficence.<br />

Nonmaleficence.<br />

Truth-telling.<br />

Confidentiality.<br />

Respect for autonomy (informed choice).<br />

SPECIFIC LEARNING OBJECTIVES:<br />

A. KNOWLEDGE: Students should be able to define, describe, and discuss:<br />

1. Basic ethical principles (autonomy, beneficence, nonmaleficence, truth-telling, confidentiality,<br />

and autonomy). (MK)<br />

2. The patient’s right to refuse care. (MK)<br />

3. The unique nature <strong>of</strong> a fiduciary relationship. (MK)<br />

4. Basic elements <strong>of</strong> informed consent. (MK)<br />

5. Circumstances under which informed consent is necessary and unnecessary (MK)<br />

6. Basic concepts <strong>of</strong> treatment efficacy, quality <strong>of</strong> life, and societal demands. (MK)<br />

7. Potential conflicts between individual patient preferences and societal demands (MK)<br />

8. The role <strong>of</strong> the physician in making decisions about the use <strong>of</strong> expensive or controversial tests<br />

and treatments. (MK)<br />

9. Bioethical concerns regarding genetic information, privacy issues in particular. (MK)<br />

10. The unique bioethical concerns regarding end-<strong>of</strong>-life care. (MK)<br />

11. Circumstances when withholding or withdrawing care is acceptable. (MK)<br />

12. The role <strong>of</strong> federal and state legislation in governing health care. (MK)<br />

13. Circumstances when it may be unavoidable or acceptable to breach the basic ethical<br />

principles. (MK)<br />

B. SKILLS: Students should be able to demonstrate specific skills, including:<br />

1. Participating in a discussion about advance directives with a patient. (PC, CS)<br />

2. Participating in obtaining informed consent for a procedure. (PC, CS)<br />

3. Participating in explaining and obtaining informed consent for genetic testing (PC, CS)<br />

4. Participating in a preceptor’s discussion with a patient about a requested<br />

treatment that may not be considered appropriate (e.g., not cost-effective). (PC, CS)<br />

5. Participating in family and interdisciplinary team conferences discussing end<strong>of</strong>-life<br />

care and incorporating the patient’s wishes in that discussion. (PC, CS, SBP)<br />

6. Obtaining additional help from ethics experts in conflict resolution. (PC, SBP)<br />

C. ATTITUDES AND PROFESSIONAL BEHAVIORS: Students should be able to:<br />

209


1. Demonstrate ongoing commitment to self-directed learning regarding bioethics. (PLI)<br />

2. Recognize the importance <strong>of</strong> patient preferences, perspectives, and perceptions regarding<br />

health and illness. (P)<br />

3. Demonstrate a commitment to caring for all patients, regardless <strong>of</strong> the medical diagnosis,<br />

gender, race, socioeconomic status, intellect/level <strong>of</strong> education, religion, political affiliation,<br />

sexual orientation, ability to pay, or cultural background. (P)<br />

4. Recognize the importance <strong>of</strong> allowing terminally ill patients to die with comfort and dignity<br />

when that is consistent with the wishes <strong>of</strong> the patient and/or the patient’s family. (P)<br />

5. Recognize the potential conflicts between patient expectations and medically appropriate care.<br />

(P)<br />

6. Respond appropriately to patients who are nonadherent to treatment. (P)<br />

7. Demonstrate respect for the patient’s privacy and confidentiality when dealing with protected<br />

health information and follow HIPAA standards. (P)<br />

8. Appreciate the psychological impact genetic information may have on patients. (P)<br />

D. REFERENCES:<br />

Bioethics Resources on the Web<br />

Inter-Institute Bioethics Interest Group<br />

National Institutes <strong>of</strong> Health<br />

U.S. <strong>Department</strong> <strong>of</strong> Health and Human Services<br />

www.nih.gov/sigs/bioethics/<br />

<strong>University</strong> <strong>of</strong> Pennsylvania Center for Bioethics<br />

www.bioethics.upenn.edu<br />

Snyder L, Leffler C, Ethics and Human Rights Committee, American College <strong>of</strong> Physicians.<br />

Ethics manual: fifth edition. Ann Intern Med. 2005;142:560-82.<br />

World Medical Association Ethics Unit<br />

www.wma.net/e/ethicsunit/resources.htm<br />

Ethics in <strong>Medicine</strong> <strong>University</strong> <strong>of</strong> Washington School <strong>of</strong> <strong>Medicine</strong><br />

eduserv.hscer.washington.edu/bioethics<br />

Program in Ethics In Science and <strong>Medicine</strong> <strong>University</strong> <strong>of</strong> Texas Southwestern Medical Center<br />

www3.utsouthwestern.edu/ethics/<br />

Virtual Mentor American Medical Association<br />

www.ama-assn.org/ama/pub/category/3040.html<br />

Bioethics Interest Group American Medical Student Association<br />

www.amsa.org/bio/index.cfm<br />

210


#8 SELF-DIRECTED LEARNING<br />

GENERAL CLINICAL CORE COMPETENCIES<br />

RATIONALE:<br />

Because <strong>of</strong> the breadth <strong>of</strong> the problems encountered in clinical practice, internists face an<br />

extraordinary challenge to keep up with the burgeoning amount <strong>of</strong> new information relevant to<br />

providing high quality care. Therefore, they must master and practice self-directed life-long learning,<br />

including the ability to access and utilize information systems and resources efficiently.<br />

PREREQUISITES:<br />

Prior knowledge, skills, and attitudes acquired during the pre-clinical experience should include:<br />

Basic library skills, including the ability to perform an electronic literature search.<br />

Critical appraisal skills.<br />

Understanding <strong>of</strong> basic concepts <strong>of</strong> biostatistics and clinical epidemiology including: sensitivity,<br />

specificity, positive predictive value, negative predictive value, absolute risk reduction, relative<br />

risk reduction, number needed to treat, likelihood/odds ratios, and tests <strong>of</strong> significance.<br />

SPECIFIC LEARNING OBJECTIVES:<br />

A. KNOWLEDGE: Students should be able to define, describe, and discuss:<br />

1. Key sources for obtaining updated information on issues relevant to the medical management<br />

<strong>of</strong> adult patients. (MK, PLI)<br />

2. A system for managing information from a variety <strong>of</strong> sources. (MK, PLI)<br />

3. The concept <strong>of</strong> the focused clinical question. (MK, PLI)<br />

4. Key questions to ask when critically appraising articles on diagnostic tests:<br />

Was there an independent, blind comparison with a reference (“gold”) standard? (MK,<br />

PLI)<br />

Was the diagnostic test evaluated in an appropriate spectrum <strong>of</strong> patients (like those in<br />

whom it would be used in practice)? (MK, PLI)<br />

Was the reference standard applied regardless <strong>of</strong> the diagnostic test result? (MK, PLI)<br />

What were the results <strong>of</strong> the study (e.g. sensitivity, specificity, likelihood ratios, and/or<br />

pre- and post-test probabilities)? (MK, PLI)<br />

5. Key questions to ask when critically appraising articles on medical therapeutics:<br />

Was the assignment <strong>of</strong> patients to treatments randomized? (MK, PLI)<br />

Were all patients who entered the trial properly accounted for at the conclusion <strong>of</strong> the<br />

study and analyzed in the group they were randomized to? (MK, PLI)<br />

Were patients and study personnel blind to the treatment? (MK, PLI)<br />

Were the groups similar at the start <strong>of</strong> the trial? (MK, PLI)<br />

Aside from the experimental intervention, were the groups treated equally? (MK, PLI)<br />

What were the results <strong>of</strong> the trial (e.g. relative risk reduction, absolute risk reduction, and<br />

“number needed to treat”)? (MK, PLI)<br />

B. SKILLS: Students should be able to demonstrate specific skills, including:<br />

211


1. Performing a computerized literature search to find articles pertinent to a focused clinical<br />

question. (PLI)<br />

2. Demonstrating critical review skills. (PLI)<br />

3. Reading critically about issues pertinent to their patients. (PLI)<br />

4. Assessing the limits <strong>of</strong> medical knowledge in relation to patient problems (PLI)<br />

5. Using information from consultants critically. (PLI)<br />

6. Recognizing when additional information is needed to care for the patient (PLI)<br />

7. Asking colleagues (students, residents, nurses, faculty) for help when needed (PLI, SBP)<br />

8. Making use <strong>of</strong> available instruments to assess one’s own knowledge base (PLI, P)<br />

9. Summarizing and presenting to colleagues what was learned from consulting the medical<br />

literature. (PLI, CS)<br />

C. ATTITUDES AND PROFESSIONAL BEHAVIORS: Students should be able to:<br />

1. Demonstrate self-directed learning in every case. (PLI, P)<br />

2. Acknowledge gaps in knowledge to both colleagues and patients and request help. (PLI, P)<br />

3. Seek feedback regularly and respond appropriately and productively. (P)<br />

4. Recognize the value and limitations <strong>of</strong> other health care pr<strong>of</strong>essionals when confronted with a<br />

knowledge gap. (PLI, P, SBP)<br />

D. REFERENCES:<br />

Users' guides to the medical literature. I. How to get started. The Evidence-Based <strong>Medicine</strong><br />

Working Group. JAMA. 1993;270:2093-5.<br />

Users' guides to the medical literature. II. How to use an article about therapy or prevention. A.<br />

Are the results <strong>of</strong> the study valid? Evidence-Based <strong>Medicine</strong> Working Group. JAMA.<br />

1993;270:2598-601.<br />

Users' guides to the medical literature. II. How to use an article about therapy or prevention. B.<br />

What were the results and will they help me in caring for my patients? Evidence-Based<br />

<strong>Medicine</strong> Working Group. JAMA. 1994;271:59-63.<br />

Users' guides to the medical literature. III. How to use an article about a diagnostic test. A. Are<br />

the results <strong>of</strong> the study valid? Evidence-Based <strong>Medicine</strong> Working Group. JAMA.<br />

1994;271:389-91.<br />

Users' guides to the medical literature. III. How to use an article about a diagnostic test. B.<br />

What are the results and will they help me in caring for my patients? The Evidence-Based<br />

<strong>Medicine</strong> Working Group. JAMA. 1994;271:703-7.<br />

Strauss SE, Richardson WS, Glasziou P, Haynes RB. Evidence Based <strong>Medicine</strong>: How to<br />

Practice and Teach EBM. 3rd ed. New York, NY: Churchill Livingstone; 2005.<br />

Advancing Education in Practice-Based Learning and Improvement An Educational Resource<br />

from the ACGME Outcome Project<br />

www.acgme.org/outcome/implement/complete_PBLIBooklet.pdf<br />

#9 PREVENTION<br />

GENERAL CLINICAL CORE COMPETENCIES<br />

RATIONALE:<br />

One <strong>of</strong> the most important responsibilities <strong>of</strong> primary care physicians is to promote health and prevent<br />

disease in a cost-effective manner. Appropriate care by internists includes not only recognition and<br />

212


treatment <strong>of</strong> disease but also the routine incorporation <strong>of</strong> the principles <strong>of</strong> preventive health care into<br />

clinical practice. All physicians should be familiar with the principles <strong>of</strong> preventive health care to<br />

ensure their patients receive appropriate preventive services.<br />

PREREQUISITES:<br />

Prior knowledge, skills, and attitudes acquired during the pre-clinical experience should include:<br />

Introductory course in clinical epidemiology and biostatistics.<br />

Introductory course in health promotion and disease prevention.<br />

Ability to perform a complete medical history and physical exam.<br />

Ability to communicate with patients <strong>of</strong> diverse backgrounds.<br />

SPECIFIC LEARNING OBJECTIVES:<br />

A. KNOWLEDGE: Students should be able to define, describe, and discuss:<br />

1. Primary, secondary, and tertiary prevention. (MK)<br />

2. Criteria for determining whether or not a screening test should be incorporated into the periodic<br />

health assessment <strong>of</strong> adults. (MK)<br />

3. General types <strong>of</strong> preventive health care issues that should be addressed on a routine basis in<br />

adult patients (i.e., cancer screening; prevention <strong>of</strong> infectious diseases, coronary artery<br />

disease, osteoporosis, and injuries; and identification <strong>of</strong> substance abuse). (MK)<br />

4. Vaccines that have been recommended for routine use in at least some adults (i.e., influenza,<br />

pneumococcal, measles, mumps, rubella, tetanus-diphtheria, hepatitis). (MK)<br />

5. Indications for endocarditis prophylaxis. (MK)<br />

6. Methods for counseling patients about risk-factor modification, including the “stages <strong>of</strong> change”<br />

approach to helping patients change behavior. (MK)<br />

7. Influence <strong>of</strong> age and clinical status on approach to prevention. (MK)<br />

8. General categories <strong>of</strong> high-risk patients in whom routine preventative health care must be<br />

modified or enhanced (e.g., family history, travel to an underdeveloped area, occupational<br />

exposures, etc.). (MK)<br />

9. The major areas <strong>of</strong> controversy in screening. (MK)<br />

10. The potential roles and limitations <strong>of</strong> genetic testing in disease<br />

prevention/early detection. (MK)<br />

B. SKILLS: Students should be able to demonstrate specific skills, including:<br />

1. Obtaining a patient history, including a detailed family history, vaccination history, travel<br />

history, sexual history, and occupational exposures. (PC)<br />

2. Identifying patients at high risk for developing diabetes, dyslipidemia, coronary artery disease,<br />

cancer, osteoporosis, influenza, pneumonia, hepatitis, HIV infection, and tuberculosis by<br />

screening for major risk factors. (PC)<br />

3. Obtaining a Pap smear and interpreting its results. (PC)<br />

4. Performing a breast examination. (PC)<br />

5. Instructing patients to perform breast self-examination. (PC, CS)<br />

6. Interpreting the results <strong>of</strong> a mammogram. (PC)<br />

7. Performing a digital rectal examination. (PC)<br />

8. Interpreting the results <strong>of</strong> a PSA test and understand its limitations. (PC)<br />

9. Performing a testicular examination. (PC)<br />

10. Interpreting the results <strong>of</strong> a bone densitometry test. (PC)<br />

11. Interpreting the results <strong>of</strong> a fasting lipid pr<strong>of</strong>ile. (PC)<br />

213


12. Interpreting the results <strong>of</strong> a fasting glucose test. (PC)<br />

13. Counseling patients about safe-sex practices, smoking cessation, alcohol abuse, weight loss,<br />

healthy diet, exercise, and seat belt use. (PC, CS)<br />

14. place and interpret a PPD. (PC)<br />

15. Locating recently published recommendations as well as original data regarding measures that<br />

should be incorporated into the periodic health assessment <strong>of</strong> adults. (PLI)<br />

C. ATTITUDES AND PROFESSIONAL BEHAVIORS: Students should be able to:<br />

1. Address preventive health care issues as a routine part <strong>of</strong> their assessment <strong>of</strong> patients. (P)<br />

2. Encourage patients to share responsibility for health promotion and disease prevention. (P)<br />

3. Recognize the importance <strong>of</strong> patient preferences when recommending preventive health<br />

measures. (P)<br />

4. Understand the patient’s right to refuse preventive health measures. (P)<br />

5. Demonstrate commitment to using risk-benefit, cost-benefit, and evidence-based<br />

considerations in the selection preventive health measures. (PLI, P)<br />

6. Demonstrate ongoing commitment to self-directed learning regarding preventive health<br />

measures. (PLI, P)<br />

D. REFERENCES:<br />

Pomrehn PR, Davis MV, Chen DW, Barker W. Prevention for the 21st century: setting the<br />

context through undergraduate medical education. Acad Med. 2000;75(7 Suppl):S5-13.<br />

Guide to Clinical Preventive Services<br />

U.S. Preventative Services Task Force (USPSTF)<br />

Agency for Healthcare Research and Quality<br />

U.S. <strong>Department</strong> <strong>of</strong> Health and Human Services<br />

USPSTF Recommendation: Screening for Cancer<br />

www.ahrq.gov/clinic/cps3dix.htm#cancer<br />

USPSTF Recommendation: Screening for Lipid Disorders<br />

www.ahrq.gov/clinic/ajpmsuppl/lipidrr.htm<br />

USPSTF Recommendation: Screening for High Blood Pressure<br />

www.ahrq/clinic/3rduspstf/highbloodsc/hibloodrr.htm<br />

USPSTF Recommendations Statement: Counseling to prevent tobacco use and tobaccocaused<br />

disease<br />

www.ahrq.gov/clinic/3rduspstf/tobaccoun/tobcounrs.htm<br />

Screening for Prostate Cancer. American College <strong>of</strong> Physicians. Ann Int Med 1997; 126: 480-<br />

484.<br />

Summary <strong>of</strong> Recommendations for Adult Immunization Immunization Action Coalition Bulletin<br />

Adapted from the recommendations <strong>of</strong> the Advisory Commttee on Immunization Practices<br />

(ACIP), August 2005<br />

www.immunize.org/acip<br />

Martin GJ. Screening and prevention <strong>of</strong> disease. In Kasper DL, Braunwald EB, Fauci AS,<br />

Hauser SL, Longo DL, and Jameson JL eds. Harrison’s Principles <strong>of</strong> Internal <strong>Medicine</strong>. 16th<br />

ed. New York, NY: McGraw-Hill; 2005:26-28.<br />

214


#10 COORDINATION OF CARE<br />

GENERAL CLINICAL CORE COMPETENCIES<br />

RATIONALE:<br />

The task <strong>of</strong> coordinating a patient’s care is central to the role <strong>of</strong> the internist, and involves<br />

communication with the patient and his or her family, colleagues, consultants, nurses, social workers,<br />

and community-based agencies. It is essential for the student to learn that the physician’s<br />

responsibility toward the patient does not stop at the end <strong>of</strong> the <strong>of</strong>fice visit or hospitalization but<br />

continues in collaboration with other pr<strong>of</strong>essionals to ensure that the patient receives optimal care.<br />

PREREQUISITES:<br />

Prior knowledge, skills, and attitudes acquired during the pre-clinical experience should<br />

include:<br />

Ability to perform patient-centered interviewing to determine the patients’ needs and<br />

communicate about diagnostic and therapeutic plans, transitions <strong>of</strong> care, and end-<strong>of</strong>-life care.<br />

Ability to identify community resources for care and strategies for coordination <strong>of</strong> care.<br />

Health Information Portability and Accountability Act (HIPAA) training to promote patient<br />

privacy.<br />

Required introductory courses in interviewing/physical examination with emphasis on doctorpatient<br />

communication and health care delivery.<br />

SPECIFIC LEARNING OBJECTIVES:<br />

A. KNOWLEDGE: Students should be able to define, describe and discuss:<br />

1. The role <strong>of</strong> consultants and their limits in the care <strong>of</strong> a patient. (MK, SBP)<br />

2. Key personnel and programs in and out <strong>of</strong> the hospital that may be able to contribute to the<br />

ongoing care <strong>of</strong> an individual patient for whom the student has responsibility (e.g. home health<br />

providers, social workers, case coordinators/managers, community health organizations, etc.).<br />

(MK, SBP)<br />

3. The role <strong>of</strong> the primary care physician in coordinating the comprehensive and longitudinal<br />

patient care plan, including communicating with the patient and family (directly, telephone, or<br />

email) and evaluating patient well-being through home health and other care providers. (MK,<br />

SBP)<br />

4. HIPAA guidelines to promote patient privacy. (MK, SBP)<br />

5. The role <strong>of</strong> the primary care physician in the coordination <strong>of</strong> care during key transitions<br />

(e.g. outpatient to inpatient, inpatient to skilled nursing facility, inpatient to hospice, etc.). (MK,<br />

SBP)<br />

6. The role <strong>of</strong> clinical nurse specialists, nurse practitioners, physicians assistants, and other<br />

allied health pr<strong>of</strong>essionals in co-managing patients in the outpatient and inpatient setting. (MK,<br />

SBP)<br />

7. The importance <strong>of</strong> reconciliation <strong>of</strong> medications at all transition points <strong>of</strong> patient care. (MK,<br />

SBP)<br />

8. The rationale for a standardized approach to all “hand <strong>of</strong>f” communications (MK, SBP)<br />

B. SKILLS: Students should be able to demonstrate specific skills, including:<br />

1. Discussing with the patient and their family ongoing health care needs; using appropriate<br />

language, avoiding jargon, and medical terminology. (PC, CS)<br />

215


2. Participating in requesting a consultation and identifying the specific question to be<br />

addressed. (PC, CS, .SBP)<br />

3. Participating in the discussion <strong>of</strong> the consultant’s recommendations. (PC, CS, SBP)<br />

4. Participating in developing a coordinated, ongoing care plan in the community. (PC, SBP)<br />

5. Obtaining a social history that identifies potential limitations in the home setting which may<br />

require an alteration in the medical care plan to protect the patient’s welfare. (PC, CS)<br />

6. Reconciling patient medications at key transition points in care. (PC, SBP)<br />

7. Conveying accurately vital patient information at all care “hand-<strong>of</strong>f” points (PC, CS, SBP)<br />

C. ATTITUDES AND PROFESSIONAL BEHAVIORS: Students should be able to:<br />

1. Demonstrate teamwork and respect toward all members <strong>of</strong> the health care team. (P, SBP)<br />

2. Demonstrate responsibility for patients’ overall welfare. (P)<br />

3. Participate, whenever possible, in coordination <strong>of</strong> care and in the provision <strong>of</strong> continuity. (P,<br />

SBP)<br />

D. REFERENCES:<br />

Goldman L, Lee, T, Rudd P. Ten commandments <strong>of</strong> effective consultation. Arch Intern Med.<br />

1983;143:1753-5.<br />

Stille CJ, Jerant A, Bell D, Meltzer D, Elmore JG. Coordinating care across diseases, settings,<br />

and clinicians: a key role for the generalist in practice. Ann Intern Med. 2005;142:700-708.<br />

Haggerty JL, Reid RJ, Freeman GK, Starfield BH, Adair CE, McKendry R. Continuity <strong>of</strong> care:<br />

a multidisciplinary review. BMJ. 2003;327:1219-21.<br />

Wenger NS, Young R. Quality indicators <strong>of</strong> continuity and coordination <strong>of</strong> care for vulnerable<br />

elder persons. Rand Corporation, 2004.<br />

www.rand.org/pubs/working_papers/2004/RAND_WR176.pdf<br />

Building a Case for Medication Reconciliation Institute for Safe Medication Practices<br />

www.ismp.org/Newsletters/acutecare/articles/20050421.asp<br />

Reconcile Medications at All Transition Points Institute for Healthcare Improvement<br />

www.ihi.org/IHI/Topics/PatientSafety/MedicationSystems/Changes/Reco<br />

ncile+Medications+at+All+Transition+Points.htm<br />

Healthcare Communications Toolkit to Improve Transitions <strong>of</strong> Care <strong>Department</strong> <strong>of</strong> Defense<br />

Patient Safety Program<br />

https://patientsafety.satx.disa.mil/ ContentStore/2005_12-<br />

8%20Hand<strong>of</strong>f%20Toolkit%20FINAL.htm<br />

216


GENERAL CLINICAL CORE COMPETENCIES<br />

#11 GERIATRIC CARE<br />

RATIONALE:<br />

Geriatric patients <strong>of</strong>ten have multiple, chronic illnesses which may present with atypical symptoms.<br />

Management strategies need to take into account the effects <strong>of</strong> aging on multiple organ systems and<br />

socioeconomic factors faced by our elderly society. As the number <strong>of</strong> geriatrics patients steadily<br />

rises, the internist will devote more time to the care <strong>of</strong> these patients.<br />

PREREQUISITES:<br />

Required courses in anatomy, physiology, pathophysiology, physical examination, and nutrition<br />

with attention to specific considerations in the elderly.<br />

Ability to perform a complete medical history and physical.<br />

Ability to communicate with patients <strong>of</strong> diverse backgrounds.<br />

SPECIFIC LEARNING OBJECTIVES:<br />

A. KNOWLEDGE: Students should be able to define, describe, and discuss:<br />

1. Functional implications <strong>of</strong> aging on each major organ system. (MK)<br />

2. Nutritional needs <strong>of</strong> the elderly and adaptations needed in the presence <strong>of</strong> chronic illness.<br />

(MK)<br />

3. Key illnesses in the elderly, focusing on their <strong>of</strong>ten atypical presentation, including:<br />

Cardiovascular and cerebrovascular disease. (MK)<br />

Diabetes. (MK)<br />

Urinary tract infection. (MK)<br />

Pneumonia. (MK)<br />

Substance abuse. (MK)<br />

Depression. (MK)<br />

Thyroid disease. (MK)<br />

Fluid and electrolyte disturbances. (MK)<br />

Arthritis. (MK)<br />

Constipation. (MK)<br />

Acute abdomen. (MK)<br />

Depression. (MK)<br />

4. The common “geriatric syndromes” (i.e. symptoms and conditions common in the elderly and<br />

<strong>of</strong>ten multifactorial in origin), including:<br />

Immobility. (MK)<br />

Falls/gait and balance problems. ( MK)<br />

Dizziness. (MK)<br />

Incontinence. (MK)<br />

Weight loss/failure to thrive/malnutrition. (MK)<br />

Sleep disturbance. (MK)<br />

Dementia/delirium. (MK)<br />

Osteoporosis. (MK)<br />

Hearing and visual impairment. (MK)<br />

Pressure ulcers. (MK)<br />

217


5. Basic treatment plans for illness in the elderly, with an awareness <strong>of</strong> the pharmacokinetic and<br />

pharmacodynamic changes seen as we age. (MK)<br />

6. Principles <strong>of</strong> screening in the elderly, including immunizations, cardiovascular risk, cancer,<br />

substance abuse, mental illness, osteoporosis, and functional assessment. (MK)<br />

7. Factors that contribute to polypharmacy in the elderly. (MK)<br />

8. Principles <strong>of</strong> Medicare (including who and what services are covered) and prescription drug<br />

coverage (who and what drugs are covered). (MK, SBP)<br />

B. SKILLS: Students should be able to demonstrate specific skills, including:<br />

1. Taking a complete and focused history from a geriatric patient with attention to current<br />

symptoms, chronic illnesses, and physical and mental functioning (PC, CS)<br />

2. Always obtaining historical information from collateral source, whenever possible. (PC, CS)<br />

3. Performing a physical examination and functional assessment on an elderly patient, adapting it<br />

to a patient's symptoms, chronic illness, and possible conditions <strong>of</strong> frailty, immobility, hearing<br />

loss, memory loss, and other impairments. (PC)<br />

4. Performing a mental status examination to evaluate confusion and/or memory loss in an<br />

elderly patient. (PC)<br />

5. Identifying patients at high risk for falling. (PC)<br />

6. Developing a diagnostic and management plan for patients with the with symptoms/conditions<br />

common in the geriatric population. (PC, MK)<br />

7. Communicating the diagnosis, treatment plan, and subsequent follow-up to the patient and<br />

their family. (PC, CS)<br />

8. Eliciting input and questions from the patient and their family about the diagnostic and<br />

management plan. (PC, CS)<br />

9. With guidance and direct supervision, participating in discussing basic issues regarding<br />

advance directives with patients and their families. (CS)<br />

10. With guidance and direct supervision participating in discussing basic end-<strong>of</strong>-life issues with<br />

patients and their families. (CS)<br />

11. Actively attempting to limit polypharmacy whenever possible. (PC)<br />

12. Participating in an interdisciplinary approach to management and rehabilitation <strong>of</strong> elderly<br />

patients. (PC, SBP)<br />

13. Determine when to obtain consultation from a geriatric specialist. (PC, SBP)<br />

14. Accessing and using appropriate information systems and resources to help delineate issues<br />

related to the common geriatric syndromes. (PC, PLI)<br />

15. Incorporating patient needs and preferences. (PC, P)<br />

C. ATTITUDES AND PROFESSIONAL BEHAVIORS: Students should be able to:<br />

1. Respect the increased risk for iatrogenic complications among elderly patients by always<br />

taking into account risks and monitoring closely for complications (P)<br />

2. Demonstrate respect to older patients, particularly those with disabilities, by making efforts to<br />

preserve their dignity and modesty. (P)<br />

3. Always treat cognitively impaired patients and patients at the end <strong>of</strong> their lives with utmost<br />

respect and dignity. (P)<br />

4. Demonstrate commitment to using risk-benefit, cost-benefit, and evidence-based<br />

considerations in the selection <strong>of</strong> diagnostic and therapeutic interventions for the common<br />

geriatric syndromes. (PLI, P)<br />

5. Recognize the importance <strong>of</strong> patient needs and preferences when selecting among diagnostic<br />

and therapeutic options for the common geriatric syndromes. (P)<br />

218


6. Demonstrate ongoing commitment to self-directed learning regarding care <strong>of</strong> the geriatric<br />

patient. (P, PLI)<br />

7. Appreciate the impact the common geriatric syndromes have on a patient’s quality <strong>of</strong> life, wellbeing,<br />

and the family. (P)<br />

8. Recognize the importance <strong>of</strong> and demonstrate a commitment to the utilization <strong>of</strong> other health<br />

care pr<strong>of</strong>essionals in the diagnosis and treatment <strong>of</strong> geriatric patients. (P, SBP)<br />

D. REFERENCES:<br />

The American Geriatrics Society<br />

www.americangeriatrics.org/<br />

Guidelines and Position Statements<br />

www.americangeriatrics.org/products/positionpapers/<br />

Portal <strong>of</strong> Geriatric Online Education (POGOe)<br />

In association with AAMC MedEdPORTAL<br />

www.pogoe.org<br />

Tinetti ME. Clinical practice. Preventing falls in elderly persons. N Engl J Med.<br />

2003;348(1):42-9.<br />

Fuller GF. Falls in the elderly. Am Fam Physician. 2000;61:2159-68, 2173-4<br />

Miller KE, Zylstra RG, Standridge JB. The geriatric patient: a systematic approach to<br />

maintaining health. Am Fam Physician. 2000;61:1089-104.<br />

Willlams CM. Using medications appropriately in older adults. Am Fam Physician.<br />

2002;66:1917-24.<br />

Huffman GB. Evaluating and treating unintentional weight loss in the elderly. Am Fam<br />

Physician. 2002;65:640-50.<br />

Sloane PD, Coeytaux RR, Beck RS, Dallara J. Dizziness: state <strong>of</strong> the science. Ann Intern<br />

Med. 2001;134:823-32.<br />

Cummings JL. Alzheimer's disease. N Engl J Med. 2004;351:56-67.<br />

Lembo A, Camilleri M. Chronic constipation. N Engl J Med. 2003;349:1360-8.<br />

Resnick NM, Dosa D. Geriatric medicine. In Kasper DL, Braunwald EB, Fauci AS, Hauser SL,<br />

Longo DL, Jameson JL eds. Harrison’s Principles <strong>of</strong> Internal <strong>Medicine</strong>. 16th ed. New York,<br />

NY: McGraw-Hill; 2005:43-53.<br />

219


#12 BASIC PROCEDURES<br />

GENERAL CLINICAL CORE COMPETENCIES<br />

RATIONALE:<br />

For many students, the internal medicine <strong>clerkship</strong> is where the basic procedural skills required in<br />

other <strong>clerkship</strong>s, subinternships, and residencies are learned.<br />

PREREQUISITES:<br />

Prior knowledge, skills, and attitudes acquired during the pre-<strong>clerkship</strong> experience should include:<br />

Pertinent anatomic considerations, including venous anatomy <strong>of</strong> the extremities (for<br />

venipuncture and IV placement), arterial anatomy <strong>of</strong> the wrist and groin (for blood gases),<br />

vaginal/vulvar anatomy (for urethral catheterization in women as well as pap smear) and<br />

prostate anatomy in men (for prostate exam), rectal anatomy (for digital rectal exam) and<br />

surface anatomy and electrical vector orientation <strong>of</strong> the heart (for EKG placement).<br />

The fundamental tenants <strong>of</strong> informed consent.<br />

Basic training in body substance isolation procedures and sterile technique.<br />

SPECIFIC LEARNING OBJECTIVES:<br />

A. KNOWLEDGE: Students should be able to define, describe, and discuss:<br />

1. Key indications, contraindications, risks to patients and health care providers, benefits, and<br />

techniques for each <strong>of</strong> the following basic procedures:<br />

Venipuncture. (MK)<br />

Blood culture. (MK)<br />

ABG. (MK)<br />

ECG. (MK)<br />

Chest radiography. (MK)<br />

Nasogastric tube placement. (MK)<br />

Urethral catheterization. (MK)<br />

Peripheral intravenous catheter insertion. (MK)<br />

Throat culture. (MK)<br />

PAP smear. (MK)<br />

Digital rectal examination. (MK)<br />

Urine dipstick. (MK)<br />

Stool occult blood testing. (MK)<br />

Subcutaneous injection. (MK)<br />

Intramuscular injection. (MK)<br />

Wound culture. (MK)<br />

Dressing change. (MK)<br />

PPD placement. (MK)<br />

2. Alternatives to a given procedure. (MK)<br />

3. The patient’s experience <strong>of</strong> the procedure. (MK)<br />

B. SKILLS: Students should be able to demonstrate specific skills, including:<br />

220


1. Obtaining informed consent, when necessary, for basic procedures, including the explanation<br />

<strong>of</strong> the purpose, possible complications, alternative approaches, and conditions necessary to<br />

make the procedure as comfortable, safe, and interpretable as possible. (PC, CS)<br />

2. Explaining what the patient’s experience is likely to be in understandable terms. (CS)<br />

3. Demonstrating step-by-step performance <strong>of</strong> basic procedures with technical pr<strong>of</strong>iciency. (PC)<br />

4. Demonstrating proper sterile technique and body substance isolation procedures. (PC)<br />

5. Appropriately documenting, when required, how the procedure was done, any complications,<br />

and results. (CS)<br />

C. ATTITUDES AND PROFESSIONAL BEHAVIORS: Students should be able to:<br />

1. Appreciate the fear and anxiety many patients have regarding even simple procedures. (P)<br />

2. Make efforts to maximize patient comfort during a procedure. (P)<br />

3. Appreciate the patient’s right to refuse procedures. (P)<br />

4. Regularly seek feedback regarding procedural skills and respond appropriately and<br />

productively. (P)<br />

D. REFERENCES:<br />

Guide to procedures. In Lin GA, Lin TL, Sakurai KA, De Fer TM, eds. The Washington Manual<br />

Internship Survival Guide. 2nd ed. Philadelphia, PA: Lippincott Williams and Wilkins; 2005:178-<br />

210.<br />

Chen H, Sonneday CJ, Lillemoe KD eds. Manual <strong>of</strong> Common Bedside Surgical Procedures.<br />

2nd ed. Philadelphia, PA: Lippincott Williams and Wilkins; 2000.<br />

Ferri FF. Procedures and interpretation <strong>of</strong> results. In Ferri FF, ed. Practical Guide to the Care<br />

<strong>of</strong> the Medical Patient. 6th ed. St. Louis, MO: Mosby; 2004:903-934.<br />

221


GENERAL CLINICAL CORE COMPETENCIES<br />

#13 NUTRITION<br />

RATIONALE:<br />

Despite the importance <strong>of</strong> nutritional factors in health and illness, physicians frequently have been<br />

criticized for giving these factors inadequate attention. Internists, by virtue <strong>of</strong> their dedication to<br />

providing comprehensive care to their patients, must assess nutritional factors on a routine basis.<br />

Medical students should be prepared to provide patients with basic advice regarding ways to optimize<br />

their nutritional status. Students also need to have at least a basic working knowledge <strong>of</strong> the<br />

principles <strong>of</strong> nutritional assessment and intervention.<br />

PREREQUISITES:<br />

Prior knowledge, skills, and attitudes acquired during the pre-<strong>clerkship</strong> experience should include:<br />

Ability to perform a complete medical history.<br />

Ability to communicate with patients <strong>of</strong> diverse backgrounds.<br />

Knowledge <strong>of</strong> body metabolism, the respective roles <strong>of</strong> dietary fats, carbohydrates, and<br />

protein, and the need for vitamins and minerals for maintenance <strong>of</strong> health.<br />

SPECIFIC LEARNING OBJECTIVES:<br />

A. KNOWLEDGE: Each student should be able to define, describe, and discuss:<br />

1. The relationship between diet and disease. (MK)<br />

2. Common medical problems that can cause nutritional deficiencies. (MK)<br />

3. Contributions <strong>of</strong> nutrition to medical problems such as obesity,<br />

hyperlipidemia, diabetes, and hypertension. (MK)<br />

4. How to perform a nutritional assessment and assist the patient in setting goals for dietary<br />

improvement. (MK)<br />

5. Daily caloric, fat, carbohydrate, protein, mineral, and vitamin requirements; adequacy <strong>of</strong> diets<br />

in providing such requirements; evidence <strong>of</strong> need for and potential risks <strong>of</strong> supplements (e.g.<br />

calcium, antioxidants). (MK)<br />

6. Common dietary supplements and their known adverse and beneficial effects on health. (MK)<br />

7. The consequences <strong>of</strong> poor nutrition on a critically ill patient, such as poor wound healing,<br />

increased risk <strong>of</strong> infection, and increased mortality. (MK)<br />

8. Nutritional needs <strong>of</strong> the elderly and adaptations needed in the presence <strong>of</strong> chronic illness.<br />

(MK)<br />

9. The indications for enteral and parenteral nutrition. (MK)<br />

B. SKILLS: Student should be able to demonstrate specific skills, including:<br />

1. Obtaining a nutritional history for all patients, with additional focus on those with chronic<br />

disease (obesity, hyperlipidemia, diabetes mellitus, hypertension, alcoholism, cancer, COPD,<br />

CHF, renal, and GI disease), giving attention to weight change, appetite, eating habits,<br />

digestive problems, dental problems, physical handicaps, psychiatric problems, socioeconomic<br />

factors, alcohol use, medications, and physical activity. (PC, CS)<br />

2. Identifying physical exam abnormalities that may suggest malnutrition, such as muscle<br />

wasting, decreased adipose stores, as well as stigmata <strong>of</strong> vitamin/mineral or protein-calorie<br />

222


malnutrition (e.g. alopecia, ecchymoses, angular chelosis, glossitis, peripheral neuropathy,<br />

edema, etc.). (PC)<br />

3. Calculating a patient’s body mass index (BMI) and measuring waist circumference. (PC)<br />

4. Ordering appropriate tests for evaluating a patient’s nutritional status, including albumin,<br />

prealbumin, serum chemistries and coagulation pr<strong>of</strong>ile(PC)<br />

5. Performing basic nutritional counseling with patients with obesity, diabetes mellitus,<br />

hyperlipidemia, hypertension, heart failure, and coronary artery disease. (PC, CS)<br />

6. Identifying barriers that prevent a patient from successfully adhering to a recommended diet.<br />

(PC, CS)<br />

7. Determining when to obtain consultation from a dietician. (PC, SBP)<br />

8. Incorporating patient needs and preferences. (PC, P)<br />

C. ATTITUDES AND PROFESSIONAL BEHAVIORS: Students should be able to:<br />

1. Demonstrate commitment to using risk-benefit, cost-benefit, and evidence-based<br />

considerations in the selection <strong>of</strong> diagnostic and therapeutic interventions for malnutrition. (PLI,<br />

P)<br />

2. Recognize the importance <strong>of</strong> patient preferences and cultural factors when selecting nutritional<br />

counseling. (PLI, P)<br />

3. Respond to patients who are non-adherent to recommendations for appropriate nutritional<br />

intake. (CS, P)<br />

4. Demonstrate ongoing commitment to self-directed learning regarding nutrition. (PLI, P)<br />

5. Appreciate the impact malnutrition has on a patient’s quality <strong>of</strong> life, wellbeing, ability to work,<br />

and the family. (P)<br />

6. Recognize the importance <strong>of</strong> involving other healthcare pr<strong>of</strong>essionals when appropriate. (P,<br />

SBP)<br />

D. REFERENCES:<br />

Division <strong>of</strong> Nutrition and Physical Activity National Center for Chronic Disease Prevention and<br />

Health Promotion<br />

Centers for Disease Control and Prevention<br />

U.S. <strong>Department</strong> <strong>of</strong> Health and Human Serviced<br />

www.cdc.gov/nccdphp/dnpa/<br />

Dietary Guidelines for Americans<br />

U.S. <strong>Department</strong> <strong>of</strong> Agriculture<br />

U.S. <strong>Department</strong> <strong>of</strong> Health and Human Services<br />

www.health.gov/dietaryguidelines/<br />

American Dietetic Association<br />

www.eatrigh.org<br />

Food and Nutrition Information Center<br />

U.S. <strong>Department</strong> <strong>of</strong> Agriculture<br />

www.nal.usda.gov/fnic/<br />

Dwyer J. Nutritional requirements and dietary assessment. In Kasper DL, Braunwald EB,<br />

Fauci AS, Hauser SL, Longo DL, Jameson JL eds. Harrison’s Principles <strong>of</strong> Internal <strong>Medicine</strong>.<br />

16th ed. New York, NY: McGraw-Hill; 2005:399-403.<br />

Halsted CH. Malnutrition and nutritional assessment. In Kasper DL, Braunwald EB, Fauci AS,<br />

Hauser SL, Longo DL, Jameson JL eds. Harrison’s Principles <strong>of</strong> Internal <strong>Medicine</strong>. 16th ed.<br />

New York, NY: McGraw-Hill; 2005:411-415.<br />

Howard L. Enteral and parenteral nutrition therapy. In Kasper DL, Braunwald EB, Fauci AS,<br />

223


Hauser SL, Longo DL, Jameson JL eds. Harrison’s Principles <strong>of</strong> Internal <strong>Medicine</strong>. 16th ed.<br />

New York, NY: McGraw-Hill; 2005:415-422.<br />

224


#14 COMMUNITY HEALTH CARE<br />

GENERAL CLINICAL CORE COMPETENCIES<br />

RATIONALE:<br />

The increasing number <strong>of</strong> physicians practicing under managed care and in community-oriented<br />

primary care practices necessitates expanding medical education to prepare graduates for<br />

population-based clinical practice. In a managed care setting, population-based clinical practice<br />

includes the health <strong>of</strong> an enrolled population. In a community-based setting, population-based clinical<br />

practice includes the health <strong>of</strong> a population in addition to the health <strong>of</strong> the individual patient through<br />

concern with resource allocation, epidemiology, and the care <strong>of</strong> patients whose needs are not<br />

currently met by the health care system.<br />

PREREQUISITES:<br />

Prior knowledge, skills, and attitudes acquired during the pre-<strong>clerkship</strong> experience should<br />

include:<br />

Required introductory coursework in health care delivery (with an emphasis on medical<br />

sociology and health care delivery to at risk populations).<br />

Required introductory course in clinical epidemiology and biostatistics.<br />

Required introductory coursework in population health (with an emphasis on differences<br />

between individuals and populations).<br />

SPECIFIC LEARNING OBJECTIVES:<br />

A. KNOWLEDGE: Students should be able to define, describe, and discuss:<br />

1. The concepts <strong>of</strong> rate, incidence, and prevalence to characterize the health <strong>of</strong> a population.<br />

(MK)<br />

2. How to gather health information about a population. (MK)<br />

3. How disease epidemiology in a community differs from that experienced in an <strong>of</strong>fice or hospital<br />

practice. (MK)<br />

4. How health care financing and health care delivery systems affect individual physicians,<br />

patients, and communities. (MK, SBP)<br />

5. How community and individual responses to health problems may be affected by both<br />

individual and community socio-cultural characteristics. (MK)<br />

6. Local government, social service, or community organizations that provide links between the<br />

underserved members <strong>of</strong> the community and the medical care systems. (MK, SBP)<br />

7. barriers faced by his or her patients in the community setting. (MK)<br />

B. SKILLS: Students should be able to demonstrate specific skills, including:<br />

1. Defining and describing a population, its demography, culture, socioeconomic makeup, and<br />

health status. (PC)<br />

2. Identifying the unique characteristics <strong>of</strong> a population that affect the health <strong>of</strong> the population and<br />

individuals within that population. (PC)<br />

3. Considering how the socio-cultural characteristics <strong>of</strong> a particular community may affect that<br />

population’s attitudes toward health care. (PC)<br />

4. Using, in daily patient care, an understanding <strong>of</strong> the community and sociocultural context that<br />

may affect an individual patient’s health care decisions and health-related behaviors. (PC)<br />

225


5. Identifying patients whose illnesses may put the community at risk. (PC, MK)<br />

6. Incorporating a population-based perspective in analyzing clinical problems (PC)<br />

7. Reading critically clinical studies and applying findings to health care decisions involving real<br />

patients and populations <strong>of</strong> patients. (PC, MK, PLI)<br />

8. Incorporating principles <strong>of</strong> disease prevention and behavioral change appropriate for specific<br />

populations <strong>of</strong> patients within a community. (PC, MK)<br />

9. Attempting to develop solutions for barriers to health care delivery (e.g. sociocultural, financial,<br />

and system-based) that affect individual patients. (PC, SBP)<br />

10. Functioning effectively as a member <strong>of</strong> a health care team. (PC, P, SBP)<br />

11. Using, when appropriate, local government, social service, and community organizations to<br />

improve the health <strong>of</strong> individuals and populations. (PC, SBP)<br />

12. Accessing and utilizing appropriate information systems and resources to help delineate issues<br />

related to population health. (PC, PLI)<br />

C. ATTITUDES AND PROFESSIONAL BEHAVIORS: Students should be able to:<br />

1. Demonstrate respect for cultural and socioeconomic diversity. (P)<br />

2. Show willingness to accept at least partial responsibility for the health <strong>of</strong> populations. (P)<br />

3. Respond non-judgmentally to an individual whose socio-cultural and community-based<br />

background result in seemingly counterproductive heath care decisions and health-related<br />

behaviors. (P)<br />

4. Value the unique contributions <strong>of</strong> all members <strong>of</strong> the health care team. (P)<br />

5. Demonstrate ongoing commitment to self-directed learning regarding population/community<br />

health issues. (PLI, P)<br />

D. REFERENCES:<br />

Contemporary Issues in Medical Education: Quality <strong>of</strong> Care<br />

Medical Informatics and Population Health, June, 1998<br />

American Association <strong>of</strong> Medical Colleges<br />

www.aamc.org/meded/msop/msop2.pdf<br />

Population Health Forum<br />

<strong>University</strong> <strong>of</strong> Washington<br />

School <strong>of</strong> Public Health and Community <strong>Medicine</strong><br />

depts.washington.edu/eqhlth/index.htm<br />

Behavioral Risk Factor Surveillance System Division <strong>of</strong> Adult and Community Health National<br />

Center for Chronic Disease Prevention and Health Promotion Centers for Disease Control and<br />

Prevention<br />

U.S. <strong>Department</strong> <strong>of</strong> Health and Human Services<br />

www.cdc.gov/brfss/index.htm<br />

Healthy People 2010 National Center for Health Statistics<br />

Centers for Disease Control and Prevention<br />

U.S. <strong>Department</strong> <strong>of</strong> Health and Human Services<br />

www.cdc.gov/nchs/hphome.htm<br />

Rhyne RL, Bogue R, Kukulka G, Fulmer H, eds. Community-Oriented Primary Care: Health<br />

Care for the 21st Century. Washington, DC: American Public Health Association; 1998.<br />

Strelnick AH. Community-oriented primary care: the state <strong>of</strong> an art. Arch Fam Med.<br />

1999;8:550-2.<br />

Fox DM. The relevance <strong>of</strong> population health to academic medicine. Acad Med. 2001;76:6-7.<br />

Kindig D, Stoddart G. What is population health?. Am J Public Health. 2003;93:380-3.<br />

226


GENERAL CLINICAL CORE COMPETENCIES<br />

#15 CONTINUOUS IMPROVEMENT IN SYSTEMS OF MEDICAL PRACTICE<br />

RATIONALE:<br />

In the past clinical education had emphasized the role <strong>of</strong> the physician as an individual decision<br />

maker. Problems with cost and quality <strong>of</strong> care had usually been attributed to errors in individual<br />

decision making. In recent years, it has become clear that the individual does not function in isolation<br />

but within the context <strong>of</strong> a health care system and a health care team whose structure ranges from<br />

simple to complex. The way the system functions is critical to achieving high quality patient care,<br />

ensuring patient safety, reducing sources <strong>of</strong> errors in medicine, and promoting an environment that<br />

respects disclosure without blame. Furthermore, we have begun to focus on the patient as the center<br />

<strong>of</strong> the health care delivery system and to assess quality from the perspectives <strong>of</strong> the patient and the<br />

physician. With the patient as the center <strong>of</strong> the health care delivery system, the physician becomes a<br />

collaborative partner with other health pr<strong>of</strong>essionals who share a common goal <strong>of</strong> providing safe,<br />

accessible, high quality, evidence-based care.<br />

PREREQUISITES:<br />

Prior knowledge, skills and attitudes acquired during the pre-<strong>clerkship</strong> experience should include:<br />

Required introductory course in clinical epidemiology and biostatistics.<br />

Required introductory course in health care delivery.<br />

Required introductory course in bioethics and pr<strong>of</strong>essionalism.<br />

SPECIFIC LEARNING OBJECTIVES:<br />

A. KNOWLEDGE: Students should be able to define, describe, and discuss:<br />

1. The concept <strong>of</strong> systems-based practice. (MK, SBP)<br />

2. How patient care is affected by other pr<strong>of</strong>essionals, organizations, and society. (MK, SBP)<br />

3. The principles <strong>of</strong> clinical quality improvement, including the notion <strong>of</strong> variation in practice as a<br />

quality issue and the concept <strong>of</strong> medical care as a process which can be studied and<br />

improved. (MK, SBP)<br />

4. The analysis and improvement <strong>of</strong> systems to address common quality problems (e.g.,<br />

treatment delays, medication errors, failure to use evidence-based diagnostics/treatments,<br />

failure to provide preventive care, etc.). (MK, SBP)<br />

5. Principles <strong>of</strong> medical record organization in both inpatient and ambulatory settings. (MK, SBP)<br />

6. The importance <strong>of</strong> complete medical documentation in the context <strong>of</strong> measuring quality <strong>of</strong><br />

care, avoiding redundancy, preventing medical errors, and improving patient safety. (MK, SBP)<br />

7. The need for a multidimensional approach to the assessment <strong>of</strong> quality, including the patient’s<br />

perspective <strong>of</strong> quality. (MK, SBP)<br />

8. The relationship <strong>of</strong> quality and cost in health care from the standpoint <strong>of</strong> the individual, health<br />

care systems, and society. (MK, SBP)<br />

9. Major health care safety concerns (e.g., medication errors, wrong-site procedures, patient<br />

misidentification, miscommunication among health care givers, nosocomial infections, falls,<br />

use <strong>of</strong> restraints, etc.). (MK, SBP)<br />

10. Potential benefits and pitfalls <strong>of</strong> critical pathways/practice guidelines intended to improve the<br />

quality <strong>of</strong> care. (MK, SBP)<br />

11. Basic organizational structures and financing streams <strong>of</strong> the U.S. health care system. (MK,<br />

SBP)<br />

227


12. The fundamentals <strong>of</strong> the various type <strong>of</strong> health insurance (e.g., fee-for-service, preferred<br />

provider organization, health maintenance organization, point-<strong>of</strong>service). (MK, SBP)<br />

13. The fundamentals <strong>of</strong> Medicare and Medicaid. (MK, SBP)<br />

B. SKILLS: Students should be able to demonstrate specific skills, including:<br />

1. Using hospital-based support systems to assist in making clinical decisions (e.g., antibiotic<br />

control program, critical pathways/practice guidelines, etc.). (PC, PLI, SBP)<br />

2. Recognizing system flaws in the delivery <strong>of</strong> care (e.g., inability to arrange a post-discharge<br />

appointment within a needed time frame, delays in obtaining test results, inaccessibility <strong>of</strong><br />

medical records, etc.). (SBP)<br />

3. Using patient education materials to facilitate patients’ participation in their own care. (CS,<br />

SBP)<br />

4. Using the medical records system efficiently to produce medical notes that communicate<br />

information clearly. (PC, CS, SBP)<br />

5. Maintaining accurate documentation <strong>of</strong> preventive health measures. (PC, CS, SBP)<br />

6. Working collaboratively with other health pr<strong>of</strong>essionals in the delivery <strong>of</strong> quality care. (PC, P,<br />

SBP)<br />

7. Assessing the patients’ needs from the standpoint <strong>of</strong> the individual, family, and community.<br />

(PC, SBP)<br />

8. Identifying resource available to patients within the health care system. (PC, SBP)<br />

9. Reporting patient safety concerns and medical errors to the appropriate individuals. (CS, SBP)<br />

10. Using resources, appropriate information systems, and the tenants <strong>of</strong> evidence-based<br />

medicine to assess systems-based practice issues. (PLI, SBP)<br />

C. ATTITUDES AND PROFESSIONAL BEHAVIORS: Students should be able to:<br />

1. Recognize the importance <strong>of</strong> systems, particularly inter-pr<strong>of</strong>essional collaboration, in delivering<br />

high quality patient care. (P, SBP)<br />

2. Strive to improve the timeliness diagnostic and therapeutic decision making in order to improve<br />

quality <strong>of</strong> care, increase patient satisfaction, and reduce health care costs. (PLI, P, SBP)<br />

3. View the patient as the center <strong>of</strong> the health care delivery system. (P, SBP)<br />

4. Advocate for patients in the health care system. (P, SBP)<br />

5. Appreciate that medical error prevention and patient safety are the responsibility <strong>of</strong> all health<br />

care providers and systems and accept the appropriate degree <strong>of</strong> responsibility at the medical<br />

student level. (P, SBP)<br />

6. Appreciate the importance teamwork in delivering high quality care. (P, SBP)<br />

7. Respect other health care pr<strong>of</strong>essionals as colleagues on a patient-centered health delivery<br />

team and as mutual contributors to high quality patient care. (P, SBP)<br />

D. REFERENCES:<br />

GENERAL:<br />

Agency for Healthcare Research and Quality<br />

U.S. <strong>Department</strong> <strong>of</strong> Health and Human Services<br />

ahrq.gov<br />

QUALITY OF CARE:<br />

Institute for Healthcare Improvement<br />

228


www.ihi.org/ihi<br />

Crossing the Quality Chasm: A New Health System for the 21st Century<br />

Committee on Quality Health Care in America<br />

Institute <strong>of</strong> <strong>Medicine</strong><br />

National Academies Press, 2001<br />

National Committee for Quality Assurance<br />

www.ncqa.org<br />

National Guideline Clearing House<br />

Agency for Healthcare Research and Quality<br />

U.S. <strong>Department</strong> <strong>of</strong> Health and Human Services<br />

www.guideline.gov<br />

MEDIAL ERRORS AND PATIENT SAFETY:<br />

To Err Is Human: Building a Safer Health System<br />

Institute <strong>of</strong> <strong>Medicine</strong><br />

www.iom.edu/?id=4117&redirect=0<br />

Patient Safety Network<br />

Agency for Healthcare Research and Quality<br />

U.S. <strong>Department</strong> <strong>of</strong> Health and Human Services<br />

psnet.ahrq.gov<br />

National Patient Safety Foundation<br />

www.npsf.org<br />

Facts About Patient Safety<br />

Joint Commission on Accreditation <strong>of</strong> Healthcare Organizations<br />

www.jcaho.org/accredited+organizations/patient+safety/facts+about+pati<br />

ent+safety.htm<br />

HEALTH INSURANCE AND FINANCE:<br />

Understanding Managed Care<br />

Institute for Health Care Studies<br />

Michigan State <strong>University</strong><br />

www.ihcs.msu.edu/modules/UMC2003/UMC2003.pdf<br />

The Official U.S. Government Site for People with Medicare<br />

U.S. <strong>Department</strong> <strong>of</strong> Health and Human Services<br />

www.medicare.gov<br />

Checkup on Health Insurance Choices<br />

Agency for Healthcare Research and Quality<br />

www.ahrq.gov/consumer/insuranc.htm<br />

229


#16 OCCUPATIONAL HEALTH CARE<br />

GENERAL CLINICAL CORE COMPETENCIES<br />

RATIONALE:<br />

Despite increasing recognition <strong>of</strong> the health hazards found in living and working environments,<br />

physicians have traditionally received little formal training in the assessment and management <strong>of</strong><br />

occupational and environmental health problems.<br />

PREREQUISITES:<br />

Prior knowledge, skills, and attitudes acquired during the pre-<strong>clerkship</strong> experience should<br />

include:<br />

Required introductory course work in clinical epidemiology and biostatistics.<br />

Required introductory course work in the fundamental principles <strong>of</strong> public health.<br />

Ability to perform a complete medical history and physical exam.<br />

Ability to communicate with patients <strong>of</strong> diverse backgrounds.<br />

SPECIFIC LEARNING OBJECTIVES:<br />

A. KNOWLEDGE: Each student should be able to define, describe, and discuss :<br />

1. Common environmental diseases that are likely to be encountered by an internist and the<br />

principal etiologic agents associated with them. (MK)<br />

2. Pathogenesis <strong>of</strong> specific occupational diseases and the types <strong>of</strong> risks that may be encountered<br />

in the home or at the work site:<br />

Musculoskeletal/ergonomic or “repetitive stress” disorders (e.g. low back pain, carpal<br />

tunnel syndrome, etc.). (MK)<br />

Work related lung disorders (e.g. occupational asthma, particulate inhalation, etc.). (MK)<br />

Noise related hearing loss. (MK)<br />

Skin disorders (e.g. latex allergy and other forms <strong>of</strong> occupational dermatitis). (MK)<br />

Infectious disease exposure (e.g. hepatitis, HIV, TB, etc.). (MK)<br />

Psychological/stress related disorders (MK)<br />

3. Information sources for determining the risk <strong>of</strong> specific environmental and occupational health<br />

hazards. (MK)<br />

4. Purpose <strong>of</strong> Occupational Safety and Health Act (OSHA) regulations and the function <strong>of</strong> the<br />

National Institute for Occupational Safety and Health. (NIOSH). (MK, SBP)<br />

B. SKILLS: Students should be able to demonstrate specific skills, including:<br />

1. Obtaining an appropriate occupational history on all patients and identifying those patients<br />

whose health may have been adversely affected by their living conditions or work environment.<br />

(PC, CS)<br />

2. Considering the possibility that the patient’s illness may be related to their home or work<br />

environment. (PC)<br />

3. Providing patients with sound advice on the prevention <strong>of</strong> occupational and environmentalrelated<br />

diseases. (PC, CS)<br />

4. Accurately diagnosing and developing a cost-effective basic management plan for common<br />

occupational health problems (e.g. carpal tunnel syndrome, asthma, asbestosis). (PC, MK,<br />

SBP)<br />

230


5. Determining when to obtain consultation from an environmental and occupational medicine<br />

specialist. (PC, SBP)<br />

6. Accessing and utilizing appropriate information systems and resources to help delineate issues<br />

related to occupational health problems. (PC, PLI)<br />

C. ATTITUDES AND PROFESSIONAL BEHAVIORS: Students should be able to:<br />

1. Demonstrate an understanding that physicians have a duty and pr<strong>of</strong>essional responsibility to<br />

follow-up on conditions that are suspected <strong>of</strong> causing occupational or environmental-related<br />

illnesses. (P, SBP)<br />

2. Demonstrate commitment to using risk-benefit, cost-benefit, and evidence-based<br />

considerations in the selection <strong>of</strong> diagnostic and therapeutic interventions for occupational<br />

health problems. (PLI, P)<br />

3. Recognize the importance <strong>of</strong> patient needs and preferences when selecting among diagnostic<br />

and therapeutic options for occupational health problems. (P)<br />

4. Demonstrate ongoing commitment to self-directed learning regarding occupational health<br />

problems. (PLI, P)<br />

5. Appreciate the impact occupational health problems have on a patient’s quality <strong>of</strong> life, wellbeing,<br />

ability to work, and the family. (P)<br />

6. Recognize the importance <strong>of</strong> and demonstrate a commitment to the utilization <strong>of</strong> other health<br />

care pr<strong>of</strong>essionals in the diagnosis and treatment <strong>of</strong> occupational health problems. (P, SBP)<br />

D. REFERENCES:<br />

Occupational Safety and Health Administration<br />

U.S. <strong>Department</strong> <strong>of</strong> Labor<br />

www.osha.gov<br />

National Institute for Occupational Safety and Health<br />

Centers for Disease Control and Prevention<br />

U.S. <strong>Department</strong> <strong>of</strong> Health and Human Services<br />

www.cdc.gov/niosh/homepage.html<br />

231


#17 ADVANCED PROCEDURES<br />

GENERAL CLINICAL CORE COMPETENCIES<br />

RATIONALE:<br />

A number <strong>of</strong> advanced procedures may be performed by general internists, and occasionally thirdyear<br />

medical students under their supervision. In either case, knowledge <strong>of</strong> the key indications,<br />

contraindications, risks, and benefits <strong>of</strong> these procedures is essential for high quality patient care.<br />

Physicians, regardless <strong>of</strong> specialty, must be able to explain to their patients, in understandable terms,<br />

what will be experienced during a procedure.<br />

PREREQUISITES:<br />

Prior knowledge, skills, and attitudes acquired during the pre-<strong>clerkship</strong> experience should include:<br />

Pertinent anatomic considerations, including vascular anatomy <strong>of</strong> the extremities, wrist/hand,<br />

neck, subclavian area and groin.<br />

Pertinent anatomic landmarks important for the safe performance <strong>of</strong> thoracentesis,<br />

paracentesis, lumbar puncture, and arthrocentesis.<br />

Required introductory course in interviewing and physical examination.<br />

The fundamental tenants <strong>of</strong> informed consent.<br />

Basic training in body substance isolation procedures and sterile technique.<br />

SPECIFIC LEARNING OBJECTIVES:<br />

A. KNOWLEDGE: Students should be able to define, describe, and discuss:<br />

1. Key indications, contraindications, risks, benefits, techniques <strong>of</strong> each <strong>of</strong> the following advanced<br />

procedures:<br />

Arthrocentesis. (MK)<br />

o Elbow (olecranon bursa). (MK)<br />

o Wrist. (MK)<br />

o Knee. (MK)<br />

o Ankle. (MK)<br />

Central venous catheterization. (MK)<br />

o Internal jugular vein. (MK)<br />

o Subclavian vein. (MK)<br />

o Femoral vein. (MK)<br />

Arterial line placement. (MK)<br />

o Radial artery. (MK)<br />

o Femoral artery. (MK)<br />

Lumbar puncture. (MK)<br />

Thoracentesis. (MK)<br />

Paracentesis. (MK)<br />

2. Potential alternatives to the listed procedures. (MK)<br />

3. The patient’s probable experience during these procedures. (MK)<br />

4. Indications for and efficacy <strong>of</strong> intra-articular corticosteroid injections. (MK)<br />

B. SKILLS: Each student should be able to demonstrate specific skills, including:<br />

232


1. Participating in obtaining informed consent for advanced procedures, including the explanation<br />

<strong>of</strong> the purpose, possible complications, alternative approaches, and conditions necessary to<br />

make the procedure as comfortable, safe, and interpretable as possible. (PC, CS)<br />

2. Explaining the patient’s probable experience during the procedure in understandable terms.<br />

(PC, CS)<br />

3. Helping to position the patient and make them as comfortable as possible during the<br />

procedure. (PC)<br />

4. Assisting (under supervision, when appropriate) in the performance <strong>of</strong> the procedure. (PC)<br />

5. Demonstrating proper sterile technique and body substance isolation procedures. (PC)<br />

6. Appropriately documenting, when required, how the procedure was done as well as any<br />

complications and results. (CS)<br />

7. Ordering and interpreting appropriate diagnostic tests on fluids removed from the patient (e.g.<br />

synovial fluid, cerebrospinal fluid, pleural fluid, and ascitic fluid). (PC, MK)<br />

C. ATTITUDES AND PROFESSIONAL BEHAVIOR: Students should be able to:<br />

1. Demonstrate commitment to using risk-benefit, cost-benefit, and evidence-based<br />

considerations in the selection <strong>of</strong> procedures to be performed. (PLI, P)<br />

2. Appreciate the fear and anxiety many patients have regarding these procedures. (P)<br />

3. Make efforts to maximize patient comfort during a procedure. (P)<br />

4. Appreciate the patient’s right to refuse procedures. (P)<br />

5. Seek feedback regularly regarding procedural skills and respond appropriately and<br />

productively. (P)<br />

D. REFERENCES:<br />

Guide to procedures. In Lin GA, Lin TL, Sakurai KA, De Fer TM, eds. The Washington Manual<br />

Internship Survival Guide. 2nd ed. Philadelphia, PA: Lippincott Williams and Wilkins; 2005:178-<br />

210.<br />

Chen H, Sonneday CJ, Lillemoe KD eds. Manual <strong>of</strong> Common Bedside Surgical Procedures.<br />

2nd ed. Philadelphia, PA: Lippincott Williams and Wilkins; 2000.<br />

Ferri FF. Procedures and interpretation <strong>of</strong> results. In Ferri FF, ed. Practical Guide to the Care<br />

<strong>of</strong> the Medical Patient. 6th ed. St. Louis, MO: Mosby; 2004:903-934.<br />

233


LIST OF ERROR-PRONE ABBREVIATIONS, SYMBOLS, AND DOSE<br />

DESIGNATIONS<br />

234


November 27, 2003 Volume 8 Issue 24<br />

ISMP List <strong>of</strong> Error-Prone Abbreviations, Symbols, and<br />

Dose Designations<br />

It’s been over 2 years since we published a list <strong>of</strong> abbreviations, symbols, the organization’s list, we’ve highlighted these items with a<br />

double and dose designations that have contributed to medication errors. Now, asterisk (**). Also, effective April 1, 2004, each<br />

organization must include with the 2004 JCAHO National Patient Safety Goals calling for organiza-at least three additional items on<br />

their list. However, we hope that you tional compliance with a list <strong>of</strong> prohibited “dangerous” abbreviations, will consider others<br />

beyond the minimum JCAHO requirement. Selections acronyms and symbols, we thought an updated list would be useful. can be<br />

made from the attached list. These items should be considered Since JCAHO has specified that certain abbreviations must appear on<br />

for handwritten, preprinted, and electronic forms <strong>of</strong> communication.<br />

Abbreviations Intended Meaning Misinterpretation Correction<br />

µg Microgram Mistaken as “mg” Use “mcg”<br />

AD, AS, AU<br />

Right ear, left ear, each Mistaken as OD, OS, OU (right eye, left eye, each Use “right ear,” “left ear,” or<br />

ear<br />

eye)<br />

“each ear”<br />

OD, OS, OU<br />

Right eye, left eye, each Mistaken as AD, AS, AU (right ear, left ear, each Use “right eye,” “left eye,” or<br />

eye<br />

ear)<br />

“each eye”<br />

BT Bedtime Mistaken as “BID” (twice daily) Use “bedtime”<br />

cc Cubic centimeters Mistaken as “u” (units) Use “mL”<br />

D/C Discharge or<br />

discontinue<br />

Premature discontinuation <strong>of</strong> medications if D/C<br />

(intended to mean “discharge”) has been<br />

misinterpreted as “discontinued” when followed by<br />

a list <strong>of</strong> discharge medications<br />

235<br />

Use “discharge” and<br />

“discontinue”<br />

IJ Injection Mistaken as “IV” or “intrajugular” Use “injection”<br />

IN Intranasal Mistaken as “IM” or “IV” Use “intranasal” or “NAS”<br />

HS hs Half-strength At Mistaken as bedtime Mistaken as half-strength Use “half-strength” or “bedtime”<br />

bedtime, hours <strong>of</strong> sleep<br />

IU** International unit Mistaken as IV (intravenous) or 10 (ten) Use “units”<br />

o.d. or OD Once daily Mistaken as “right eye” (OD-oculus dexter), leading Use “daily”<br />

to oral liquid medications administered in the eye<br />

OJ Orange juice Mistaken as OD or OS (right or left eye); drugs Use "orange juice"<br />

meant to be diluted in orange juice may be given in<br />

the eye<br />

Per os By mouth, orally<br />

The “os” can be mistaken as “left eye” (OS-oculus Use “PO,” “by mouth,” or<br />

sinister)<br />

“orally”<br />

q.d. or QD** Every day Mistaken as q.i.d., especially if the period after the Use “daily”<br />

“q” or the tail <strong>of</strong> the “q” is misunderstood as an “i”<br />

qhs At bedtime Mistaken as “qhr” or every hour Use “at bedtime”<br />

qn Nightly Mistaken as “qh” (every hour) Use “nightly”<br />

q.o.d. or QOD** Every other day Mistaken as “q.d.” (daily) or “q.i.d. (four times Use “every other day”<br />

daily) if the “o” is poorly written<br />

q1d Daily Mistaken as q.i.d. (four times daily) Use “daily”<br />

q6PM, etc. Every evening at 6 PM Mistaken as every 6 hours<br />

Use “6 PM nightly” or “6 PM<br />

daily”<br />

SC, SQ, sub q Subcutaneous SC mistaken as SL (sublingual); SQ mistaken as “5<br />

every;” the “q” in “sub q” has been mistaken as<br />

Use “subcut” or<br />

“subcutaneously”<br />

“every” (e.g., a heparin dose ordered “sub q 2 hours<br />

before surgery” misunderstood as every 2 hours<br />

before surgery)<br />

ss Sliding scale (insulin) or<br />

½ (apothecary)<br />

Mistaken as “55”<br />

Spell out “sliding scale;” use<br />

“one-half” or “½”<br />

SSRI SSI<br />

Sliding scale regular<br />

insulin Sliding scale<br />

insulin<br />

Mistaken as selective-serotonin reuptake inhibitor<br />

Mistaken as Strong Solution <strong>of</strong> Iodine (Lugol's)<br />

Spell out “sliding scale (insulin)”<br />

i/d One daily Mistaken as “tid” Use “1 daily”<br />

TIW or tiw 3 times a week Mistaken as “3 times a day” or “twice in a week” Use “3 times weekly”


U or u** Unit Mistaken as the number 0 or 4, causing a 10-fold Use “unit”<br />

overdose or greater (e.g., 4U seen as “40” or 4u seen<br />

as “44”); mistaken as “cc” so dose given in volume<br />

instead <strong>of</strong> units (e.g., 4u seen as 4cc)<br />

Dose Designations<br />

and Other<br />

Information<br />

Intended Meaning Misinterpretation Correction<br />

Trailing zero<br />

after decimal<br />

point (e.g., 1.0<br />

mg)**<br />

No leading zero<br />

before a decimal<br />

dose (e.g., .5<br />

mg)**<br />

1 mg Mistaken as 10 mg if the decimal point is not seen<br />

0.5 mg Mistaken as 5 mg if the decimal point is not seen<br />

Do not use trailing zeros for<br />

doses expressed in whole<br />

numbers<br />

Use zero before a decimal point<br />

when the dose is less than a<br />

whole unit<br />

_ ISMP 2003<br />

ISMP MedicationSafetyAlert!<br />

November 27, 2003 Volume 8 Issue 24<br />

Dose Designations Intended Meaning Misinterpretation Correction<br />

and Other Information<br />

Drug name and dose<br />

run together<br />

(especially<br />

Inderal 40 mg Tegretol<br />

300 mg<br />

Mistaken as Inderal 140 mg Mistaken as<br />

Tegretol 1300 mg<br />

Place adequate space between the<br />

drug name, dose, and unit <strong>of</strong><br />

measure<br />

problematic for drug<br />

names that end in<br />

“L” such as<br />

Inderal40 mg;<br />

Tegretol300 mg)<br />

Numerical dose and<br />

The “m” is sometimes mistaken as a zero or Place adequate space between the<br />

unit <strong>of</strong> measure run<br />

two zeros, risking a 10- to 100-fold overdose dose and unit <strong>of</strong> measure<br />

10 mg 100 mL<br />

together (e.g., 10mg,<br />

100mL)<br />

Abbreviations such<br />

The period is unnecessary and could be Use mg, mL, etc. without a<br />

as mg. or mL. with a<br />

mistaken as the number 1 if written poorly terminal period<br />

mg mL<br />

period following the<br />

abbreviation<br />

Large doses without<br />

properly placed<br />

commas (e.g., 100000<br />

units; 1000000 units)<br />

100,000 units 1,000,000<br />

units<br />

100000 has been mistaken as 10,000 or<br />

1,000,000; 1000000 has been mistaken as<br />

100,000<br />

Use commas for dosing units at<br />

or above 1,000, or use words<br />

such as 100 "thousand" or 1<br />

"million" to improve readability<br />

Drug Name<br />

Abbreviations<br />

Intended Meaning Misinterpretation Correction<br />

ARA A vidarabine Mistaken as cytarabine (ARA C) Use complete drug name<br />

AZT zidovudine (Retrovir) Mistaken as azathioprine or aztreonam Use complete drug name<br />

CPZ<br />

Compazine<br />

(prochlorperazine)<br />

Mistaken as chlorpromazine<br />

Use complete drug name<br />

DPT<br />

Demerol-Phenergan- Mistaken as diphtheria-pertussis-tetanus<br />

Thorazine<br />

(vaccine)<br />

Use complete drug name<br />

DTO Diluted tincture <strong>of</strong> opium, Mistaken as tincture <strong>of</strong> opium<br />

Use complete drug name<br />

or deodorized tincture <strong>of</strong><br />

opium (Paregoric)<br />

HCl hydrochloric acid or<br />

hydrochloride<br />

Mistaken as potassium chloride (The “H” is<br />

misinterpreted as “K”)<br />

Use complete drug name unless<br />

expressed as a salt <strong>of</strong> a drug<br />

HCT hydrocortisone Mistaken as hydrochlorothiazide Use complete drug name<br />

HCTZ hydrochlorothiazide<br />

Mistaken as hydrocortisone (seen as HCT250<br />

mg)<br />

Use complete drug name<br />

MgSO4** magnesium sulfate Mistaken as morphine sulfate Use complete drug name<br />

MS, MSO4** morphine sulfate Mistaken as magnesium sulfate Use complete drug name<br />

MTX methotrexate Mistaken as mitoxantrone Use complete drug name<br />

PCA procainamide Mistaken as Patient Controlled Analgesia Use complete drug name<br />

236


PTU propylthiouracil Mistaken as mercaptopurine Use complete drug name<br />

T3<br />

Tylenol with codeine No.<br />

3<br />

Mistaken as liothyronine<br />

Use complete drug name<br />

TAC triamcinolone Mistaken as tetracaine, Adrenalin, cocaine Use complete drug name<br />

TNK TNKase Mistaken as “TPA” Use complete drug name<br />

ZnSO4 zinc sulfate Mistaken as morphine sulfate Use complete drug name<br />

Stemmed Drug Names Intended Meaning Misinterpretation Correction<br />

“Nitro” drip nitroglycerin infusion Mistaken as sodium nitroprusside infusion Use complete drug name<br />

“Norflox” norfloxacin Mistaken as Norflex Use complete drug name<br />

“IV Vanc” intravenous vancomycin Mistaken as Invanz Use complete drug name<br />

Symbols Intended Meaning Misinterpretation Correction<br />

Dram Minim<br />

Symbol for dram mistaken as “3” Symbol for Use the metric system<br />

minim mistaken as “mL”<br />

x3d For three days Mistaken as “3 doses” Use “for three days”<br />

> and < Greater than and less than Mistaken as opposite <strong>of</strong> intended; mistakenly Use “greater than” or “less than”<br />

use incorrect symbol; “< 10” mistaken as “40”<br />

/ (slash mark) Separates two doses or<br />

indicates “per”<br />

Mistaken as the number 1 (e.g., “25 units/10<br />

units” misread as “25 units and 110” units)<br />

Use “per” rather than a slash<br />

mark to separate doses<br />

@ At Mistaken as “2” Use “at”<br />

& And Mistaken as “2” Use “and”<br />

+ Plus or and Mistaken as “4” Use “and”<br />

° Hour Mistaken as a zero (e.g., q2° seen as q 20) Use “hr,” “h,” or “hour”<br />

_ ISMP 2003<br />

** Identified abbreviations above are also included on the JCAHO's "minimum list" <strong>of</strong> dangerous abbreviations, acronyms and symbols<br />

that must be included on an organization's "Do NotUse" list, effective January 1, 2004. An updated list <strong>of</strong> frequently asked questions<br />

about this JCAHO requirement can be found on their website at www.jcaho.org.<br />

ISMP MedicationSafetyAlert!<br />

237


COMPREHENSIVE WRITE-UPS<br />

Definition <strong>of</strong> Comprehensive Write-up<br />

A Comprehensive Write-up is a complete history and physical or a problem-focused note in<br />

standard Problem-Oriented Medical Record form (see following examples) and includes:<br />

a) a complete Problem List, with problems designated as “active” or<br />

“inactive/resolved” with dates <strong>of</strong> onset and resolution, respectively<br />

b) a comprehensive Assessment <strong>of</strong> at least three (3) problems from the Problem<br />

List, with discussion <strong>of</strong> differential diagnoses <strong>of</strong> undiagnosed problems (including rationale for<br />

including/excluding diagnoses) or discussion <strong>of</strong> diagnosed problems (such as course,<br />

complications, control and compliance)<br />

c) Plans, divided into Diagnostic, Therapeutic and Education Plans<br />

d) Resources and References<br />

e) Student’s name, printed and signed, followed by “MS3”<br />

f) Abbreviations: Since medical records communicate important information and<br />

may be scrutinized, abbreviations should not be used because their meaning is not universal<br />

among all readers <strong>of</strong> the medical records. Specifically, students should not use error-prone<br />

abbreviations, symbols, and dose designations (see Appendix).<br />

Submission Requirements <strong>of</strong> Comprehensive Write-ups<br />

a) 6B and 6L students on Inpatient <strong>Medicine</strong> are required to submit 3 write-ups by halfway<br />

through the inpatient rotation (averaging 1 write-up each week). The Hospital Site Coordinator<br />

will decide whether these 3 write-ups are satisfactory. If they are deemed satisfactory, then<br />

the student will not be required to submit any more write-ups. If, however, they are deemed<br />

unsatisfactory, then the student will be required to submit 1 – 3 additional write-ups, as<br />

determined by the Hospital Site Coordinator, up to a maximum <strong>of</strong> 6 write-ups.<br />

b) 6B students on Ambulatory <strong>Medicine</strong> are required to submit 2 write-ups each week for a<br />

total <strong>of</strong> 10 write-ups to their Ambulatory Preceptor.<br />

c) 6L students on Ambulatory <strong>Medicine</strong> are required to submit 2 write-ups each month for<br />

a total <strong>of</strong> 10 write-ups to their Ambulatory Preceptor.<br />

d) All write-ups, either originals with corrections/comments from the preceptor or copies <strong>of</strong><br />

the originals with corrections/comments, must be turned in by the last day <strong>of</strong> the respective<br />

inpatient or ambulatory component <strong>of</strong> the <strong>clerkship</strong>.<br />

238


Example <strong>of</strong> Inpatient History and Physical<br />

Date <strong>of</strong> Admission: 11/01/02<br />

Date <strong>of</strong> Exam: 11/01/02<br />

ID: 78-yo Japanese woman who is a widow and a retired hotel worker<br />

S/R: Patient and her daughter, who are fair historians. Medical records not available.<br />

RE: Admission to Progressive Care Unit at Kuakini Medical Center<br />

CC: Three episodes <strong>of</strong> "Bloody stools" since last night<br />

HPI: The patient is a 78-year old woman with history <strong>of</strong> hypertension, hypercholesterolemia and two<br />

previous "mild strokes", who was in her usual state <strong>of</strong> health until October 29, 2002, three days prior<br />

to admission, when she began passing bright red blood per rectum along with "dark black clots" and<br />

"black stools", The patient claims to have had more than 10 bowel movements <strong>of</strong> this kind within 10<br />

hours. She admits to feeling weak at this time with a "near-fainting" episode after which she found<br />

herself drenched with sweat, as if "someone dumped a bucket <strong>of</strong> water over my head". The patient's<br />

daughter claims that the patient may have experienced a brief loss <strong>of</strong> consciousness (less than 30<br />

seconds.) during this "near-fainting" episode, as she stopped talking for a short period <strong>of</strong> time. On the<br />

morning <strong>of</strong> October 30, 2002, the patient went in to see her physician, Dr. Shozo Ogawa. At this time<br />

the bleeding had ceased and her only complaint was weakness. Dr. Ogawa found her to be stable,<br />

and found her hemoglobin to be 12.2, and arranged for her to follow-up with a gastroenterologist. On<br />

October 31 st , at around 5:00pm, the patient again began to pass bright red blood per rectum, but<br />

without black clots. She says she passed 3 bloody bowel movements since that time until she<br />

presented to the Emergency Room on the morning <strong>of</strong> November 1st. She admits to feeling weak and<br />

"lousy" and again feeling faint, but not actually fainting.<br />

The patient denies any prior episodes <strong>of</strong> rectal bleeding. She had experienced some constipation the<br />

week before, and had used Metamucil, which had given relief. She denies fever, nausea, vomiting,<br />

diarrhea, sick contacts, chest pain, shortness <strong>of</strong> breath, recent weight changes or changes in<br />

appetite. She claims to have a chronic mild right lower quadrant abdominal pain which she attributes<br />

to her degenerative vertebral disc disease. She also admits to having "sour stomach" every few<br />

months, but no history <strong>of</strong> previously diagnosed GERD or peptic ulcer disease.<br />

In 2000, the patient was diagnosed with "degenerative disc disease" in her lower spine. She claims to<br />

have had back pain since age 17, and utilizes a back brace and cane to assist in mobilization. She<br />

has been using aspirin for the pain, with her last dose taken 10 days ago. She was told by her<br />

physician at the <strong>of</strong>fice visit three days ago that "aspirin wasn't good for her stomach" and so she has<br />

since been taking Extra-Strength Tylenol, with gives her only partial relief.<br />

PMH:<br />

Childhood illnesses: Not asked<br />

Immunizations: Not asked<br />

Adult illnesses: Two "mild strokes" (Had slurred speech) in 1970s<br />

Cervical cancer – had possible cone biopsy in 1970s<br />

Hypertension- first aware <strong>of</strong> diagnosis in 1970s<br />

Degenerative disc disease- told <strong>of</strong> diagnosis in 2000; uses back brace, cane, occasional<br />

acupuncture<br />

Cataracts bilaterally- date <strong>of</strong> diagnosis unknown<br />

Hypercholesterolemia- date <strong>of</strong> diagnosis unknown<br />

The patient has no history <strong>of</strong> bleeding disorder, liver disease, diabetes mellitus, myocardial infarction<br />

or renal disease.<br />

Hospitalizations/Surgeries:<br />

1950s Kapiolani Medical Center- birth <strong>of</strong> her children<br />

239


1960s Kapiolani Medical Center- Hysterectomy, reason for surgery unknown.<br />

1998 Queens Medical Center- "surgery for the insides coming out the vagina"<br />

Transfusions: Not asked<br />

Current medications: Covera HS (verapamil) 240mg qd<br />

Tylenol 1 tab every 4-6 hrs pm back pain, last dose taken 1 day ago<br />

Aspirin- dosage unknown, last dose taken 10 days ago<br />

Allergies: Penicillin reaction unknown; told by doctor not to take penicillin<br />

Sulfa acute onset <strong>of</strong> "red rash and skin peeling in sheets" (1960s)<br />

Cortisone face swelling<br />

FH: Patient's father died in his 40s from “stomach cancer." Mother died in her 90s <strong>of</strong> a "stroke, and<br />

had diabetes mellitus. The patient has numerous siblings, some <strong>of</strong> whom are step-siblings. One sister<br />

has diabetes. Two <strong>of</strong> the step-siblings have died, causes unknown. Health status <strong>of</strong> the other siblings<br />

are unknown. There is no family history <strong>of</strong> bleeding disorders.<br />

40s<br />

“Stomach ca”<br />

90<br />

“Stroke”<br />

“DM”<br />

Personal Pr<strong>of</strong>ile/SH: The patient is a retired hotel worker and widow. She now lives with her<br />

daughter. She denies the use <strong>of</strong> cigarettes, alcohol or illicit drugs. Her diet consists mostly <strong>of</strong> cereals,<br />

oatmeal and bread. She doesn't each much fruit or vegetables.<br />

ROS:<br />

General: See HPI. Denies weight loss, fever.<br />

Skin: Has no rash, itching, bruising.<br />

Eyes: Reports no blurry vision, other visual disturbances.<br />

Ears: Reports no hearing loss, tinnitus, pain, discharge, vertigo.<br />

Nose: Has "allergies” which cause runny nose, sneezing, cough.<br />

Mouth: Has no gingivitis, sore tongue, taste changes, dental problems<br />

Throat: Reports no pain, voice changes<br />

Pulmonary: Reports no chest pain, pneumonia, SOB, DOE, wheezing, sputum, hemoptysis<br />

Circulatory system: Has no chest pain, palpitations, dyspnea, PND, orthopnea, edema, syncope<br />

GI: See HPI; has no change in appetite, dysphagia, nausea, vomiting, rectal pain, hematemesis,<br />

diarrhea, peptic ulcer disease<br />

GU: Reports no frequency, nocturia, polyuria, urgency, dysuria, hematuria, hesitancy, urinary flow<br />

240


changes, retention, incontinence; has no history <strong>of</strong> kidney problems<br />

Female genitalia: See PMH.<br />

Breast: Not asked<br />

Sexual Hx: Not asked<br />

Endocrine system: Reports no neck mass, thyroid problems, exophthalmos, heat/cold intolerance,<br />

thirst changes<br />

Hemopoietic system: Has no lymph node enlargement, excessive bleeding, bruising, anemia<br />

Musculoskeletal system: See HPI and PMH. Has other joint or muscle pain.<br />

Nervous system: Has no history <strong>of</strong> head trauma, headaches, numbness, paralysis, convulsions,<br />

seizures, tremor, gait disturbances, coordination changes<br />

Mood: Not asked.<br />

PE:<br />

General Appearance: Patient appears well-nourished, appearing her stated age. She is lying<br />

comfortably in bed, in no evident distress. She is alert, oriented and cooperative.<br />

Vital Signs: Temp 96.0; Respirations 11; Oxygen sat 99% on Room Air; Supine- HR 89, BP 147/64;<br />

Standing- HR 110, BP 131/54<br />

Skin: Warm, dry, pale<br />

Head: Nontender over scalp<br />

Eyes: Acuity not tested. PERRL. Extraocular mucles function intact. Fundi not visualized due to<br />

cataracts.<br />

Ears: Acuity not tested. Pinna- no lesions, nontender. Canals- no bleeding. TMs not visualized due to<br />

cerumen obscuring view. Nose: normal pink mucosa nontender. no discharge.<br />

Mouth: Mucosa pink, moist, slightly pale. No lesions or bleeding. No tonsillar erythema or exudates.<br />

Neck: Supple. No thyromegaly, enlarged lymph nodes, jugular venous distention; no carotid bruits.<br />

Pulmonary: Lungs clear to percussion and ausculatation bilaterally, anteriorly and posteriorly<br />

Cardiac: No thrills, lifts or heaves. PMI palpated in left 5th ICS at the midclavicular line, non-bounding.<br />

Rate and rhythm are regular, normal S1 and S2. No murmurs, extra heart sounds heard.<br />

Abdomen: S<strong>of</strong>t, non-tender, non-distended. Normoactive bowel sounds in four quadrants. No<br />

hepatosplenomegaly by palpation..<br />

Rectal (done by ER physician- reported as showing no masses, with “pink stool”. Bright red blood on<br />

glove which tested positive with hemoccult.<br />

Extremities: Full motion in all extremities. No clubbing, cyanosis, edema. Patient was slow to stand,<br />

and had an antalgic gait she attributed to back pain.<br />

Neuro: Alert and oriented x 3. Cranial Nerves: II - XII grossly intact. Speech slightly slurred, difficult to<br />

comprehend at times. Sensation normal to light touch and 10 gram mon<strong>of</strong>ilament; motor 5/5 in all<br />

extremities. DTRs 2+ in biceps, triceps, knees and ankles. No Babinski response is noted.<br />

Admission lab results:<br />

CBC: WBC 7.9, differential: Bands 7, Segs 50, Lymphs 37, Monocytes 5, Eos O, Baso 1<br />

Hgb 9.8 (was noted to have been 12.2, 2 days PTA),<br />

Hct 28.4<br />

MCV 93.9<br />

Platelet count 238<br />

BMP: Na 140 BUN 20 PT 12.3<br />

K 3.6 Creatinine 0.7 INR 1.0<br />

Cl 110 Glucose 135 PTT 22<br />

Bicarb 25<br />

241


Problem List<br />

Problem<br />

No.<br />

Date Onset Active Problems Date Resolved Inactive/Resolved<br />

Problems<br />

1. 1960s Allergic reaction to sulfa (rash,<br />

peeling skin)<br />

2. 1970s 1970s Hx <strong>of</strong> cervical cancer<br />

s/p TAHBSO<br />

3. 1970s Hx <strong>of</strong> “mild stroke x2<br />

slurred speech<br />

4. 1970s Hypertension<br />

5. 1998 Bladder prolapse s/p<br />

corrective surgery<br />

6. 2000 Degenerative disc disease <br />

chronic lower back pain<br />

7. 10-29-02 GI bleed<br />

8. 11-01-02 Anemia<br />

9. Unknown Hypercholesterolemia<br />

10. Unknown Unknown Bilateral cataracts<br />

s/p cataract surgery<br />

11. Unknown Allergy to penicillin<br />

(unknown reaction), cortisone<br />

(facial swelling)<br />

Problem #1: Bleeding per rectum<br />

Assessment: The patient has experienced 2 episodes <strong>of</strong> bloody stools in the past three days, during<br />

which time she has had more many bloody bowel movements. During the first episode, she describes<br />

bright red blood as well as “black clots.” In the most recent episode, she reports only bright red<br />

blood. She has been feeling weak since these episodes. In addition, there is evidence that this patient<br />

has had substantial blood loss including: (1) a decrease in hemoglobin from 12.2 to 9.8 in two days<br />

with normal MCV, which suggest an acute bleed; (2) orthostatic changes with an increase in HR <strong>of</strong><br />

>20 and decrease in Systolic BP > 15mmHg suggest that the patient is hypovolemic, with a blood<br />

loss <strong>of</strong> greater than 1 Liter. In trying to identify the source <strong>of</strong> the bleed we must attempt to differentiate<br />

an upper GI bleed from a lower GI bleed, as well as confirm that the bleeding is from the rectum and<br />

not from the vagina or urethral orifice. Because the rectal exam confirms that there is blood in the<br />

rectum, we can assume the patient does have a GI bleed. With the reports <strong>of</strong> “bright red blood", it is<br />

likely that the patient is having a lower GI bleed, although a brisk upper GI bleed could also present<br />

as bright red blood per rectum. The “black clots” seen in the first episode suggest a possible upper GI<br />

bleed, in which the “clots” may have been melenic stool.<br />

The differential diagnosis <strong>of</strong> lower GI bleed include diverticulosis, colon cancer or polyps, ulcerative<br />

colitis, angiodysplasia, and hemorrhoids. Diverticulosis is likely as it most common in the elderly and<br />

can present<br />

with painless bright red blood per rectum that can result in massive hemorrhage. A diverticular<br />

242


hemorrhage is usually caused by erosion <strong>of</strong> a blood vessel by a fecalith within the diverticular sac.<br />

The patient did have some recent constipation which could have lead to the development <strong>of</strong> a<br />

fecolith. She took Metamucil for relief. It is unknown if the patient then had diarrhea, which can occur<br />

with Metamucil; but if she did, it is possible that the<br />

diarrhea in its rapid transit through the colon could have dislodged the fecalith resulting in injury to a<br />

blood<br />

vessel and hemorrhage. Diverticular bleeding stops spontaneously in the majority <strong>of</strong> patients. This<br />

was not the case for this patient. This could be due to the rupture <strong>of</strong> a large blood vessel in the colon,<br />

or it could be due to the patient's chronic use <strong>of</strong> aspirin (which can inhibit platelet aggregation<br />

resulting in decreased clotting ability for about 7-10 days, however, she claims to have not had<br />

aspirin for 10 days prior to admission).<br />

Colon cancer is also possible. Like diverticular disease, it is more common in the elderly. The patient<br />

displays signs and symptoms that are characteristic <strong>of</strong> a left colonic cancer- change in bowel habits<br />

(constipation, diarrhea) and bright red blood per rectum. Intestinal obstruction is also common with a<br />

left colon cancer, however the patient did not complain <strong>of</strong> abdominal pain or tenderness which would<br />

be expected with obstruction. A right colon carcinoma would present with an iron deficiency anemia<br />

due to chronic blood loss, which can be occult and thus unknown to the patient, and a dull vague<br />

abdominal pain. The patient did have a chronic dull lower right quadrant abdominal pain. She also<br />

had anemia (Hgb 9.8), however this anemia appears to be due to the acute blood loss and not iron<br />

deficiency as her Hgb was 12.2 two days prior to admission, and her MCV was normal.<br />

Ulcerative colitis should also be considered, although more commonly diagnosed in younger adults,<br />

there is also a small peak incidence among the elderly, ages 50-65. Although the patient is beyond<br />

this peak, UC must still be considered given that it's hallmark is bloody diarrhea. Most patients with<br />

UC will also have fever and weight loss, which this patient did not have. Angiodysplasia/AVM is also<br />

possible as it is most commonly seen in the elderly and presents with painless bright red blood per<br />

rectum. Hemorrhoids can be a cause <strong>of</strong> rectal bleeding, however there is usually associated pain<br />

and a palpable rectal mass, which this patient did not have. Infectious colitis is less likely in this<br />

patient as she is has no fever, abdominal pain or leukocytosis, all <strong>of</strong> which would be expected with an<br />

infectious process. The patient also denies any sick contacts.<br />

A brisk Upper GI bleed could also be responsible for this patient's rectal bleeding, however with such<br />

a large amount <strong>of</strong> blood loss, one would also expect some hematemesis, which did not occur. The<br />

differential would include gastritis, vascular ectasia, peptic ulcer disease and ruptured varices.<br />

Gastritis is very likely given the patient's chronic use <strong>of</strong> aspirin. However, she does not have<br />

abdominal pain which might be expected if it were severe enough to have caused this patient's<br />

bleeding. Peptic ulcer disease is less likely as pain is its predominant symptom, and the patient<br />

denied any abdominal or chest pain. It is important to rule-out a gastric ulcer in this patient, or to<br />

diagnose and treat it, as an untreated ulcer can increase her risk <strong>of</strong> developing gastric cancer. She is<br />

already at risk given her Japanese ethnicity and positive family history <strong>of</strong> gastric cancer. Ruptured<br />

varices is unlikely as the patient has no hematemesis and no history <strong>of</strong> liver disease which would<br />

cause the formation <strong>of</strong> varices. Her normal coagulation studies make liver disease unlikely. Given the<br />

numerous possible diagnoses for this patient's GI bleed it is imperative to have the patient to undergo<br />

both upper endoscopy and colonoscopy, especially given her history <strong>of</strong> both bright red blood per<br />

rectum and “black clots".<br />

Plan: Admit to acute care hospital with careful monitoring <strong>of</strong> vital signs.<br />

Plans:<br />

Diagnostic:<br />

-Place NG tube to assess for gastric bleeding<br />

243


-Consult a gastroenterologist for upper endoscopy and colonoscopy<br />

Treatment:<br />

-Type and crossmatch. Transfuse 2 units packed red blood cells to replace blood loss, since she is at<br />

risk to cntinue bleeding<br />

-No food or drink in preparation for endoscopy according to gastroenterologist instructions<br />

-Further treatment dependent on endoscopic findings. Consider initiate gastric acid blocking regimen<br />

prophylactically<br />

Patient education:<br />

-Inform patient <strong>of</strong> the possible diagnoses and the need for careful monitoring and testing<br />

-Inform patient <strong>of</strong> endoscopic procedures, explain risks and benefits, obtain informed consent<br />

-Inform patient <strong>of</strong> need for transfusion, explain risks and benefits, obtain informed consent<br />

Problem #2: Anemia<br />

Assessment: The patient has developed an acute anemia. Her hemoglobin had dropped 2.4 G/dL<br />

(from 12.2 to 9.8), which correlates with a decrease in hematocrit <strong>of</strong> approximately 7%. Hematocrit<br />

generally falls 2-3 points for every 500 mL <strong>of</strong> blood lost, making the estimated blood loss in this<br />

patient 3.5L. The normal MCV also suggests that the anemia is due to an acute blood loss. The<br />

normal coagulation studies rule-out a coagulopathy that may complicate the patient's GI bleed,<br />

although platelet dysfunction due to the patient's chronic aspirin use may exacerbate the bleed. The<br />

patient's orthostatic changes in heart rate and systolic BP is consistent with blood loss <strong>of</strong> greater than<br />

1 liter resulting in hypovolemia. The patient appears pale with pale mucous membranes and has<br />

complaints <strong>of</strong> weakness and light-headedness, which are all due to her anemia. It is important to<br />

transfuse this patient and increase her hemoglobin in order to avoid the complications associated with<br />

anemia and hypovolemia, such as high output cardiac failure and organ hypoperfusion, which may be<br />

<strong>of</strong> greater risk in the elderly.<br />

Plans:<br />

Diagnostic:<br />

-Monitor blood count every 4 hrs for continued bleeding and decrease in hemoglobin<br />

Treatment:<br />

- Place adequate intravenous access (2 large bore peripheral catheters)<br />

-Type and crossmatch. Transfuse 1 unit packed red cellss now.<br />

-Intravenous fluids: normal saline at 100cc/hour<br />

-Transfuse packed red cells to keep hemoglobin > 10 G/dL<br />

Patient education:<br />

-Inform patient <strong>of</strong> need for transfusion, explain risks and benefits, obtain informed consent<br />

-Inform patient <strong>of</strong> signs and symptoms <strong>of</strong> worsening anemia that she should be aware <strong>of</strong>, such as<br />

worsening orthostatic hypotension, weakness, faintness, pallor, tachycardia.<br />

Problem #3: Back pain<br />

Assessment: The patient has been previously diagnosed with degenerative disc disease and suffers<br />

from chronic back pain. She regularly uses a back brace and cane to assist with mobilization. She<br />

occasionally has acupuncture treatments to relieve the pain. She had been taking aspirin on a regular<br />

basis for pain, and recently changed to Extra-Strength Tylenol which gives only partial relief. Given<br />

the possibility <strong>of</strong> gastritis, it is best for the patient to avoid aspirin and any other NSAID, especially<br />

with an active GI bleed. Once the bleeding has resolved, a specific COX-2 inhibitor could be<br />

considered, although there is still a risk <strong>of</strong> GI irritation with COX2 inhibitors, the risk is less than that <strong>of</strong><br />

nonspecific NSAIDs. Given the patient's allergic reaction to sulfa, celecoxib is contraindicated;<br />

however, r<strong>of</strong>ecoxib can be used. In clinical trials with r<strong>of</strong>ecoxib, 3.9% <strong>of</strong> the patients had a reported<br />

sulfa sensitivity, none developed anaphylactic reactions. Another option would be to administer<br />

misoprostol along with the NSAID, in order to further protect the gastric mucosa. In order to<br />

244


completely avoid NSAIDs but still provide complete relief, tramadol could be considered. tramadol is<br />

a centrally acting analgesic which acts as an opiate agonist, although it is not opioid-derived.<br />

Tramadol does not irritate gastric mucosa, thus not causing an increased risk for GI bleed. Side<br />

effects reported for tramadol include constipation, dizziness, nausea, dry mouth, sweating and<br />

minimal cardiovascular effects including hypotension, tachycardia and syncope. Given the patients<br />

current gastritis, recent GI bleed and sulfa allergy, tramadol might be the best choice for pain relief in<br />

this patient should she request more complete relief than she is currently receiving with Tylenol; while<br />

being aware <strong>of</strong> possible hypotensive effects <strong>of</strong> this drug.<br />

Plans:<br />

Diagnostic: No further diagnostic studies at this time.<br />

Treatment:<br />

-Continue Tylenol 1 gram every 4-6 hrs as needed for back pain, not to exceed 4g daily<br />

-If better pain control is desired, consider starting tramadol 25 mg daily<br />

Patient education:<br />

-Inform patient <strong>of</strong> the need to avoid NSAIDs<br />

-Inform patient <strong>of</strong> the daily maximum dose <strong>of</strong> Tylenol (4 G per day) and the adverse effects that could<br />

occur in the event <strong>of</strong> toxicity<br />

-Inform patient that a trial <strong>of</strong> tramadol could be started, with attention to added side effects, if better<br />

pain control is needed<br />

--------------------------------------------------------------------------------------<br />

Resources:<br />

AHFS Drug Information, 2002<br />

Ferri. Practical Guide to the Care <strong>of</strong> the Medical Patient, 5th ed., 2001<br />

Myers. NMS <strong>Medicine</strong>, 4th ed" 2001<br />

<br />

Toby Best, MS 3<br />

245


Example <strong>of</strong> Ambulatory Note<br />

Problem List<br />

Problem<br />

#<br />

Date <strong>of</strong><br />

onset<br />

Active Problems Date<br />

resolved<br />

Inactive/Resolved<br />

Problems<br />

1. 9/14/04 Pharyngitis<br />

2. 1998 Hypertension<br />

3. 1994 Obesity<br />

4. 1973 S/P Appendectomy<br />

Date <strong>of</strong> Examination: September 17, 2004<br />

ID: 40 year old Part <strong>Hawaii</strong>an woman<br />

SR: History obtained from the patient and medical records, which are reliable.<br />

RE: Scheduled <strong>of</strong>fice visit<br />

CC: Sore throat and fever x3 days<br />

Problem #1: Pharyngitis<br />

S: This 40 year old woman presents to the <strong>of</strong>fice per chief complaint. 3 days ago patient began<br />

having fever, chills, sore throat, rhinorrhea, myalgia, and fatigue. She denies cough, dyspnea,<br />

wheezing, otalgia, dysphagia, abdominal pain, nausea, vomiting, diarrhea or constipation. She has<br />

taken Advil, which improves symptoms. There are no sick contacts. Patient is able to swallow solids<br />

and liquids.<br />

Patient is taking oral contraceptives, and has no medication allergies.<br />

O:<br />

General: Well-groomed, cheerful woman who appears stated age.<br />

Not in acute distress.<br />

VS: T 97.5°F; P 78; R 12; BP 112/68; Wt 159 lbs.; Ht. 61 in.<br />

Skin: Warm, dry, intact. No rashes, petechiae, or bruising noted.<br />

Ears: Symmetrical auricles, canals nontender. Auditory canals clear and<br />

non-erythematous bilaterally. Tympanic membrane intact and pearly with clear cone <strong>of</strong> light<br />

bilaterally.<br />

Nose:<br />

Nares patent, septum midline, clear discharge.<br />

Mouth/Throat: Erythema and yellowish exudate present on posterior pharynx and<br />

tonsils bilaterally. Mucous membranes moist. Lips without cyanosis or lesions. Uvula midline.<br />

Tongue pink, moist. Parotids<br />

nonpalpable.<br />

Neck:<br />

Supple, no lymphadenopathy, no masses, non-tender to palpation.<br />

Trachea midline.<br />

Lungs: Clear to percussion and auscultation bilaterally. No wheezes, rales or coarse breath<br />

sounds.<br />

Heart:<br />

No thrills or heaves felt. RRR. S1 and S2 <strong>of</strong> equal intensity. No<br />

murmurs or rubs heard.<br />

246


A: While the differential diagnosis <strong>of</strong> acute pharyngitis in adults includes a number <strong>of</strong> bacterial and<br />

viral pathogens, the objective in evaluating a patient with pharyngitis is to identify those with group A<br />

streptococcus (GAS) pharyngitis. The patient’s symptoms and physical findings suggest possible<br />

group A streptococcal pharygitis. Although viruses are the most common cause <strong>of</strong> acute pharyngitis,<br />

Group A strep pharyngitis accounts for approximately 10% <strong>of</strong> acute pharyngitis in adults. It presents<br />

with the sudden onset <strong>of</strong> sore throat, tonsillar exudate, tender cervical adenopathy, and fever.<br />

Malaise, headache, abdominal pain and vomiting may also occur. The absence <strong>of</strong> fever or presence<br />

<strong>of</strong> cough, conjunctivitis, hoarseness, coryza, viral exanthem, and diarrhea strongly suggest a viral<br />

rather than a streptococcal etiology. The Centor criteria for predicting GAS have been the most widely<br />

used and accepted and include, 1) tonsillar exudate; 2) tender anterior cervical adenopathy; 3) fever<br />

by history; 4) absence <strong>of</strong> cough. If three or four <strong>of</strong> these criteria are met, the positive predictive value<br />

is 40% to 60%. However, the presence <strong>of</strong> less than three has a negative predictive value <strong>of</strong> 80%. By<br />

these criteria, this patient has a 40-60% probability <strong>of</strong> having GAS pharyngitis.<br />

Throat cultures have always been the "gold standard" for diagnosing GAS pharyngitis with a<br />

sensitivity <strong>of</strong> 90% and specificity <strong>of</strong> 95-99% if collected properly. However, cultures take 24 to 48<br />

hours to grow, therefore cannot be used to decide which patients merit antibiotic therapy. Thus, rapid<br />

antigen detection test (RADT) has assumed greater importance in managing acute pharyngitis.<br />

RADT uses enzyme or acid extraction <strong>of</strong> antigen from throat swabs followed by latex agglutination,<br />

co-agglutination, or enzyme linked immunoabsorbent assay (ELISA) procedures to demonstrate<br />

antigen-antibody complexes. The majority <strong>of</strong> RADTs that are currently available have a sensitivity <strong>of</strong><br />

80-90% and specificity <strong>of</strong> >95%. It is also advantageous because it can yield results within<br />

approximately 10 minutes. Rapid identification and treatment <strong>of</strong> those with strep pharyngitis can<br />

reduce the risk <strong>of</strong> its spread, allowing the patient to return to school or work sooner, result in faster<br />

resolution <strong>of</strong> signs and symptoms (by 1-2 days), and can reduce the risk <strong>of</strong> suppurative complications<br />

and acute rheumatic fever. A positive result <strong>of</strong> either throat culture or RADT provides adequate<br />

confirmation <strong>of</strong> the presence <strong>of</strong> GAS in the pharynx. In children, a negative RADT should be<br />

confirmed with a throat culture result while in adults a negative RADT is sufficient to exclude this<br />

diagnosis, because the risk for acute rheumatic fever in adults is extremely low. Serologic tests for<br />

antistreptococcal antibody titers reflect past and not present immunologic events and are <strong>of</strong> no value<br />

in the diagnosis <strong>of</strong> acute pharyngitis.<br />

The major goal <strong>of</strong> identifying patients with GAS pharyngitis is to prescribe antibiotics to these patients<br />

and not treat the others, especially those with pharyngitis caused by a virus in order to limit antibiotic<br />

resistance. Recent guidelines from the Infectious Diseases Society <strong>of</strong> America (IDSA) regarding<br />

principles <strong>of</strong> management in cases <strong>of</strong> sore throat include: (1) use <strong>of</strong> clinical and epidemiologic<br />

features to distinguish patients who may have GAS pharyngitis; and (2) antibacterial treatment only<br />

for cases confirmed with a laboratory test (culture or rapid test). In contrast, a position paper by the<br />

American College <strong>of</strong> Physicians–American Society <strong>of</strong> Internal <strong>Medicine</strong>/American Academy <strong>of</strong> Family<br />

Physicians/US Centers for Disease Control and Prevention, while endorsing the IDSA approach in<br />

children, recommends a departure from the principle <strong>of</strong> laboratory confirmation <strong>of</strong> all adult cases. The<br />

suggested strategies are: empirically treat patients who have all four Centor clinical criteria (fever,<br />

tonsillar exudate, tender anterior cervical adenopathy, and absence <strong>of</strong> cough); do not treat nor<br />

perform diagnostic tests on patients with zero or one criterion; perform RADT on those with two or<br />

three criteria and use antibiotic treatment for patients with positive RADT results. Empiric treatment<br />

<strong>of</strong> all patients with 3 or 4 Centor criteria can result in unnecessary antibiotic exposure to at least 50<br />

percent <strong>of</strong> patients. Some authorities, including the Infectious Diseases Society <strong>of</strong> America, find this<br />

unacceptable and recommend antibiotics only if there is a positive RADT or culture. This criterion will<br />

result in undertreatment <strong>of</strong> 10-20%, but the consequences <strong>of</strong> undertreatment in adults are rare. This<br />

patient has three out <strong>of</strong> the four Centor criteria, so under both current recommendations patient<br />

should have a RADT done to determine if antibiotic therapy is warranted.<br />

247


Most oral antibiotics must be administered for the conventional 10-day regimen. Penicillin remains<br />

the treatment <strong>of</strong> choice in treating acute streptococcal pharyngitis because <strong>of</strong> its proven efficacy,<br />

safety, narrow spectrum, and low cost. Amoxicillin is <strong>of</strong>ten used in young children, while erythromycin<br />

is a suitable alternative for patients who are allergic to penicillin. First-generation cephalosporins are<br />

also acceptable for patients allergic to penicillin who do not manifest immediate-type hypersensitivity<br />

to beta-lactam antibiotics. Intramuscular benzathine penicillin G is preferred for patients who are<br />

unlikely to complete a full 10-day course <strong>of</strong> oral therapy. Antibiotics can affect the metabolism and/or<br />

efficacy <strong>of</strong> oral contraceptives, which the patient is using. Antibiotic therapy with penicillin should be<br />

started if patient’s RADT is positive.<br />

Symptomatic treatment, including antipyretics, fluids, and gargles, can be helpful. One study showed<br />

that herbal tea (Throat coat), containing ingredients such as licorice root that may relieve irritation,<br />

provided significant temporary relief to the intensity <strong>of</strong> sore throat with swallowing compared to<br />

placebo tea. However herbal teas should be used with caution in this patient because <strong>of</strong> her<br />

hypertension and potential medication interactions. Licorice may cause hypokalemia, so should<br />

probably not be used by a patient taking a thiazide diuretic. Regardless <strong>of</strong> RADT results, patient<br />

should be advised to use acetaminophen for pain and fever, drink increased amounts <strong>of</strong> fluid, and<br />

possibly gargle for symptomatic relief. Side effects <strong>of</strong> NSAIDs can worsen hypertension, and there<br />

are potential drug interactions with antihypertensive medications including β-blockers. Also, use<br />

caution in recommending a decongestant for this patient because <strong>of</strong> her hypertension. However, her<br />

hypertension is well-controlled, so it would probably be safe to use medications such as<br />

pseudephedrine.<br />

Although complications <strong>of</strong> GAS are rare in adults, they include acute rheumatic fever, acute<br />

glomerulonephritis, toxic shock syndrome, tonsillopharyngeal abscess, otitis media, sinusitis,<br />

necrotizing fasciitis, bacteremia, and meningitis. If patient tests positive for GAS, she should be<br />

educated on the possible complications and the signs and symptoms to watch for. She should also<br />

be reminded to return if her symptoms become worse or do not improve within a few days.<br />

P. Diagnostic<br />

Run a rapid antigen detection test to check for Group A Streptococcus pharyngitis.<br />

Therapeutic<br />

If RADT positive: oral penicillin 250 mg PO three times a day for 10 days. Pt should be cautioned use<br />

a back up method <strong>of</strong> contraception while taking the antibiotics because <strong>of</strong> the potential drug<br />

interactions.<br />

Acetaminophen 500 mg PO every 4-6 hours as needed for fever<br />

Salt gargles, tea to relieve sore throat.<br />

Patient Education<br />

If RADT negative: Explain rationale for not using antibiotics.<br />

If RADT positive: Stress importance <strong>of</strong> finishing the 10-day course <strong>of</strong> antibiotics, remind patient that<br />

she is still contagious until she has been on antibiotics for 24 hours, and educate her on the signs and<br />

symptoms <strong>of</strong> possible complications.<br />

Encourage patient to drink adequate amounts <strong>of</strong> fluid. Educate on “recipe” <strong>of</strong> NaCl solution for<br />

gargle- ¼ tsp table salt to a full cup <strong>of</strong> water, and remind her not to swallow this salty solution!.<br />

Reassure patient that asymptomatic household contacts do not need a routine culture <strong>of</strong> throat swab<br />

specimen, unless there is a “ping-pong” spreading <strong>of</strong> GAS pharyngitis within the family.<br />

Return for a follow-up visit if symptoms do not improve within a few days.<br />

Follow-up in 2-3 days if symptoms do not improve.<br />

248


Problem #2: Hypertension<br />

S. No chest pain, palpitations, dizziness, headaches, shortness-<strong>of</strong>-breath, wheezing, orthopnea,<br />

PND or edema. Pt is taking Ziac (bisoprolol/hydrochlorothiazide) 5 mg/6.25 mg, two tablets in the<br />

morning, and denies any symptomatic side effects from the medication. She exercises by walking for<br />

30 minutes three times a week. She eats a well-balanced diet and limits her sodium intake.<br />

O:<br />

VS: As above<br />

Eyes: Sclera clear, conjunctiva pink, no discharge. Extraocular muscles<br />

intact, pupils equal, round, reactive to light and accommodation.<br />

No papilledema, nicking, or cotton wool spots.<br />

Neck: No jugular venous distention<br />

Lungs:<br />

As above<br />

Heart: As above<br />

Circulation: Posterior tibial and pedal pulses 2+ bilaterally. No jugular venous<br />

distention.<br />

Extremities: No edema.<br />

A: Patient’s blood pressure is well-controlled with lifestyle modifications and Ziac. The JNC VII<br />

guidelines indicate that she is below goal blood pressure <strong>of</strong>


W., et al. JAMA. 2004 Apr 7;291(13):1587-95.<br />

“Principles <strong>of</strong> Appropriate Antibiotic Use for Acute Pharyngitis in Adults” Snow et al. Ann Intern<br />

Med.2001; 134: 506-508.<br />

Practice Guidelines for the Diagnosis and Management <strong>of</strong> Group A Streptococcal Pharyngitis,<br />

Infectious Disease Society <strong>of</strong> America.<br />

Harrison’s Online<br />

<br />

N.O. Itall, MS 3<br />

250


CLINICAL SKILLS EXAMINATION (CSE)<br />

STUDENT INSTRUCTIONS<br />

(Revised 6/12)<br />

DESCRIPTION<br />

The CSE is a practical examination consisting <strong>of</strong> 5 - 10 clinical problems including patient<br />

encounters and writing stations. The examination requires you to interact with standardized patients<br />

who volunteer for this exercise. These patients may have real clinical findings or are carefully trained<br />

to simulate patients with “real” medical problems. You should interview and examine these patients<br />

as if they are being seen in an actual patient care situation.<br />

GENERAL PROCEDURES<br />

You will have 15 minutes with each patient. You are responsible for pacing your visit with the patient.<br />

The timer will announce the beginning <strong>of</strong> each patient session. You will be warned when there are 5<br />

minutes remaining in each station, when to rotate to and begin the next station. Do not enter an exam<br />

room until told to begin. Each <strong>of</strong> you will start at a different point along the exam route and will<br />

continue to rotate until all stations are completed. If 2 (or more) students end up in the same station<br />

at the same time, notify the exam monitor immediately.<br />

Please knock before entering a patient's room. If you are finished evaluating the patient before the<br />

15 minutes are up, you may leave the station and begin answering questions in the writing station.<br />

Introduce yourself to the standardized patients as you would introduce yourself to any patient. End<br />

the encounter as you would end any encounter. On the door <strong>of</strong> every patient's room you will find<br />

student instructions in a RED FOLDER. Do not open the RED FOLDER until the timer says, “begin<br />

the encounter.” Reading the student instructions is considered part <strong>of</strong> your 15 minutes with the<br />

patient. In general, there will be enough time to review the instructions for 1 to 2 minutes and still<br />

complete your task with the patient.<br />

The instructions will give you the patient’s name, age, gender and reason for the visit. The<br />

instructions may also indicate where you are seeing the patient (outpatient clinic, emergency room,<br />

etc.), and the time <strong>of</strong> day (if it is different from the current time). The vital sign including the<br />

temperature, heart rate, blood pressure and respiratory rate may also be given. You should accept<br />

the vital signs as accurate and you do not need to repeat them unless you feel the case specifically<br />

requires it.<br />

Your task with the patient will be specifically defined. For example the instructions may tell you to<br />

take an appropriate history and counsel the patient . . . or take an appropriate history and perform a<br />

focused physical examination. You are to perform a focused examination <strong>of</strong> each patient that is<br />

consistent with the instructions provided. Hand washing is an essential practice when seeing patients<br />

and should be considered as a requirement during this examination. Facilities for washing your<br />

hands are available in each examining room.<br />

READ THE INSTRUCTIONS CAREFULLY BEFORE ENTERING EACH STATION. Do not take the<br />

RED FOLDER containing the student instructions into the examination room. A copy <strong>of</strong> the<br />

instructions will be placed in each room for your reference. If you feel it is necessary, you may take<br />

notes on the blank sheets provided on your clipboard. PLEASE DO NOT WRITE ON THE STUDENT<br />

INSTRUCTIONS.<br />

251


You should not perform rectal, genital, breast or corneal reflex examinations on your patients. If you<br />

feel that one or more <strong>of</strong> these maneuvers should be included as part <strong>of</strong> the work up for the patient<br />

you should inform the patient (e.g. “Mrs. Smith, the next step would be to do a pelvic and rectal<br />

examination on you”) and/or include it as part <strong>of</strong> your proposed diagnostic evaluation. As part <strong>of</strong> the<br />

encounter, you should provide the patient with your initial impression and initial management plan.<br />

End the encounter with the patient as you would end any patient encounter.<br />

You will receive a supply <strong>of</strong> “BUSINESS CARDS” pre-printed with your Examinee Identification<br />

Number. Before leaving the patient's rooms please leave one card with the patient (e.g. "If you have<br />

any questions or need to get in touch with me, here is my card"). This will allow the standardized<br />

patient to complete your evaluation forms. If you forget to give your card to the patient please leave<br />

it with the monitor and he/she will bring it in to the patient. ONCE YOU LEAVE THE PATIENT'S<br />

ROOM, YOU ARE NOT ALLOWED TO RE-ENTER FOR ANY REASON.<br />

WRITING STATIONS (PATIENT NOTE):<br />

Immediately after each patient encounter will begin a “writing station” where you will answer specific<br />

questions related to the encounter. Depending on the station, you will be writing your answers on<br />

paper or entering them on a computer.<br />

For stations requiring paper and pen, take one TEST BOOKLET from the blue folder located at the<br />

writing station. WRITE IN YOUR ASSIGNED STUDENT EXAMINEE ID# ON YOUR TEST<br />

BOOKLET and then begin answering the questions. You will have 10 minutes to answer the<br />

questions and complete any evaluation forms. Remember to write clearly, as we must be able to<br />

read your writing to grade it. When you have finished, turn your paper faced down on the desk. A<br />

proctor will pick up these forms.<br />

For stations requiring computer entry, log on to the computer using your UH USERNAME AND<br />

PASSWORD. Enter the Patient’s ID Number (located in a blue folder next to the computer monitor)<br />

to access the entry form for the station and begin answering the questions. You will have 10 minutes<br />

to answer the questions and complete any evaluation forms. When you have finished, remember to<br />

LOGOUT.<br />

In most stations, you will be asked to describe the significant positive and negative clinical findings<br />

uncovered during your interaction with the patient that will allow you to make clinical decisions, your<br />

differential diagnosis in the order <strong>of</strong> likelihood, and/or your INITIAL DIAGNOSTIC management plan<br />

(See Attached Sample Patient Note form). LIST only one item for each numbered blank line. Be as<br />

specific as possible.<br />

BREAKS<br />

You are asked not to leave the examination area during the break times and to refrain from<br />

discussing the cases with each other.<br />

CONTENT OF EXAM<br />

The clinical situations for this examination reflect the “General Core Clinical Competencies and<br />

“Training Problems” described in the <strong>Medicine</strong> 531/532 Student Handbook. You are being evaluated<br />

based on checklists and rating scales completed by the patient and by the responses you provide in<br />

the test booklet regarding your findings (writing stations). You are not allowed to consult any medical<br />

references during the exam.<br />

252


PAGERS/CELLULAR PHONES<br />

Cellular phones and pagers will not be allowed inside the examination area. We can hold<br />

them for you while you take the examination, but we will not be responsible for loss or damages.<br />

Please plan accordingly.<br />

EQUIPMENT<br />

1. STETHOSCOPE<br />

2. REFLEX HAMMER<br />

3. PENS with black ink<br />

4. NAME TAG<br />

5. PROPER ATTIRE. You should dress as you would to see patients.<br />

6. Otoscopes, ophthalmoscopes, sphygmomanometers, tongue blades, reflex hammers, drapes, clip<br />

boards and other supplies will be available in each examination room.<br />

GRADING AND FEEDBACK<br />

Student performance on each station is scored based on the following component scores, weighted<br />

as follows:<br />

Communication/Interpersonal Skills 30-40%<br />

Hx-Taking/Physical Exam/Counseling 40-50%<br />

Problem Solving (writing stations) 10-30%<br />

The Total Examination Score is calculated based on the average <strong>of</strong> the scores on each station.<br />

Students must achieve Clerkship Level Competency on this examination to receive credit for Med<br />

531, and Graduation Level Competency to receive credit for Med 541.<br />

Each student will receive a feedback letter with their Total Score and breakdown <strong>of</strong> their component<br />

scores. In addition, the average ratings by the standardized patients on specific communication skills<br />

(introducing self, active listening, showing interest etc), will also be provided. Unfortunately, we<br />

cannot return the actual checklists and written test materials without compromising the cases which<br />

may be used on future examinations.<br />

IMPORTANT REMINDER:<br />

You must have your UH Login and Password to complete this examination.<br />

253


FREQUENTLY ASKED QUESTIONS:<br />

Q1: How can I best prepare for this examination?<br />

A1: Review the Training Problems and General Core Clinical Competencies sections <strong>of</strong> your<br />

Med531/532 Student Handbook. For each <strong>of</strong> the Training Problems listed, be able take an<br />

appropriate history and focused physical examination. Understand how specific signs and symptoms<br />

relate to the differential diagnosis for each problem. Most importantly, review and practice good<br />

communication and interpersonal skills when seeing patients during the rotation.<br />

Q2: Are all the stations equal in length and difficulty?<br />

A2: Some stations, especially those requiring both a focused history and focused physical<br />

examination, seem to take everyone longer than other stations, although there is great variability<br />

among students. All <strong>of</strong> the stations are designed to be completed in the time allotted. The more<br />

difficult stations require students to focus their examination on obtaining the most relevant clinical<br />

information. The passing score for each station is adjusted for case difficulty.<br />

Q3: What does the interstation exercise (writing station or patient note) look like?<br />

A3: See example below.<br />

254


JOHN DOE -- PATIENT NOTE<br />

EXAMINEE ID # ____ ____<br />

DATE: __/__/__<br />

HISTORY: Include significant positives and negatives from history <strong>of</strong> present illness, past medical<br />

history, review <strong>of</strong> system(s), social history and family history.<br />

PHYSICAL EXAMINATION: Indicate only pertinent positive and negative findings related to the<br />

patient’s chief complaint.<br />

DIFFERENTIAL DIAGNOSES: In order <strong>of</strong><br />

likelihood (with 1 being most likely), list up to<br />

5 potential or possible diagnoses for this<br />

patient’s presentation (in many cases, fewer<br />

than 5 diagnoses are likely).<br />

1.<br />

2.<br />

3.<br />

4.<br />

5.<br />

DIAGNOSTIC WORK UP: List immediate<br />

plans (up to 5) for further diagnostic workup.<br />

1.<br />

2.<br />

3.<br />

4.<br />

5.<br />

255


NBME SUBJECT EXAM IN INTERNAL MEDICINE<br />

256


MEDICINE<br />

General Principles 1%-5%<br />

Organ Systems 95%-99%<br />

Immunologic Disorders 5%-10%<br />

Diseases <strong>of</strong> the Blood and Blood-forming Organs<br />

5%-10% Diseases <strong>of</strong> the Nervous System and Special Senses<br />

5%-10% Cardiovascular Disorders<br />

15%-20% Diseases <strong>of</strong> the Respiratory System<br />

15%-20% Nutritional and Digestive Disorders<br />

10%-15% Gynecologic Disorders<br />

1%-5%<br />

Renal, Urinary, and Male Reproductive System<br />

10%-15% Disorders <strong>of</strong> the Skin and Subcutaneous Tissues<br />

5%-10% Diseases <strong>of</strong> the Musculoskeletal System and Connective<br />

Tissue 5%-10% Endocrine and Metabolic Disorders<br />

Physician Tasks 5%-10%<br />

Promoting Health and Health Maintenance<br />

10%-15% Understanding Mechanisms <strong>of</strong> Disease<br />

20%-25% Establishing a Diagnosis<br />

40%-45% Applying Principles <strong>of</strong> Management<br />

20%-25%<br />

1. A 22-year-old woman with a 10-year history <strong>of</strong><br />

asthma comes to the physician because she has had to<br />

increase her use <strong>of</strong> her albuterol inhaler during the past 6<br />

weeks. Her asthma was previously well controlled with<br />

inhaled glucocorticoids. She has a 2-year history <strong>of</strong><br />

generalized anxiety disorder controlled with fluoxetine and a<br />

5-year history <strong>of</strong> migraines. The migraines were well<br />

controlled with sumatriptan until 4 months ago when she<br />

began to have headaches twice weekly; propranolol was<br />

added to her regimen at that time. She has been taking an<br />

oral contraceptive for the past year. She says she has been<br />

under increased stress at graduate school and in her<br />

personal life during the past 3 months; during this period,<br />

she has been drinking an average <strong>of</strong> four cups <strong>of</strong> c<strong>of</strong>fee<br />

daily (compared with her usual one cup daily). She does not<br />

drink alcohol or use illicit drugs. She appears mildly anxious<br />

but is not in respiratory distress. Scattered end-expiratory<br />

wheezes are heard. The remainder <strong>of</strong> the examination<br />

shows no abnormalities. Which <strong>of</strong> the following is the most<br />

likely cause <strong>of</strong> the exacerbation <strong>of</strong> this patient's asthma?<br />

(A) Fluoxetine therapy<br />

(B) Increased caffeine intake<br />

(C) Oral contraceptive therapy<br />

(D) Propranolol therapy<br />

(E) Sumatriptan therapy<br />

2. A 28-year-old woman has palpitations that occur<br />

approximately once a week, last 1B5 minutes, and consist <strong>of</strong><br />

rapid, regular heart pounding. The episodes start and stop<br />

suddenly and have not been associated with chest discomfort<br />

or dyspnea. There is no history <strong>of</strong> heart problems. She drinks<br />

two to three cups <strong>of</strong> c<strong>of</strong>fee daily. She rarely drinks alcohol<br />

and does not smoke. Her pulse is 96/min and regular, and<br />

blood pressure is 120/88 mm Hg. A stare and lid lag are<br />

noted. The thyroid gland is firm and 1.5 times larger than<br />

normal. There is a midsystolic click at the apex and a grade<br />

2/6, early systolic murmur at the upper left sternal border. An<br />

ECG is normal except for evidence <strong>of</strong> sinus tachycardia.<br />

Which <strong>of</strong> the following is the most appropriate next step in<br />

diagnosis?<br />

(A) Ambulatory ECG monitoring<br />

(B) Measurement <strong>of</strong> serum thyroidstimulating<br />

hormone concentration<br />

(C) Measurement <strong>of</strong> urine<br />

catecholamine concentration<br />

(D) MUGA scan<br />

(E) Echocardiography<br />

257


3. A study is conducted to assess the benefits <strong>of</strong> a new drug to 5. Two days after receiving 3 units <strong>of</strong> packed red blood cells for<br />

reduce the recurrence <strong>of</strong> colonic polyps. The results postpartum hemorrhage, a 24-year-old woman has<br />

show a number needed to treat (NNT) <strong>of</strong> 16. Which <strong>of</strong> fatigue and slight jaundice. Laboratory studies show:<br />

the following is the most accurate interpretation <strong>of</strong> this<br />

result?<br />

Hemoglobin 8.8 g/dL<br />

(A) For every 16 patients treated, 1 would<br />

Hematocrit 28% Serum total<br />

benefit from the new drug<br />

bilirubin 5 mg/dL<br />

(B) For every 100 patients treated, 16 would<br />

benefit from the new drug<br />

(C) The new drug is 1.6 times more<br />

beneficial than a placebo<br />

(D) 93% <strong>of</strong> patients taking the new drug<br />

would benefit from it<br />

(E) 84% <strong>of</strong> patients taking the new drug<br />

would not have any benefit from it<br />

4. A previously healthy 57-year-old woman comes to<br />

the physician 1 week after noticing a lump under her right<br />

arm. She is concerned that it is breast cancer because both<br />

her mother and maternal aunt died <strong>of</strong> breast cancer. She<br />

does not smoke, drink alcohol, or use illicit drugs. She has<br />

avoided the sun for the past 10 years. She notes that her skin<br />

has never tanned but always burned and freckled when<br />

exposed to the sun. She exercises daily on a stationary<br />

bicycle and eats a well-balanced diet. Her temperature is<br />

37°C (98.6°F), pulse is 82/min and regular, respirations are<br />

14/min, and blood pressure is 130/74 mm Hg. There are<br />

numerous freckles over the entire body. Examination <strong>of</strong> the<br />

right breast shows a 0.6-cm, flat, brown lesion; the lesion is<br />

mottled with deep purple and black areas and has an irregular<br />

border. There are no breast masses, dimpling, peau d'orange,<br />

or nipple discharge. The patient says that the lesion has been<br />

present for 1 year, but she has never had it examined. There<br />

is a firm, nontender mass in the right axilla. Examination<br />

shows no other abnormalities. Which <strong>of</strong> the following is the<br />

most likely diagnosis?<br />

(A) Basal cell carcinoma<br />

(B) Fibrocystic changes <strong>of</strong> the<br />

breast (C) Malignant melanoma<br />

(D) Mastitis<br />

(E) Pigmented nevus<br />

(F) Port-wine stain<br />

(G) Squamous cell<br />

carcinoma (H) Superficial breast<br />

carcinoma<br />

Liver tests are otherwise within normal limits. Which <strong>of</strong><br />

the following is the most appropriate next step in<br />

diagnosis?<br />

(A) Cytomegalovirus antibody titer<br />

(B) Direct and indirect antiglobulin<br />

(Coombs) tests<br />

(C) Monospot test<br />

(D) Serology for hepatitis B<br />

markers (E) Ultrasonography <strong>of</strong> the<br />

gallbladder<br />

6. A 30-year-old man has had nausea, vomiting, and<br />

severe colicky right flank pain radiating into the thigh for 4<br />

hours. He is afebrile. There is right costovertebral angle<br />

tenderness. Urinalysis shows RBCs too numerous to count<br />

and no bacteria. Which <strong>of</strong> the following is the most likely<br />

diagnosis?<br />

(A) Acute glomerulonephritis<br />

(B) Bacterial cystitis<br />

(C) Benign prostatic hyperplasia<br />

(D) Bladder carcinoma<br />

(E) Renal cell carcinoma<br />

(F) Urinary tract tuberculosis<br />

(G) Urolithiasis<br />

7. A 66-year-old woman comes to the emergency<br />

department 1 hour after the sudden onset <strong>of</strong> retrosternal<br />

chest discomfort accompanied by nausea and diaphoresis.<br />

She has hypotension, jugular venous distention, and a<br />

murmur <strong>of</strong> tricuspid regurgitation. An ECG shows STsegment<br />

elevation in the right precordial leads. Which <strong>of</strong> the<br />

following is the most likely diagnosis?<br />

(A) Constrictive pericarditis<br />

(B) Dissecting aortic aneurysm<br />

(C) Pericardial tamponade<br />

(D) Pulmonary emboli<br />

(E) Right ventricular infarction<br />

258


8. A 20-year-old African American woman comes to the physician because <strong>of</strong> a 6-month history <strong>of</strong> diffuse joint pain,<br />

especially in her hips and knees. During this period, she occasionally has had a rash on her nose and cheeks. She has no<br />

history <strong>of</strong> serious illness and takes no medications. Her temperature is 38.1°C (100.5°F). Examination shows warmth and<br />

swelling <strong>of</strong> the knees. Laboratory studies show:<br />

Which <strong>of</strong> the following is the most likely diagnosis?<br />

(A) Ankylosing spondylitis<br />

(B) Gouty arthritis<br />

(C) Psoriatic arthritis<br />

(D) Reactive arthritis<br />

(E) Rheumatoid arthritis<br />

(F) Septic arthritis<br />

(G) Systemic lupus<br />

erythematosus<br />

Hemoglobin 10.5 g/dL Erythrocyte sedimentation rate 40 mm/h<br />

Serum<br />

Urea nitrogen 30 mg/dL Creatinine 1.8<br />

mg/dL<br />

9. A 37-year-old man with type 1 diabetes mellitus<br />

comes to the physician for a routine examination. His only<br />

medication is insulin. His pulse is 72/min, respirations are<br />

12/min, and blood pressure is 138/88 mm Hg. Funduscopic<br />

examination shows microaneurysms and hemorrhages.<br />

Sensation to vibration and light touch is decreased over the<br />

lower extremities. His serum creatinine concentration is 1.6<br />

mg/dL. A 24-hour urine collection shows 550 mg <strong>of</strong> protein.<br />

Treatment with which <strong>of</strong> the following is most likely to slow<br />

progression <strong>of</strong> this patient's renal disease?<br />

(A) Atenolol<br />

(B) Clonidine<br />

(C) Hydralazine<br />

(D) Hydrochlorothiazide<br />

(E) Lisinopril<br />

10. A 50-year-old man is admitted to the hospital within<br />

2 hours <strong>of</strong> the onset <strong>of</strong> nausea, vomiting, and acute<br />

crushing pain in the left anterior chest. He has a family<br />

history <strong>of</strong> early coronary artery disease. The pain does not<br />

subside with the administration <strong>of</strong> nitroglycerin, sublingually.<br />

An ECG shows ST-segment elevation in leads aVL and V2<br />

through V4. Which <strong>of</strong> the following is the most appropriate<br />

management to decrease myocardial damage and<br />

mortality?<br />

(A) Administration <strong>of</strong> digitalis<br />

(B) Administration <strong>of</strong> lidocaine<br />

(C) Administration <strong>of</strong> quinidine<br />

(D) Coronary artery bypass grafting within 1<br />

week<br />

(E) Thrombolytic therapy<br />

11. A previously healthy 67-year-old woman comes to<br />

the physician with her husband because <strong>of</strong> a 4-month history<br />

<strong>of</strong> a resting tremor <strong>of</strong> her right arm. Her husband reports that<br />

her movements have been slower and that she appears less<br />

stable while walking. Examination shows increased muscle<br />

tone in the upper extremities that is greater on the right than<br />

on the left. There is decreased right arm swing. Her gait is<br />

slow and shuffling. Which <strong>of</strong> the following is the most likely<br />

explanation for this patient's symptoms?<br />

(A) Bilateral frontal lobe<br />

degeneration<br />

(B) Decreased dopaminergic input to the<br />

(C) striatum Decreased serotonergic activity in the brain<br />

stem<br />

(D) Excessive output <strong>of</strong> oxytocin<br />

(E) Excessive thalamic output <strong>of</strong><br />

norepinephrine<br />

12. A 47-year-old man comes to the physician because<br />

<strong>of</strong> a 4week history <strong>of</strong> increased thirst and urination. He has<br />

had a 23-kg (50-lb) weight gain during the past 2 years. He<br />

has no history <strong>of</strong> serious illness and takes no medications.<br />

His mother and maternal grandfather have type 2 diabetes<br />

mellitus. The patient does not smoke and drinks one beer<br />

every night. He is 175 cm (5 ft 9 in) tall and now weighs 104<br />

kg (230 lb); BMI is 34 kg/m 2 . His pulse is 90/min, and blood<br />

pressure is 150/88 mm Hg. The remainder <strong>of</strong> the<br />

examination shows no abnormalities. His serum glucose<br />

concentration is 330 mg/dL. Which <strong>of</strong> the following is the<br />

most likely underlying cause <strong>of</strong> this patient's increased<br />

serum glucose concentration?<br />

(A) Autoimmune destruction <strong>of</strong> islet cells<br />

(B) Chronic pancreatitis<br />

(C) Exogenous production <strong>of</strong><br />

corticosteroids<br />

(D) Insulin resistance<br />

(E) Pancreatic cancer<br />

259


13. A previously healthy 39-year-old woman is brought<br />

to the physician because <strong>of</strong> a tingling sensation in her<br />

fingers and toes for 2 days and rapidly progressive<br />

weakness <strong>of</strong> her legs. She had an upper respiratory tract<br />

infection 2 weeks ago. She was unable to get up from bed<br />

this morning. Examination shows weakness <strong>of</strong> all four<br />

extremities, distal greater than proximal. Deep tendon<br />

reflexes are absent. Sensation is mildly decreased over the<br />

feet. Which <strong>of</strong> the following is the most likely diagnosis?<br />

(A) Guillain-Barré syndrome<br />

(B) Multiple sclerosis<br />

(C) Myasthenia gravis<br />

(D) Poliomyelitis<br />

(E) Tick paralysis<br />

14. A previously healthy 77-year-old woman who<br />

resides in a skilled nursing care facility is brought to the<br />

emergency department 6 hours after the onset <strong>of</strong> acute<br />

midback pain that began while lifting a box. The pain does<br />

not radiate, and she has no other symptoms. She continues<br />

to carry out her daily activities. She appears to be in mild<br />

distress. She is 157 cm (5 ft 2 in) tall and weighs 47 kg (104<br />

lb); BMI is 19 kg/m 2 . Examination shows mild tenderness<br />

over T11. There is no tremor. Serum studies show a calcium<br />

concentration <strong>of</strong> 9.1 mg/dL, a urea nitrogen concentration <strong>of</strong><br />

12 mg/dL, and a creatinine concentration <strong>of</strong> 0.5 mg/dL. An x-<br />

ray <strong>of</strong> the dorsal and lumbar spine shows an anterior wedge<br />

fracture <strong>of</strong> T11. In addition to treating the pain,<br />

supplementation with which <strong>of</strong> the following is most likely to<br />

improve this patient's underlying condition?<br />

(A) 25-Hydroxycholecalciferol<br />

(B) Levothyroxine<br />

(C) Selenium<br />

(D) Vitamin C<br />

(E) Vitamin E<br />

15. A 52-year-old woman comes to the physician<br />

because <strong>of</strong> a 3-month history <strong>of</strong> diarrhea and intermittent<br />

abdominal pain that radiates to her back. The pain is<br />

exacerbated by eating. She describes her stools as greasy,<br />

foul-smelling, and difficult to flush. She has had a 4.5-kg<br />

(10-lb) weight loss during the past 4 months. She has a<br />

history <strong>of</strong> chronic alcohol abuse. Examination shows mild<br />

epigastric tenderness. An x-ray <strong>of</strong> the abdomen shows<br />

calcifications in the epigastrium. Which <strong>of</strong> the following is<br />

the most likely diagnosis?<br />

(A) Bacterial overgrowth<br />

(B) Celiac disease<br />

(C) Lactose intolerance<br />

(D) Malabsorption <strong>of</strong> bile salts<br />

(E) Pancreatic insufficiency<br />

16. A 67-year-old woman comes to the physician<br />

because <strong>of</strong> an 8-month history <strong>of</strong> progressive shortness <strong>of</strong><br />

breath. The shortness <strong>of</strong> breath initially occurred only with<br />

walking long distances but now occurs after walking ¼ mile<br />

to her mailbox. She also has a daily morning cough<br />

productive <strong>of</strong> whitish tan sputum. She has had no chest pain,<br />

palpitations, orthopnea, or paroxysmal nocturnal dyspnea.<br />

She has smoked one pack <strong>of</strong> cigarettes daily for 52 years.<br />

Her pulse is 88/min, respirations are 20/min, and blood<br />

pressure is 144/90 mm Hg. Examination shows a barrelshaped<br />

chest. Breath sounds are decreased, and faint<br />

expiratory wheezes are heard in all lung fields. There is no<br />

peripheral edema. An x-ray <strong>of</strong> the chest shows no<br />

abnormalities except for hyperinflation. Which <strong>of</strong> the<br />

following is the most likely diagnosis?<br />

(A) Angina pectoris<br />

(B) Asthma<br />

(C) Chronic obstructive pulmonary<br />

disease<br />

(D) Chronic pulmonary embolism<br />

(E) Congestive heart failure<br />

(F) Panic disorder<br />

17. A 22-year-old woman comes to the physician<br />

because <strong>of</strong> a 10-day history <strong>of</strong> pain in multiple joints. She<br />

first had pain in her right elbow, and then her right shoulder,<br />

and now has pain, redness, and swelling in her left knee that<br />

began 2 days ago. She currently has no pain in the right<br />

shoulder and elbow. There is no history <strong>of</strong> trauma. She is<br />

sexually active, and she and her partner use condoms for<br />

contraception inconsistently. Examination <strong>of</strong> the left knee<br />

shows warmth, erythema, tenderness, and s<strong>of</strong>t-tissue<br />

swelling. Range <strong>of</strong> motion <strong>of</strong> the knee is limited to 10<br />

degrees <strong>of</strong> flexion. The remainder <strong>of</strong> the examination,<br />

including pelvic examination, shows no abnormalities.<br />

Arthrocentesis <strong>of</strong> the knee joint yields 10 mL <strong>of</strong> cloudy fluid<br />

with a leukocyte count <strong>of</strong> 18,300/mm 3 (97% segmented<br />

neutrophils). Microscopic examination <strong>of</strong> the leukocytes<br />

within the joint fluid is most likely to show which <strong>of</strong> the<br />

following?<br />

(A) Acid-fast bacteria<br />

(B) Cuboidal positively birefringent crystals<br />

(C) Gram-negative diplococci<br />

(D) Gram-positive cocci in clusters<br />

(E) Needle-shaped negatively birefringent<br />

crystals<br />

18. A 47-year-old woman comes to the physician for a<br />

routine health maintenance examination. She feels well and<br />

has no history <strong>of</strong> serious illness. Her mother, brother, and<br />

sister have hypertension. The patient's pulse is 84/min, and<br />

blood pressure is 138/85 mm Hg. Examination shows no<br />

abnormalities. The most appropriate recommendation is<br />

decreased intake <strong>of</strong> which <strong>of</strong> the following?<br />

(A) Calcium<br />

(B) Carbohydrates<br />

(C) Potassium<br />

(D) Protein<br />

(E) Sodium<br />

260


19. A 32-year-old man comes to the physician because<br />

<strong>of</strong> a 12day history <strong>of</strong> abdominal cramps and bloating,<br />

diarrhea, and flatulence. He says that he started a new<br />

exercise program 2 weeks ago and has been consuming a<br />

high quantity <strong>of</strong> yogurt bars, peanut butter, and protein- and<br />

calorie-enriched milk shakes to "bulk up." He has no history<br />

<strong>of</strong> serious illness and takes no medications. His temperature<br />

is 37°C (98.6°F). The abdomen is distended, nontender, and<br />

tympanitic to percussion. Bowel sounds are increased. The<br />

remainder <strong>of</strong> the examination shows no abnormalities. Which<br />

<strong>of</strong> the following is the most likely cause <strong>of</strong> this patient's<br />

symptoms?<br />

(A) Allergy to peanuts<br />

(B) Fungal overgrowth in the small<br />

bowel<br />

(C) Incarcerated hernia<br />

(D) Irritable bowel syndrome<br />

(E) Lactase deficiency<br />

20. A 22-year-old college student comes to student<br />

health services because <strong>of</strong> a 7-day history <strong>of</strong> low-grade<br />

fever, sore throat, fatigue, and general malaise. One month<br />

ago, she had a painless vulvar ulcer that resolved<br />

spontaneously; she has been otherwise healthy. Her last<br />

menstrual period was 3 weeks ago; she uses tampons<br />

regularly. She is sexually active and has had three partners<br />

since the age <strong>of</strong> 15 years; she uses an oral contraceptive.<br />

Her temperature is 38°C (100.4°F), pulse is 100/min, and<br />

blood pressure is 110/60 mm Hg. Examination shows a rash<br />

over the palms and soles and mild cervical<br />

lymphadenopathy. Pelvic examination shows no<br />

abnormalities. Which <strong>of</strong> the following is the most appropriate<br />

pharmacotherapy?<br />

(A) Acyclovir<br />

(B) Dexamethasone<br />

(C) Interferon<br />

(D) Penicillin<br />

(E) Zidovudine (AZT)<br />

261


Answer Form for <strong>Medicine</strong> Subject Test Sample Questions<br />

(Questions 1-20)<br />

1. _____<br />

2. _____<br />

3. _____<br />

4. _____<br />

5. _____<br />

6. _____<br />

7. _____<br />

8. _____<br />

9. _____<br />

10. _____<br />

11.<br />

12.<br />

13.<br />

14.<br />

15.<br />

16.<br />

17.<br />

18.<br />

19.<br />

20.<br />

262


Answer Key for <strong>Medicine</strong> Subject Test Sample Questions<br />

(Questions 1-20)<br />

1. D 11. B<br />

2. B 12. D<br />

3. A 13. A<br />

4. C 14. A<br />

5. B 15. E<br />

6. G 16. C<br />

7. E 17. C<br />

8. G 18. E<br />

9. E 19. E<br />

10. E 20. D<br />

263

Hooray! Your file is uploaded and ready to be published.

Saved successfully!

Ooh no, something went wrong!